The Official Study Guide For All SAT Subject Tests PDF

You might also like

Download as pdf
Download as pdf
You are on page 1of 650
acl m Ul Makers of the SAT Subject Tests” PULP en eed (CottegeBoard S > The Official Study Guide for all SAT Subject Tests The College Board The College Board is a not-for-profit membership association whose mission is to connect students to college success and opportunity. Founded in 1900, the College Board is composed of more than 5,600 schools, colleges, universities and other educational organizations. Each year, the College Board serves seven million students and their parents, 23,000 high schools, and 3,800 colleges through major programs and services in college admissions, quidance, assessment, financial aid, enrollment, and teaching and learning. Among its best-known programs are the SAT®, the PSAT/NMSQT® and the Advanced Placement Program® (AP®). The College Board is committed to the principles of excellence and equity, and that commitment is embodied in all of its programs, services, activities and concerns. For further information, visit www.collegeboard.com. Copies of this book are available from your bookseller or may be orcered from College Board Publications, P.O, Box 88900, Plano, TX 86901. 800 326-7155. The book may also be ordered online through the Celloge Board Store at wwnw-collegeboard com. The price is $18.95. Editorial inquiries concerning this book should be addressed to the College Board, SAT Program, 45 Columbus Avenue, New York, New York 10023-5992, (© 2006 The Collec Board All rights reserved, College Board, Advanced Placement Program, AP, SAT, and the acomn logo ate registered trademarks ofthe College Board. inspiring minds, SAT Preparation Booklet, SAT Reasoning Test, SAT Subjact Testa, Score Choice, The Official SAT Question of the Day, The Official SAT Subject Tests in Mathematics Levels 1 & 2 Study Guide, The Official SAT Subject Tests in U.S. and World History Study Guid, and Tho Official Study Guide forall SAT Subject Tosts aro trademarks owned by the College Board. PSATINMSOT ie a registered trademark of the College Board and National Merit Scholarship Corporation, All other products and services may be trademarks oftheir respective owners, Visit the College Boatd on the Web: wivw.collegeboard.com, 1SBN-49; 978-0-87447-756-6 ISBN-10:0-87447-756-5 Library of Congress Card Catalog Number: 2006016433 Pinte in the United States of America Distributed by Macmillan 1987654 Contents The SAT Subject Tests™ . You're in Good Company Who Develops the Tests? Keep the Tests in Perspective Who Requires the SAT Subject Tests When to Take the Tests How to Register for the Tests Score Choice™: A New Score Reporting Feature collegeboard.com........... Tests You Can Take : Which Tests Should You Take? Get Ready. . ‘Avoid Surprises......... A Practice Test Can Help How to Keep Your Nerves in Check Checklist: Acceptable Photo IDs Checklist: What | Need on Test Day On Test Day 10 Tips for Taking the Test . 7 Ways to Pace Yourself... . After the Tests What's Your Score? Should You Take the Tests Again? Contacting the College Board SAT Subject Tests Schedule Chapter Chapter Chapter Chapter Rowen Chapter Chapter Chapter Chapter ecariwe|®s Chapter Chapter 10 Chapter 11 Chapter 12 Chapter 13 Chapter 14 Chapter 15 Chapter 16 Answer Sheets div Literature. . United States History World History Mathematics Mathematics Level 1 Mathematics Level 2, Biology E/M. Chemistry ...0..eeccececeees Physics. Chinese with Listening ........ French . French with Listening German . German with Listening Italian Japanese with Listening. ..... Korean with Listening Latin Modern Hebrew .......... . Spanish Spanish with Listening The SAT Subject Tests” The SAT Subject Tests” give you the opportunity to show colleges what you know about specific subjects and how well you can apply that knowledge. They provide a fair, reliable ‘measure of your academic achievement in high school and are good predictors of your academic performance in college. Your SAT Subject Test scores are just one factor that colleges consider in the admissions process. They also look at your high school grades, SAT" scores, teacher recommendations, etc, Many colleges use SAT Subject Tests for course placement and selection. This book provides information and guidance to help you study for and familiarize yourself with the SAT Subject Tests. You'll also find official practice tests that will help you get comfortable with the tests’ format, so you feel better prepared on test day. You're in Good Company Every year, hundreds of thousands of college-bound students take SAT Subject Tests. ‘The SAT Subject Tests are part of the SAT Program of the College Board, a not-for-profit, membership association of more than 5,600 schools, colleges, universities, and other educational associations. Every year, the College Board serves seven million students and their parents, 23,000 high schools, and 3,800 colleges through major programs and services in college admissions, guidance, assessment, financial aid, enrollment, and teaching and learning. Who Develops the Tests? ‘Test development committees typically composed of teachers and college professors are appointed for the different Subject Tests. The test questions are written and reviewed by each Subject Test Development Committee, outside experts, and external staff. The SAT Subject Tests are not expected to change in major ways during the next few years, but minor revisions may be made in the content, types of questions, or testing schedule. REMEMBER ‘The Official Study Guide for All SAT Subject Tests Keep the Tests in Perspective Colleges that require Subject Test scores do so because the scores are useful in making admissions or placement decisions. Even schools that don't require the tests often review them during the application process because the scores can give a fuller picture of your academic achievement. The Subject see eee Tests are a particularly helpful tool for admissions and placement programs because the tests aren't tied to specific textbooks, grading procedures, or Many colleges and universities use instruction methods, but are still tied to the subject's curriculum, The tests provide level ground on which the Subject Test colleges can compare your scores with those of - students who come from schools and backgrounds scores to assist that may be far different from yours. with placement and Your test scores are only one of the criteria that help - colleges decide whether to admit you. Most college guidance, rather admissions officers pay more attention to high school than foradmissions, _s#des than to test results. Your extracurricular activities and letters of recommendation are also considered Who Requires the SAT Subject Tests Most college catalogs include information about admissions requirements, including which Subject Tests are needed or recommended. In addition, the College Board provides a number of resources where you can search for information about Subject Test requirements at specific colleges: + Visit the Web + Visit College Search at www.collegeboard.com. + Purchase a copy of the The College Board College Handbook. (es of the colleges and universities that interest you. Ifyou have questions or concerns about admissions policies, contact college admissions officers at individual schools. They are usually pleased to meet with students interested in their schools. o2 ‘The SAT Subject Tests When to Take the Tests Most students take the Subject Tests toward the end of their junior year or at the start of their senior year. If possible, take tests such as United States history, world history, biology, chemistry, and physics right after your courses end, when the content is still fresh in your mind. For other subjects, such as languages, you would probably do better afier you have studied the subject for several years. This book suggests ways you can prepare for each Subject Test. Before taking a test ina subject you haven't studied recently, ask your teacher's advice about the best time to take the test. Then begin to thoroughly review the course material several weeks before the exam. How to Register for the Tests There are several ways to register for the SAT and the SAT Subject Tests. + Visit the College Board’s Web site: www.collegeboard.com. + Register by telephone (for a fee) if you have registered previously for an SAT or an SAT Subject Test. Call toll free from anywhere in the United States 866-756-7346. From outside the You may take the United States, call 212-713-7789. + If you don't have ready access to the Internet, test again if you you can find registration forms in the SAT Paper don't do as well as Registration Guide. You can find the booklet in a guidance office at any high school or by writing to: you expected. College Board SAT Program eee eee eee P.O. Box 025505 ‘Miami, FL 33102 You will need to indicate the specific Subject Tests you plan to take on the test date you select. You may take one, two, or three tests on any given test date; your testing fee will vary accordingly. Except for the Language ‘Tests with Listening, you may change your mind on the day of the test and instead select from any of the other Subject Tests offered that day. You will be asked to fill out or update a Student Questionnaire when you register for the test. Although it’s optional, the questionnaire can help you compare your college plans and preferences to the characteristics of the colleges to which you sent scores. Your answers will not influence your test score and the information will be provided to colleges only with your permission. ‘The Listening Tests require special material, so they are only offered once a year, in November. You can take one only if you register for it by the regular deadline. Go to wwwcollegeboard.com or look in the current Registration Booklet for information. 32> UsAaNANAY ‘The Official Study Guide for All SAT Subject Tests Score Choice™: A New Score Reporting Feature ‘The College Board has introduced Score Choice™, a new feature that gives you the option to choose the scores you send to colleges by test date for the SAT and by individual test for the SAT Subject Tests—at no additional cost. Designed to reduce your test day stress, Score Choice gives you an opportunity to show colleges the scores you feel best represent your abilities. Score Choice is optional, so if you don't actively choose to use it, all of your scores will be sent automatically with your score report. Since most colleges only consider your best scores, you should still feel comfortable reporting scores from all of your tests. collegeboard.com On collegeboard.com, you can register for the SAT, find information about other tests and services, try The Official SAT Question of the Day”, browse the College Board Store (where you can order The College Board College Handbook, The Official SAT Subject Tests in Mathematics Levels I & 2 Study Guide", or The Official SAT Subject Tests in U.S. and World History Study Guide"), and send e-mails with your questions and concerns. Once you create a free online account, you can print your SAT admission ticket, see your scores, and send them to schools. Which colleges are right for you? College Search, at www.collegeboard.com, offers ‘two ways to find out, The College MatchMaker finds colleges that meet all of your needs. If you are already familiar with a school, use the College QuickFinder for updates of essential information. Both methods help you to find the latest information on more than 3,800 colleges, as well as other easy access to related tools. How will you pay for college? While you're at the College Board Web site, look at the Financial Aid EasyPlanner to help you organize your college finances, It can help you find answers to such questions as: What does the school of your choice cost? How much can you save? How much can you and your family afford to pay? How much can your family afford to borrow for your education? What scholarships are available to you? ‘The SAT Subject Tests Tests You Can Take SAT Subject Tests measure how well you know a particular subject area and your ability to apply that knowledge. SAT Subject Tests aren't connected to particular textbooks or teaching methods. The content of each test evolves to reflect the latest trends in teaching and learning and to reflect high school curricula. The tests fall into five general subject areas: English Languages Literature co Sees History Franch Ccinese United States History German French World History Modern Hebrew German Mathematics Italian Japanese Mathomatics Level 1 Latin Korean Mathematics Level 2 Spanish Spanish Science Biology E/M Chemistry Prysies ‘The Language Tests with Listening include Chinese, French, German, Japanese, Korean, and Spanish, These tests have a listening section that takes about 20 minutes and a reading section that takes 40 minutes. Modern Hebrew, Italian, and Latin are offered as reading only tests. French, German, and Spanish are offered as reading only or reading and listening tests. The chart on page 14 lists specifics about all the Language Tests. The Language with Listening Tests are offered only in November at designated test centers. Calculators are required for the Subject Tests in Mathematics. You cannot use a calculator for any other SAT Subject Test. The Subject Tests in Mathematics have been developed with the expectation that most students use graphing calculators. ‘The Biology E/M test lets you choose questions with an ecological emphasis (Biology E) or a molecular emphasis (Biology M) in the same test. This means you can answer questions in the area in which you feel best prepared. Biology E/M has 80 questions: 60 are for everyone who takes this test. The remaining 20 questions emphasize either ecology or molecular biology. 5) REMEMBER ‘The Official Study Guide for All SAT Subject Tests Which Tests Should You Take? Before deciding which tests to take, make a list of all the colleges you're thinking about attending. Then review their catalogs to find out whether they require Subject Test scores for admission and, if so, how many and which ones. : Use your list of colleges and their admissions If you are strong in requirements to help plan your testing schedule. You may want to adjust your schedule to meet colleges’ a particular subject, requirements. For example, a college you're planning you may want to to apply to may require a language Subject Test score for admission, or the college might exempt you from add that Subject a freshman course requirement if you do well on a particular Subject Test. Test score to your And don't forget, you can enhance your college college portfolio. portfolio by taking Subject Tests in subject areas that see eee eee youknow very well. a6 ‘The SAT Subject Tests Get Ready Give yourself several weeks before the tests to read the course material and the suggestions in this book. The rules are different for the SAT Subject Tests than for the tests you've taken in high school. You're probably used to answering questions in order, spending more time answering the hard questions and, in the hopes of getting at least partial credit, showing all your work. When you take the SAT Subject Tests, it’s OK to move around within the test section and to answer questions in any order. Keep in mind that the questions go from easier to harder, You receive one point for each question answered correctly. For each question that you answer incorrectly, a fraction of a point is subtracted from the total number of correct answers. No points are added or subtracted for unanswered questions. If your final raw score includes a fraction, the score is rounded to the nearest whole number. Avoid Surprises Know what to expect. Become familiar with the test and test-day procedures. You'll boost your confidence and feel a lot more relaxed + Know how the tests are set up. All SAT Subject Tests are one-hour multiple- choice tests. ‘The first page of each Subject Test includes a background questionnaire. You will be asked to fill it out before taking the test. The information is for statistical purposes only. It will not influence your test score. Your answers to the questionnaire will assist us in developing future versions of the test. + Learn the test directions. The directions for answering the questions in this book are the same as those on the actual test. If you become familiar with the directions now, you'll leave yourself more time to answer the questions when you take the test. + Study the sample questions. The more familiar you are with question formats, the more comfortable you'll feel when you see similar questions on the actual test. + Get to know the answer sheet, At the back of this book, you'll find a set of sample answer sheets. The appearance of the answer sheet in this book may differ from the answer sheet you see on test day. + Understand how the tests are scored. You get one point for each right answer and lose a fraction of a point for each wrong answer. You neither gain nor lose points for omitting an answer. Hard questions count the same amount as easier questions, A Practice Test Can Help Find out where your strengths lie and which areas you need to work on. Do a run-through of a Subject Test under conditions that are close to what they will be on test day. + Set aside an hour so you can take the test without interruption. Make sure you can complete the test in one sitting. ? ‘The Official Study Guide for All SAT Subject Tests + Prepare a desk or table that has no books or papers on it. No books, including dictionaries, are allowed in the test room. + Read the instructions that precede the practice test. On test day, you will be asked to do this before you answer the questions. + Remove and fill in an answer sheet from the back of this book. You can use one answer sheet for up to three Subject Tests. + For the Mathematics Tests, use the calculator that you plan to use on test day. + For the Language with Listening Tests, familiarize yourself with the CD player you will use on test da + Use a clock or kitchen timer to time yourself. This will help you pace yourself and get used to taking a test in 60 minutes. How to Keep Your Nerves in Check It’s natural to be nervous. A bit of a nervous edge can keep you sharp and focused. Do brief review on the day before the test. Look through the sample questions, explanations, and test directions in this book or on the College Board Web site, Keep the review brief; cramming the night before the tests is unlikely to help your performance and might even make you more anxious. The night before test day, prepare everything you need to take with you. You will need: + your admission ticket + an acceptable photo ID (see page 9) + Two No. 2 pencils and soft erasers. Do not bring pens or mechanical pencils. + a watch without an audible alarm + an acceptable calculator with fresh batteries (if you are taking the Mathematics Tests) + a portable CD player with fresh batteries (if you are taking a Language with Listening Test) Know the route to the test center and any instructions for finding the entrance. Check the time your admission ticket specifies for arrival. Arrive a little early to give yourself time to settle in. Get a good night’s sleep. ‘The SAT Subject Tests Acceptable Photo IDs Driver's license (with your photo) State-issued ID Valid passport Schoo! ID card Student ID form that has been prepared by your school on schoo! stationery and includes your photo and the school seal (Go to www.collegeboard,com to see an example.) "REMINDER What I Need on Test Day | Make a copy of this box and post it somewhere noticeable. I Need I Have Appropriate photo ID Admission ticket ‘Two No. 2 pencils with clean soft erasers Calculator with fresh batteries (for the Mathematics Tests only) CD player with fresh batteries (for the Listening Tests only) Watch (without an audible alarm) ‘Snack Directions to the test center Instructions for finding the entrance on weekends | am leaving the house at ___ am | I | I ! \ ' | I | | I \ \ \ **Be on time or you can't take the test.** ! | bee EE EEE REMEMBER ‘The Official Study Guide for All SAT Subject Tests On Test Day You have good reason to feel confident. You're thoroughly prepared. You're familiar with what this day will bring. You are in control. Keep in Mind ‘You must be on time or you can't take the test. Leave yourself plenty of time for mishaps and emergencies, Think positively. If you are worrying about not doing well, then your mind isn't on the test. Be as positive as possible. Stay focused. Think only about the question in front of you. Letting your mind wander will cost you time. Concentrate on your own test. The first thing some students do when they get stuck on a question is to look around to see how everyone else is doing. What they usually see is that others seem busy filling in their answer sheets, Instead of being concerned that you are not doing as well as everyone else, keep in mind that everyone works at a different pace. Your neighbors may not be working on the question that puzzled you. They may not even be taking the same test. Thinking about what others are doing distracts you from working on your own test. Make an Educated Guess Educated guesses are helpful when it comes to taking tests with multiple-choice questions; however, All questions random guesses are not a good idea. To correct for random guessing, a fraction of a point is subtracted for each incorrect answer. That means random one point. guessing—guessing with no idea of an answer that might be correct—could lower your score. The best approach is to eliminate all of the choices that you know are wrong. Make an educated guess from the remaining choices. If you can't eliminate any choice, move on. are worth Cell phones are not allowed to be used in the test center or testing room. If your cell phone is on, your scores will D0 || be canceled, d q 4a a | gi al ‘The SAT Subject Tests ys Fw 0 Tips FOR TAKING THE TEST Read carefully. Consider all the choices in each question. Avoid careless mistakes that will cost you points. Answer the easier questions first. Work on less time-consuming questions before moving on to the more difficult ones. Eliminate choices that you know are wrong. Cross them out in your test book so that you can clearly see which choices are left. Make educated guesses or skip the question. If you have eliminated the choices -- iH You are in control. 2 that you know are wrong, guessing is your best Come prepared. [rj strategy. If you cannot eliminate any of the answer choices, it is better to skip the question. Pace yourself. = Keep your answer sheet neat. The answer sheet is scored by a machine, which can't tell the difference between an answer and a doodle. If the machine reads what look like two answers for one question, it will consider the question unanswered. Use your test booklet as scrap paper. Use it to make notes or write down ideas. No one else will look at what you write. Check off questions as you work on them, This will save time when you go back to questions you've skipped, Check your answer sheet regularly. Make sure you are in the right place. Check the number of the question and the number on the answer sheet every few questions. This is especially important when you skip a question, Losing your place on the answer sheet will cost you time and even points. Work at an even, steady pace and keep moving. Each question on the test takes a certain amount of time to read and answer. Good test-takers develop a sense of timing to help them complete the test. Your goal is to spend the most time on the questions that you are most likely to answer correctly. Keep track of time, During the hour that each Subject Test takes, check your progress occasionally so that you know what point you are at and how much time is left. Leave a few minutes for review toward the end of the testing period. If you erase all of your answers to a Subject Test, that's the same as a request to cancel the test. All Subject Tests taken with the erased test will also be canceled nud = IMPORTANT. Guess wisely. 0 +» wm REMEMBER ‘The Official Study Guide for All SAT Subject Tests More About Pacing No matter how much time a test takes, the experience will be enhanced and give you an even stronger chance at success if you pace yourself. a Ways O PACE YOURSELF Set up a schedule. Know when you should be a quarter of the way through and halfway through. Every now and then, check your progress against your schedule. Begin to work as soon as the testing time begins. Reading the instructions and getting to know the test directions ahead of time will allow you to do that. Work at an even, steady pace. After you answer the questions you are sure of, move on to those for which you'll need more time. Skip questions you can’t answer. You might have time to return to them, Remember to mark them in your test booklet so you'll be able to find them later. As you work on a question, cross out the answers you can eliminate. Go back to the questions you skipped. Guess, if you can, to eliminate some of the answer choices. Leave time in the last few minutes to check your answers to avoid mistakes. Check your answer sheet. Make sure your answers are dark and completely filled in. Erase completely. Diz ‘The SAT Subject Tests After the Tests Most, but not all, scores will be reported online several weeks after the test date. A few days later, a full score report will be available to you online. Your score report will also be mailed to you and your high school, and to the colleges, universities, and scholarship programs that you indicated on your registration form or on the correction form attached to your admission ticket, The score report includes your scores, percentiles, and interpretive information. What's Your Score? Scores are available for free at www.collegeboard.com several weeks after each SAT is given. You can also get your scores—for a fee—by telephone. Call Customer Service at 866-756-7346 in the United States. From outside the United States, dial 212-713-7789, Some scores may take longer to report. If your score report is not available online when expected, check back the following week. If you have not received your mailed score report by eight weeks after the test date (by five weeks for online reports), contact, Customer Service by phone at 866-756-7346 or by e-mail at sat@info.collegeboard.org. Should You Take the Tests Again? Before you decide whether to retest, you need to evaluate your scores. The best way to evaluate how you really did on a Subject Testis to compare your scores to the admissions or placement requirements, or average scores, of the colleges to which you are applying ‘You may decide that with additional work you could do better by taking the test again. Contacting the College Board If you have comments or questions about the tests, please write to us at The College Board SAT Program, | P.O. Box 025505, Miami, FL 33102; or e-mail us at sat@info.collegeboard.org 13) ‘The Official Study Guide for All SAT Subject Tests SAT Subject Tests Schedule Subject Literature United States . . . . . . History Word History * * Mathematics Level 1[ * * * * * Mathematics Level 2 | # * * * * . Biology E/M * * * * * * Chemistry * * * * * * Physics * * * * * * res French * * * * * Geman * ‘Modern Hebrew * tralian * Latin * * Spanish * * > * > Chinese * French * Geiman > Japanese * Korean * Spanish * NOTES 1. You can take up to three SAT Subject Tests on a single test day 2, You must indicate which test or tests you plan to take when you register, but, except forthe Language Tests with Listening, you can change which tests you actualy take. 43, You may only use calculator for Mathematics Level | and Mathematics Level 2 Subject Tests, You can take these tests without a calculator, but that will put you ata disadvantage. 4. You must bring an acceptable CD player ifyou are taking a Language with Listening test. p14 Chapter 1 Literature Purpose ‘The Subject Test in Literature measures how well you have learned to read literary works from different periods and cultures, There is no prescribed or suggested reading list. Format This one-hour test consists of approximately 60 multiple-choice questions based on six to eight reading selections. About half of the selections are poetry and half are prose. Selections include complete short poems or excerpts from various works, including longer poems, stories, novels, nonfiction writing, and drama, You are not expected to have read or studied particular poems or passages that appear on the test. Extensive knowledge of literary terminology is not essential, but the test does assume a good working knowledge of basic terminology. All questions are based on selections from original works written in English from the Renaissance to the present. The date printed at the end of each passage or poem is the original publication date, or in some cases, the estimated date of composition. The set of 4 to 12 questions per selection usually covers these aspects of a text: + meaning—overall effect and argument or theme + form—structure and genre (how one part develops from or differs from another) + narrative voice—the characterization of the speaker, the possible distinction between the speaker and the author, the speaker's attitude and tone + characters represented—distinguishing traits and the techniques by which the character is presented and the traits revealed + characteristic use of language—imagery, figures of speech, and diction + contextual meaning—specific words, phrases, and lines within a passage or poem 15D ‘The Official Study Guide for All SAT Subject Tests Content All of the questions are five-choice completion questions, which fall into three categories: + Regular multiple-choice questions ask you to choose the best response + NOT or EXCEPT questions ask you to select the inappropriate choice from five choices + Roman numeral questions ask you which statement or combi be the best response ation of statements may Source of Questions Sharia ast Sai os Renaissance and 17th Century 30 20th Century 40 Sans on Approximate Percentage of Test” The distribution of passages may vary in different editions of the test. The chart abowe indicates typical cor average content. The Subject Test in Literature included in this book contains 61 questions based on seven selections—"A Divine Mistress” by Thomas Carew, an excerpt from Middlemarch by George Eliot, an excerpt from Our Country's Good by Timberlake Wertenbaker, an excerpt from the Autobiography of Benjamin Franklin, an excerpt from Invisible Man by Ralph Ellison, “Of English Verse” by Edmund Waller, and “Daybreak” by Gary Soto. As frequently happens when tests are composed of lengthy sets of questions based on relatively few selections, the distribution of passages in a particular test differs somewhat from the typical or average content summarized in the chart. The test in this book, for example, contains one more selection from English Literature than from American ature. How to Prepare + Read widely. It is best to prepare by close, critical reading in English and American literature from a variety of historical periods and genre. + There is no suggested reading list. + Have a working knowledge of basic literary terminology, such as speaker, tone, image, irony, alliteration, stanza, and so on. D16 Literature + Familiarize yourself with the test directions in advance. The directions given in this book are identical to those that appear on the test. Score ‘The total score is reported on the 200-to-800 scale. m ‘The Official Study Guide for All SAT Subject Tests Sample Questions ‘The James Merrill poem below and many of the questions that follow it are fairly easy: however, some of the other passages and questions used in the Subject Test in Literature are likely to be more difficult. James Merrill was. twentieth-century American poet; therefore, according to the content chart on page 16, all of the questions on this poem would be classified as American Literature, ‘Twentieth Century, Poetry. ‘The directions used in the test book precede the poem, Directions: This test consists of selections from literary works and questions on their content, form, and style, After reading each passage or poem, choose the best answer to each question and fill in the corresponding circle on the answer sheet. Note: Pay particular attention to the requirement of questions that contain the words NOT, LEAST, or EXCEPT. Questions 1-6. Read the following poem carefully before you choose your answers. Kite Poem “One is reminded of a certain person,” Continued the parson, settling back in his chair With a glass of port, “who sought to emulate Line The sport of birds (it was something of a chore) (5) By climbing up on a kite. They found his coat Two counties away; the man himself was missing.” His daughters tittered: it was meant to be a lesson To them—they had been caught kissing, or some such nonsense, The night before, under the crescent moon. (10) So, finishing his pheasant, their father began This thirty-minute discourse ending with A story improbable from the start. He paused for breath, Having shown but a fow of the dangers. However, the wind Blew out the candles and the moon wrought changes (15) Which the daughters felt along their stockings. Then, Thus persuaded, they fled to their young men Waiting in the sweet night by the raspberry bed, And kissed and kissed, as though to escape on a kite. Dig Literature 1. The attitude of the parson (line 2) toward the “certain person” (lines 1-6) is one of (A) admiration (B) anxiety () di (D) curiosity () grief dain Choice (C) is the correct answer to question 1. In order to warn his daughters of the danger of imprudent behavior, the parson uses the tale of the person who climbed up on akite. It is unlikely, given this purpose, that he would feel either “admiration,” “anxiety,” “curiosity,” or “grief” for the man, and nothing in the pocm suggests that the parson had any of these feelings. His attitude is one of disdain for a person whose behavior he regards as foolish, 2, The descriptive detail “settling back in his chair/With a glass of port” (lines 2-3) underscores the parson's (A) authority (B) complacency (©) hypocrisy (D) gentleness (E) indecisiveness Choice (B) is the correct answer to question 2. The poem suggests that the parson is a rather rigid, formal man focused on his own comforts and ignorant of his daughters’ needs. Itcan be inferred from the context that complacency—unaware self-satisfaction—is. one element of his character. There is no evidence in the poem that the parson is either hypocritical, gentle, or indecisive. Out of context, the quotation from the poem might be interpreted as behavior associated with someone in a position of authority. In context, however, the parson is more notable for his lack of authority—his daughters titter when he lectures and ignore his advice. 19) ‘The Official Study Guide for All SAT Subject Tests 3. The chief reason the parson’s daughters “tittered” (line 7) is that they (A) were embarrassed to have been caught kissing (B) knew where the missing man in their father’s story was (©) wanted to flatter their father (D) did not take their father's lecture seriously (E) took cruel pleasure in the kite flyer’s disaster Choice (D) is the correct answer to question 3. It is the most plausible explanation of why the daughters “tittered”—they did not take their father’s lecture seriously. This view is supported by the daughters’ actions—as soon as their father paused for breath, they did what his “thirty-minute discourse” warned them not to do, There is no indication in the poem that choices (B) or (E) are true, and if the daughters had wanted to flatter their father, as choice (C) claims, they certainly would not have tittered during his serious lecture. If choice (A) were true, itis unlikely that the daughters would have “fled to their young men” so quickly the second time. 4, The speaker's tone suggests that the reader should regard the parson’s “thirty-minute discourse” (Line 11) as (A) scholarly and enlightening (B) serious and important (C)_ entertaining and amusing (D) verbose and pedantic (E) grisly and morbid Choice (D) is the correct answer to question 4. The speaker's tone suggests that the reader should regard the parson’s “thirty-minute discourse” as “verbose and pedantic.” The parson is presented as one who speaks at length, telling “improbable” stories and taking 30 minutes to show “but a few of the dangers" he wanted to warn his daughters about. He uses lengthy phrases such as “emulate/The sport of birds” when a simple verb such as “fly” would have sufficed. The parson might well have intended his discourse to seem “scholarly and enlightening,” choice (A), and “serious and important,” choice (B), but neither the daughters nor the speaker suggests that the parson succeeded, and the reader has no reason to assess the effectiveness of the discourse differently from the speaker and the daughters. The reader may be entertained and amused by the speaker's account of the discourse, but that response is not the same as being amused by the discourse itself, as choice (C) states. Choice (E) is implausible. 20 Literature 5. The daughters are “persuaded” (line 16) by (A) their own fear of danger (B) the fate of the kite flyer (©) their own natural impulses (D) the parson’s authority (BE) respect for their father Choice (C) is the correct answer to question 5. The daughters are “persuaded” by their own natural impulses. According to the poem, “the moon wrought changes/Which the daughters felt along their stockings” (lines 14-15), These natural impulses were, ironically, more persuasive than the long discourse delivered by their father in an attempt to dissuade them. The daughters, like the kite flyer, are attracted to the possibility of “escape on a kite” (line 18) and are not deterred by solemn and tedious warnings of danger. 6. All of the following are elements of opposition in the development of the poem EXCEPT (A) indoors . . outdoors (B) talking. . kissing (©) caution. adventure (D) work. . play (B) settling back ... flying Choice (D) is the correct answer to question 6. It is the only opposition that is not evident in the poem. Actions such as “climbing up on a kite” and “kissing...under the crescent moon” might be regarded as forms of play, but the poem really does not offer any contrasting examples of work. Choices (A), (B), (C), and (E) illustrate the contrasting actions and attitudes of the parson on the one hand and the daughters or the kite flyer, or both, on the other. ap Literature Test Practice Helps The test that follows is an actual, recently administered SAT Subject Test in Literature. To get an idea of what it’s like to take this test, practice under conditions that are much like those of an actual test administration. dz Set aside an hour when you can take the test uninterrupted. Make sure you complete the test in one sitting Sit at a desk or table with no other books or papers. Dictionaries, other books, or notes are not allowed in the test room. Tear out an answer sheet from the back of this book and fill it in just as you would on the day of the test. One answer sheet can be used for up to three Subject Tests. Read the instructions that precede the practice test. During the actual administration you will be asked to read them before answering test questions. Time yourself by placing a clock or kitchen timer in front of you. After you finish the practice test, read the sections "How to Score the SAT Subject Test in Literature” and “How Did You Do on the Subject Test in Literature?” The appearance of the answer sheet in this book may differ from the answer sheet you see on test day. LITERATURE TEST The top portion of th Wn of the answer sheet that you will use in taking the Literature Test must be filled in exactly as shown in the illustration below, Note carefully that you have to do all of the following on your answer sheet. 1. Print LITERATURE on the line under the words “Subject Test (print).” 2. In the shaded box labeled “Test Code” fill in four circles: —Fill in cirele 3 in the row labeled V. —Fill in circle 1 in the row labeled W. —Fill in circle 1 in the row labeled X. —Fill in circle D in the row labeled Y. ‘Subject Test (pin) LITERATURE 3. Please answer the two questions below by filling in the appropriate circles in the row labeled Q on the answer sheet. The information you provide is for statistical purposes only and will not affect your How many semesters of English courses that were predominantly devoted to the study of literature have you taken from grade 10 (o the present? (If you are studying literature in the current semester, count the ‘current semester as a full semester.) Fill in only one circle of circles 1-3 © One semester or less —Pill in circle 1 © Two semesters —Fill in circle 2. ‘© Three semesters or more —Fill in circle 3, jon Il Of the following, which content areas made up a significant patt (at least 10 percent) of the literature ‘you read in your English classes in grades 10-12 ? Fill in as many circles as apply. ‘© British and/or North American writers writing before 1800 —Fill in circle 4. ‘© European writers in translation —Fill in circle 5. ‘© African American and Black writers —Fill in circle 6, © Ethnic American writers (Hispanic American, Asian American, American Indian, etc.) —Fill in circle 7. ‘© Latin American writers in translation —Fill in circle 8, ‘© Writers from Africa or India writing in English —Fill in cirele 9. When the supervisor gives the signal, turn the page and begin the Literature Test. There are 100 numbered circles on the answer sheet and 61 questions in the Literature Test. Therefore, use only circles | to 61 for recording your answers. othe pe gal 23 \s on their content, form, and style. After fion and fill in the corresponding cirele on the LITERATURE TE: Directions: ‘This test consists of selections from literary works and questi reading each passage or poem, choose the best answer to each q answer sheet. Note: Pay particular attention to the requirement of questions that contain the words NOT, LEAST, or EXCEPT. Questions 1-7. Read the following poem carefully 2. The word “framed” in Line 5 is particularly appro- before you choose your answers. priate in this context because it suggests the A Divine Mistress (A) deceitfulness and evil intentions (B) imagination and fertility (C) virtue and benevolence (D) fickleness and ethereal nature (E) physical shape and aesthetic completeness In Nature’s pieces stil I see ‘Some error that might mended be; ‘Something my wish could still remove, Alter or add; but my fair love 5 Was framed by hands far more divine For she hath every beauteous line. bine 3. Which of the following could be substituted for “had been” (line 7) without changing the meaning? ‘Yet I had been far happier Had Nature, that made me, made her. (A) was ‘Then likeness might (that love creates) (B) will be 10 Haye made her love what now she hates; (C) have been Yet, | confess, | cannot spare (D) would have been From her just shape the smallest hairs (E) ought to be Nor need I beg from all the store Of heaven for her one beauty more. 4. All of the following contrasts appear in the first 15, She hath too much divinity for me: ten Hines of the poem EXCEPT You gods, teach her some more humani y (A) nature and divinity (B) error and perfection (tet) (©) similavty and difference (D) love and hate () innocence and experience 1. Which of the following best restates the meaning of ines I and 2? (A) The natural world contains imperfections. (B) The natural world has only the meaning that poets give it. (C) The natural world has been systematically destroyed by humans. (D) The natural world was an accident of divinity. (E) The poetic imagination can ereate o destroy the natural world. 3XAC2 rauorized spying ot uso oi GO ONTO THE NEXT PAGE [sy pt of ts pope nega D2 = (S LITERATURE TEST—Continued 5. Which of the following best states the wish of the 7. The unannounced intention of the speaker inthis speaker in lines 7-14 ? pocm is to (A) He wants the woman to be even more beau (A) commend a woman for her impeceable virtue tiful than she is, (B) praise a woman for her unequaled beauty (B) He wants the woman to ignore other men. (C) make a woman more receptive to his passion (C) He wants the woman to be both beautiful and (D) delude a woman into thinking that he loves her accessible. (E) flatter a woman so that she will have a better (D) He does not want the woman to Tove him in opinion of herself the same way he loves her. (B) He does not want the woman to be s0 vain. 6. The speaker's tone in lines 15-16 is best described as (A) bitter sarcasm (B) amused indifference (C) dignified solemnity (D) playful exasperation (E) cold rationality GO ONTO THE NEXT PAGE > 25D Une Questions 8-17. Read the following passage carefully before you choose your answers. He had not had much foretaste of happiness in his previous life. To know intense joy without a strong, bodily frame, one must have an enthusiastic soul Mr. Casaubon had never had a strong bodily frame, and his soul was sensitive without being enthusiastic: it was too languid to thrill out of self-conseiousness into passionate delight; it went on fluttering in the ‘swampy ground where it was hatched, thinking oF its ‘wings and never flying. His experience was of that pitiable kind which shrinks from pity, and fears most Of all that it should be known: it was that proud narrow sensitiveness which has not mass enough to spare for transformation into sympathy, and quivers thread-like in small currents of self-preoccupation or at best of an egoistic scrupulosity. And Mr. Casaubon hhad many scruples: he was capable of a severe self- restraint; he was resolute in being a man of honour according to the code; he would be unimpeachable by any recognised opinion. In conduct these ends had been attained; but the difficulty of making his Key fo all Mythologies unimpeachable weighed like lead upon his minds and the pamphlets—or “Parerga hie called them—by which he tested his public and deposited small monumental records of his march, were far from having been seen in all their signifi- cance. He suspected the Archdeacon of not having read them; he was in painful doubt as to what was really thought of them by the leading minds of Brasenose,? and bitterly convinced that his old acquaintance Carp had been the writer of that depre~ iatory recension which was kept locked in a small drawer of Mr. Casaubon's desk, and also in a dark closet of his verbal memory. These were heavy impressions to struggle against, and brought that melancholy embitterment which is the consequence of all excessive claim: even his religious faith wavered ‘with his wavering trust in his own authorship, and the consolations of the Christian hope in immortality ‘seemed to Iean on the immortality of the still unwritten Key to all Mythologies. ast) "Greek erm for supplementary or econdary works 2 geollege at Oxford LITERATURE TEST—Continued 8, The passage is best described as an example of (A) character analysis (B) historical commentary (C) allegorical drama (D) interior monologue (E) political satire 9, By the end of the passage, Casaubon (A) crude and inconsiderate (B) insecure and self-centered (©) temperamental and rebel (D) sensitive but self-confident (E) ambitious but generous 10, In the context of the passage, the image of the fluttering bird “thinking of its wings and never flying” (lines 8-9) is most suggestive of (A) Casaubon’ lifelong aversion to physic: activities (B) Casaubon’s control over his imagination and ‘emot (C) the limiting effect of Casaubon’s self- (D) the nobility of Casaubon’s physical and striving (E) the liberating influ arly intellect al \ce of Casaubon’ s schol- 11, Casaubon’s struggle to make “his Key 10 all Mythologies unimpeachable” (lines 20-21) can be best viewed as an example of his (A) dedication to an outdated code of honor (B) enthusiasm only for intellectual pursuits (C) unrealistic expectations of achievement (D) rivalry with the Archdeacon (E) tendency toward procrastination GO ONTO THE NEXT PAGE > 12, The phrase Casaubon sted his public” (line 23) means that (A) tried the patience of those who were eagerly waiting for his Key to all Mythologies (B) evaluated his popularity with the general reading public (C) examined the public on its knowledge of mythological literature (D) attempted to confirm publicly the validity of his scholarly project (E) compared the publ phlets with that of the scholarly community 13, The statement “These wer ‘lines 33-36) can be bes excessive claim” interpreted as (A) 4 paraphrase of the negative responses to Casaubon’s work (B) a generalization about human nature appli- ‘eable to Casaubon’s personality (©) anallusion to Casaubon’s earlier years of ‘unhappiness (D) a denunciation of harsh crities like Carp (E) a plea for sympathy for Casaubon 14, The repeated reference to smallness—“shrinks” ine 10), “has not mass enough’ (line 12), "small ccurrents” (line 14), “small monumental record: line 24), and “small drawer" (lines 31-32)—has the cumulative effect of reinforcing the theme of Casaubon’s (A) aptitude for analyzing only the small details. in his life (B) intellectual and emotional Limitations (C) modesty and lack of idealism (Dy hetoie struggle against the weight of public opin (E) inability to live up to his reputation as an ‘eminent scholar [ny pt of ts pope egal” LITERATURE TEST—Continued 15, 16. In context, the comment “the consolations of the Christian hope in immortality seemed to lean on the immortality of the still unwritten Key 10 all “Mythologies” (lines 37-40) suggests the narrator's ef that (A) Casaubon’ scholarly work would be a contri- ‘bution to the Christian community (B) Casaubon hoped that his work, when con pleted, would be as widely read as the Bible uubon relied desperately on his religious faith to help him complete his manuscript (D) the importance Casaubon ascribed to his work was greatly inflated (E) the suffering and humiliation endured by Casaubon would make his work immortal © Which of the following references is NOT me phorical but actually describes a physical act performed by Casaubon? (a) (B) luticring in the swampy ground” (lines 7-8) uivers thread-like in small currents” (ines 13-14) (© “weighed like lead” (line 21) (D) “deposited small monumental records of his march’ (line 24) (£) “locked in a small drawer” (lines 31-32) The narrator's attitude toward Casaubon is primarily one of (A) ambivalence (B) puzzlement (©) revulsion (D) bitter disparagement (E) incisive eriticism GO ONTO THE NEXT PAGE > 27) Line 1s ‘Questions 18-27. Read the following dramatic excerpt carefully before you choose your answers. In this scene, the somewhat inebriated officers of an eighteenth-century Australian penal colony debate the merits of Second Lieutenant Ralph Clark's proposal to stage a play—George Farquhar’s “The Recruiting Officer” —using the conviets as actors. REVEREND JOHNSON: What is the plot, Ralph? RALPH: I's about this recruiting officer and his friend, and they are in love with these two young ladies from Shrewsbury and after some difficulties, they marry them, REV, JOHNSON: It sanetions Holy Matrimony then? RALPH: Yes, yes, it does. REV. JOHNSON: That wouldn't do the convicts any harm. I'm having such trouble getting them to marry instead of this sordid cohabitation they're so used 10. ROSS: Marriage, plays, why not a ball for the conviets! CAMPBELL: Euuh, Boxing. ARTHUR PHILLIP: Some of these men will have finished their sentence in a few years. They will become members of Society again, and help create a new society in this colony. Should we not encourage ‘them now to think in a free and responsible manner? TENCH: | don’t see how a comedy about two lovers will do that, Arthur ARTHUR PHILLIP: The theatre is an expression of civilisation, We belong to a great country which has, spawned great playwrights: Shakespeare, Marlow Jonson, and even in our own time, Sheridan. The conviets will be speaking a refined, literate language and expressing sentiments of a delicacy they are not used to. It will remind them that there is more to life than crime, punishment. And we, this colony of a hundred will be watching this together, for a few hours, ‘we will no longer be despised prisoners and hated. gaolers. We will laugh, we may be moved, we may even think a little. Can you suggest something else that will provide such an evening, Watkin? lay pat of tus page ees | D28 LITERATURE TEST—Continued DAWES: Mapping the stars gives me more enjoy- ‘ment, personally, ‘TENCH: I'm not sure it’s a good idea having the conviets laugh at officers, Arthur. CAMPBELL: No. Pheeoh, insubordination, heh, ehh, no discipline. ROSS: You want this viee: themselves? jdden vermin to enjoy COLLINS: They would only laugh at Sergeant Kite. RALPH: Captain Plume is a most attractive, noble fellow. REV. JOHNSON: He's not loose, is he Ralph? I hear many of these plays are about rakes and encourage Ioose morals in women. They do get married? Before, that is, before. And for the right reasons. RALPH: They marry for love and to secure wealth. REV. JOHNSON: That's all right. TENCH: I would simply say that if you want to build civilisation there are more important things than a play. If you want to teach the conviets something, teach them to farm, to build houses, teach them a sense of respect for property, teach them thrift so they don’t ‘eat a week’s rations in one night, but above all, teach them how to work, not how to sit around laughing. at comedy, ARTHUR PHILLIP: The Greeks believed that it was duty to watch a play. It was a kind of work in that it required attention, judgement, patience, all social virtues. TENCH: And the Greeks were conquered by the ‘more practical Romans, Arthur. COLLINS: Indeed, the Romans built their bridges, but they also spent many centuries wishing they were Greeks. And they, after all, were conquered by bar- Darians, or by their own corrupt and small spirits TENCH: Are you saying Rome would not have fallen iff the theatre had been better? GO ONTO THE NEXT PAGE > RALPH (very loud): Why not? (Everyone looks at him and he continues, fast and nervously.) In my own small way, in just a few hours, [ have seen something change. I asked some of the convict women to re ‘me some lines, these women who behave often no better than animals. And it scemed to me, as one or ‘two—I’m not saying all of them, not at all—but ‘one or (Wo, saying those well-balanced lines of Mr. Farquhar, they seemed to acquire a dignity, they seemed— they seemed to lose some of their corruption, There was one, Mary Brenham, she read so well, perhaps this play will keep her from selling herself to the first marine who offers her bread. FADDY (under his breath): She'll sell he him, instead. Ato ROSS: So that’s the way the wind blows — CAMPBELL: Hooh. A tempest. Hooh, RALPH: (over them): I speak about her, but in a small way this could affect all the convicts and even, ourselves, we could forget our worries about the sup- plies, the hangings and the floggings, and think of ourselves at the theatre, in London with our wives and children, that is, we could, ewh— ARTHUR PHILLIP: Transcend— RALPH: Transcend the darker, euh—transeend, the— JOHNSTON: Brutal— RALPH: The brutality and remember— remember our better nature COLLINS: England, RALPH: England, (1988) [Urautovied copying oy vuoe ol ‘sry poof hs page siege LITERATURE TEST—Continued 18. The positions articulated by Reverend Johnson. and Arthur Phillip are alike in that both men (A) believe that great artis defined by its morality (B) assume the convicts will value the beliefs of the characters they observe (C) see entertainment as a distraction that will pacify the convicts (D) think that presenting harsh social realities will lead to moral reformation (E) rely on empirical evidence for their credibility 19. Arthur Phillip’s invocation of Shakespeare, Marlowe, Jonson, and Sheridan (lines 23-25) ‘ean be most aptly described as (A) an irrefutable argument about the value of ‘drama (B) a pedantic displ (C) an irrelevant aside (D) an appeal to a tradition of national culture (E) a justification of lovers’ comedies of expert learning 20. The arguments advanced about performing a play invoke all of the following issues EXCEPT the (A) representation of immoral behavior (B) desirability of reforming convicts (C) values of the colonizing country (D) possibility of transcending local circumstances (E) merit of staging plays about convicts 21. If Tench’s position in lines 52-59 is valid, then, by contrast, the views of Ralph and Arthur Phillip are (A) without historical precedent (B) not sufficiently pragmatic (©) morally isresponsible (D) philosophically questionable (B) self-interested 22. The tone of Tench's question in lines 70-71 can best be described as (A) sardonically contentious (B) dispassionately curious (C) sympathetically supportive (D) personally offended (E) humorously credulous GO ONTO THE NEXT PAGE > 290 ® This passage is reprinted for your In this scene, the somewhat inebriated officers of an eighteenth-century Australian penal colony debate the merits of Second Lieutenant Ralph Clark's proposal to stage a play—George Farquhar’s “The Recruiting Officer” —using the convicts as actors. REVEREND JOHNSON: What is the plot, Ralph? RALPH: It’s about this recruiting officer and his friend, and they are in love with these two young ladies from Shrewsbury and after some difficulties, they marry them, REV. JOHNSON: then? It sanetions Holy Matrimony RALPH: Yes, yes, it does, REV. JOHNSON: That wouldn’t do the conviets any harm. I'm having such trouble getting them to marry instead of this sordid cohabitation they're so used to, ROSS: Marriage, plays, why not a ball for the convicts! CAMPBELL: Euuh. Boxing. ARTHUR PHILLIP: Some of these men will have finished their sentence in a few years, They will become members of Society again, and help create a new society in this colony. Should we not encourage them now (o think in a free and responsible manner? TENCH: | don’t see how a comedy about wo lovers, will do that, Arthur. ARTHUR PHILLIP: ‘The theatre is an expression of civilisation, We belong to a great country which has, spawned great playwrights: Shakespeare, Marlowe, Jonson, and even in our own time, Sheridan, The convicts will be speaking a refined, literate language and expressing sentiments of a delicacy they are not used (0. I¢ will remind them that there is more (o Tif than crime, punishment. And we, this colony of a few hhundred will be watching this together, for a few hours. ‘we will no longer be despised prisoners and hated. gaolers. We will laugh, we may be moved, we may even think a little, Can you suggest something else that will provide such an evening, Watkin’? [Grated copying aroun ol ary ar of hs pogo sea »30 LITERATURE TEST—Continued in answering the remaining questions DAWES: Mapping the stars gives ment, personally. TENCH: I’m not sure it’s a good ide: conviets laugh at officers, Arthur. CAMPBELL: No. Pheeoh, insubordination, heh, ehh, no discipline. ROSS: You want this vice-ridden vermin to enjoy themselves? COLLINS: They would only laugh at § ergeant Kite, RALPH: Captain Plume is a most atiractive, noble fellow. REV. JOHNSON: He’s not loose, is he Ralph? I hear many of these plays are about rakes and encourage loose morals in women. They do get married’? Before, that is, before. And for the right reasons. RALPH: They marry for love and to secure wealth, REV. JOHNSON: That's all right TENCH: | would simply say that if you want to build «civilisation there are more important things than a play. If you want to teach the convicts something, teach ‘them to farm, to them a sense of respect for property, teach them thrift so they don’t ceat a week's rations in one night, but above all, teach them how to work, not how to sit around laughing at comedy, ARTHUR PHILLIP: The Greeks believed that it was, acitizen’s duty to watch a play, It was a kind of work in that it required attention, judgement, patience, all social virtues. ‘TENCH: And the Greeks were conquered by the more practical Romans, Arthur. COLLINS: Indeed, the Romans built their bridges, but they also spent many centuries wishing they were Greeks. And they, after all, were conquered by bar- barians, or by their own corrupt and small spirits. TENCH: Are you saying Rome would not have fallen if the theatre had been better? GO ONTO THE NEXT PAGE > 100 RALPH (very loud): Why not? (Everyone looks at him and he continues, fast and nervously.) In my own small way, in just a few hours, I have seen something. change. I asked some of the convict women to read, ‘me some lines, these women who behave often no better than animals, And i two—T'm not saying all of them, not at all—but cone or two, saying those well-balanced lines of Mr. Farquhar, they seemed to acquire a dignity, they seemed—they seemed to lose some of their corruption. There was one, Mary Brenham, she read so well, perhaps this play will keep her from selling herself to the first marine who offers her bread— cemed to me, as one or FADDY (under his breath): She'll sell herself to him, instead, ROSS: So that’s the way the wind blows— CAMPBELL: Hooh. A tempest. Hooh, RALPH: (over them): I speak about her, but in a ‘small way this could affect all the convicts and even, ‘ourselves, we could forget our worries about the sup- plies, the hangings and the floggings, and think of ourselves at the theatre, in London with our wives and children, that is, we could, eu ARTHUR PHILLIP: ‘Transcend— RALPH; Transcend the darker, cuh—wanscend, the— JOHNSTON: Brutal— RALPH: The brutality —remember our better nature and remember— COLLINS: England, RALPH: England. (1988) LITERATURE TEST—Continued 2. 26. 27. ® Faddy’s interruption (Iines 85-86) of Ralph's reflections functions as which of the following? 1 Acomic aside IL A cynical deflation of pretension IIL A reprimand for poor taste (A) Lonly. (B) Ml only (©) Land TI only (D) Land IIL only ( Land ut Ross's comment in line 87 most probably refers this (A) appreciation of Faddy’s sense of humor (B) assessment of Mary Brenham's character (C) perception of Ralph's underlying motives (D) recognition of the validity of Ralph's ‘argument (E) indifference to the topic at hand The moral effect of speaking the well-written language of the play is most persuasively argued by (A) Reverend Johnson (B) Tench (C) Campbell (D) Collins (E) Ralph ‘The characters want to “remember . . . England” ines 99-102) because for them England is (A) no Tonger the country that Shakespeare knew (B) the ideal civilization (C) the home of the theatre’s most skilled performers () a reminder of their authority (E) the country in which comedy serves a social purpose ‘The excerpt thematically explores the (A) history of drama in recent centuries (B) importance of wholesome entertainment (C) need for reform in government (D) nature and purpose of drama itself (E) tendency of people everywhere to engage in acting GO ONTO THE NEXT PAGE > 3D Line Questions 28-35. Read the following passage carefully before you choose your answers. I elieve I have omitted mentioning that in my fist Voyage from Boston, being becal’d off Block Island, ‘our People set about eatching Cod and had up a ‘reat many. Hitherto I had stuck to my Resolution of not eating animal Food; and on this Occasion, I consider'd with my Master Tryon,” the taking every Fish as a kind of unprovok'd Murder, since none of them had or ever could do us any Injury that might justify the Slaughter. Al this seem'd very reasonable. But [had formerly been a great Lover of Fish, and when this came hot out of the Frying Pan, it smelt admirably well. I balane’d some time between Principle and Inclination: till T recollected, that when the Fish were opened, I saw smaller Fish taken ut oftheir Stomachs: Then thought I, if you eat one another, [don't see why we mayn'teat you. So I din'd upon Cod very heartily and continu'd to eat with other People, returning only now and thea occasionally to 1 vegetable Diet. So convenient a thing it isto be a reasonable Creature, since it enables one to find o ‘make a Reason for every thing one has a mind to do. «791 "The author ofa bock espousing vegetarianism 28. Asit is used in line 1 (A) leaning, bending (B) slant, slope (C) bowing, noddi (D) disposition, preference (B) decision, determination inclination” means LITERATURE TEST—Continued 29. 30. a1 Which of the following best describes the tone of the sentence “Then thought I, if you eat one nother, I don’t see why we mayn’t eat you” ines 15-16) ? (A) Witty (B) Inquiring (C) Critical (D) Defiant (E) Sincere As used in lines 20-21, the phrase a Reason for” means (A) show enthusiasm for (B) understand the outcome of (©) think of an excuse to justify (D) examine the motivation behind (E) weigh the advantages and disadvantages of In the final sentence of the passage, the speaker can be best described as (A) humorously self-aware (B) objective and matter-of-fact (C) slightly befuddled (D) selfish and immodest (E) thoroughly disillusioned GO ONTO THE NEXT PAGE > 32. Which statements about the speaker's vegetari- anism can be inferred from the passage?” 1. Ithad been adopted in response to reading Tryon’s book. II, It was based on a concern for the just treatment of animals. IIL It was chosen because it seemed to be rational bel (A) Lonly (B) only (C) Land I only (D) Hand I only (B) 1.1, and It 33. Which of the following shows the use of hyperbole? (A) “great many” (line 4) (B) “Murder” (line 7) and “Slaughter” (line 9) (C) “Injury” (line 8) and “reasonable” (line 9) (D) “admirably well” (line 12) (E) “very heartily” (line 17) = (S LITERATURE TEST—Continued 34. The tone of the final sentence is established by which of the following?” I. The use of the word “convenient” TI. The italicization of “reasonable Creature” IIL. The use of the phrase “to find or make a Reason” (A) Honly (B) Tand only (©) Land Ut only (D) I and 111 only (©) 1,1, and ttt 35. The speaker actually abandons vegetarianism, because the speaker (A) cannot find a reasonable argument for con tinuing it (B) has never been convinced by the arguments for it (C) is convinced that the big fish deserves to be (D) loves cod more than meat (E) has an appetite that outweighs abstract, principles GO ONTO THE NEXT PAGE > 33D Line wo 1s Questions 36-44. Read the following passage carefully before you choose your answers. Tam not ashamed of my grandparents for having been slaves. Iam only ashamed of myself for having at one time been ashamed. About eighty-five years ago they were told that they were i others of our country in everything pertaining to the common good, and, in everything social, separate like the fingers of the hand. And they believed it. They exulted in it, They stayed in their place, worked hard, and brought up my father to do the same. But my grandfather is the one. He was an odd old guy, my grandfather, and I am told I take after him. It was he ‘who caused the trouble. On his deathbed he called my father to him and said, “Son, after I'm gone I want you to keep up the good fight, I never told you, but our life is a war and [have been a traitor all my born days, a spy in the enemy’s country ever since I give up my gun back in the Reconstruction, Live with your head in the lion’s mouth, I want you to overcome “em with yeses, undermine "em with grins, agree ‘em to death and destruction, let ‘em swoller you till they ‘vomit or bust wide open.” They thought the old man had gone out of his mind, He had been the meekest of, ‘men, The younger children were rushed from the room, the shades drawn and the flame of the lamp turned so low that it sputtered on the wick like the old man’s breathing, “Learn it to the younguns,” he whispe1 fiercely; then he died, But my folks were more alarmed over his last ‘words than over his dying. It was as though he had not died at all, his words caused so much anxiety. I ‘was warned emphatically to forget what he had said and, indeed, this is the first time it has been mentioned outside the family circle. It had a tremendous effect, upon me, however. I could never be sure of what he meant. Grandfather had been a quiet old man who never made any trouble, yet on his deathbed he had called himself a traitor and a spy, and he had spoken of his meekness as a dangerous activity. It became a constant puzzle which lay unanswered in the back of my mind, And whenever things went well for me I remembered my grandfather and felt guilty and uncomfortable, It was as though I was carrying out his advice in spite of myself. And to make it worse, everyone loved me for it. I was praised by the most D34 LITERATURE TEST—Continued so “o g lily-white men of the town. I was considered an example of desirable conduct —just as my grand- father had been. And what puzzled me was that th: old man had defined it as treachery. When I was, praised for my conduct I felt a guilt that in some w I was doing something that was really against the wishes of the white folks, that if they had understo they would have desired me to act just the opposit: that I should have been sulky and mean, and that t really would have been what they wanted, even tho they were fooled and thought they wanted me to a as { did. It made me afraid that some day they wou look upon me as a traitor and I would be lost, Still ‘was more afraid to act any other way because they didn’t like that at all. The old man’s words were curse. On my graduation day I delivered an oration Which I showed that humility was the secret, indec the very essence of progress. (Not that I believed t —how could I, remembering my grandfather? —T only believed that it worked.) It was a great sticces Everyone praised me and I was invited to give the at a gathering of the town’s leading white citizens. It was a triumph for our whole communit (1952) 36. The narrator's central concern in the passage i (A) curiosity about his family history (B) uneasiness about his family’s care of his ds ‘grandfather (C) frustration with the limitations imposed by parents (D) a sense of being betrayed by the leading citizens of the town (E) uncertainty about how he should act 37. The simile of the hand (line 7) suggests (A) acceptance of change in social worlds (B) a rationale for a segregated social system, (C) a symbol of racial pride (D) hard work as the basis for economic prospx (E) a physical basis for similarities and differe 38. In the context of lines 1-9, the narrator is, suggesting that his grandparents (A) built an ideal life after they had been freed (B) were proud of the efforts they made to achieve their freedom (C) appeared to have adopted socially approved values (D) were unusual among the former slaves of U ‘generation (B) lived in the past rather than the present 39. The grandfather's injunction “to overcome “em with yeses, undermine “em with grins” (lines 18-19) asks for (A) optimism in the face of adversity (B) resignation when change is impossible (C) subtle imitation as a way to gain favor (D) seeming acquiescence as a means of rebellion (E) unforced graciousness toward defeated ‘opponents. 40. The fact that the narrator h mentioned his grandfather's dying words outside the family circle suggests that he has never, before now, (A) been deliberately disrespectful to his ‘grandfather (B) felt that the words were entrusted to him (C) concluded that no one would be interested in them, (D) not bothered to think about them (E) felt profoundly anxious about them 41. The lifelong behavior and the deathbed words of his grandfather, taken together, puzzle the narrator Decause they (A) imply that the grandfather was not devoted to his family (B) require the narrator to assume a position of leadership (C) seem to reflect contradictory impulses (D) prove that direct confrontations are undesirable (E) suggest that unquali ed victory is attainable = (S LITERATURE TEST—Continued 42. It can be inferred from the passage that the grand. father regarded what he called treachery as (A) an affirmative act, because the deception allows you to prevail (B) a.useless act, because those who are betrayed are too obtuse to notice (C) an innocent act, because no one is misled by it (D) an honorable act, because the behavior exhibited is friendly and agreeable (©) an unintent , because no one would knowingly engage in such dangerous behavior 43. By “worked” (line 64), the narrator means (A) pleased the leading citizens of the community (B) brought about intellectual progress (C) shocked the graduating class (D) encouraged frank discussion of bias (E) openly challenged racist assumptions 44. In the context of the passage, the sentence “It was a tiumph for our whole community” (line 67) suggests that (A) the triumph wi seemed (B) humility and triumph are irreconcilable (C) the grandfather's battle has temporarily been halted (D) formal education will reduce racial discrimination (E) the nature of language necessarily what it ssentially deceptive GO ONTO THE NEXT PAGE > 35D Une ® Questions 45-51, Read the following poem carefully before you choose your answers. Of English Verse Poets may boast, as safely vain, ‘Their works shall with the world remain; Both, bound together, live or die, ‘The verses and the prophecy. But who can hope his lines should long Last in a daily changing tongue? While they are new, envy prevails; And as that dies, our language fails. When architects have done their part, ‘The matter may betray their art; Time, if we use ill-chosen stone, ‘Soon brings a well-built palace down. Poets that lasting marble se Must carve in Latin or in Greeks We write in sand, our language grows, And, like the tide, our work o"erflows. Chaucer! his sense ean only boast, “The glory of his numbers lost Years have defaced his matchless strain, And yet he did not sing in vain, ‘The beauties which adored that age, ‘The shining subjects of his rage? Hoping they should immortal prove, Rewarded with suecess his love This was the generous poet's scope, And all an English pen can hope, ‘To make the fair approve his flame, ‘That can so far extend their fame. Verse, thus designed, has no ill fate IFit arrive but atthe date Of fading beauty; if it prove But as long-lived as present love. (1668) " Founentrcentury poet whose works, writen in Midi English refet features tat Engh as ost, 2 Poetic ispitation sry pat of is page loge 236 LITERATURE TEST—Continued 45. 47 48, In the context of line English language as ils" presents the (A) possessing a vocabulary too narrow to express the richness of human experience (B) reflecting the central weakness of a society ‘consumed by jealousy of talent (C) containing too few beauties of sound for spoken poetry to please listeners (D) imitating the worst features of languages like Latin and Greek (E) undergoing too many transformations to pre~ serve all the original qualities of a poem In the argument of the poem, the function of the third stanza is to show that (A) the art of poetry is superior to the art of architecture (B) architecture requires artistic skills as great as those of poetry (C) worldly pomp is subject to the power of time (D) art lasts only as long as its materials do (B) the choice of subject may determine the usefulness of a work of art In the fourth stanza, all of the following words are used metaphorically EXCEPT. (A) “Poets” (ine 13) (B) “marble” (line 13) * (line 14) (ine 15) ‘erflows” (line 16) In line 14, the speaker refers to Latin and Greek because (A) classical civilization is noted for its marble temples and statues (B) they are thought of as unchanging languages (C) the greatest poetry has been written in Latin and Greek: (D) time renders all languages obsolete (B) the inscriptions on tombs are frequently writ ten in Latin and Greek GO ONTO THE NEXT PAGE > 49. 50. In ines 15-16, “grows! which of the following? plies that English does (A) Becomes more refined in its vocabulary. (B) Changes inevitably with the passage of time. (C) Alters imperceptibly to reflect social transformations, (D) Evolves away from its 0 and simplicity. ns a new power of expression. sal purity ©) According to lines 29-32, what trait does “Verse” share with “fading beauty” and “present love” ? (A) Sentimental appeal to nostalgi (B) Dazzling effect on the speaker (C) Lack of recognition by fashionable society (D) Beauty that must endure hardship before triumphing (E) Value that can last only a limited time temperament LITERATURE TEST—Continued 51. Which of the following does the poe frequently employ? (A) Hyperbote (B) Apostrophe (C) Amtithesis (D) Euphemi (2) Metaphor GO ONTO THE NEXT PAGE > 37D 1s Questions $261. Read the following poem, carefully before you choose your answers. Daybreak In this moment when the light starts up In the east and rubs ‘The horizon until it catches fire, We enter the fields to hoe, Row after row, among the small flags of onion, Waving off the dragonflies ‘That ladder the air. ‘And tears the onions raise Do not begin in your eyes but in ours, In the salt blown From one blister into another; ‘They begin in knowing ‘You will never waken to bear ‘The hour timed to a heart beat, ‘The wind pressing us closer to the ground. ‘When the season ends, And the onions are unplugged from their sleep, ‘We won't forget what you failed 10 see, And nothing will heal Under the rain’s broken fingers. (1977) 52. The tone of the poem is best described as one of | (A) shock (B) anxiety (C) rationalization (D) bitemess and pain (E) resignation and apathy 53. The basie opposition in the poem is between (A) the employed and the unemployed. (B) time and timelessness (C) worker and consumer (D) misery and elation (2) machines and laborers LITERATURE TEST—Continued 34. 55. 56. 37. In Hight of the poem as a whole, the figurative language of the first stanza sets a scene with an image of (A) peace (B) creativity (C) affection (D) friction (E) chaos ‘The effect of lines 14 and 15 isto (A) illustrate the conditions that the workers have to endure (B) show how nature both helps and hinders those who work in the fields (C) suggest that while nature is changeable, human will is constant (D) imply that those who study nature will even- tually realize their own shortcomings (E) suggest that workers who have pride in what they do can withstand adversity ‘The relation between the third and fourth stanzas ight best be described as a (A) change from one explanation for the tears to another (B) contrast between experienced and anticipated pain (©) progression in time from the past to the present (D) movement from the concerns of consumers to the concerns of workers (E) shift in tone from acceptance to denial In line 17, “unplugged from their sleep” means (A) planted (B) cultivated (C) harvested (D) consumed (E) replenished GO ONTO THE NEXT PAGE > 58. Which of the following will not “heal” line 19)? 1. The problems of the workers UL The tears of “you TIL. The rift between “we” and “you” (A) Tonly (B) Land Il only (C) Land II only (D) Mand IM only (B) 1.0, and It 59. In context, the image of the “rain’s broken finger (line 20) calls attention to the (A) decreasing profit margin of farming, (B) disappointment of the workers when a erop is poor (C) difficulty of the workers’ situation (D) inconveniences that adverse weather condi- tions produce in modern life (E) failure of science in predicting the weather STOP LITERATURE TEST—Continued 660. The physical labor the speaker describes ol © pre sented as (A) an occupation that ruthlessly exploits natural (B) adifficult but satisfying way of earning a living (C) an opportunity to be at one with nature (D) a way of life that is about to become outdated (E) @ painful and unappreciated endeavor . The speaker suggests that the "you" referred to in the poem can best be characterized as (A) ignorant or unseeing (B) sentimental and foolish (©) greedy or wasteful (D) physically exhausted (E) emotionally unstable IF YOU FINISH BEFORE TIME IS CALLED, YOU MAY CHECK YOUR WORK ON THIS TEST ONLY. DO NOT TURN TO ANY OTHER TEST IN THIS BOOK. [amy prof ts poe eg 39D ACKNOWLEDGMENTS From NEW AND SELECTED POEMS by Gary Soto © 1995. Reprinte permission of Chronicle Books, San Francisco. Literature How to Score the SAT Subject Test in Literature When you take an actual SAT Subject Test in Literature, your answer sheet will be “read” bya scanning machine that will record your responses to each question. Then a computer will compare your answers with the correct answers and produce your raw score. You get one point for each correct answer. For each wrong answer, you lose one-fourth of a point. Questions you omit (and any for which you mark more than one answer) are not counted. This raw score is converted to a scaled score that is reported to you and to the colleges you specify. Worksheet 1. Finding Your Raw Test Score STEP 1; Table A lists the correct answers for all the questions on the Subject Test in Literature that is reproduced in this book. It also serves as a worksheet for you to calculate your raw score. + Compare your answers with those given in the table, Puta check in the column marked “Right” if your answer is correct. + Puta check in the column marked “Wrong” if your answer is incorrect. Leave both columns blank if you omitted the question. STEP Count the number of right answers. Enter the total here: STEP 3: Count the number of wrong answers. Enter the total here: STEP 4: Multiply the number of wrong answers by .250. Enter the product here: STEP 5: Subtract the result obtained in Step 4 from the total you obtained in Step 2. Enter the result here: STEP Round the number obtained in Step 5 to the nearest whole number. Enter the result heres The number you obtained in Step 6 is your raw score. a> ‘The Official Study Guide for All SAT Subject Tests Table A Answers to the Subject Test in Literature, Form 3XAC2, and Percentage of Students Answering Each Question Correctly Perry ern A 34 32 E 44 E 6 33 8 50 D B 34 E 40 E a1 35 e a c 44 36 E 86 0 32 37 B 66 c 5 38 c 46 A a 39 0 60 a 6 40 € 50 c 68 4 c a 1 c 7 a2 A 7 2 D 0 43 A 66 B a 8 44 A 49 4 a 49 45 E 70 15 o 34 46. o ST 16 E 59 a7 A 5 ” E 4 48 8 ™ 8 a 62 49 8 55 18 o 82 50 e 50 ao E ST 51 E i} 2 a 52 52 D 78 2 A 8 53 c 1 2B c 55 54 D 52 4 c 3 55 A 63 B E 6 56 A 29 6 a 70 7 c 2 a o 70 58 c 68 2 0 n 59 c 66 2 A 54 60 E 5 30 c 2 6 A 88 31 A 69 * These percentages are based on an analysis of the answer sheets of a representative sample of 12,064 students who took the original form of this test in December 2001, and whose mean score was 579. They may be used. asan indication ofthe relative difficulty ofa particular question. Each percentage may alsa be uscd to predict the likelihood that a typical SAT Subject Test in Literature candidate wll answer that question correctly on this edition ofthe tet. baz Literature Finding Your Scaled Score When you take SAT Subject Tests, the scores sent to the colleges you specify are reported on the College Board scale, which ranges from 200-800. You can convert your practice test score to a scaled score by using Table B. To find your scaled score, locate your raw score in the left-hand column of Table B; the corresponding score in the right-hand column is your scaled score. For example, a raw score of 21 on this particular edition of the Subject Test in Literature corresponds to a scaled score of 500 Raw scores are converted to scaled scores to ensure that a score earned on any one edition of a particular Subject Test is comparable to the same scaled score earned on any other edition of the same Subject Test. Because some editions of the tests may be slightly easier or more difficult than others, College Board scaled scores are adjusted so that they indicate the same level of performance regardless of the edition of the test taken and the ability of the group that takes it. Thus, for example, a score of 400 on one edition of a test taken at a particular administration indicates the same level of achievement as a score of 400 on a different edition of the test taken at a different administration, When you take the SAT Subject Tests during a national administration, your scores are likely to differ somewhat from the scores you obtain on the tests in this book. People perform at different levels at different times for reasons unrelated to the tests themselves. The precision of any test is also limited because it represents only a sample of all the possible questions that could be asked. 43D ‘The Official Study Guide for All SAT Subject Tests Table B Scaled Score Conversion Table Subject Test in Literature (Form 3XAC2) 81 800 32 590 3 360 80 800 3 580 2 350 59 800 30 570 1 340 58 790 29 560 a 330 57 730 28 550 4 320 56 780 7 550 2 320 58 770 6 540 3 310 54 780 Fy 530 4 300 53 750 2 520 5 230 82 780 2 510 6 280 51 740 2 510 7 280 50 730 a 500 8 270 49 720 20 490 4 260 48 70 19 480 40 20 ol 70 18 470 a 240 46 700 7 470 a2 730 45 690 6 460 13 220 4a 680 5 450 14 210 43 680 4 440 15 200 42 70 3 430. a 680 12 430 40 650 " 420 39 640 0 410 38 640 3 400 a 620 8 390 36 620 7 390 % 610 6 380 34 600 5 370 % 590 4 360 Daa Literature How Did You Do on the Subject Test in Literature? After you score your test and analyze your performance, think about the following questions: Did you run out of time before reaching the end of the test? If so, you may need to pace yourself better. For example, maybe you spent too much time on one or two hard questions. A better approach might be to skip the ones you can't answer right away and try answering all the questions that remain on the test. Then if there's time, go back to the questions you skipped. Did you take a long time reading the directions? You will save time when you take the test by learning the directions to the Subject Test in Literature ahead of time, Each minute you spend reading directions during the testis. a minute that you could use to answer questions. How did you handle questions you were unsure of? Ifyou were able to eliminate one or more of the answer choices as wrong and guess from the remaining ones, your approach probably worked to your advantage. On the other hand, making haphazard guesses or omitting questions without trying to eliminate choices could cost you valuable points. How difficult were the questions for you compared with other students who took the test? Table A shows you how difficult the multiple-choice questions were for the group of students who took this test during its national administration. ‘The right-hand column gives the percentage of students that answered each question correctly. A question answered correctly by almost everyone in the group is obviously an easier question. For example, 84 percent of the students answered question 8 correctly. But only 20 percent answered question 12 correctly. Keep in mind that these percentages are based on just one group of students. They would probably be different with another group of students taking the test. If you missed several easier questions, go back and try to find out why: Did the questions cover material you haven't yet reviewed? Did you misunderstand the directions? 45D Chapter 2 United States History Purpose ‘The emphasis of the Subject Test in United States History is on United States history from pre-Columbian times to the present as well as basic social science concepts, methods, and generalizations as they are found in the study of history. It is not tied to any single textbook or instructional approach, Format This is a one-hour test with 90 to 95 multiple-choice questions. The questions cover political, economic, social, intellectual, and cultural history as well as foreign policy. Content ‘The questions may require you to: + recall basic information and require you to know facts, terms, concepts, and generalizations + analyze and interpret material such as graphs, charts, paintings, text, cartoons, photographs, and maps + understand important aspects of U.S. history + relate ideas to given data + evaluate data for a given purpose, basing your judgment either on internal evidence, such as proof and logical consistency, or on external criteria, such as comparison with other works, established standards, and theories 47) ‘The Official Study Guide for All SAT Subject Tests Mat Paoltical History 1 Covered* ‘Approximate Percentage of Test Economic History Social History Intellectual and Cultural History Foreign Policy Periods Covered Pre-Columbian history to 1789 20 1790 to 1998 40 1899 to the present 40 * Social science concepts methods and generalizations are incorporated in this material How to Prepare ‘The only essential preparation is a sound, one-year course in U.S. history at the college- preparatory level. Most of the test questions are based on material commonly taught in US. history courses in secondary schools, although some of the material may be covered in other social studies courses. Knowledge gained from social studies courses and from ‘outside reading could be helpful. No one textbook or method of instruction is considered better than another. Familiarize yourself with the directions in advance. The directions in this book are identical to those that appear on the test. Score ‘The total score is reported on the 200-to-800 scale. Sample Questions The types of questions used in the test and the abilities they measure are described below. Questions may be presented as separate items or in sets based on quotations, maps, pictures, graphs, or tables. Directions: Each of the questions or incomplete statements below is followed by five suggested answers or completions. Select the one that is best in each case and then fill in the corresponding circle on the answer sheet. Some questions require you to know facts, terms, concepts, and generalizations, They test your recall of basic information and your understanding of significant aspects of US. history and the social studies. Question 1 is a sample of this type. United States History 1. Harriet Tubman was known as the “Moses” of her people because she (A) helped slaves escape from the South (B) was instrumental in bringing about suffrage reform, (©) advocated emigration to Africa for Black people (D) organized mass civil rights demonstrations (BE) traveled as a lay minister preaching the gospel Choice (A) is the correct answer to question 1, To answer this question you need to know that Harriet Tubman was a notable African American abolitionist. As the use of the name “Moses” may help you to remember, Tubman led many slaves to freedom in the North along the route of the Underground Railroad as referred to in choice (A). The other choices only describe activities that Tubman did not pursue. Some questions require you to analyze and interpret materials. Question 2, based on the chart below, illustrates a question that tests your ability to use these skills. Popular Vote for Presidential Electors, Georgia, 1848 and 1852 Democratic Electors Whig Electors Webster Electors 1848 44808 47538 = 1052 40516 16,660 5.324 2. Using the table above, one might conclude that the most plausible explanation for the Georgia Democrats’ victory in 1852, following their defeat in 1848, was that (A) many new voters increased the turnout in 1852, to the advantage of the Democrats (B) many voters abstained from voting in 1852, to the disadvantage of the Whigs (C)_ Webster, who had not run in 1848, drew sufficient votes from the Whigs to cost them the election of 1852 (D) the Democrats, who had run a highly unpopular candidate in 1848, ran a highly popular candidate in 1852 (E) the Democrats cast fraudulent ballots to increase their share of the votes in 1852 Choice (B) is the correct answer to question 2. To answer this question, you must analyze the electoral data given for 1848 and 1852, noting that the voter turnout dropped dramatically in 1852 and that the Whigs suffered a much larger decline in voter turnout than did the Democrats. As a consequence, the Whigs lost their majority position, Choices (A), (C), (D), and (E) are not logically consistent with this data. For example, choice (C) is incorrect because the table shows that the Democratic Electors received more votes than the Whig and Webster Electors combined. Choice (B) is the correct agp ‘The Official Study Guide for All SAT Subject Tests answer because it is the only plausible explanation for the change in the fortunes of the Georgia Democrats. Other questions test your ability to analyze material as well as your ability to recall information related to the materials, or to make inferences and interpolations based on the material. Questions 3, 4, 5, and 6 are illustrations of questions that test a combination of interpretation and recall. 3. “What is man born for but to be a reformer, a remaker of what man has made; a renouncer of lies; a restorer of truth and good, imitating that great Nature which embosoms us all, and which sleeps no moment on an old past, but every hour repairs herself, yielding every morning a new day, and with every pulsation a new life?” ‘These sentiments are most characteristic of (A) fundamentalism (B)_ Social Darwinism (© pragmatism (D)_neoorthodoxy (E)_transcendentalism Choice (E) is the correct answer to question 3. Several elements in the quotation suggest this as the correct answer. The emphasis that the quotation places on reform, on nature as a source of moral truth, and on the infinite possibilities open to people mark it as an example of the thought of the transcendentalist movement. This combination of elements is not pertinent to any of the other choices. Even if you do not know the source of the material, your understanding of the nature of transcendentalism should lead you to choice (E). »50 United States History Questions 4-5 refer to the following map. Saisie 4 Canada a Mepican Territory: 4, The controversy with Great Britain over control of the shaded section was settled during the presidency of (A) John Quincy Adams (B) James K. Polk (©) Franklin Pierce (D) James Buchanan (6) Andrew Johnson Choice (B) is the correct answer to question 4, To answer this question, you must interpret the map and recognize the shaded section as part of the Oregon territory. The Oregon dispute with Great Britain was settled during the presidency of James K. Polk. 5. To the northwest of the area shown on the map is a continental territory purchased by Secretary of State William H. Seward from. (A) Great Britain (B) Canada (C) Russia (D) France © spain Choice (C) is the correct answer to question 5. To answer this question, you must go beyond the content of the map in order to determine that the territory referred to in the question is Alaska. If you recall that Secretary of State Seward purchased the territory from Russia in 1867, you will choose the correct answer. 51D ‘The Official Study Guide for All SAT Subject Tests OF VETO MEMORY. BORN TO COMMAND. 1 avi agLiasnoo Nag KING ANDREW THE FIRST. ps2 Courtesy of the New York Historical Society United States History 6. The point of view expressed by this cartoon would probably have met with the approval of (A) Daniel Webster (B) James K. Polk (©) Martin Van Buren (D) Roger B. Taney (£) Stephen A. Douglas Choice (A) is the correct answer to question 6. To answer this question, you must first note the anti-Jackson tone of the cartoon, which portrays King Andrew the First trampling the Constitution of the United States. You must then decide which of the choices given opposed Jackson's use of the veto to return important bills to Congress. Only the Whig Daniel Webster fits that description, The others were Democrats who either supported Andrew Jackson or were politically active at a later time, Some questions require you to select or relate hypotheses, concepts, principles, or generalizations to given data. The questions may begin with concrete specifics and ask {for the appropriate concept, or they may begin with a concept and apply it to particular problems or situations, Thus, you may need to use inductive and deductive reasoning. Questions 7 and 8 are examples of questions in this category. 7. From 1870 to 1930, the trend in industry was for hours to be generally reduced, while both money wages and real wages rose. What factor was primarily responsible for this trend? (A) A reduction in profit margins (8) Minimum wage laws (©) Restriction of the labor supply (D) Increased output per hour of work (BE) Right-to-work legislation Choice (D) is the correct answer to question 7. To arrive at this answer, you must be aware that the trend referred to in the question came about primarily because of technological advances that resulted in increased productivity. None of the other answer choices satisfactorily accounts for all the conditions described in the question. 53D ‘The Official Study Guide for All SAT Subject Tests 8. Which of the following wars of the United States would fit the description of a war neither lost nor won? 1, The War of 1812 Il. ‘The Mexican War UI, ‘The Spanish-American War IV. The Second World War (A) Tonly (B) Ionly (©) Land I only (D) Hand IV only (E) Hand IV only Choice (A) is the correct answer to question 8. In answering this question, you must recognize that a war not won, though not necessarily lost, is one in which a country either fails to achieve clear victory on the battlefield or fails to sign a peace treaty that is definitive and fulfills its goals. Only the War of 1812 is an illustration of the kind of war defined by the question. That war was ended by The Treaty of Ghent, which provided for the Status quo ante bellum, or a return to things as they had been before the war. Some questions require you to judge the value of data for a given purpose, either basing your judgment on internal evidence, such as accuracy and logical consistency, or on external criteria, such as accepted historical scholarship. Question 9 is an illustration of this kind of question. 9, Which of the following would most probably provide the widest range of information for a historian wishing to analyze the social composition of an American city in the 1880s? (A) The minutes of the city council (B) A debutante’s diary (©) A manuscript census tabulating the residence, ethnicity, occupation, and wealth of each city resident (D) Precinctlevel voting returns in a closely contested mayoral election held in a presidential election year (E) A survey of slum housing conditions carried out by a Social Gospel minister in the year following several epidemics Choice (C) is the correct answer to question 9. In answering this question, you must be able to eliminate from consideration choices that offer information about the city that is either irrelevant or less relevant than other options to understanding the social composition of the city, choices (A) and (D). You must also eliminate choices that offer p54 United States History relevant information but are limited to a particular section of the population of the city, choices (B) and (E). Choice (C) contains the widest range of information about the social composition of a city. 55D United States History Test Practice Helps The test that follows is an actual, recently administered SAT Subject Test in United States History. To get an idea of what it's like to take this test, practice under conditions that are much like those of an actual test administration 56 Set aside an hour when you can take the test uninterrupted. Make sure you complete the test in one sitting Sit at a desk or table with no other books or papers. Dictionaries, other books, or notes are not allowed in the test room. Tear out an answer sheet from the back of this book and fill it in just as you would on the day of the test. One answer sheet can be used for up to three Subject Tests. Read the instructions that precede the practice test. During the actual administration you will be asked to read them before answering test questions. Time yourself by placing a clock or kitchen timer in front of you. After you finish the practice test, read the sections "How to Score the SAT Subject Test in United States History” and “How Did You Do on the Subject Test in United States History?” The appearance of the answer sheet in this book may differ from the answer sheet you see on test day. UNITED STATES HISTORY TEST ‘The top portion of the section of the answer sheet that you will use in taking the United States History Test must be filled in exactly as shown in the illustration below. Noie carefully that you have to do all of the following on your answer sheet |. Print UNITED STATES HISTORY on the line under the words, “Subject Test (print).” . In the shaded box labeled “Test Cod —Fill in circle 4 in the row labeled V. Fill in circle 7 in the row labeled W. —Fill in circle 4 in the row labeled X. Je A in the row labeled Y. 2" fill in four circles: ‘Subject Test (pin) UNITED STATES HISTORY 3. Please answer the «wo questions below by filling in the appropriate circles in the row labeled Q on the answer sheet. formation you provide is for statistical purposes 01 ot affect you ‘score on the test. ion | How many semesters of United States History have you taken from grade 9 to the present? (If you are taking United States History this semester, count it as a full semester.) Fill in only one cirele of circles 1-4. . —Fill in circle 1 . —Fillin cirele 2, © Three semesters —Fill in cirele 3. ‘© Four or more semesters —Fill in cirele 4. Question © One or more semesters of government —Fillin circle 5, ‘© One or more semesters of economics Fill in circle 6, © One or more semesters of geography Fill in circle 7, One or more semesters of psychology in circle 8, ‘© One or more semesters of sociology or anthropology —Fillin circle 9, If you have taken none of these social studies courses, leave the circles 5 through 9 blank. ‘When the supervisor gives the signal, turn the page and begin the United States History Test. There are 100 numbered circles on the answer sheet and 90 questions in the United States History Test. Therefore, use only cireles 1 to 90 for recording your answers. ry pr ofthis poe seg 57D s: Each of the que: 1. During the seventeemth and eighteenth cen- turies, the English colonial system was based ‘most explicitly on the economic and political principles of (A) mereantilism (B) free trade (C) salutary neglect (D) enlightened despotism (B) physiocracy 2. The concept of the separation of powers, as articulated by the framers of the Constitution, (A) right of free speech (B) right of freedom of assembly (C) organization of the national government three branches (D) separation of church and state (E) political rights of confederated states 3WAC2 Dnoutorzed spying or eure dss UNITED STATES HISTORY TEST S jons or incomplete statements completions. Select the one that is best in each ease and then fill in the correspondin slow is followed by fi suggested answers or circle on the answer sheet 3. The Trail of Tears refers to the (A) movement of slaves from eastern states into the West after the 1820's (B) relocation of Cherokee Indians from the Southeast to settlements in what is now Oklahoma: (C) difficult movement of s Oregon Trail (D) Lewis and Clark's expedition during the Jefferson presidency (B) movement of thousands of people across the Great Plains during the California gold rush ters over the 4. All of the following reformers are correctly paired ‘with the reform issue with which they were most involved EXCEPT (A) Elizabeth Cady Stanton. . suffrage (B) Sojourner Truth .. antislavery (C) Harriet Beecher Stowe (D) Emma Willard (E) Dorothea Dix . . treatment of people with ‘mental and emotional disabi GO ONTO THE NEXT PAGE > 5 UNITED STATES HISTORY TEST—Continued BLACK POPULATION IN THE UNITED STATES, 1820 ~ 1850 FREE AND SLAVE (in thousands) 1800 1,603. 1,602 1500 7 1247 1241 1200 5 900 600 300 ] 200 “|226 208, 120 113 166 153 186 1820 1830 i840 1850) Census Year EES] Fematesin slavery [7] Free femates I Mates in stavery [ZZ] Free mates 5. Which of the following statements about the period from 1820 to 1850 is supported by the diagram above? (A) The percentage ofthe Black population held in slavery declined (B) The ratio of Black males to Black females retained feily constant (©) Black males were more ikely than Black females tobe fee (©) The numberof Black females doubled every 20 years (©) The'otl Black population in each census exceeded two million [mero tis pepe ge GO ONTO THE NEXT PAGE 59D © UNITED STATES HISTORY TEST—Continued © vac Le 6. The cartoon above illustrates popular reaction 0 publication of a theory by (A) Malthus (B) Darwin (©) Marx 7. An the early years of the twentieth century, the 8, President Franklin D. Roosevelt attempted to majority of female workers employed outside “pack” the Supreme Court in 1937 for which Of the home were of the following reasons? (A) widowed (A) He wanted to make sure that New Deal laws (B) divorced would be found constitutional (C) married with young children (B) He believed that additional conservative ried with grown children justices would balance the Court (©) He owed favors to many political friends who were trained lawyers (D) He wanted to inerease minority repre: fon the Court (B) He wanted socialists and communists to be represented on the Court. ary ar of ths page sea GO ONTO THE NEXT PAGE > D60 tation ® UNITED STATES HISTORY TEST—Continued © TARDLTED iy ALL my LIFES 9, The cartoon above makes which of the following points about federal aid policies in the years following the Second World War? (A) The federal government has always been reluctant to offer financial aid fo farmers. (B) American farmers have never needed government support to main- tain self-sufficieney (©) Much federal aid goes to individuals in forms other than welfare payments, (D) Congress should cease paying both welfare and price supports. (E) Price supports paid to farmers are not a significant percentage of the federal budget. Urauinored coping oc any bar ofthis ge fea GO ONTO THE NEXT PAGE > 61D 10. The United States supported the Bay of Pigs invasion in 1961 in an attempt to overthrow (A) Nikita Khrushehev (B) Gamal Abdel Nasser (C) Fidet Castro (D) Chiang Kai-shek (8) Ngo Dinh Diem 11. Single women and widows in the eighteenth- century British North American colonies had the legal right to (A) hold political offi (B) serve as Protestant ministers ©) vore (D) own property (B) serve on juries 12. Which of the following was most responsible for the repeal of the Stamp Act in 1766? (A) The dumping of the East India Company's {ea into Boston Harbor (B) Petitions by the First Continental Congress to Parliament (C) The boycott of British imports (D) Acceptance by the Massachusetts colonists of alternate taxation (B) Pressure on Parliament by the king [Drawtberied sonrng a reuse ary ar of ths page sea 62 UNITED STATES HISTORY TEST—Continued B 15 ® Marbury v. Madison was a significant turning point in the interpretation of the United States Constitution because it (A) upheld the separation of church and state (B) validated the principle of the free press (C) established the practice of judicial review (D) abolished the slave trade (B) overturned the Alien and Sedition Acts ‘After the Civil War, sharecropping was an impor: tant element in the agricultural economy of which of the following regions? (A) The Middle Atlantic states (B) The South (©) The Great Plains, (D) The West Coast (B) New England “Texas has been absorbed into the Union in the inevitable fulfillment of the general law which is rolling our population westward. ... It was disintegrated from Mexico in the natural course of events, by a process perfectly legitimate on its own part, blameless on ours. ... [ts] incor poration into the Union was not only inevitable, but the most natural, right and proper thing in the world.” The statement above an expression of (A) Social Darwinism (B) antiabolitionism: (C) federalism (D) Manifest Destiny (B) self-determination GO ONTO THE NEXT PAGE UNITED STATES HISTORY TEST—Continued Regent by permisson ofthe New York Historie Socksy 16. The nineteenth-century cartoon above supports which of the following conclusions about the United States economy? Unauthorized spying or eu ary ar ofthis pogo eat (A) The emergence of strong unions resulted in loss of productivity. (B) The emergence of big government resulted in loss of liberties. (©) Railroad corporations wielded tremendous power in American society. (D) Southern planters wielded tremendous power in the Senate. (E) Rapid urbanization led to unsanitary conditions in ‘many cities GO ONTO THE NEXT PAGE > 63D ES HISTORY TEST—Continued g UNITED ST 17. “There was never the least attention paid to what MAJOR HOUSEHOLD EXPENDITUR! ‘was cut up for sausage; there would come all the 1900 and 1928 way back from Europe old sausage that had been rejected, and that was moldy and white—it would 1900 be dosed with borax and glycerine, and dumped > Bicycles $0 into the hoppers, and made over again for home Wrinser ‘and washboard $5 consumption. ... There would be meat stored in Ce ane $5 great piles in rooms; and the water from leaky Sowing muchine (mechanical) $25 roofs would drip over it, and thousands of rats would race about on it.” Total $105 The above . 1928 We passage above is most likely excerpted fron (A) John Steinbeck’s The Grapes of Wrath Automobile 3 Radio $75 (B) Theodore Dreiser's An American Tragedy Phonograph $50 (C) Jane Addams’ Twenty Years at Hull-House Warhine pnachine $150 (D) Lincoln Steffens’ The Shame of the Cities alin cleaner 3 (E) Upton Sinclair's The lunal ‘Vacuum cleaner $50 Upton Sinclair's The Jungle Sewing machine (electric) $60 Other electrical equipment $25 Telephone (year) $35 | Pot STS 18, The chart above shows the major household expenditures of a middle-class American fami in 1900 and a similar family in 1928. Which of the following is an accurate statement supporte by the chart? (A) Families needed more mechanical help wit housework in 1928 than they did in 1900 because they had less domestic help. (B) Inflation caused a significant increase in th rices of most household goods by 1928. (C) Many families moved from rural to urban areas between 1900 and 1930 in search of employment opportunities (D) By 1928 more consumer goods were avail able to families than had been available it 1900. (E) Increased consumer spending was a major ‘cause of the stock market erash of 1929. ONTOTHE NEXT PA UNITED STATES HISTORY TEST—Continued S 19. “Rosie the Riveter” was a nickname given during the Second World War to (A) American women who did industrial work in the 1940°s (B) American women who cared for soldiers ‘wounded in battle (©) amachine that increased the speed of construc- tion work (D) awoman who was a popular radio talk-show host of the 1940°s (E) awoman who broadea pat ofthe page aga GO ONTO THE NEXT PAGE > 65D UNITED STATES HISTORY TEST—Continued UPMCorbis-Betmann 20. The picture above illustrates efforts in the 1960"s to organize a boycott that focused attention on the (A) long hours of grocery clerks and stock clerks. (B) problems of Mississippi Valley fruit growers (C) labor shortages in produce transport companies (D) plight of migrant farmworkers (B) problems of West Coast win [Uratnarieed coping or reusina [so owroTHEwexTPace sry ar of his pe seg GO ONTO THE NEXT PAGE D66 21. Which of the following statements the response of Native Americans to the con- tinued settlement of Europeans in North America during the eighteenth century? (A) Native Americans traded with the French and the English as a means of maintaining their autonomy. (B) Native Americans in the southern part of New France negotiated treaties with the French that allowed the peaceful expansion of the European timber trad (©) Some Native Americans created a horse based nomadic culture in the Northeast. (D) Native Americans in the Great Plains assim ilated with the European settlers (E) The Iroquois did not adopt European firearms and metal tools, in an effort maintain their ‘own traditions. Unauthorized springs sry bar of hs page eg UNITED STATES HISTORY TEST—Continued 23. In the seventeenth century, the British colon the Chesapeake Bay region became economically viable due to the (A) adoption of representative government (B) introduction of tobacco cultivation (C) flourishing trade with American Indians (D) export of dried cod and whale tallow (B) cultivation of cotton All of the following were aspects of the Con- stitution that was submitted to the states for ratification in 1787 EXCEPT (A) the ability to levy taxes (B) congressional authority to declare war (©) a two-term limit for Presidents (D) provision for impeachment of the President (E) provision for presidential State of the Union messages GO ONTOTHE NEXT PAGE > 67) UNITED STATES HISTORY TEST—Continued 24, 25. Somme Dos “The St, Lowis Act Mvscam, Gift Bank of Ames The painting above, which shows an antebellum election, site, supports which of the following statements? (A) Women were equal participants in the voting process. (B) The sale and provision of liquor was prohibited on lection day (©) Party workers had to remain at least 50 yards away from the polling place. (D) There were no property restrictions for male voters. (B)_Blections were a welcome social event as well asa political obligation. The introduction of canals, railroads, and new factory technology in the mid-nineteenth century affected which of the following regions LEAST? (A) New England (B) New York and Pennsylvania (©) New Jersey and Delaware (D) The South, (B) The Midwest GO ONTO THE NEXT PAGE > (Questions 26-27 are based on 1 © passage below 26. “Unsanitary housing, poisonous sewage, contam- inated water, infant mortality, the spread of contagion, adulterated food, impure milk, smoke-laden air, ill+ ventilated factories . .. unwholesome crowding, pros titution and drunkenness are the enemies which the ‘modern cities must face and overcome, would they survive, Logically their electorate should be made up of those who . . .have at least attempted to care for children, to clean houses, to prepare foods, to isolate the family from moral dangers. ... To test the elector’s fitness to deal with this situation by ability to bear arms is absurd. .. . City housekeeping hhas failed partly because women, the traditional housekeepers, have not been consulted as to its multiform activities. The men have been carelessly indifferent to much of this civic housekeeping, as they have been carelessly indifferent to the details of the household. Jane Addams, 1906, anv pat of ts page UNITED STATES HISTORY TEST—Continued . The st reflects the main Which of the following argument of the passage? (A) Men should spend less time away from home and participate more fully in domestic life. (B) Women should be able to vote in order to apply their proven housekeeping abilities to the civie sphere. (C) Military solutions to social problems are ineffective because they ignore moral issues, (D) Solving the problems of cities mostly depends, ‘on providing for poor children. (E) Modem cities have been saved from ruin only by the involvement of women in civic assage above suggests that Jane Addams would probably have supported all of the following EXCEPT (A) military preparedness (B) woman suffrage (C) prohibition (D) settlement houses (B) the Pure Food and Drug Act GO ONTO THE NEXT PAGE > UNITED STATES HISTORY TEST—Continued PITTSBURG: A: CITY ASHAMED ae : : LINCOLN STEFFENS'S exposure ‘of another type of municipal graf- ing; how Pittsburg differs from St. Louis and Minneapolis. 4) THE END OF THE WORLD, by Professor Neweumb. A power fol story, yet a sc prediction; pictures by the famous Fzench artist, Henri Lanos, IDA M. TARBELL on the Stand> ard tactics which brought on the famous oil crisis of 1878. SIX SHORT STORIES Culver Pictures In 28. The articles appearing in this 1905 issue of MeClure’s Magazine illu Of the following trends in the early twentieth-century United States EXCEPT (A) Popular magazines were begin reform. (B) Reform of municipal city governments was a growing concer. (C) Exposure of monopolistic business practices Was beginning to draw public attention, jentific methods were increasingly called on to lend credibility to all, sorts of theories. (E) Reformers of both goverment and society enjoyed widespread support among leading indust ing to turn their altention to issues of ©) sv pat of tis ge ts egal GO ONTO THE NEXT PAGE 70 S SS) UNITED STATES HISTORY TEST—Continued ‘Questions 29-30 are based on the chart below IMMIGRATION TO THE UNITED STATES BY AREA OF ORIGIN Year AllCoumries Europe Asia Ame Africa 1921 805,228 652,364 25,034 124,118 1,301 1922 309,556 216385 14,263 71448 520 1924 706.896 364,339 22,065, 318,855 900 1925 294,314 148,366 3,578 141,496 412 1927 335,175 168,368 3,669) 161,872 920 1928 307,255 158,513 3,380 144281 475 1929 279,678 158,598 3,798 116,177 509 636 29. Which of the Following areas of origin showed the greatest percentage decline in the number ‘of immigrants to the United States between 1921 and 1929 30. Which of the following best accounts for the trend in immigration shown in the chart? (A) Improved economic conditions in many areas of origin (A) Europe (B) Warfare in several areas of the world during (B) Asia this period (C) The Americas (C) New United States immigration legislation (D) Africa (D) Economic instability in the United States (E) Australasia (E) Increased immigration (other areas of North Sy bar ofthis pode faa ‘America GO ONTO THE NEXT PAGE nm 31. In the seventeenth century, some Pueblo Indians 34, of the desert Southwest adopted Christianity as (A) an added dimension to their own religious ‘culture, adding the Christian God as another deity (B) evidence of an ancient European culture that they were willing to embra (©) a means of improving their agricultural practic (D) asa means of establishing greater equality within their community (&) a means of direct communication with the afterlife through the practice of Christian prayer 32. Of the following, who challenged the religious, establishment in Puritan New England? (A) Coton Mather (B) Thomas Hutchinson (©) Anne Hutchinson (D) John Winthrop (©) Abigail Adams 35, 33, Henry Clay's “American System” included which of the following? (A) A protective tariff that would fund internal improvements (B) Restriction on the use of federal money for national defense (C) Restriction on immigration from Asian ‘countries (D) Blimination of the national bank (B) Protection of the property rights of Native Americans, amy par ths ge sea d72 UNITED STATES HISTORY TEST—Continued ® In 1860 a southern writer, D. R. Hundley, wrote: constitute a separate class to themselve Southern Yeomen are as distinet from them as the Southern Gentleman is from the Cotton Snob.” Which of the following characterizations would Hundley probably accept as best describing the southern yeoman? (A) A class of White plantation employees who oversaw slave labor (B) A group of landowners who generally owned more than 100 slaves and who formed the elite of southern society (©) A group of independent farmers who owned small plots and few, if any, slaves (D) A small group of farmers who believed that there were few, if any, class distinctions in the South (B) A class of people known for their poor ‘manners and lack of education ‘The Exelusion Act of 1882 prohibited the immigration of which of the following groups? (A) Irish (B) Mexicans (©) Eastern European Jews (D) Japanese (©) Chinese GO ONTO THE NEXT PAGE UNITED STATES HISTORY TEST—Continued © MONTHLY WAGES AND SEXUAL COMPOSITION OF THE WORKFORCE IN SELECTED TRADES IN NEW YORK CITY, 1850 Average Average Percent Percent Trade Male Wage Female Wage Male Female Clothing and tailors $9.75 5 6.99 485 51S Hats and millinery 2751 1714 43s 565 Shoes and boots 24.32 10.43 75.2 248 Printing 36.28 14.48 13 28.7 36. Which of the following statements about the trades listed above is supported by the data in the table?” (A) The majority of female workers were in the hats and millinery trade, (B) Both men and women received wages that were inadequate to support their families (C) Intrades where women were in the majority, the difference between men’s and women’s wages was less than in trades where women were in the minority. (D) The trades in which women were most highly represented had the lowest wages in the economy. (E) The most skilled female workers were paid less than unskilled male workers. GO ONTOTHE NEXT PAGE > 730 © UNITED STATES HISTORY TEST—Continued © about the g the Second World 37. All of the following stat ‘American home front di War are correct EXCEPT: (A) The government instituted direct price controls to halt inf (B) The Supreme Court upheld the forced relocation of Japanese Am West Coast (C) Black workers migrated in large numbers from the rural South to the industrial cities of the North and West. (D) Unemployment continued at Depression-era levels. (B) Business leaders served as heads of the federal war-mobilization programs, ins on the 38. Which of the following events of the civil rights ‘movement best illustrates the concept of “non: violent civil disobedience"? (A) The Brown v. The Board of Education of Topeka caso of 1954 (B) The lunch-counter sit-ins of the early 1960°s, (C) The March on Washington, D.C., in 1963 (D) The formation of the Black Panther party in 196 (B) The desegregation of Little Rock, Arkansas, Central High School 39. In the 1950's John Kenneth Galbraith’s The Affluent Society and W. H. Whyte’s The jion Man were significant because (A) criticized Amer belief that all problems (B) challenged the American view that the Soviet Union was responsible for the Cold Ws (C) advocated the nationalization of basie ind tries to inerease production and profits (D) were novels describing life among the “beat generation’ (B) urged a greater role for religion in American life and acceptance of Christian ethies by business executives om “The Oda Couple 0. Which of the following policies is the subject of the cartoon above? (A) Vietnamization (B) Containment (C) Détente (D) Interventionism (B) Isolationism 41. The Halfway Covenant adopted by many Puritan congregations in the late seventet which of the following? th century did (A) Strengthened the Anglican church in New ngland. (B) Undermined religious toleration in New England. (©) Promoted Christianity among American Indians in New England, (D) Eased the requirements for church mem. bership. (B) Encouraged belief in the doctrine of predestination, GO ONTO THE NEXT PAGE > 42, Atthe time of the American Revolution, the ‘most valuable cash crop produced in the southern states was, (A) cotton (B) corn (C) sugar (D) wheat (B) tobacco 43, The War of 1812 resulted in (A) an upsurge of nationalism in the United States (B) the acquisition of territories from Great Britain (C) the strengthening of Napoleon in Europe (D) the large-scale emigration of Europeans to the United States (E) the elimination of United States shipping from European waters 44, Which of the following provides the best evidence ‘of Lincoln's talents as a political leader? (A) His success in getting his Reconstruction policies passed by Congress (B) His skill in getting the South to acknowl edge responsibility for the outbreak of the Civil War (C) His success in securing adoption of the Fifteenth Amendment (D) His ability to keep his party relatively united despite its internal cont (B) His success in winning public support for a military draft Sy bar ofthis pode faa UNITED STATES HISTORY TEST—Continued 45. 46. All of the following situations contributed to agrarian discontent in the late nineteenth century EXCEPT: (A) Cotton averaged 5.8 cents a pound between 1894 and 1898, whereas it had been 15.1 cents a pound between 1870 and 1873. (B) Short-haul railroad rates rose 60 percent in the 1890's. (C) Farmers borrowed more heavily from banks than they had before the Civil War. (D) European countries raised duties on agri- cultural products in the 1880"s. (©) The wheat harvest in Europe declined 30 percent in 1897, At the beginning of the twentieth century, erties labeled individuals who exploited workers, ‘charged high prices, and bribed public officials as (A) robber barons. (B) free silverites. (C) knights of labor (D) captains of industry (E) muckrakers| GO ONTO THE NEXT PAGE a) UNITED STATES HISTORY TEST—Continued DISTRIBUTION OF TOTAL PERSONAL INCOME AMONG THE UNITED STATES POPULATION, 1950-1970. Second Poorest Poorest Middle Wealthiest Year Fifth Fiah Fifth Fifth 1950 3.1% 10.5% 17.3% 45.0% 1960 3.2% 10.6% 17.6% 44.0% 1970 3.6% 10.3% 17.2% 44.1% 47. The chart above supports which of the following statements? (A) Federal antipoverty programs in the 1960s had little impact on the national distribution of income. (B) Between 1950 and 1970, children tended to remain in the same socioeconomic groups as their parents. (C) The wealthiest people earned about the same amount ‘of money in 1970 as they earned in 1960. (D) The increased number of women in the labor force in the 1970's had litle effect on the amount of total family (&) The number of people in the “poorest fifth” remained about the same from 1950 to 1970. (GO ONTO THE NEXT PAGE > S SS) UNITED STATES HISTORY TEST—Continued 48, “One who breaks an unjust law must do so ‘openly, lovingly, and with a willingness to accept the penalty. 1 submit that an individual who breaks the law that conscience tells him is unjust, and who willingly accepts the penalty ‘of imprisonment in order to arouse the conscience of the community over its injustice, is in reality ‘expressing the highest respect for the law.” ‘The quotation above most clearly expresses the views of (A) Maleolm X (B) Phyllis Schlafly (©) Martin Luther King, Jr (D) Douglas MacArthur (E) Barry Goldwater 49, Rachel Carson’s book Silent Spring was a (A) forestry manual (B) description of deaf people's perception of the changing seasons. (©) protest against noise pollution (D) protest against overuse of chemical insecticides (B) protest against the Vietnam War 50, Which of the following was a consequence of President Lyndon B. Johnson's Great Society program’? (A) An end to the urban population decline in the East and Midwest (B) Full employment until the end of the 1960's (C) The near elimination of urban and rural poverty (D) A major redistribution of the income tax burden (B) An increase in federal spending on social Sy bar ofthis pode faa “For we must consider that we shall be as a city upon a hill, the eyes of all people are upon us. So that if we shall deal falsely with our God in this work we shall have undertaken, and so cause Him {© withdraw His present help from us, we shall be made a story and a by-word through the world.” ‘The statement above was made by (A) Jonathan Edwards preaching to a congregation during the Great Awakenin; (B) John Winthrop defining the purpose of the Puritan colony (©) Thomas Jefferson on the adoption of the Declaration of Independence (D) William Penn defining the purpose of the Pennsylvania colony (©) Benjamin Franklin gathering support for the American Revolution 2. Which of the following best characterizes the Anti-Federalists? (A) They wanted a strong executive branch. (B) They were loyal supporters of the Crown, (C) They drew support primarily from rural areas. (D) They favored universal suffrage (E) They favored rapid industrial development. ‘The Missouri Compromise was, in part, an effort to maintain the balance between the number of northerners and southerners in which of the fol- Jowing United States institutions? (A) The Senate (B) The House of Representatives (©) Congress (D) The Supreme Court (E) The electoral college GO ONTO THE NEXT PAGE 7 54. - Skilled male Workers felt threater Which of the following is true of the Black Codes of the Reconstruction era” (A) They promised every adult male former slave “forty acres and a mule.” (B) They were Andrew Johnson's response to criticism that he was not doing enough for former slaves. (©) They were the result of joint actions by seala- wags and carpethaggers in the southern states, (D) They were passed by the Radical Republicans, in Congress to ensure the rights of former slay (B) They were passed by Souther state legisla- tures to restriet the rights of former slaves. Advocates of a free silver policy argued that the free coinage of silver would (A) increase the supply of money and end economic depressions (B) facilitate free trade between countries (C) limit the market power of farmers (D) stabilize the value of gold in relation to silver (B) increase the value of the dollar in relation to currencies of foreign countries ed by all of the following changes that occurred in the United States economy between 1890 and 1920 EXCEPT the (A) arrival of large numbers of immigrants from southern Europe, eastern Europe, and Mexico (B) introduction of “scientific management” to increase factory production and lower Iabor costs: (C) growing power of major corporations. (D) increasingly widespread distribution of inexpensive consumer goods (B)_ growing presence of women workers in industry ar of ths pode se D783 UNITED STATES HISTORY TEST—Continued 57, 38. © Which of the following was demonstrated by the outcome of the presidential election of 1928 ? (A) The nation had become convinced of the futility of Prohibition, (B) “Republican prosperity” was a pers campaign slogan. (C) Ethnic and religious differences among ‘Americans exerted litle influence on their voting behavior. (D) Great numbers of ethnic minority-group voters switched from the Democratic to the Republican Party (®) The Ku Klux Klan was the commanding force in United States politics during the 1920's. “[The American] is intensely and cocksurely moral, but his morality and his self-interest are crudely identical. He is emotional and easy to scare, but his imagination cannot grasp an abstraction. He is a violent nationalist and patriot, but he admires rogues in office and always beats the tax-collector if he ean. He is violently jealous. ‘of what he conceives to be his rights, but brutally disregardful of the other fellow’s, ‘The author of the quotation above is the noted journalist and satirist (A) Dorothy Thompson (B) Lillian Hellman (©) HL. Mencken (D) Will Rogers (6) Pearl Buck GO ONTO THE NEXT PAGE > 59, Which of the following contributed most to ending the post-Second World War economie boom? (A) Women leaving the workforce (B) Development of the computer (C) Consolidation of agriculture (D) A shift in population to the Sunbelt (B) The Arab oil embargo 60. ‘The United States of the 1970's was characterized by an increase in all of the following EXCEPT (A) computer technology and marketing (B) an awareness of the rights of minorit (C) the migration of Americans from the Frostbelt to the Sunbelt (D) the strength of political party attachments (E) the number of multinational corporations, 61. Colonists in eighteenth-century South Carolina benefited from the knowledge of Africans about the cultivation of (A) tobacco (B) rice (©) sugar (D) cotton (E) wheat 62, In the hundred years prior to 1776, which of the following had the LEAST influence on the ‘emergence of the movement for independence in England's North American colonies? (A) The control of money bills by colonial legislatures. (B) The long period of conflict betwe England and France (©) The models provided by the autonomous ‘governments of other English colonies (D) The distance between England and its colonies (E) Constitutional developments in England Sy bar ofthis pode faa UNITED STATES HISTORY TEST—Continued 63 “To maintain the existing relations between the inhabiting that section of the Union, is indispensable to the peace and happiness of both. It cannot be subyerted without drenching the ‘country in blood, and extirpating one or the other of the races.” ‘The statement above was most likely made by which of the following? >. Calhoun to the United States Senate lerick Douglass to the Anti-Slavery Society (©) Daniel Webster to the South Carolina legis- lature (D) John Brown at Harpers Ferry (E) Abraham Lincoln in Springfield, Illinois “In the late nineteenth century, the federal govern- ment followed a laissez-faire policy toward the ‘economy, A historian could argue against this thesis using all of the following pieces of evidence EXCEPT (A) tariff laws protecting various industries from European competition (B) laws granting land to the transcontiner railroad corporations (©) government policy toward the unemployed during the depression of the 1890°s (D) the Bland-Alfison Act of 1878 and the ‘Sherman Silver Purchase Act of 1890 (E) the Interstate Commerce Act of 1887 Booker T. Washington encouraged Black people to pursue all of the following EXCEPT (A) accommodation to White society (B) racial solidarity (©) industrial education (D) economic self-help (E) public political agitation GO ONTO THE NEXT PAGE 79D 66. “What we want to consider is, first, to make our employment more secure, and, Secondly, 10 ‘make wages more permanent. ... I say the labor ‘movement is a fixed fact. [thas grown out of the necessities of the people, and, although some may desire to see it fail, still the labor movement will be found to have a strong lodgment in the hearts, of the people, and we will go on until success has. been achieved.” ‘The quotation above best reflects the philosophy of which of the following organizations around 19007 (A) Industrial Workers of the World (B) National Labor Union (©) American Federation of Labor (D) Congress of Industrial Organizations (E) Knights of Labor 67. Theodore Roosevelt issued his corollary to the Monroe Doetrine primarily because n Manchuria had violated (B) United States protection was needed by the colonies acquired in the Spanish- American War (C) the Filipino people revolted against United ates rule (D) the financial difficulties of Caribbean nations, threatened to bring about European inter- vention (B) the declining toll revenue from the Panama Canal threatened Panamanian stability Ursutvised copying rung any bart this pode sean Deo UNITED STATES HISTORY TEST—Continued 68. “The problem lay buried, unspoken, for many years in the minds of American women. It was ‘strange stirring, a sense of dissatisfaction, a yearning that women suffered in the middle of the twentieth century in the United States. Each suburban wife struggled with it alone. As she ‘made the beds, shopped for groceries, ate peanut butter sandwiches with her children, chauffeured Cub Scouts and Brownies, she was afraid to ask even of herself the silent question—"Is this all?" ‘The passage above supports which of the fol- lowing statements about women in the middle of the twentieth century? (A) Women were no longer activities. (B) Feminism tended to be a middle-class move- ment, (C) Feminism renewed interest in religion among (D) There were very few educational opportunities for women, (E) Most women supported the feminist move- ment. rested in political 69. “Government is not the solution to our problems Government is the problem.” ‘The statement above was made by (A) John F, Kennedy, asserting that the govern- ment did not do enough for the people (B) Dwight D. Eisenhower, arguing that the ‘government interfered with the military's ‘operations (©) Jimmy Carter, claiming that the government \vas inefficient and unfair (D) Gerald Ford, charging that the government was corrupt (B) Ronald Reagan, contending that the govern- ‘ment had taken on functions properly belonging to the private sector GO ONTO THE NEXT PAGE ® UNITED STATES HISTORY TEST—Continued © “\ DON'T KNOW WHY THEY DON'T SEEM TO HOLD BN ‘AWE THE WAY THEY USED To* From Herblock At Large (Pantheoe, 1987) 70. Which of the following best summarizes the idea expressed in the 1987 cartoon above? (A) In the 1980s, States. (B) President Reagan expected tha ‘economic summit would enable the Unit to solve its financial problems. (C) In the 1980°s, the United States could not look to its ‘economic partners for help in solving its economic problems. (D) The economic problems of th 1980's resulted from European ec (B) Had it not been for the Iran-Contra States could have solved its ecor States mic policies. ndal, the United problems, [any prt ofthis poses eas ratraed opying oe using GO ONTO THE NEXT PAGE aid 71. “No man was a warmer wisher for reconci tion than myself, before the fatal nineteenth of April 1775, but the moment the event of that day ‘was made known, I rejected the hardened, sullen tempered Pharaoh of England for ever; and disdain the wretch, that with the pretended title of FATHER OF HIS PEOPLE, can unfeelingly hear of their slaughter, and composedly sleep with their blood upon his soul.” 74, ‘The passage above comes from (A) the Declaration of Independence (B) The Federalist papers (©) Letters from a Farmer in Pennsylvania (D) the Virginia Resolves against the Stamp Act ©) Common Sense 7s Alexander Hamilton’s plan for stimulating eco- nomic growth in the United States included all of the following EXCEPT (A) acquisition of additional teritory (B) a protective tariff (C) expansion of manufacturing (D) establishment of a national bank (E) federal assumption of debts incurred by states during the Revolutionary War 16. 73. The first American party system, which devel- oped in the 1790's, maintained party discipline al the federal level primarily by means of (A) caucuse (B) nominating conventions (C) rotation in office (D) restrictive primaries (B) “pork barrel” legislation amy par ths ge sea Daz UNITED STATES HISTORY TEST—Continued Which of the following was true of the Jacksonian Democrats in the 1830's? (A) They were the minority party in the nation. (B) They opposed a national bank. (C) They supported South Carolina’ nullification, of the protective tariff. (D) They were stronger in New England than in the West (E) They generally repudiated the ideas of the Jeffersonian Republicans. “We hold these truths to be self-evident: th: ‘men and women are created equal. ... The of mankind is a history of repeated injuries and usurpations on the part of man toward woman, haying in direct object the establishment of an absolute tyranny over her. ‘The quotation above is excerpted from the (A) Seneca Falls Declaration of Sentiments and Resolutions (B) United States Declaration of Independence (C) United States Constitution (D) Declaration of Rights and Grievances (B) Equal Rights Amendment (ERA) Which of the following is true of the Pullman strike of 1894 2 (A) It brought a substantial portion of American railroads to a standstill. (B) It started when Pullman workers were fired after the Haymarket riot. (C) Twas caused by grievanes working conditions. (D) tended when the government forced ‘management to settle with the union, (B) Tended when the courts issued a blanket injunction against management. about unsafe GO ONTO THE NEXT PAGE > ust be the great arsenal of democracy. For this is an emergeney as serious as wae itself. We ‘must apply ourselves to our task with the same resolution, the same sense of urgency, the same spirit of patriotism, and sacrifice, as we would show were wo at war.” ‘The emergeney to which the speaker refers was (A) German U-boat attacks in 1917 (B) the Spanish Civil War in 1936 (©) German warfare against Britain in 1940 (D) the Berlin Blockade of 1948 (B) the Cuban missile crisis of 1962 78, The legislation passed between 1935 and 1937 dealing with the role of the United States in future wars seemed to refleet a belief that (A) totalitarianism dircetly threatened the security. of the United States. (B) the United States should quickly intervene in ‘any future world wars (C) the United States had made a mistake in not joining the League of Nations (D) the United States should not have become involved in the First World War (E) the United States should take a position of leadership in world affairs 79. Civil rights organizations in the 1950°s and 1960°s based their court suits primarily on the (A) five freedoms of the First (B) Fourteenth Amendment (C) Thirteenth Amendment (D) “necessary and proper” elause of the Constitution (B) Preamble to the Constitution sndment Sy bar ofthis pode faa UNITED STATES HISTORY TEST—Continued 80. 8 82. The Nixon administration differed from previous, administrations in adopting which of the fol- Iowing Vietnam War policies? I. The bombing of North Vietnam Il, The use of American combat troops IIL. The invasion of Cambodia IV. The mining of North Vietnamese harbors (A) Lonly: (B) Land IIL only (©) Mand It only (D) Hand 1V only (E) Hand 1V only Which of the following political ideas or philos- ‘ophies inspired the American revolutionaries of the eighteenth century? (A) Progre: (B) Populism (©) Manifest Destiny (D) Republicanism (E) The Social Gospel ‘The first major nineteenth-century political conflict over the issue of slavery was settled. by the (A) Alien and Sedition Acts (B) Kentucky and Virginia Resolutions (C) Missouri Compromise (D) Kansas-Nebraska Act (B) Dred Scou decision GO ONTO THE NEXT PAGE 83D 83. During the 1850's, Kansas became a significant issue for which of the following reasons’ (A) The territory was an important way station in the Underground Railroad, (B) Northern and southern the first transcontis Kansas, (©) Kansas served as a center for the Peoples (Populist) Party’s agitation against railroads and banks, (D) John Quincy Adams invoked the gag rule to prevent the discussion of slavery in the Senate, (B) led (a divisive debate over the expansion of slavery into the territories 84. Which of the following was a sig ‘ment in American literature during the late nineteenth century? jeant move- (A) Creationiss (B) Modemi (©) Romanticism (D) Classicism (B) Realism 85. Edward Bellamy’s Looking Backward, written in the 1880"s, was a utopian reaction to which of the following? (A) The disillusionment with an incre ‘competitive and industrial society (B) The plight of farmers who were driven off their land during the Great Depression (C) The disillusionment of the planter aristocracy in the post-Civil War era (D) The growing number of immigrants who regretted leaving their homes in Europe (©) Increasing concerns over the growth and power of labor unions in the railroad industry ingly amy par ths ge sea UNITED STATES HISTORY TEST—Continued 86. 87 88. ® (A) Marcus Garvey’s “back to Africa” crusade (B) the reemergence of the Ku Klux Klan as a force in American politics (©) writers and artists in New York who expressed pride in their African American culture (D) American expatriate writers living in Paris who wrote critically of American society (B) the political success of the Democratic Party in northern urban neighborhoods ‘The Harlem Renaissance refers 10 The Federal Reserve Act of 1913 established a (A) single central bank like the Bank of England (B) method of insuring bank deposits against loss (C) system to guarantce the continued existence of the gold standard. (D) system of local national banks (B) system of distriet banks eoordins central board cd by a ‘The Korean War and the Vietnam War differed in that only one involved (A) a formal declaration of war (B) a communist-led government (C) troops under United Nations auspices (D) Soviet arms support to one of the belligerents (E) United States air and ground forces GO ONTO THE NEXT PAGE ® UNITED STATES HISTORY TEST—Continued “1 NEVER TIRE OF WATCHING THEM" COMPUTER we oon HERS 89. Which of the following best summarizes the idea expressed in the cartoon above? (A) Most people are too dependent on computers in their daly lives. (B) The amount of information available via is so overwhelming that people are no longer able to use the information effectively (C) Individual privacy is being threatened by the computerization of personal information, (D) Many industries in the United States are threatened with significant layoff as com- puters replace workers, (B) People in the United reluctant to begin using computers than have people in other parts of the world Pamera near GO ONTO THE NEXT PAGE 85D ates have been more UNITED STATES HISTORY TEST—Continued 90. Which of the following is an accurate statement about the Equal Rights Amendment to the Con- stitution proposed in the 1970°s? (A) It was opposed primarily by those who feared loss of political power. (B) It guaranteed equal opportunity for wom the workplace. (©) Itbecame a part of the Constitution in 1978. (D) Itrepresemted the first effort to enfranchise (B) It failed to gain the necessary votes for rati- fication within the constitutional time li STOP IF YOU FINISH BEFORE TIME IS CALLED, YOU MAY CHECK YOUR WORK ON THIS TEST ONLY DO NOT TURN TO ANY OTHER TEST IN THIS BOOK. United States History How to Score the SAT Subject Test in United States History When you take an actual SAT Subject Test in United States History, your answer sheet will be “read” by a scanning machine that will record your responses to each question. ‘Then a computer will compare your answers with the correct answers and produce your raw score. You get one point for each correct answer. For each wrong answer, you lose one-fourth of a point. Questions you omit (and any for which you mark more than one answer) are not counted. This raw score is converted to a scaled score that is reported to you and to the colleges you specify. Worksheet 1. Finding Your Raw Test Score STEP 1: Table A lists the correct answers for all the questions on the Subject Test in United States History that is reproduced in this book. It also serves as a worksheet for you to calculate your raw score, + Compare your answers with those given in the table. + Puta check in the column marked “Right” if your answer is correct. + Puta check in the column marked “Wrong” if your answer is incorrect. + Leave both columns blank if you omitted the question, STEP 2: Count the number of right answers. Enter the total here: STEP 3: Count the number of wrong answers. Enter the total here: STEP : Multiply the number of wrong answers by .250. Enter the product here: STEP 5: Subtract the result obtained in Step 4 from the total you obtained in Step 2. Enter the result here: STEP 6: Round the number obtained in Step 5 to the nearest whole number. Enter the result here: The number you obtained in Step 6 is your raw score. 87) ‘The Official Study Guide for All SAT Subject Tests Table A Answers to the Subject Test in United States History, Form 3WAC2, and. Percentage of Students Answering Each Question Correctly ee errr Bec of Studer ponents the Question | Question | Correct wrong | Correctly" ‘Table A continued on next page United States History Table A continued from previous page see fend 65 E 4 78 D 30 6 c 6 73 8 6 6) oD Fz 80 E " Cy 8 32 at D a cl E a a2 c 38 70 a 40 a3 E a n E 38 a4 E 35 n A 36 85 A a B A 2 86 c a2 ms a 6 a E 16 % a 3 88 c 18 % A a a9 c 68 n c 2 90 E 4 These percentages are based on an analysis ofthe answer sheets of a representative sample of 8,509 students ‘who took the original form ofthis test in December 2000, and whose mean score was 5M. They may be used as an indication of the relative difficulty ofa partcwlar question. Each percentage may also be used to predict the likelihood that a typical SAT Subject Test in United States History candidate wll answer that question correctly ‘on this edition of the test. ‘The Official Study Guide for All SAT Subject Tests Finding Your Scaled Score When you take SAT Subject Tests, the scores sent to the colleges you specify are reported on the College Board scale, which ranges from 200-800. You can convert your practice test score to a scaled score by using Table B, To find your scaled score, locate your raw score in the left-hand column of Table B; the corresponding score in the right-hand column is your scaled score. For example, a raw score of 39 on this particular edition of the Subject Test in United States History corresponds to a scaled score of 560. Raw scores are converted to scaled scores to ensure that a score earned on any one edition of a particular Subject Test is comparable to the same scaled score earned on any other edition of the same Subject Test. Because some editions of the tests may be slightly easier or more difficult than others, College Board scaled scores are adjusted so that they indicate the same level of performance regardless of the edition of the test taken and the ability of the group that takes it. Thus, for example, a score of 400 on one edition of a test taken ata particular administration indicates the same level of achievement as a score of 400 on a different edition of the test taken at a different administration, When you take the SAT Subject Tests during a national administration, your scores are likely to differ somewhat from the scores you obtain on the tests in this book. People perform at different levels at different times for reasons unrelated to the tests themselves. The precision of any test is also limited because it represents only a sample of all the possible questions that could be asked. D390 United States History Table B ‘Scaled Score Conversion Table ct Test in United States History (Form 3WAC2) 0 52 630 4 430 9 St 630 13 430 8 50 620 12 420 87 43 620 " 420 % 48 610 10 410 6 a 610 8 410 84 48 600 8 400 83 5 600 7 400 2 4a 590 6 390 81 43 590 5 390 80 a 580 4 380 3 a 50 3 380 18 40 570 2 370 7 39 560 1 370 18 38 560 o 360 5 a7 580 1 380 m 38 580 2 350 B 35 540 a 350 R 34 540 4 340 1 33 530 8 330 70 32 530 8 330 6 31 520 7 320 6 30 520 8 310 @ 29 510 9 310 6 28 510 0 300 5 2 500 T] 290 64 6 490 2 230 6 B 490 43 280 e 2 480 14 270 61 2 480 6 270 0 22 470 16 260 59 a 470 7 280 58 20 460 8 250 7 8 460 3 250 56 8 450 20 200 5 7 450 2 230 54 6 440 2 230 53 5 440. oD ‘The Official Study Guide for All SAT Subject Tests How Did You Do on the Subject Test in United States History? After you score your test and analyze your performance, think about the following question Did you run out of time before reaching the end of the test? If'so, you may need to pace yourself better. For example, maybe you spent too much time on one or two hard questions. A better approach might be to skip the ones you can't answer right away and try answering all the questions that remain on the test. Then if there’s time, go back to the questions you skipped. Did you take a long time reading the directions? You will save time when you take the test by learning the directions to the Subject Test in United States History ahead of time. Each minute you spend reading directions during the test is a minute that you could use to answer questions, How did you handle questions you were unsure of? If you were able to eliminate one or more of the answer choices as wrong and guess from the remaining ones, your approach probably worked to your advantage. On the other hand, making haphazard guesses or omitting questions without trying to eliminate choices could cost you valuable points. How difficult were the questions for you compared with other students who took the test? Table A shows you how difficult the multiple-choice questions were for the group of students who took this test during its national administration. The right-hand column. gives the percentage of students that answered each question correctly. A question answered correctly by almost everyone in the group is obviously an easier question, For example, 85 percent of the students answered question 2 correctly, But only 19 percent answered question 58 correctly. Keep in mind that these percentages are based on just one group of students. They would probably be different with another group of students taking the test. If you missed several easier questions, go back and try to find out why: Did the questions cover material you haven't yet reviewed? Did you misunderstand the directions? Daz Chapter 3 World History Purpose ‘The Subject Test in World History measures your understanding of the development of major world cultures and your use of historical techniques, including the application and weighing of evidence and the ability to interpret and generalize. The test covers all historical fields: + political and diplomatic + intellectual and cultural + social and economic Format This one-hour test consists of 95 multiple-choice questions. Many of the questions are global in nature, dealing with issues and trends that have significance throughout the modern world. Content The questions test your: + familiarity with terms commonly used in the social sciences, + understanding of cause-and-effect relationships + knowledge of the history and geography necessary for understanding major historical developments + grasp of concepts essential to historical analysis + capacity to interpret artistic materials + ability to assess quotations from speeches, documents, and other published materials + ability to use historical knowledge in interpreting data based on maps, graphs, and charts 93D ‘The Official Study Guide for All SAT Subject Tests Chronological Material Covered Approximate Percentage of Test Prehistory and Civilizations to 500 Comman Era (C.E* % 500-1500 0 20 1500-1900 CE % Post-1900 C.E 20 Cross-chronological 10 Geographical Material Covered Global or Comparative 2 Europe % latica 10 Southwest Asia 10 South and Southeast Asia 10 East Asia 10 IThe Americas 10 The SAT Subject Test in World History uses the chronological designations B.C.E. (before common era) and G.E. (Common era). These labels correspond to B.C. (before Christ) and A.D. (annto Domini), which are used in some world history textbooks. How to Prepare You can prepare academically for the test by taking a one-year comprehensive course in world or global history at the college-preparatory level and through independent reading of materials on historic topics. Because secondary school programs differ, the Subject Test in World History is not tied to any one textbook or particular course of study. Familiarize yourself with the directions in advance. The directions in this book are identical to those that appear on the test. Score ‘The total score is reported on the 200-to-800 scale. doa World History Sample Questions All questions on the Subject Test in World History are multiple-choice, requiring you to choose the BEST response from five choices. The following sample questions illustrate the types of questions on the test, their range of difficulty, and the abilities they measure. Questions may be presented as separate items or in sets based on quotations, maps, pictures, graphs, or tables. Directions: Each of the questions or incomplete statements below is followed by five suggested answers or completions. Select the one that is best in each case and then fill in the corresponding circle on the answer sheet. Note: The World History Test uses the chronological designation B.C.E. (before common era) and C.E. (common era). These labels correspond to B.C. (before Christ) and A.D. (anno Domini), which are used in some world history textbooks. Some questions require you to know social science terms, factual cause-and-effect relationships, geography, and other data necessary for understanding major historical developments. Questions I and 2 fall into this category. 1. Which of the following was immediately responsible for precipitating the French Revolution? (A). The threat of national bankruptcy (B) Anattack upon the privileges of the middle class (©) The desire of the nobility for a written constitution (D)_ The suffering of the peasantry (E) The king's attempt to restore feudalism Choice (A) is the correct answer to question 1. To answer this question, you need to recall the circumstances that led in May 1789 to the first meeting of the French Estates-General in over a century and a half, an event that arrayed the Third Estate against the nobility and Louis XVI in the first stage of a political struggle that was to evolve into the French Revolution. With his debt-ridden government brought to a halt, the king, by mid-1788, was left with no other recourse than a promise to convene the Estates-General in the months ahead. 95D ‘The Official Study Guide for All SAT Subject Tests ATLANTIC OCEAN CASPIAN SCALE OF MILES 2. The shaded area in the map above shows the extent of which of the following? (A) Irrigation agriculture in 1000 B.C.E. (B)_ Greek colonization in 550 B.C.E. (©) Alexander the Great's empire in 323 BCE. (D) The Roman Empire in 117 C.E. (E) The Byzantine Empire in 565 C.E. Choice (D) is the correct answer to question 2. This question tests your knowledge of both history and geography. To answer it you must know something about the extent to which irrigated farming was practiced in Africa, Europe, and Southwest Asia three thousand years ago, and you need to have a general idea of the extent of the territory controlled by four major ancient civilizations at specific points in time. Choice (A) can be eliminated because irrigation in this early period would have been confined to the regions along the major rivers of the Middle East and Southwest Asia. Choice (B) can be eliminated because Greek colonization was confined primarily to the eastern Mediterranean and did not extend as far north in Europe as shown in the shaded areas of the map. Choice (©) can be eliminated because Alexander the Great's empire did not D96 World History extend into either the western Mediterranean or northwestern Europe. Choice (E) can be eliminated because the Byzantine Empire, with its capital in Constantinople, was confined primarily to the eastern Mediterranean area. Only choice (D) outlines the greatest extent of territory controlled by the Roman Empire under the Emperor Trajan in the second century C.E. Some questions test your understanding of concepts essential to history and social science, your capacity to interpret artistic materials, and your ability to assess quotations from speeches, documents, and other published materials. Questions 3-6 fall into this category. 3. Which of the following was introduced into the diet of Europeans only after European contact with the Americas in the fifteenth century? (A) Tea (B) Rice (©) Cinnamon (D) Sugar (E) Potatoes Choice (E) is the correct answer to question 3. It is correct because the potato is native to the Peruvian-Bolivian Andes and, after its “discovery” by the Europeans in the fifteenth century, became a staple of the European diet. The potato is now a major food crop worldwide. To answer this question you need to have some basic information about what has come to be known as the “Columbian Exchange,” i., the enormous biological transfer that occurred as a result of the fifteenth- and sixteenth-century European voyages of discovery. Choice (A) can be ruled out because tea comes from China, not from the Americas, and was not widely used in Europe until the mid-seventeenth century. Choice (B) can be eliminated because the origin of rice culture has been traced to India, Rice was introduced into southern Europe in medieval times. Choice (C) can be eliminated because the cinnamon tree is native to South Asia and, like rice, has been known in Europe since medieval times. Choice (D) can be eliminated because sugarcane originated in what is now known as New Guinea, followed human migration routes from Southeast Asia through Southwest Asia to Europe and, although rare and expensive, was known to the European aristocracy in medieval times. 97D ‘The Official Study Guide for All SAT Subject Tests Giraudon/Art Resource 4, The nineteenth-century wood block print above is associated with the culture of (A) Japan (B) India © tran (D) Myanmar (Burma) (©) Thailand Choice (A) is the correct answer to question 4, The scene depicted in this dramatic picture is world-famous. Although the spatial arrangement and perspective are generally East Asian and the title at the upper left-hand corner is written in Chinese characters, there are a number of characteristics that identify the picture as Japanese, The dramatic subject matter, with Mount Fuji in the background, is Japanese. In addition, colorful wood-block prints depicting famous scenery, beautiful women, warriors, and well-known theater subjects were popular in Japan from the seventeenth to the nineteenth centuries because they were widely affordable. This work is by the nineteenth-century artist Hokusai. World History Questions 5 and 6 refer to the following passage. We have heard that in your country opium is prohibited with the utmost strictness and severity—this is a strong proof that you know full well how hurtful it is to humankind, Since then you do not permit it to injure your own country, you ought not to have the injurious drug transferred to another country, and above all others, how much less to the Middle Kingdom! 5. The author of the diplomatic dispatch above lived in which of the following countries? (A) Ghana (B) The Netherlands (©) Iran (D) China (E) Germany Choice (D) is the answer to question 5. The above discussion of the forced importation of opium suggests China's struggle against Great Britain, culminating in the Opium War of the mid-nineteenth century. The tone of the dispatch, expressing indignation at Great Britain's flaunting of Chinese law, is consistent with Chine’s concern over growing opium addiction in China and with Chinese resistance to the British. From your study of China you will also remember that the Chinese used to refer to their country as the Middle Kingdom. 6. The country which went to war in the nineteenth century over the issue raised in the dispatch was (A) France (B) Egypt (©) Great Britain (D) India (E) Japan Choice (C) is the correct answer to question 6, Great Britain was expanding its Asian trade and needed a product to exchange for Chinese goods, Opium from India was Great Britain's answer to this dilemma, The dispatch above was sent by a representative of the Chinese emperor to Queen Victoria shortly before the Opium War (1839-1842), in which China was defeated by the British and therefore was not able to enforce its prohibition against the importation of opium, ‘The Official Study Guide for All SAT Subject Tests Questions posed in the negative, like question 7, account for at most 25 percent of the test questions. Variations of this question format employ the capitalized words NOT or LEAST, as in the following examples: “Which of the following is NOT true?” “Which of the following is LEAST likely to occur?” 7. All of the following are “Pillars of Islam” EXCEPT (A) giving alms for the support of society's poor (B) praying five times a day in the direction of Mecca (©) fasting for one month of the year (D) making a pilgrimage to Mecca at least once during a lifetime (E) attending mosque prayers daily Choice (E) is the correct answer to question 7. This question asks you to identify the exception in a series of true statements. In other words, you are being asked to locate the false answer among the five options. To answer this question, you need to draw on your knowledge of Islam. Choices (A) through (D) are true because they refer to four of the five “Pillars of Islam.” Choice (E) is false because Muslims are not required to attend mosque prayers daily. The fifth pillar actually is the “profession of faith.” > 100 World History Questions based on graphs, charts, or cartoons require you to use historical knowledge in interpreting data. Questions 8-10 fall into this category. ANNUAL PRODUCTION OF STEEL (in thousands of metric tons) 97 169 370 8. Read from left to right, the column headings for the table above should be (A) Great Britain, United States, Germany, and France (B) Italy, Great Britain, Russia, and Germany (© Germany, Great Britain, Russia, and France (D)_ Great Britain, United States, France, and Germany (E) Germany, Russia, Great Britain, and United States Choice (A) is the correct answer to question 8. To answer this question, you need to know in which country the Industrial Revolution began and which other countries caught up early or late. Great Britain was industrialized by 1850, the United States and Germany, were next, and France, Italy, and Russia followed later in the nineteenth century. Questions containing charts and graphs require careful study and therefore may be more time-consuming than other types of questions. Remember to budget your time accordingly. 101) ‘The Official Study Guide for All SAT Subject Tests Questions 9 and 10 are based on the August 1914 Punch cartoon below. BRAVO, BELGIUM! 9. The “No Thoroughfare” sign in the cartoon is a reference to (A) an international treaty guaranteeing the neutrality of Belgium (B) the heavy defensive fortifications built by Belgium in the preceding decade (©) bilateral nonaggression pact between Belgium and Germany (D)_ analliance between Belgium and France (E) the treacherous, swampy terrain on the Belgian-German border D102 World History Choice (A) is the correct answer to question 9, In this question set you are asked to interpret a British political cartoon published during the tense diplomatic period before the outbreak of the First World War. Choice (A) refers to treaties signed by the Great Powers in 1839 guaranteeing the neutrality of Belgium and Luxembourg in the event of war. 10. This cartoon is a comment on Germany's attempt to (A) acquire valuable mineral resources in Belgium (B) invade France through Belgium (©) force Belgium to repeal tariffs on German goods (D) intimidate Belgium into signing a military alliance with Germany (E) _ pressure Belgium into withdrawing from the Triple Alliance Choice (B) is the correct answer to question 10. This question focuses on Belgium's resistance to the more powerful Germany's threat of aggression if Belgium, situated between Germany and France, will not give transit to German troops. Other types of questions rely on knowledge of historical methodology or a grasp of important issues still affecting the world today. Questions 11 and 12 fall into this category. 11. Which of the following statements would be most difficult for historians to prove true or false? (A). There was little organized education in Europe during the Middle Ages. (B) Greece contributed more to Western civilization than Rome. (©) The invention of the steam engine influenced the way people lived. (D) Russia is territorially the largest country in the world, (E) The tourist industry in Europe increased markedly after the Second World War, Choice (B) is the correct answer to question 11. The assertion that Greece’s contribution to Western civilization was greater than Rome’s requires the most justification. In a methodology question such as the one above, you must make the distinction between statements that are verifiable by fact and statements that are based on judgments. The latter are more difficult than the former to prove true or false because they are evaluations. In this question, choice (B) is the most opinionated of the statements and therefore the one most difficult to prove or disprove. 103) ‘The Official Study Guide for All SAT Subject Tests 12.The term “green revolution” refers to (A) protests against the placement of nuclear weapons in Europe (B) ecological changes in the ocean because of algae growth (©) increased agricultural output resulting from development of hybrid seeds and chemical fertilizers (D) expanded irrigation farming made possible by the construction of large dams (E) thinning of the atmospheric ozone layer resulting in changes in the growing season Choice (C) is the correct answer to question 12. To answer this question, you need to know about modern scientific breakthroughs in agricultural research that have allowed countries like India, formerly subject to terrible famines, to become self-sufficient in grain production. D104 World History Test Practice Helps The test that follows is an actual, recently administered SAT Subject Test in World History, To get an idea of what it's like to take this test, practice under conditions that are much like those of an actual test administration Set aside an hour when you can take the test uninterrupted. Make sure you complete the test in one sitting Sit at a desk or table with no other books or papers. Dictionaries, other books, or notes are not allowed in the test room, Tear out an answer sheet from the back of this book and fill it in just as you would on the day of the test. One answer sheet can be used for up to three Subject Tests. Read the instructions that precede the practice test. During the actual administration you will be asked to read them before answering test questions. Time yourself by placing a clock or kitchen timer in front of you. After you finish the practice test, read the sections “How to Score the SAT Subject Test in World History” and “How Did You Do on the Subject Test in World History?” The appearance of the answer sheet in this book may differ from the answer sheet you see on test day. 105) WORLD HISTORY TEST ‘The top portion of the section of the answer sheet that you will use in taking the World History Test must be filled in exactly as shown in the illustration below. Note carefully that you have to do all of the following on your answer sheet. 1, Print WORLD HISTORY on the line under the words “Subject Test (print).” 2. In the shaded box Inbeled “Test Code” fill in four circles: —Fill in cirele 1 in the row labeled V. —Fill in circle 7 in the row labeled W ‘Subject Test pit) WORLD HISTORY ape = 3. Please answer the three questions below by filling in the appropriate circles in the row labeled Q on the answer sheet. The information you provide is for statistical purposes only and will not affect your score on th 15 of world history, world cultu grade 9 to the present? (If you are taking a course this seme: only one circle of circles 1-2, or European history have you taken from count it a8 a full semester.) Fill in '* One semester or less ‘+ Two semesters or more Question IL For the courses in world history, world cultures, or European history you have taken, which of the following geographical areas did you study? Fill in all of the circles that apply. © Africa —Fill in cirete 3, © Asia —Fillin circle 4. © Europe —Fill in circle 5. © Latin America —Fill in circle 6. = Middle East —Fill in circle 7. Question II How recently have you studied world history, world cultures, or European history? ‘+ [am currently enrolled in or have just completed such a course. —Fill in circle 8. ‘© Thave not studied this subject for 6 months or more. —Fill in circle 9. When the supervisor gives the signal, turn the page and begin the World History Test. There are 100 numbered circles on the answer sheet and 95 questions in the World History Test. Therefore, use only circles 1 to 95 for recording your answers, D106 WORLD HISTORY TEST © S Directions: Each of the questions or incomplete statements below is followed by five suggested answers or completions. Select the one that is best in each case and then fill in the corresponding circle on the answer sheet. Note: The World History Test uses the chronological designations B.C.E. (before common era) and CE. (common era). These labels correspond to B.C. (before Christ) and A.D. (anno Domini), which are used in some world history textbooks. 1. Which of the following was true of both Greece, and China in the period around 500 B.C.E.? (A) Both fostered vibrant philosophical schools that debated the human condition. (B) Both were threatened by more powerful neighboring civilizations, (C) Both experienced economic revolutions brought on by the discovery of iron. (D) Both underwent social revolutions that led to the seclusion of women. (B) Both suffered from overpopulation that led to class warfare and massive emigration. 2. Which of the following is true of the epie poems the Mahabharata, the Miad, and the Tales of the Heike? (A) All three were influenced by Chinese literary forms. (B) All three stress the exploits of a warrior elite. (C) All three were written down at first and later transmitted orally. (D) All three stress humanity's inde} from the influence of the gods. (B) Historians have conclusively identified the authors of the three works. ndence 4BAC snp of hs pope ae 3. Which of the following statements about the effects of Muhammad's teaching is true? (A) Islam initially attracted many fotlowers, but ‘gradually became less popular. (B) Muhammad believed that social differen needed to be preserved, which encouraged divisions in society. (C) Islam affected every aspect of life and encouraged unity among converts with widely diverse backgrounds. (D) Muhammad believed that wealth should be renounced; thus Islam did not attempt to expand. (E) Muslims set up a complex priesthood that ‘mediated the contact between Allah and individual believers. 4, The military campaigns of the Huns under Auila contributed to which of the following? (A) The introduction of the bubonic plague to Asia (B) The fall of the western Roman Empit (C) The division of Charlemagne’ empi (D) The introduction of horse domestication into western Europe (E) The defeat of the Muslims in Spain GO ONTO THE NEXT PAGE 107 ® WORLD HISTORY TEST—Continued 5. After the fall of the Han dynasty, the nomadic peoples who invaded China did which of the following? (A) They attempted to restore the Han dynasty to power. (B) They tried unsuecessfully to convert the Chinese to Islam. (©) They outlawed the use of the Chinese Ianguage by governing officials. (D) They launched an invasion of Japan (E) They adopted Chinese culture and customs. 6. Mahavira and Buddha were lar in that both (A) were successful military leaders who conquered most of India (B) resisted the spread of Islam in India (C) were theorists who pioneered new ‘mathematical concepts (D) led religious movements that challenged the social order of Hinduism (E) were martyred for their beliefs 7. “Warfare in nineteenth-century southern Africa ‘was revolutionized with the development of the short, stabbing spear, the body shield, and a {actical formation known as the ox’s horns.” The above de: cloped by the (A) Zulu (B) Xhosa (© Sotho (D) Shona (B) Ibo ibes innovations de' 8. All of the following are central to the practice of Islam EXCEPT. (A) Observation of Ramadan through fa (B) Monotheism (C) Prayer five times a day facing Mecca (D) Making a pilgrimage to Mecca at least once (E) Realistic representations of people in art ‘ing D108 9. Which of the following is a pair of neighboring, countries both of which had acquired the cap: bility of exploding nuclear weapons by the late 1990's? (A) Argentina and Chile (B) Mexico and the United States (C) The Crech Republic and Germany (D) India and Pakistan (B) North Korea and South Korea 10. Which of the following best describes the eco: nomic strategy of the Soviet Union under Stalin? (A) Development of a mixed economy (B) Creation of a landowning peasant class (C) Production for export (D) Centralized economie planning (B) Government encouragement of free enterprise 11. Historiography is (A) a single, accurate account of events in past time (B) the study of how historical accounts are produced (C) a chronological chart of historical events (D) ahistorical account based only on written records, (E) the official record of past events, usually produced by a government 12. Which of the following describes the primary role Of the scholar-gentry in imperial Chin (A) The mainstay of the imperial bureaucracy (B) The development of political revolution (C) The education of China's peasantry (D) The dissemination of European culture in China (E) The advancement of engineering and agricultural science GO ONTO THE NEXT PAGE > B Is, V7. ‘The Japanese victory in the Russo-Japanese War demonstrated to other non-Western peoples that (A) successful modernization was not a strictly ‘Western phenomenon, (B) countries that held traditional values could not defeat a European power (C) passive resistance could be effectively employed in the defeat of a European power (D) the distance between Asia and Europe would make Asian industrialization difficult (E) further expansion by Russia in Asia was inevitable The defeat of the Umayyads by the Abbasids in 750 C.E. led to the relocation of the caliphate and of the primary center of Islamic culture from (A) Mecea to Medina (B) Jerusalem to Cairo (C) Damascus to Baghdad (D) Constantinople to Beirut (E) Cérdoba to Alexandria Which of the following is an example of an ancient megalithic structure? (A) Stonehenge (B) The Coliseum (C) Angkor Wat (D) The Acropolis (6) Great Zimbabwe ‘The Aztec viewed the Toltee as (A) barbarians who lacked eulture (B) slaves, fit only for conquest (C) the givers of civilization (D) heretics who practiced a forbidden religion (E) the greatest rivals to the Aztce dominance of the valley of Mexico ‘The terms Indo-European and Bantu were created to describe (A) biological races (B) language groups (C) religious movements (D) artistic styles (B) ancient empires WORLD HISTORY TEST—Continued Is 19, According to one theory of state formation, large states first developed in river valleys because (A) coordination of large-scale irrigation projects created the need for more complex organizations (B) the healthier climates of river valle large populations to develop there (C) river valleys provided the best natural defensive barriers for growing states (D) river valley the only sources of drinking water large enough to support concentrated populations. (E) river valleys were the best sources of metal ores for weapons and tools allowed In England in the late ninetwenth century it was. socially acceptable for young working-class ‘women to take jobs as domestic servants because (A) many of their employers allowed them 10 do volunteer work among the urban poor on evenings and weekends (B) this work was believed to contribute to habits of hard work, cleanliness, and obedience, which were Seen as good preparation for marriage (C) such jobs provided opportunities for them to meet and marry men from higher social classes (D) the training they received in household man- ‘agement provided them with skills needed for later careers in business (E) residence in middle- or upper-class homes contributed to their political education c history, the phrase “Mandate of refers to the (A) divine selection of China as the holiest place in the world. (B) obligation of eact religious teachings dividual to obey (C) divine favor enjoyed by wise and benevolent rulers (D) Chinese version of the Ten Commandments (E) most important of the Confucian writings GO ONTO THE NEXT PAGE > 109) ® 21. Which of the following is the commonly accepted meaning of the term Homo sapiens (A) The southern apes (B) The upright-walking humans (©) The consciously thinking humans (D) The animal with a large brain (B) The missing link In their original form, all of the following major religions focused on humanity’ s relationship to a god or gods EXCEPT (A) Buddhism (B) Christianity (©) Hindui (D) Islam (E) Judaism 23. Social Darwinism is most closely associated with the idea that (A) government should provide support for disad vantaged members of society (B) competition is natural to society (©) revolution is inevitable (D) imperialistic expansion will inerease economic pressures on citizens (E) technological development will decrease the gap between rich and poor 24. Which of the following led Great Britain and France to declare war on Germany in 1939 ? (A) Hitler established a fascist dictatorship in ermany. (B) Germany occuy (©) Germany annexed Austria. (D) Germany invaded Poland. (E) Germany passed anti-Semitic laws. 25. The pyramids in ancient Bgypt were built to function primarily as (A) temples (B) tombs (©) watehtowers (D) astronomical observatories (E) sundials eirper crate rape Dato WORLD HISTORY TEST—Continued 26, 20 29. In the Hindu caste system, members of the Brahman caste originally served as (A) priests (B) farmers (C) warriors, (D) merchants (B) herders Which of the following is attributed to Alexander the Great? (A) Three centuries of political stability in the Middle East (B) The establishment of the basic political forms of the Roman Empire (C) The concept of kingship limited by elected. representative: (D) The spread of Greek cultural forms into, western Asia () The extension of property and inheritance rights to women Prior to the Ron Britain, the (A) Celts (B) Goths (C) Greeks. (D) Mongols (B) Seythians in conquests of Gaul, Spain, and areas were inhabited primarily by Which of the following major ancient civilizations did NOT originate along a river valley? (A) Chinese (B) Indian (C) Egyptian (D) Mesopotamian (B) Greek GO ONTOTHE NEXT PAGE ) 30. Which of the following changes best characterizes the commercial revolution that accelerated during the 1400’s and continued throughout the Age of Exploration? (A) The growth of capitalism as an economic system (B) The shift of the center of trading from the Mediterranean Sea to the Indian Ocean (©) The development and application of (D) The decline in the production of consumer goods (E) The loss of overseas em European nations ss by western 31. The words “alchemy,” “algebra,” “assassin,” “sugar.” “zenith,” and “zero” entered the English Tanguage as a result of the influence on Europe of which of the following cultures? (A) Arabic (B) Turkish (©) Indian (D) Aramaic (&) Hebrew 32. The principal development during the Neolithic Age was the (A) disappearance of the Neanderthals (B) invention of writing (C) beginning of metallurgy (D) domestication of animals and plants (B) appearance of craft specialization WORLD HISTORY TEST—Continued © 33. Cultivation of which of the following crops most drastically changed the geographical distribution of human populations? (A) Sugar (B) Opium (C) Tobacco (D) Tea (B) Cotton 34, Which of the following crops originated in ‘Mesoamerica and spread to South America and the present-day United S pre-Columbian period? (A) Maize (B) Oats (©) Peanuts (D) Potatoes. (E) Wheat 35. The African kingdoms of Mali and Ghana acquired much of their wealth from (A) trade across the Sahara (B) trade with Portuguese ships along the Atlantic coast (C) trade across the Atlantic with the Maya and Aztecs (D) production of food for export to Europe (E) tribute from the Islamic states north of the Sahara GO ONTO THE NEXT PAGE > mp WORLD HISTORY TEST—Continued 36. The map above shows the route of (A) Marco Polo on his travels to the court of Kublai Khan (B) Ibn Battutah on his travels through Dar al-Islam (C) Zheng He in his seafaring voyages from China (D) the Arab slave traders (E) the Buddhist pilgrim Xuanzang GO ONTOTHE NEXT PAGE > ® 37 38 Which of the following was NOT a Swahi state? (A) Zimbabwe (B) Mogadishu (©) Kilwa (D) Mombasa (B) Sofala city- ‘The large earthen mounds built in North America between the tenth and the thirteenth centuries CE, such as those at Cahokia, are most likely evidence for (A) the use of communal dwellings (B) the importance of trade and commerce (C) a large-scale commitment to road-building (D) the importance of religious ceremonies and tals (E) a democratic form of government WORLD HISTORY TEST—Continued 39. 40. ® Following the First World War, the governments ‘of many of the world’s industrialized nations urge women to (A) leave the paid workforce (B) provide food and shelter for disabled veterans (C) take advantage of new opportunities for higher education (D) join the army to offset war losses (E) volunteer their services in understaffed hospitals and rehabilitation centers Which of the following Southeast Asian nations has an Islamic majority? (A) Singapore (B) Indonesia (C) The Philippines (D) Vietnam (E) Thailand GO ONTO THE NEXT PAGE > 113) © WORLD HISTORY TEST—Continued © 41, The four differently shaded land regions on the map above were collectively known as the (A) Quadruple Alliance (B) Hellenistic Kingdoms (C) Mongo! Khanates (D) Mamluk Sultanates (B) Tetrarchy (Uravihard copying aroun al GO ONTO THE NEXT PAGE sy pa oft pg oan ou 2 43. 4s. ‘Mayan civilization differed from Aztec civilization in that (A) nobles governed the Aztec empire, whereas. priests dominated the Mayan society (B) the Aztecs had more peaceful relations with neighboring groups than did the Maya (C) the Aztecs had a much longer period of predominance than did the Maya (D) Mayan cities were generally independent, but “Aztec cities were not (&) Mayan society was much more expansionist than Aztec society In the period before 1500 CE., the two primary trading groups in the Indian Ocean were the (A) Africans and Portuguese (B) Arabs and Indians (C) Arabs and Portuguese (D) Chinese and Europeans (E) Chinese and Indians ‘The first armed attempt to gain Mexican independence from Spain was led by (A) Simén Bolivar (B) Antonio Lépez de Santa Anna (C) Bemardo O° Higgins. (D) Father Miguel Hidalgo (B) José de San Martin ‘The feudal periods in Japan and western Europe ‘were similar in that both (A) coincided with a period of growth in the ‘money economy (B) were characterized by frequent warfare (C) saw the development of strong monarchies (D) were marked by greater freedom for women. than had existed previously (B) were dominated by religious strife Which of the following population in 1000 cE. ities had the largest (A) Constantinople (B) London (C) Paris (D) Rome (E) Toledo WORLD HISTORY TEST—Continued 47, 49, Which of the following is true of the legal status of Jews and Christians in early Islamic society? (A) They were categorically forbidden from holding any public office. (B) As “people of the book.” they were exempt from taxation, (C) They were required to serve in the army in place of Muslims. (D) They were allowed to practice their religions with some restrictions. (E) They were treated as equals of Muslim citizens and were accorded all the same rights and privileges as Muslims. “And I say unto thee, thou art Peter and upon this rock I will build my chureh.” ‘The Biblical passage cited above formed the basis: in the early Catholic church for the (A) authority of the pope (B) emphasis on clerical celibacy (C) seven sacraments (D) construction of cathedrals (B) location of the Vatican What was the most significant impact of the period of the Mongol rule on Russia? (A) The period of Mongol rule reinforced the isolation of Russia from western Europe (B) The Mongols sided the Russians in gaining political dominance over the peoples of the Central Asian steppes. (C) The period of Mongol rule introduced many Muslims into the region of Russia, (D) The Mongol domination resulted in the destruction of Eastern Orthodoxy and the rise of Nestorian Christianity (E) Russians’ admiration of Mongol culture led them to abandon their Byzantine roots. GO ONTO THE NEXT PAGE > 115) ® 50. Which of the following was an important characteristic of the Inea road system? (A) It was well equipped for even the heaviest wheeled wagons, (B) It was kept up by privately owned ‘commercial companies. (©) I required frequent repair because of the high tides and salt water of the Pacific. (D) It facilitated transportation among the towns. of the high Andes mountains. (B) Itlinked independent city-states 51. As a result of the defeats of China in the first Anglo-Chinese war (1839-1842) and in later ‘contficts with Westerners, the Chinese were forced to do all of the following EXCEPT (A) allow Western mission in China (B) cede Hong Kong territory to the British (C) open numerous port cities to foreign traders (D) grant Westerners in China the privilege of cextraterritoriality (E) ban the import of opium into China ies t0 seek converts 52. In 750 C-E.,a major political difference between China and Europe was that, unlike Europe, China (A) was a unified empire (B) was a theocracy (C) was controlled by rulers who came from ‘outside its borders (D) was under threat of invasion from all sides (E) had a republican form of government 53. “When the personal life is cultivated, the family will be regulated; when the family is regulated, ‘will be in order; and when the state isin there will be peace throughout the world.” ‘The quotation above reflects a key tenet of which of the following teachings? (A) Taoism (B) Zen Buddhism (©) Mahayana Buddhism (D) Shinto (E) Confucianism WORLD HISTORY TEST—Continued 3, 56. 58. Ss Barly Roman religious ritual was heavily infl enced by the religious practices of the (A) Seythians (B) Gauls. (©) Biruscans (D) Carthaginians (®) Diuids Afier amassing the largest land empire ever Known, most of the Mongols and Turks who invaded central and south Asia converted to (A) Confucianism (D) Judaism (©) Iskm In the seventeenth century, E trade was dominated by the (A) English (B) Dutch (©) French © © s opean maritime The failure of Europe’s potato erop in the late 1840°s spurred mass emigration from (A) Sweden (B) Spain (©) Ireland (D) Italy (2) Russia Which of the following became important New World contributions to the world’s food crops? (A) Wheat and barley (B) Rice and sugarcane (C) Oats and millet (D) Com and potatoes (£) Bananas and melons GO ONTO THE NEXT PAGE ) 59, 60. 6. ‘Alexander IT emancipated the serfs and intro- duced government reforms following Russia's defeat in the (A) Balkan Wars (B) Crimean War (C) First World War (D) Russo-Turkish Wars () Russo-Japanese War ‘After independence, India pursued a foreign policy that led to which of the following? (A) lis membership inthe Soviet-backed Warsaw Pact (8) lus membership in the Southeast Asia Treaty Organization (©) Its signing of a mutual defense pact with the People’s Republic of China (D) lus emergence as a leader of the Nonaligned Movement (©) lts avoidance of armed conflict with its neighbors Which of the following best characterizes the classical economic theory of Adam Smith? (A) The demands of consumers are met most cheaply by competition among individual producers. (B) Since land is the source of value, the whole economy will benefit if small holdings are consolidated into large estates. (©) An increase in wages will increase the demand for manufactured goods, making the economy as a whole grow (D) Since the interests of businessmen and workers are necessarily in conflict, the interests of one can thrive only at the expense of the other. () To encourage the growth of infant national industries, government should protect them from unfair foreign competition by imposing tariffs WORLD HISTORY TEST—Continued 62, 63, 65. ® Which of the following countries are sof the Organization of Petroleum Exporting Countries (OPEC) ? (A) Argentina, Mexico, and Turkey (B) China, Egypt, and the United States (C) Great Britain, Canada, and Morocco (D) The Soviet Union, Syria, and Kenya (E) Venezuela, Nigeria, and Iraq ‘The navigator James Cook was most famous for (A) being the first to sal around the world (B) charting a northwest passage (C) exploring the Amtaretic continent (D) scientific observation on Caribbean islands (E) charting the seas around Australia and New Zealand In closing Japan to Europeans, the Tokugawa shogunate was motivated primarily by a desire to limit (A) the influence of Westerners on Japanese ‘government and society (B) a large influx of European immigrants (C) widespread intermarriage between Japane: and Europeans (D) the despoiling of Japan's pristine natural ‘environment by Europeans (E) the spread of industrialization to Japan Which of the following was a major consequence of the opening of large silver mines in Spanish colonies in the Americas during the 1500's? (A) The production of goods in Spain for export to its colonies in America was greatly stimulated, (B) The increased wealth circulating in Spain's colonies fueled a resurgence of Native American culture, (C) The European economy experienced an extended period of price inflation. (D) The Spanish colonies where the located were successful in declari independence from Spain. (B) Other European powers succeeded in capturing the mines from Spain GO ONTO THE NEXT PAGE > 170 ® 66. Which of the following sx dependent on livestock? (A) Aztec society (B) Chinese societ (©) Persian society (D) Tartar society (B) Roman society 's was the LEAST 67. Which of the following was called “the Sick Man ‘of Europe” in the nineteenth century? (A) lealy, (B) Spain (C) The Netherlands (D) The Ottoman Empire (E) Russia 68. The eighteenth-century philosophy of Deism was, strongly denounced by (A) Voltaire and his followers (B) the Roman Catholic church (©) essayists in Diderot’s Eneyelopedie (D) Locke and his followers (B) Frederick the Great of Prussia 69. Which of the following was one of the major effects of the spread of gunpowder technology in ie 1400°s and 1500°s CE.? jor firepower of European armies Ted to the reconquest of most lands that had been lost to the Ottoman Turks, (B) The widespread use of guns in hunting led to 4 virtual extermination of game animals and game birds in Europe. (C) The high cost of equipping armies with guns led to a strengthening of some centralized monarchies at the expense of feudal lords, (D) Fear of the new technology led to religious revivals in many areas of Europe. (E) Many European countries sought to avoid confliets with each other because gun- powder made wars more destructive, pig WORLD HISTORY TEST—Continued 70. n 2. 73, Which of the following factors contributed to the success of independence movements in Latin America during the early 1800's? (A) Military and economic aid from the United States (B) An increase in the production of precious ‘metals from Latin American mines (C) The establishment of universities throughout Latin America (D) The drain on Spain's resources caused by the Napoleonic Wars (2) Intervention by professional revolutionaries from France In the sixteenth and seventeenth centuries, the primary uuropean powers in the East Indies (A) buy rice and other food grains (B) obtain wood for shipbuilding (C) obtain spices for trading (D) to seek markets for exports (E) exploit silver mines in the area ‘The most characteristic feature of Enlightenme! thought was (A) opposition to slavery (B) antimateralism (C) opposition to religious belief (D) an emphasis on reason (&) a belief in sexual equality Mazzini, Cavour, and Garibaldi are most often associated with (A) parliamentary democracy in Italy (B) Italian unification (C) the rebuilding of Rome (D) Italian imperialism in Ethiopia (E) Tualian industrialization GO ONTO THE NEXT PAGE > m4 7 ‘When Siddhartha Gautama (the Buddha) embarked on his spiritual quest in the sixth century B.C.E., his primary concern was (A) whether there is one God or many (B) whether there is life after death (C) why humans suffer (D) how to convert nonbel (B) the relationship between religion and the state Which of the following best explains ancient Egypt's ability to support a large population’? (A) Its strategie location on the Mediterranean Sea (B) The approval its religious leaders gave to the concept of large famili (C) The yearly flooding of the Nile River (D) The early iroduction of technology 1d crop rotation system WORLD HISTORY TEST—Continued ® 76. Which of the following best characterizes demo graphic change in eighteenth century England’ (A) Destruction of the nuclear family during the Industrial Revolution caused the population to decline. (B) Unhealthy conditions in crowded cities caused the population to dectine. (C) Pressures of the enclosure movement caused the population to decline, (D) Dramatically rising birth rates caused the population to in (E) Falling death rates caused the population to GO ONTO THE NEXT PAGE > 119 WORLD HISTORY TEST—Continued ® D120 77. The cartoon above shows President Gam: Nasser (1956-1970) encouraging Egyp' the advantages of (A) maintaining equality in the workplace (B) curbing population growth (C) reducing consumption (D) increasing savings (E) legalizing unions. GO ONTO THE NEXT PAGE > © WORLD HISTORY TEST—Continued 78. Of the following Southeast Asian countries, which is NOT matched with the colonial power that dominated it during the colonial period? (A) Vietnam France (B) Burma. Germany (©) Indonesia the Netherlands (D) Malaya Great Britain (E) The Philippines the United States 79. The Boxer Rebellion was a revolt of (A) Indian soldiers against British domination (B) Vietnamese against French domination (©) Arabs against the Otoman Empire (D) Chinese against Western imperialism (E) Koreans against Japanese rule 80. The picture above, which depicts the symbolic ‘crowning of a twelfth-century king of Sicily by (Christ, reveals the cultural influence of (A) Russia. (B) Scandinavia ©) Spain (D) Islam (E) Byzantium [Unahoriaed copying op roves of GO ONTO THE NEXT PAGE > 12> ® 81. The political and religious cemter at Great Zimbabwe, which reached its height in the fifteenth century, was characterized by all of the following EXCEPT (A) long-distance trading (B) gold mining (C) significant population expa (D) a written epic tradition (E) copper and bronze ornament making 82. Which of the following art forms originated in the United States? (A) Impressionism (B) Surrealist poctry (C) Social realism (D) Jaze (E) Atonal music 83. The Provisional Government failed to keep the support of the Russian people in 1917 because it (A) executed the entire royal family (B) collectivized agriculture and industry (©) allowed Nicholas II to rule as a constitutional ‘monarch (D) suffered a humiliating defeat by the Japanese (E) continued Russia’s participation in the First World War 84. Which of the following best deseribes the Indian National Congress?” (A) The first national political organization in India to challenge British rule (B) The first all-Indian legislative body formed after independence in 1947 (©) An organization formed by Hindus that pri- ‘marily preached tolerance of Indian Muslims (D) An organization formed by Hindus and ‘Muslims that sought social reform within India (E) A conference of Muslim religious leaders that convened to discuss Indian statehood 85. OF all the dictatorial regimes established in Europe between the First and Second World Wars, the ‘one that held power the longest was that of (A) Hitler (B) Stalin (©) Mussolini (D) Pilsudski (E) Franco WORLD HISTORY TEST—Continued 86. 87, 89. ‘One of the principal strengths of the Byzantine ‘empire was its (A) constitutional monarchy (B) sound economic base (C) preference for decentralized government (D) close relationship with the Roman Catholic church, (B) orderly system of succession to the throne ‘The first significant test of the ability of the League of Nations to respond when a major nation acted as an aggressor occurred when (A) Japan invaded Manchuria (B) the Soviet Union invaded Poland (C) Japan declared war on China (D) Franco's rebels attacked Spanish loyalists (E) Hitler incorporated Austria into the Third Reich In the 1980's, which of the following Muslim ‘countries most actively promoted Islamic fundamentalism? (A) Morocco (B) Iran (C) Iraq (D) Indonesia (6) Turkey Mao Zedong revolutionized Chinese Marxist doctrine in the 1920's by advocating that the (A) Chinese Communist Party sever its ties with the Soviet Union to preserve its dependence (B) Chinese Communist Party allow its rival, the Kuomintang, to reform a separate government on the island of Taiwan (C) Chinese Communist Party renounce the use of violence to achieve revolution (D) rural peasants, not the urban proletariat, lead the revolution in China. indlord and capitalist classes be allowed (0 survive even after the communists took power © GO ONTO THE NEXT PAGE > ® 90. The economies of China, North Korea, and North Vietnam were relatively isolated from the world economy during much of the third quarter of the: ‘owentieth century primarily because of their (A) adherence to a planned Marxist economy (B) inability to recover from the devastation of the Second World War (©) subjection to a trade embargo enforced by the United Nations (D) subjection to almost continual civil wars (E) enduring extended droughts due to global climate change 91. The partition of Korea at the end of the World War in 1945 was primarily the re ‘cond. It of (A) rivalry between the United States and the Soviet Union (B) Japanese dominance of important sectors of the Korean economy (C) the emergence of China as a major world power (D) the inability of the Koreans to agree on a form of government (E) sharp cultural differences between northem and southern Korea 92. During the American occupation of Japan following the Second World War, authorities seeking to restructure Japanese society received the strongest support from which of the following Japanese groups? (A) Socialist leaders (B) Business leaders. (© Military leaders (D) Expatriates returning to Japan (E) Members of the imperial court STOP WORLD HISTORY TEST—Continued 93, 94, 95, Many historians believe that the end of the French Revolutionary era was the (A) execution of King Louis XVI (B) Reign of Terror (©) storming of the Bastille prison (D) defeat of Napoleon in Russia (B) peace settlement at the Congress of Vienna The nations that signed and confirmed the 1975 Helsinki Accords agreed to (A) establish uniform prices for crude oil (B) establish peace in the Middle Bast (©) cooperate among themselves and respect, human rights (D) end the Vietnam conflict and withdraw all foreign troops (B) end the Cold War In the late twentieth century, experts began to question the value of building large dam projects in the developing world primarily because these projects tend 10 (A) reduce the cost of electric power in the ‘countries in which they are built (B) displace people from their homes and disturb the ecology of the regions in which they are built (C) encourage separatist movements in the in whieh they are built (D) conflict with the development plans of the central governments of the countries in which they are built (E) disrupt road and rail communicati the rivers on which the dams are built as It you finish before time is called, you may check your work on this test only. Do not turn to any other section in the test. py sy ar of hs pogo ga 123) NO TEST MATERIAL ON THIS PAGE D124 World History How to Score the SAT Subject Test in World History When you take an actual SAT Subject Test in World History, your answer sheet will be “read” by a scanning machine that will record your responses to each question. Then a computer will compare your answers with the correct answers and produce your raw score. You get one point for each correct answer. For each wrong answer, you lose one-fourth of a point. Questions you omit (and any for which you mark more than one answer) are not counted. This raw score is converted to a scaled score that is reported to you and to the colleges you specify. Worksheet 1. Finding Your Raw Test Score STEP 1: Table A lists the correct answers for all the questions on the World History Test that is reproduced in this book. It also serves as a worksheet for you to calculate your raw score. + Compare your answers with those given in the table, + Puta check in the column marked “Right” if your answer is correct. «+ Puta check in the column marked “Wrong” if your answer is incorrect. + Leave both columns blank if you omitted the question. STEP Count the number of right answers. Enter the total here: STEP 3: Count the number of wrong answers. Enter the total here: STEP 4: Multiply the number of wrong answers by .250. Enter the product here: STEP 5: Subtract the result obtained in Step 4 from the total you obtained in Step 2. Enter the result here: STEP 6: Round the number obtained in Step 5 to the nearest whole number. Enter the result here: ‘The number you obtained in Step 6 is your raw score. 125) ‘The Official Study Guide for All SAT Subject Tests Table A Answers to the Subject Test in World History, Form 4BAC, and Percentage of Students Answering Each Question Correctly Brees Peet Question the Question Question | pen fetes Right | Wrong | Correctly | Number |4 Ed ita A 66 3 A 4 8 59 34 A 64 c 76 35 A 50 8 49 36 c 52 E 65 7 A 24. D 73 38 . 59 A 61 39 A 33 E 93 40 8 7 D 57 4 c 69 10 D 76 42 ° 19 1 8 35 43 B 7 2 A 54 44 D 37 18 A 80 45 8 55 4 c 4 46 A eo 15 A 51 a7 D 51 16 c 2 48 A 36 W 8 43 49 A 60 18 A 29 50 D 55 19 8 81 51 E 69 20 c n 52 A 65 21 c 47 53 E 65 2 A B 54 c 45 23 8 85 55 E 62 24 D 82 56 8 2 25 8 95 57 c 89 26 A 73 58 D 6 27 D 68 59 8 43 28 A 46 60 D 36 29 E 7 61 A 46 30 A 48 62 E 65 31 a 73 63 E 46 32 D 56 64 A 83 Table A continued on next page D126 World History Table A continued from previous page ere Breen preteens reenter tone - ore cree ee : es Right | Wro 65 66 A 88 a 0 52 68 8 a 69 c 2 70 0 3 n c a n D 53 B 8 56 m4 c 55 5 c 47 6 E 54 n 8 38 18 B 26 13 D a 80 E “These percentages are based on an analysis ofthe answer sheets of a representative sample of 745 students who took the original form ofthis test in Jane 2005, and whose mean score was 61. They may’ be used as an indication of the relative difficulty ofa particular question. Each percentage may also be used to predict the likelihood that a typical SAT Subject Test in World History tes-taker will answer that question correctly on this edition ofthe test 127) ‘The Official Study Guide for All SAT Subject Tests Finding Your Scaled Score When you take SAT Subject Tests, the scores sent to the colleges you specify are reported on the College Board scale, which ranges from 200 to 800. You can convert your practice test score to a scaled score by using Table B (on page 129). To find your scaled score, locate your raw score in the left-hand column of Table B; the corresponding score in the right-hand column is your scaled score. For example, a raw score of 39 on this particular edition of the World History Test corresponds to a scaled score of 580. Raw scores are converted to scaled scores to ensure that a score earned on any one edition of a particular Subject Test is comparable to the same scaled score earned on any other edition of the same Subject Test. Because some editions of the tests may be slightly easier or more difficult than others, College Board scaled scores are adjusted so that they indicate the same level of performance regardless of the edition of the test taken and the ability of the group that takes it. Thus, for example, a score of 400 on one edition of a test. taken ata particular administration indicates the same level of achievement as a score of 400 on a different edition of the test taken at a different administration, When you take the SAT Subject Tests during a national administration, your scores are likely to differ somewhat from the scores you obtain on the tests in this book. People perform at different levels at different times for reasons unrelated to the tests themselves. The precision of any test is also limited because it represents only a sample of all the possible questions that could be asked. D128 World History Table B ‘Scaled Score Conversion Table World History Test (Form 4BAC) Ps na] oe | 2a % 800 90 800 8 800 2 00 Ei 800 0 00 9 800 3 8 800 8 7 800 7 % 800 6 6 800 5 84 800 44 610 4 380 83 800 a 600 3 370 EA 800 a2 590 2 370 at 800 a 590 1 360 80 00 0 50 a 380 19 800 39 50 4 380 78 800 8 570 2 380 n 790 a7 570 a 30 18 790 38 560 4 340 5 780 5 560 5 330 1 770 34 580 6 330 B 770 3 580 7 330 nm 760 a2 540 8 320 1 760 3 530 9 320 70 750 30 530 40 310 6 750 2 520 a 310 68 740 28 520 12 300 7 740 a 510 43 300 66 730 2 510 a4 300 5 720 B 500 18 290 64 720 a 490 16 280 6 70 2 490 av 280 2 70 2 480 18 270 a 700 2 480 19 260 Gy 690 20 470 “20 250 59 690 19 470 2 250 58 680 18 460 2 240 57 680 v7 450 23 230 56 670 16 450 a 220 129) ‘The Official Study Guide for All SAT Subject Tests How Did You Do on the Subject Test in World History? After you score your test and analyze your performance, think about the following question Did you run out of time before reaching the end of the test? If so, you may need to pace yourself better. For example, maybe you spent too much time on one or two hard questions. A better approach might be to skip the ones you can't answer right away and try answering all the questions that remain on the test. Then if there’s time, go back to the questions you skipped. Did you take a long time reading the directions? You will save time when you take the test by learning the directions to the World History ‘Test ahead of time, Each minute you spend reading directions during the test isa minute that you could use to answer questions. How did you handle questions you were unsure of? If you were able to eliminate one or more of the answer choices as wrong and guess from the remaining ones, your approach probably worked to your advantage. On the other hand, making haphazard guesses or omitting questions without trying to eliminate choices could cost you valuable points. How difficult were the questions for you compared with other students who took the test? Table A shows you how difficult the multiple-choice questions were for the group of students who took this test during its national administration, ‘The right-hand column, gives the percentage of students that answered each question correctly. A question answered correctly by almost everyone in the group is obviously an easier question, For example, 82 percent of the students answered question 24 correctly. But only 19 percent answered question 42 correctly. Keep in mind that these percentages are based on just one group of students. They would probably be different with another group of students taking the test. If you missed several easier questions, go back and try to find out why: Did the questions cover material you haven't yet reviewed? Did you misunderstand the directions? D130 Chapter 4 Mathematics Purpose There are two, one-hour subject tests in mathematics: Mathematics Level 1 and Mathematics Level 2. The purpose of these tests is to measure your knowledge of mathematics through the first three years of college-preparatory mathematics for Level 1 and through precalculus for Level 2. Mathematics Level 1 Subject Test Format Mathematics Level 1 is a one-hour broad survey test that consists of 50 multiple-choice questions. The test has questions in the following areas: + Number and Operations + Algebra and Functions + Geometry and Measurement (plane Euclidean, coordinate, three-dimensional, and trigonometry) + Data Analysis, Statistics, and Probability How to Prepare ‘The Mathematics Level 1 Subject Test is intended for students who have taken three years of college-preparatory mathematics, including two years of algebra and one year of geometry. You are not expected to have studied every topic on the test. Familiarize yourself with the test directions in advance. The directions in this book are identical to those that appear on the test. Calculator Use Itis NOT necessary to use a calculator to solve every question on the Level | test, but it is important to know when and how to use one. Students who take the test without a calculator will be at a disadvantage. For about 50 to 60 percent of the questions, there 131 ‘The Official Study Guide for All SAT Subject Tests is no advantage, perhaps even a disadvantage, to using a calculator. For about 40 to 50 percent of the questions, a calculator may be useful or necessary. A graphing calculator may provide an advantage over a scientific calculator on some questions. However, you should bring the calculator with which you are most familiar. If you are comfortable with both a scientific calculator and a graphing calculator, you should bring the graphing calculator. Mathematics Level 2 Subject Test Format Mathematics Level 2 is also a one-hour test that contains 50 multiple-choice questions that cover the following areas: + Number and Operations + Algebra and Functions + Geometry and Measurement (coordinate, three-dimensional, and trigonometry) + Data Analysis, Statistics, and Probability How to Prepare ‘The Mathematics Level 2 Subject Test is intended for students who have taken college- preparatory mathematics for more than three years, including two years of algebra, one year of geometry, and elementary functions (precalculus) and/or trigonometry. You are not expected to have studied every topic on the test. If you have had preparation in trigonometry and elementary functions, have attained grades of B or better in these courses, and have skill in knowing when and how to use a scientific or a graphing calculator, you should select the Level 2 test. If you are sufficiently prepared to take Level 2, but elect to take Level 1 in hopes of receiving a higher score, you may not do as well as you expect. Familiarize yourself with the test directions in advance. ‘The directions in this book are identical to those that appear on the test. Calculator Use Itis NOT necessary to use a calculator to solve every question on the Level 2 test, but it is important to know when and how to use one. For about 35 to 45 percent of the questions, there is no advantage, perhaps even a disadvantage, to using a calculator. For about 55 to 65 percent of the questions, a calculator may be useful or necessary. ‘As with the Level 1 test, a graphing calculator may provide an advantage over a scientific calculator on some questions. However, you should bring the calculator with which you are most familiar. If you are comfortable with both a scientific calculator and a graphing calculator, you should bring the graphing calculator. D132 ‘Mathematics Calculator Policy: You may NOT use a calculator on any Subject Test other than the Mathematics Level 1 and Level 2 Tests. What Calculator to Bring + Bring a calculator that you are used to using. If you're comfortable with both a scientific calculator and a graphing calculator, bring the graphing calculator. + Before you take the test, make sure that your calculator is in good working order. You may bring batteries and a backup calculator to the test center. + The test center will not have substitute calculators or batteries on hand. Students may not share calculators. + If your calculator malfunctions during one of the Mathematics Level 1 or Level 2 Tests and you do not have a backup calculator, you must tell your test supervisor when the malfunction occurs. The supervisor will then give you a form that allows you to cancel the scores on that test only, if you desire to do so, What Is NOT Permitted + pocket organizers + laptops and portable handheld computers + models with typewriter keypads (eg., T1-92 Plus, Voyage 200) + electronic writing pads or pen-input/stylus-driven devices (eg., Palm, PDAs, Casio ClassPad 300) + calculators with paper tapes + calculators that make noise or “talk” + calculators that require an electrical outlet + cell phone calculators Using Your Calculator + Only some questions on these tests require the use of a calculator, First decide how you will solve a problem, then determine if you need a calculator. For many of the ‘questions, there's more than one way to solve the problem. Don’t pick up a calculator if you don’t need to—you might waste time. + The answer choices are often rounded, so the answer you get might not match the answer in the test book. Since the choices are rounded, plugging the choices into the problem might not produce an exact answer. + Don’t round any intermediate calculations. For example, if you get a result from the calculator for the first step of a solution, keep the result in the calculator and use it for the second step. If you round the result from the first step, and the answer choices are close to each other, you might choose the wrong answer. + Read the question carefully so that you know what you are being asked to do, Sometimes a result that you may get from your calculator is NOT the final answer. If 133) ‘The Official Study Guide for All SAT Subject Tests an answer you get is not one of the choices in the question, it may be that you didn't answer the question being asked. You should read the question again. It may also be that you rounded at an intermediate step in solving the problem, and that's why your answer doesn’t match any of the choices in the question. + Think about how you are going to solve the question before picking up your calculator. It may be that you only need the calculator for the final step or two and can do the rest in your test book or in your head. Don't waste time by using the calculator more than necessary. + Ifyou are taking the Level 1 test, make sure your calculator is in degree mode ahead of time so you won't have to worry about it during the test. If you're taking the Level 2 test, make sure your calculator is in the correct mode (degree or radian) for the question being asked. + For some questions on these tests, a graphing calculator may provide an advantage. If you use a graphing calculator, you should know how to perform calculations (eg., exponents, roots, trigonometric values, logarithms), graph functions and analyze the graphs, find zeros of functions, find points of intersection of graphs of functions, find minima/maxima of functions, find numerical solutions to equations, generate a table of values for a function, and perform data analysis features, including finding a regression equation, + You may not use your calculator for sharing or exchanging, or removing part ofa test book or any notes relating to the test from the test room. Such action may be grounds for dismissal and/or cancellation of scores. You do not have to clear your calculator’s memory before or after taking the test. D134 ‘Mathematics Comparing the Two Tests Although there is some overlap between Levels 1 and 2, the emphasis for Level 2 is on more advanced content. Here are the differences in the two tests. Approximate Percentage of Mathematics Test 10-14 10-14 Operations, ratio and proportion, complex numbers, counting, elementary number theory, matrices, sequences, series vectors ‘Algebra and Functions 38-42 48-52 Expressions, equations, inequalities, representation and ‘modeling, properties of functions (linear, polynomial, rational, exponential, logarithmic, tigonometnc, inverse trigonemetrc, pind, piecewise, recursive, parametric) Geometry and Measurement 38-42 28-32 Plane Euclidean Measurement 18-22 = Coordinate a2 10-14 Lines, parabolas, circle, eliases, hyperbolas, symmetry, transformations, polar coordinates Three dimensional +6 6 Solids, surface area and volume (cylinders, cones, pyramids, spheres, prisms), coordinates in three dimensions Trigonometry 8 116 Fight triangles, identities, radian measure, law of cosines, law of sines, equations, double angle formulas Data Analysis, Statistics, and Probability 640 e410 Mean, median, mode, range, interquartile range, standard D150 1 1 1 1 1 1 1 MATHEMATICS LEVEL | TEST For each of the following problems, decide which is the BEST of the choices given. IF the exact numerical value is not ‘onc of the choices, select the choice that best approximates this valuc. Then fill in the corresponding cirele on the answer sheet Notes: (1) A scientific or graphing calculator will be necessary for answering some (but not all) of the questions in this test. For each question you will have to decide whether or not you should use a calculator. (2) The only angle measure used on this test is degree measure, Make sure your calculator is in the degree mode. the n that its figure is not led to provide information useful in sol (3) Figures that accompany problems in this test ane xccurately as possible EXCEPT when itis stated in a specific prob in a plane unless otherwise indicate (4) Unless otherwise specified, the domain of any function f is assumed to be the set of all real numbers for which f(x) is a real number. The range of f is assumed to be the set of all real numbers f(x), whe the domain of (5) Reference information that may be useful in answering the questions in this test can be found on the age preceding Question |. USE THIS SPACE FOR SCRATCHWORK. 1. If xy + 7y = 84 and x47 = 3, what is the value of y? (a) -4 (B) 49 © 84 @) 12 (B) 28 3YBC GO ONTO THE NEXT PAGE > 151) 1 1 1 1 1 1 1 MATHEMATICS LEVEL 1 TEST—Continued USE THIS SPACE FOR § RATCHWORK. 2. When four given numbers are multiplied together, the product is negative. Which of the following could be true about the four numbers? (A) One is negative, wo are positive, and one is (B) Two are negative, one is positive, and one is (C) Two are negative and two are positive. (D) Three are negative and one is positive. (B) Four are negative 3. Ix + y = Sand x~ y= 3, then x? y A9 BIS Ole (25 |) 34 Y & 4, In the figure above, what fractis region with center Q is shaded? 1 L 1 Loy 3 WF BF OF OF OF of the circular GO ONTO THE NEXT PAGE > D152 1 1 1 1 1 1 1 MATHEMATICS LEVEL 1 TEST—Continued USE THIS s ACE FOR SCRATCHWORK. 5. Which of the following is the graph of a linear function with both a negative slope and a negative y-intercept? “ » @ » O * O * © y o » a * a * ©) y s—-» g 6. Ik of k? 4=4- what are all possible values (A) Oonly (B) 2only (© 4only (D) ~2 and 2 only (E) 2,0, and 2 7. 1F b+! = 6! for all values of , what is the value of ©) 3 w2 oF © GO ONTO THE NEXT PAGE > 153) 1 1 1 1 1 1 1 MATHEMATICS LEVEL 1 TEST—Continued 8. At North High School, the number of students taking French is decreasing by 20 students per year and the number of students taking Spanish is increasing by 10 students per year. This year 250 students are taking French, and 100 students are taking Spanish, Which of the following equations could be used to find the number of Years n until the number of students is the same in both courses? (A) 250 = 20n = 100 + 10n (B) 250 +10” = 100 = 20n (C) 250 + 20n = 100 = 10n (D) 20n ~ 250 = 100 + 10n (B) (250 = 20) = n(100 + 10) 9. If y = x3 = L5, for what value of x is y = 2? (A) 0.79 @) Ld (©) 152 (D) 1.87 (E) 6.50 10. The length of a rectangle is four time: the perimeter of the rectangle is 40. what is its area? (a) dom? ) 16 em © 20m? (D) 40 cm? ©) 64 em? D154 USE THIS SPACE FOR § GO ONTO THE NEXT PAGE > 1 1 1 1 1 1 1 MATHEMATICS LEVEL 1 TEST—Continued us E THIS $ ACE FOR SCRATCHWORK. 11. The function g, where g(1) = 0.0661 + 0.96, ccan be used to represent the relation between grade point average g(P) and the numberof hours # spent studying each week. Based on this function, a stu- dent with a grade point average of 3.5 studied how ‘many hours per week? (A) 0.96 (B) 12 © 145 (D) 38.5 (&) 678 12. 34 2x + x° is equivalent to wo (B) 2x* = 4x = 0 (© =F 44x-3=0 (D) 8-26 0 (B) Se4x-3=0 2 P R 13, In right triangle POR in the figure above, sin P = 0.5. What is the length of side QR? 2 B) 3 Os @) 6 © 2 GO ONTO THE NEXT PAGE > 155) 1 1 1 1 1 1 1 MATHEMATICS LEVEL 1 TEST—Continued COUNTEREXAMPLE to the statement “All odd numbers greater than 2 are prime numbers” ? A2 ®3 ©5 M7 9 15. If f(x) = (A) -120 (B) 100 ©) 100 (Dy 120 (&) 220 16. On a blueprint, 0.4 inch represents 6 feet. If the actual distance betw ‘what would be the distance between the corre- sponding buildings on the blueprint? (A) 3210 (B) S.1in ©) 127in D) 304 in (B) 31.7 in D156 + what is the value of (0.1)? en two buildings is 76 feet, USE THIS SPACE FOR s GO ONTO THE NEXT PAGE > 1 1 1 1 1 1 1 MATHEMATICS LEVEL 1 TEST—Continued 17. In the figure above, if éllm and rts, what is the value of x? (A) 65 (B) 80 © 3 (D) 95 © 15 18, For what value of x is-52* > undefined? w+ oo ©fF wf wi 19. A sales team sold an average (arithmetic mean) of 10.375 mobile phones per week during the first 8 weeks of the last quarter of the year. The mem- bers of the sales team will receive a bonus if they sell a total of 185 phones for the quarter. What ‘must their average sales, in phones per week, be for the remaining 5 weeks of the quarter if they are to receive the bonus’? (ay 42 B) 204 © 3 (D) 102 &) 1746 USE THIS SPACE FOR SCRATCHWORK. GO ONTO THE NEXT PAGE > 157) 1 1 1 1 1 1 1 MATHEMATICS LEVEL 1 TEST—Continued 20. What is the y-coordinate of the point at which the Tine whose equation is 3 crosses the y-axis? w-5 7 w 4 » 7 ©} wo 3 © 7 Bc1.4) c A x A D7.-3) 21. In the figure above, the sides of rectangle ABCD are parallel to the axes. What is the distance between point A and point C? (A) 6.07 (B) 7 ©) 10.1 @) 106 &) 15 D158 USE THIS SPACE FOR SCRATCHWORK. GO ONTO THE NEXT PAGE > 1 1 1 1 1 1 1 MATHEMATICS LEVEL 1 TEST—Continued USE THIS $ ACE FOR SCRATCHWORK. 22. Four signal flags — one red, one blue, one yellow, and one green — can be arranged from ‘op to bottom on a signal pole. Every arrangement of the four flags is a different signal. How many different signals using all four flags have the red flag at the top? (3 B)4 (6 (D) 16 (24 23. Triangle FGH is similar to triangle JKL. The length of side GH is 2.1 meters, the length of corresponding side KL is 1.4 meters, and the perimeter of AJKL is 3.6 meters. What is the perimeter of AFGH ? (a) 24m (B) 33m © 43m (D) 5.1m (&) 54m 24. Which of the following is an equation of a line that is parallel to the line with equation Qo ya7? wy (B) y Oy ) y ©» GO ONTO THE NEXT PAGE > 159) 1 1 1 1 1 1 1 MATHEMATICS LEVEL 1 TEST—Continued 25. A tetrahedron was cut from the corner of the cube shown above, with three of its vertices at the midpoints of three edges of the cube. If {etrahedrons of the same size are cut from the remaining seven corners of the cube, how many faces will the resulting solid have? A6 ®B8s O12 MM © 16 and D. Which of the following are NOT necessarily congruent? (A) ZA and ZC (B) ZB and 2D (©) AC and BD (D) AB and CD (E) AD and BC > 160 USE THIS SPACE FOR SCRATCHWORK. GO ONTO THE NEXT PAGE > 1 1 1 1 1 1 1 MATHEMATICS LEVEL 1 TEST—Continued USE THIS s ACE FOR SCRATCHWORK. 27. A car traveled 200 miles at an average speed of 45 miles per hour. Of the following, which is the closest approximation to the amount of time that could be saved on this 200-mile trip if the average speed had increased 20 percent? (A) T hour hour (B) (© $ hour (D) F hour 7 hour «& 28, If c is a negative integer, for which of the following Values of d is |c - d| greatest? @)-0 8-4 ©O M4 HO 29. In SPOR, 20 is aright angle. Which of the following is equal to cos P’? Po PR LR PQ PR OR OR PO “) (B) © ©) QR PR «& GO ONTO THE NEXT PAGE > 161) 1 1 1 1 1 1 1 MATHEMATICS LEVEL 1 TEST—Continued 30. The junior class is sponsoring a drama production to raise funds and plans to charge the same price for all admission tickets. The elass has $700 in expenses for this production. If 300 tickets are sold, the class will make a profit of $1,100, What will be the profit for the class if 500 tickets are sold? (A) $1,133 (B) $1,833 (©) $2,300 (D) $3,000 (B) $3,700 31. In the xy-plane, the point (6, 3) is the midpoint of the line segment with endpoints (x, 5) and (9, y) What is the value of x + y? A4 G9 O14 (18 © 32 3 © wo) © D162 USE THIS SPACE FOR § CRATCHWORK. GO ONTO THE NEXT PAGE > 1 1 1 1 1 1 1 MATHEMATICS LEVEL 1 TEST—Continued USE THIS $ ACE FOR SCRATCHWORK. J, K H L 33. In the figure above, HJKL is a square and JN = NO = OP = PK. What is the ratio of the area of AMNP to the area of square HJKL ? L el 3 1 WM; BOF OF OF OF 34. Which of the following numbers is NOT contained in the domain of the function f t= Ry? Ol OW 3 ) 3 @) = 35. Which of the following is the graph of all values of x for which 1 163) 1 1 1 1 1 1 1 MATHEMATICS LEVEL 1 TEST—Continued 36. The circle in the figure above has center O and radius r. If OB = OD, how many of the line segments shown (with labeled endpoints) have length r? (A) Two (B) Three (©) Four (D) Five (E) Six D Be 37. In the figure above, if AABC and AADC are right triangles, then CD = (B) feet © yerri+2 wD) yer rd © 245 D164 USE THIS SPACE FOR § CRATCHWORK. GO ONTO THE NEXT PAGE > 1 1 1 1 1 1 1 MATHEMATICS LEVEL 1 TEST—Continued USE THIS $ ACE FOR SCRATCHWORK. 1 678 910 i red candies in sample x xX x, x 567 Number 38. Each of 20 students in class took a sample of 10 candies from a large bag and counted the number of red candies in the sample. The distri- bution of red candies in their samples is shown above. If one of the students were chosen at random, what isthe probability thatthe student's, sample would have at least § red candies? 3 Ws B) © ©) © 39, The figure above shows the graphs of functions and g. What isthe value of (¢(3)) ? A2 B-1 ©O MI 2 GO ONTO THE NEXT PAGE > 165) 1 1 1 1 1 1 40. If O is the center of the circle in the figure above, 4 MATHEMATICS LEVEL 1 TEST—Continued AR i ‘what is the length of minor are AB”? (A) 0.65 B) 1.27 © 131 (D) 140 ©) 1.96 In the xy-plane, which of the following are the points of intersection of the circles whose equa- tions are x7 + y? = 4 and (x— 2) +97 = 4? () Ch). (HL 3) @®) (143) (1.-V3) © (4.3.43) @) (0.CL0) (E) (1.1). (1.-2) USE THIS SPACE FOR SCRATCHWORK. GO ONTO THE NEXT PAGE > 1 1 1 1 42, 43, |. If the positiv MATHEMATICS LEVEL 1 TEST—Continued ‘The area of one face of a cube is x square meters. Which of the following gives an expression for the volume of this cube, in cubie meters? wt (B) Wr © evr @ < ©) 30 For which of the following equations is it true that the sum of the roots equals the product of the roots? (© -4x4+4=0 (©) o+aeg with 1, are written consecutively, what will be the 90th digit written? Ao Bl Cs Ms He us E THIS $ ACE FOR SCRATCHWORK, GO ONTO THE NEXT PAGE > 167) 1 1 1 1 1 1 1 MATHEMATICS LEVEL 1 TEST—Continued 45. The function fis defined by S(s) = 8a 4x + x + 1 for 5 Sx $5. In which of the following intervals does the minimum value of f occur? (A) S 1 1 1 MATHEMATICS LEVEL 1 TEST—Continued USE THIS SPACI 49. In the figure above, all of the right circular eylin- radius of 2.5 and 5, cylinder IT is the mean of the volumes of ely. Ifthe volume of Ainders Vand IIT, what is the radius. r of cylinder II? (A) 1.98 (B) 3.75 ©) 395 (D) 4.00 (E) 15.63 50, If f and g are functions, where F(x) = x9 = 10x? + 27x — 18 and a) x? ~ 6x, which of the following gives a relationship between f and g’? (A) gi) = 3/0) (B) gi) = fy -3 © gy = soy +3 ) eo) = fe -3) © gi) = f+3) STOP FOR SCRATCHWORK, IF YOU FINISH BEFORE TIME IS CALLED, YOU MAY CHECK YOUR WORK ON THIS TEST ONLY. DO NOT TURN TO ANY OTHER TEST IN THIS BOOK. 169) ‘The Official Study Guide for All SAT Subject Tests How to Score the SAT Subject Test in Mathematics Level 1 When you take an actual SAT Subject Test in Mathematics Level 1, your answer sheet will be “read” by a scanning machine that will record your responses to each question. ‘Then a computer will compare your answers with the correct answers and produce your raw score. You get one point for each correct answer. For each wrong answer, you lose one-fourth of a point. Questions you omit (and any for which you mark more than one answer) are not counted. This raw score is converted to a scaled score that is reported to you and to the colleges you specify. Worksheet 1. Finding Your Raw Test Score STEP 1: Table A lists the correct answers for all the questions on the Subject Test in Mathematics Level | that is reproduced in this book. It also serves as a worksheet for you to calculate your raw score. + Compare your answers with those given in the table. + Puta check in the column marked “Right” if your answer is correct. «+ Puta check in the column marked “Wrong” if your answer is incorrect. + Leave both columns blank if you omitted the question. STEP 2: Count the number of right answers. Enter the total here: STEP 3: Count the number of wrong answers. Enter the total here: STEP 4: Multiply the number of wrong answers by .250. Enter the product here: STEP 5: Subtract the result obtained in Step 4 from the total you obtained in Step 2. Enter the result here; _____ STEP 6: Round the number obtained in Step 5 to the nearest whole number. Enter the result here: The number you obtained in Step 6 is your raw score. D170 Table A Answers to the Subject Test in Mathematics Level 1, Form 3YBC, and Percentage of Students Answering Each Question Correctly ery erry Bete Studen iri Rig These percentages are based on an analysis ofthe answer sheets ofa representative sample of 21,848 students ‘who took the original form ofthis test in October 2002, and whose mean score was 60S. They may be used as aan indication of the relative difficulty ofa particular question, Each percentage may also be used to predict the likelihood that a typical SAT Subject Test in Mathematics Level 1 candidate will answer that question correctly ‘on this edition af the test im? ‘The Official Study Guide for All SAT Subject Tests Finding Your Scaled Score When you take SAT Subject Tests, the scores sent to the colleges you specify are reported on the College Board scale, which ranges from 200-800. You can convert your practice test score to a scaled score by using Table B, To find your scaled score, locate your raw score in the left-hand column of Table B; the corresponding score in the right-hand column is your scaled score, For example, a raw score of 28 on this particular edition of the Subject Test in Mathematics Level 1 corresponds to a scaled score of 600. Raw scores are converted to scaled scores to ensure that a score earned on any one edition of a particular Subject Test is comparable to the same scaled score earned on any other edition of the same Subject Test. Because some editions of the tests may be slightly easier or more difficult than others, College Board scaled scores are adjusted so that they indicate the same level of performance regardless of the edition of the test taken and the ability of the group that takes it. Thus, for example, a score of 400 on one edition of a test taken ata particular administration indicates the same level of achievement as a score of 400 on a different edition of the test taken at a different administration, When you take the SAT Subject Tests during a national administration, your scores are likely to differ somewhat from the scores you obtain on the tests in this book. People perform at different levels at different times for reasons unrelated to the tests themselves. ‘The precision of any test is also limited because it represents only a sample of all the possible questions that could be asked. D172 Table B Scaled Score Conversion Table Subject Test in Mathematics Level 1 (Form 3¥BC) 50 800 28 600 390 49 800 a 580 390 48 790 26 370 380 a 780 B 560 370 46 ™ m4 580 360 6 760 a 540 360 44 750 2 530 350 8 m0 a 520 340. 42 730 20 510 2 330 a 720 19 500 a 330, 0 720 18 490 “4 320 a9 70 7 490 8 310 38 700 16 480 6 300 a 690 15 470 a 290 36 680 4 460 8 280 35 670 3 450 9 270 34 660 12 440 10 260 3 650 " 440 a 260 2 640 10 430 12 250 31 630 9 420 30 620 a 410 29 610 1 400 173) ‘The Official Study Guide for All SAT Subject Tests How Did You Do on the Subject Test in Mathematics Level 1? After you score your test and analyze your performance, think about the following question Did you run out of time before reaching the end of the test? If'so, you may need to pace yourself better. For example, maybe you spent too much time on one or two hard questions. A better approach might be to skip the ones you can’t answer right away and try answering all the questions that remain on the test. Then if there’s time, go back to the questions you skipped. Did you take a long time reading the directions? You will save time when you take the test by learning the directions to the Subject Test in Mathematics Level 1 ahead of time. Each minute you spend reading directions during the test is a minute that you could use to answer questions. How did you handle questions you were unsure of? If you were able to eliminate one or more of the answer choices as wrong and guess from the remaining ones, your approach probably worked to your advantage. On the other hand, making haphazard guesses or omitting questions without trying to eliminate choices could cost you valuable points. How difficult were the questions for you compared with other students who took the test? Table A shows you how difficult the multiple-choice questions were for the group of students who took this test during its national administration. The right-hand column. gives the percentage of students that answered each question correctly. A question answered correctly by almost everyone in the group is obviously an easier question. For example, 89 percent of the students answered question 14 correctly. But only 29 percent answered question 43 correctly. Keep in mind that these percentages are based on just one group of students. They would probably be different with another group of students taking the test. If you missed several easier questions, go back and try to find out why: Did the questions cover material you haven't yet reviewed? Did you misunderstand the directions? Daim Mathematics Level 2 Sample Questions All questions in the Mathematics Level 2 Test are multiple-choice questions in which you must choose the BEST response from the five choices offered. The directions that follow are the same as those that are in the Mathematics Level 2 test. For each of the following problems, decide which is the BEST of the choices given. If the exact numerical value is not one of the choices, select the choice that best approximates this value, Then fill in the corresponding circle on the answer sheet. Notes: (1) A scientific or graphing calculator will be necessary for answering some (but not all) of the questions in this test. For each question you will have to decide whether or not you should use a calculator. 2) For some questions in this test you may have to decide whether your calculator should be in the radian mode or the degree mode. (3) Figures that accompany problems in this test are intended to provide information useful in solving the problems. They are drawn as accurately as possible EXCEPT when it is stated in a specific problem that its figure is not drawn to scale. All figures lie in a plane unless otherwise indicated. (4) Unless otherwise specified, the domain of any function fis assumed to be the set of all real numbers x for which f(x) isa real number. The range of fis assumed to be the set of all real numbers f(x), where x is in the domain of f 6) Reference information that may be useful in answering the questions in this, test can be found on the page preceding Question 1. 175) ‘The Official Study Guide for All SAT Subject Tests Reference Information: The following information is for your reference in answering some of the questions in this test Volume ofa right circular cone with radius r and height hi: V Lateral Area of a right circular cone with circumference of the base ¢ and slant height & $= 3c 4p yar Volume of a sphere with radius r: V= Surface Area of a sphere with radius r: $ = 4nr? Volume of a pyramid with base area B and height h: V = Bh Number and Operations 1. Froma group of 6 juniors and 8 seniors on the student council, 2 juniors and 4 seniors will be chosen to make up a 6-person committee. How many different 6-person committees are possible? (A) 84 (8) 85 (©) 1,050 (0) 1710 (©) 1,890 Choice (C) is the correct answer to question 1. The 2 juniors on the committee can be chosen from the 6 juniorsin (§)=15 ways. The4 seniors on the committee can be chosen from the 8 seniors in (§) =70 ways. Therefore, there are (15)(70)=1,050 possibilities for the 6-person committee. Algebra and Functions 2, If2*= 3, what does 3* equal? (A) 57 (B) 5.2 © 20 (D) 18 (E) 16 D176 Mathematics A calculator is useful for this problem, To solve for x, you can take the natural log of both sides of the equation. xed in2 0.6931 3° 5.7045 Choice (A) is the correct answer to question 2. Since the directions to this test state, “If the exact numerical value is not one of the choices, select the choice that best approximates this value,” the correct answer is choice (A). You can also solve this problem by graphing Y1=2" and Y2=3 and finding the point of intersection of the two graphs in the standard viewing window. The two graphs intersect at the point with x-coordinate ~ 1.5850. You can store this x-value and then evaluate 3*, which gives 5.7045. Many graphing calculators retain the last calculation from the graph screen in memory. If you return to the home screen immediately after finding the point of intersection, you can use the x-coordinate (called “X” or “xc,” depending on the calculator) to evaluate 3°. axe +bxt+cx+dx'+e=0 3. Let a, b, c d, and ¢ represent nonzero real numbers in the equation above. If the equation has 2i as a root, which of the following statements must be true? (A) The only other nonreal root of the equation is ~ 2 (B) The equation has an odd number of nonreal roots. (©) The equation has exactly one real root. (D) The equation has an odd number of real roots. (6) All real roots of the equation are positive Choice (D) is the correct answer to question 3. Since ax°+ bx'+-cx?+dx?+-e=0 is a Sth- degree polynomial equation with real coefficients, the equation has exactly 5 roots in the complex number system. Because 2 is a root of the equation, -2/ is also a root. Complex roots always occur in conjugate pairs a+ bi, where a and b are real numbers and b#0. ‘There are two possibilities for the other 3 roots of the equation. () 1 real root, 2 complex (nonreal) roots (2) 3 real roots Since the equation could have 4 nonreal roots, choice (A) does not have to be true. Since nonreal roots always occur in pairs, choice (B) cannot be true. Since the equation could have 3 real roots, choice (C) does not have to be true. We do not have enough information about the polynomial equation to determine the sign of the real roots. Therefore, choice m7? ‘The Official Study Guide for All SAT Subject Tests (E) does not have to be true. Since the equation could have 1 or 3 real roots, choice (D) must be true, 4, Two environmentalists have proposed two different function models for the survival rate of a particular endangered species. Sit) = 1000.7)' g() = 100(0,999993)" For the functions f and g above, fit) and g(#) represent the percentage of the species that survive t years from a starting point t=0. Which of the following statements about the models are true? I. Both models give the same prediction at approximately t= 15 years. Il, Model g predicts that the population size will decrease most rapidly from t=0 to f=5 years. IIL, The greatest difference in the two model predictions occurs at approximately 6 years. (A) Tonly (B) Land II only (©) Land III only (D) Mand III only (©) Land Il Choice (C) is the correct answer to question 4 since statements I and III are true, You can use a graphing calculator to help you solve this problem. Enter functions f and g in the calculator as Y1 and Y2, respectively. By examining the graphs of the two functions or a table of values for the two functions, you can determine that f)=g(0 for a value between f= 15 and t= 16. Both models give the same prediction at approximately ¢= 15.024. Thus, statement I is true. By examining the graph of g or a table of values for g on the interval from 1=0 to ¢=5, you can see that the g(#) values are fairly constant and show little decrease. The function values start to decrease after =5 years. Thus, statement II is not true. You can look at the graph of Y1-¥2 or a table of values for Y1 - Y2 to determine where the greatest difference between the two model predictions occurs. The greatest difference occurs at approximately ¢= 5.976, Thus, statement IIT is true. D178 Mathematics 5. LEfla) = ot for all x#1, which of the following statements must be true? 5 1. f@) =f) IL fo) UL. f@) (A) None (B) Tony (© Monly (D) Tand TI only (E) 1,1, and IIL l-x x-1 greatly simplifies this problem. Since f(0), (2), 3), and f(4) are all equal to ~1, statements Choice (B) is the correct answer to question 5. Realizing that 1—* = -1 for all x21 1, I, and Ill are all true. If you do not realize f(x) = -1, you can easily substitute the numbers in f. Using a calculator may actually be a disadvantage to you if you spend time substituting the numbers into an expression of this kind to find the answer. However, + —* and see that the graph isa horizontal line crossing the y-axis at -1. Therefore, f(x) = -1 for all values of x except 1 using a graphing calculator, you can graph y= 6, Leth be the function defined by h(#) =|5cos{ 31) -2). What is the period of h 2 Oy) (B) © © (E) 30 i Choice (B) is the correct answer to question 6. The period of ht corresponds to the length of one cycle of the graph of ft. The smallest positive real number k such that (c+) = h(x) for every value of x in the domain of his the period. By examining the geaph of on your graphing calculator, you can see that the values of h(x) repeat every 37 units, Alternately, note that the graph of ft is obtained from the graph of y=cos¢ by applying several transformations. The vertical “stretch” by a factor of 5 units and the shift down 2 units do not affect the period of the function. The absolute value, in this case, also does not affect the period of the function. The horizontal “stretch” is a result of the 5. 179 ‘The Official Study Guide for All SAT Subject Tests ‘This affects the period. Since the period of cost is 27, the period of It can be found o o @ 2 ‘The formula for the volume of the pyramid and several other formulae are given in the reference information at the beginning of the test. The volume of a pyramid is Bh, where B is the area of the base of the pyramid and hh is its height. It may be helpful to mark the figure to indicate those parts whose lengths are given or that can be deduced, P Roa T ESE Choice (A) is the correct answer to question 9. Since PS is perpendicular to the triangular base RST, its length h is the height of the pyramid PRST. R and Tare the midpoints of the two adjacent edges of the cube; therefore, R: areais(3)(4)(4) ="§ \ce ARST is a right triangle, its de 2 Thus, the volume of PRST is ( 181) ‘The Official Study Guide for All SAT Subject Tests Geometry and Measurement: Trigonometry M c B Note: Figure not drawn to scale. 10, In ABC above, CM 1. AB If AM=9, MB=15, and the measure of Z BACis 22°, what is the length of CB: (A) 3.64 (B) 971 © 1543 (D) 1716 (2) 1749 Choice (C) is the correct answer to question 10, You can use right triangle ACM to find the length of CM. tan22° thus, CM =3.636. ‘Now you can use the Pythagorean theorem on right triangle CMB to find the length of CB. D182 Mathematics 32° 48" x Zz 5 miles x 11, The airplane in the figure above is flying directly over point Z on a straight, level road. The angles of elevation for points X and Y are 32° and 48°, respectively. If points X and Y are 5 miles apart, what is the distance, in miles, from the airplane to point X? (A) 1.60 (B) 2.40 (© 269 (D) 377 (2) 701 Choice (D) is the correct answer to question 11. Label the location of the airplane as point W. Then in AXYW, the measure of ZX is 32°, the measure of ZY is 48°, and the measure of ZWis 100°, Let x, y, and w denote the lengths, in miles, of the sides of AXYW opposite respectively. By the law ed = 2X, L¥,and ZW, respectively. By the law of sines, == p= Mp. Since w=5 and 5 yon the distance from the plane to point X is it follows that 4-5 = =... This gives int X. y=3.77 for the distance, in miles, from the plane to p 183) ‘The Official Study Guide for All SAT Subject Tests Data Analysis, Statistics, and Probability 12. "The standard deviation is least for the data shown in which of the following histograms? (A) (B) ‘Lili 10, 10 © (D) yo «®) hel 10 Choice (C) is the correct answer to question 12. The standard deviation is a measure of spread—how far the observations in a set of data are from their mean. ‘The data is closest to 10 in the histogram in choice (C), and thus has the least standard deviation. In each of the other choices, the data is further spread from 10. D184 Mathematics Level 2 Test Practice Helps The test that follows is an actual, recently administered SAT Subject Test in Mathematics Level 2. To get an idea of what it's like to take this test, practice under conditions that are much like those of an actual test administration. Set aside an hour when you can take the test uninterrupted. Make sure you complete the test in one sitting. Sit at a desk or table with no other books or papers. Dictionaries, other books, or notes are not allowed in the test room. Remember to have a scientific or graphing calculator with you. Tear out an answer sheet from the back of this book and fill it in just as you would on the day of the test. One answer sheet can be used for up to three Subject Tests. Read the instructions that precede the practice test. During the actual administration you will be asked to read them before answering test questions. Time yourself by placing a clock or kitchen timer in front of you. After you finish the practice test, read the sections “How to Score the SAT Subject Test in Mathematics Level 2” and "How Did You Do on the Subject Test in Mathematics Level 2?” The appearance of the answer sheet in this book may differ from the answer sheet you see on test day. 185) MATHEMATICS LEVEL 2 TEST ‘The top portion of the section of the answer sheet that you will use in taking the Mathematics Level 2 Test must be filled in exactly as shown in the illustration below. Noie carefully that you have to do all of the following on your answer sheet. L. Print MATHEMATICS LEVEL 2 on the line under the words “Subject Test (print).” 2. In the shaded box labeled “Test Code” fill in four circles’ ll in circle 5 in the row labeled V. ll in circle 3 in the row labeled W, lin circle 5 in the row labeled X. lin circle E in the row labeled Y. Gave 5 ‘Subject Test prin) ® MATHEMATICS LEVEL 2 o 3. Please answer Part I and Part IL below by filling in the specified circles in row Q that correspond to the ‘courses you have taken or are presently taking, and the circle that corresponds to the type of calculator you are going to use to take this test. The information that you provide is for statistical purposes only ‘and will not affect your score on Part I. Which of the following describes a mathematics course you have taken or are currently taking? (FILL IN ALL. CIRCLES THAT APPLY.) ‘Algebra Lor Elementary Algebra OR Course I ofa college preparatory mathematics sequence —Fillinerele 1 . try OR Course I ofa college preparatory mathematies sequence Fill in circle 2 Algebra Il or Intermediate Algebra OR Course Ill ofa college preparatory mathematics sequence —Fillin eile 3. Elementary Functions (Precalculus) and/or Trigonometry OR beyond Course Ill of a college ‘Preparatory mathematics sequence —Fill in civele 4 ulus BC) Fill in circle 5, Advanced Placement Mathematies (Calculus AB or Part 11, What type of calculator did you bring to use for this test? (FILL IN THE ONE CIRCLE THAT APPLIES. If ‘you did not bring a scientific or graphing calculator, do not fill in any of circles 6-9.) * Sciemtitie —Fill in cirele 6 ‘© Graphing (Fill in the cirele corresponding to the model you used.) Casio 9700, Casio 9750, Casio 9800, Casio 9850, Casio FX 1.0, Sharp 9200, Sharp 9300, Sharp 9600, Sharp 9900, TI-82, TI-83, TI-83 Plus, TI-S3 Plus Silver, TI-84 Plus, TI-84 Plus Silver, TLS5, or T-86 “Pill in cirete 7. Casio 9970, Casio Algebra FX 2.0, HP 38G, HP 39 series, HP 40G, HP 48 series, HP 49 series, TH80, or T-89 Titanium —Fill in circle 8 ‘Some other graphing calculator Fill in circle 9 ‘When the supervisor gives the signal, turn the page and begin the Mathematics Level 2 Test. There are 100 numbered circles on the answer sheet and 50 questions in the Mathematics Level 2 Test. Therefore, use only circles 1 to 50 for recording your answers. [Braud copying orous oT sr par ths ge se D186 2 2 2 2 2 2 2 MATHEMATICS LEVEL 2 TEST REFERENCE INFORMATION THE FOLLOWING INFORMATION IS FOR YOUR REFERENCE IN ANSWERING SOME OF ‘THE QUESTIONS IN THIS TEST. Volume of a right circular cone with radius rand height ft: V Lateral Area of a right circular cone with circumference of the base ¢ and slant height ¢ Volume ofa phe with ras rs V= a =4ar? Surface Area of a sphere with radius 1: Volume of a pyramid with base area B and height ht DO NOT DETACH FROM BOOK. GO ONTO THE NEXT PAGE > 187) 2 2 2 2 2 2 2 MATHEMATICS LEVEL 2 TEST For each of the following problems, decide whieh is the BEST of the choices given. Ifthe exact numerical value is not ‘one of the choices, select the choice that best approximates this value. Then fill in the corresponding cirele on the answer sheet. Notes: (1) A scientific or graphing calculator will be necessary For answering some (but not all) of the questions in this test. For each question you will have to decide whether of not you should use a calculator. (2) For some questions inthis test you may have to decide whether your calculator should be mode or the degree mod the radian (3) Figures that accompany problems in this test are intended to provide information useful in sob problems. They are drawn as accurately as possible EXCEPT when itis stated in a specific problem that its figure is not 189) 2 2 2 2 2 2 2 MATHEMATICS LEVEL 2 TEST—Continued 5. IF f(x) = 3in(x) ~ Land g(x) =e, then f(g(5)) (A) 6.83 (B) 12 © 4 (D) 45.98 (b) 568.17 6. The intersection of a cube with a plane could be which of the following? 1. A square UL. A parallelogram IML. A triangle (A) Tonly (B) Ionly © tony (D) Land IIL only (©) 11, and IT Abst O97. ‘The figure above shows a rocket taking off vertically. When the rocket reaches a height of 12 kilometers, the angles of elevation from points A and B on level ground are 84.1° and 62.7°, respectively. What is the distance between points A and B? (A) 0.97 km (B) 6.36 km (© 7.43 km (D) 22.60 km (&) 139.37 km USE THIS SPACE FOR SCRATCHWORK. GO ONTO THE NEXT PAGE 2 2 2 2 2 2 2 MATHEMATICS LEVEL 2 TEST—Continued USE THIS SPAC! FOR SCRATCHWORK. 8. What is the value of x if.x = 15 = 1 AZ B3 OF M8 © 9. ‘The points in the rectangular coordinate plane are transformed in such a way that each point P(x, ») is moved to the point P’(2x.2y), If the distance between a poi distance between the point P? and the origin is 1 Wt a P and the origin isd, then the @B) Od (D) 2a a then g(x) = (A) By Ver +1 «x (D) (e+ [Urastberied copying a veuse sry part of page eget GO ONTO THE NEXT PAGE > 191) 2 2 2 2 2 2 2 MATHEMATICS LEVEL 2 TEST—Continued II. If A is the degree measure of an acute angle and sin A 08, then cos(90°= A) = (a) 02 (B) 04 © 05 (D) 0.6 (©) 08 12. The set of points (x, y, 2) such that (A) empty (B) a point (©) asphere (D) acircle (©) apk 13, The graph of the rational function f, where (O* Beet i6 ; has a vertical (A) Oonly (B) 4only (C) Sonly (D) Oand4 only (©) 0,4, and 5 USE THIS SPACE FOR SCRATCHWORK. GO ONTO THE NEXT PAGE 2 2 2 2 2 2 2 MATHEMATICS LEVEL 2 TEST—Continued USE THIS SPACE FOR SCRATCHWORK. 14. The graph of y = x44 10x° + 10x? = 96x + ¢ is shown above. Which of the following could be the value of ©? (A) 3.240 (B) 1,080 © 7 (D) =72 (&) -3.240 15. If cos x = 0.4697, then see (A) 2.1290 (B) 2.0452 (C) 1.0818 (D) 0.9243 (B) 0.4890 GO ONTO THE NEXT PAGE > 193) 2 2 2 2 2 2 2 MATHEMATICS LEVEL 2 TEST—Continued USE THIS SPACE FOR SCRATCHWORK. 16. A club is planning a trip to a museum that has an admission price of $7 per person. The club mem- bers going on the trip must share the $20) cost of aa bus and the admission price for 2 chaperones who vill accompany them on the trip. Which of the following correctly expresses the cost, in dollars, for each club member as a function of 1, the number of club members going on the trip? 200 + 7n (A) en) = O47 (B) e(ny = 27H ©) ein) = 204.70 200 + 70 (D) en) (&) cn) = 17. Which of the following is an equation whose ‘graph is the set of points equidistant from the points (0,0) and (0,4)? ay ®) © (oy © 18, What is the sum of the infinite geometric series 1 wet ®1 © M2 HF GO ONTO THE NEXT PAGE 2 2 2 2 2 2 2 MATHEMATICS LEVEL 2 TEST—Continued 19, Which of the following is equivalent to p+s>p-s? (A) p>s (B) p>0 (C)s>p (D) 5>0 ©) <0 20. If @ and 6 are in the domain ofa funetion f and f(a) < FO), Which of the following must be true? (A) a= 0 orb =0 B)ab (Dab (B) a=b 21, In arecent survey, it was reported that 75 percent of the population of a certain state lived within ten miles of its largest city and that 40 percent of those ‘who lived within ten miles of the largest city lived. in single-family houses. Ifa resident of this state is selected at random, what is the probability that the person lives in a single-family house within ten miles of the largest city? (A) 0.10 (B) 0.15 (C) 0.30 (D) 0.35 (B) 22. To the nearest degree, what is the measure of the smallest angle in a right triangle with sides of lengths 3, 4, and 5? (A) 27° (B) 30° (©) 37° (b) 45° (B) 58° [Urastberied copying a veuse sry part of page eget USE THIS SPACE FOR SCRATCHWORK. GO ONTO THE NEXT PAGE > 195) 2 2 2 2 2 2 2 MATHEMATICS LEVEL 2 TEST—Continued USE THIS SPACE FOR SCRATCHWORK. 23. Which of the following is an equation of a line perpendicular to y = —2x +3? (A) y= 3x-2 (B) y= 2x-3 © yeless (D) y ©) y 24. What is the range of the function f, where I(x) = 4 + 3sin(2x + 52)? A) 7s si) $3 (B) ~7S f(x) S-1 (©) 35 70) 53 @) 3< fs © -1< fst GO ONTO THE NEXT PAGE 2 2 2 2 2 2 2 MATHEMATICS LEVEL 2 TEST—Continued 26. The formula A = Pe®°® gives the amount A that a savings account will he worth afer an initial investment P is compounded continuously at an annual rate of & percent for t years. Under these conditions, how many years will it take an initial investment of $1,000 to be worth approximately $5,000? (A) 44 (B) 50 (C) 87 (D) 20.1 &) 23.0 ui Vv 27. Ifsin 8 > 0 and sin @.cos 8 < 0, then 6 must be in which quadrant in the figure above? yt «By «© mm (Dy IV (B) There is no quadrant in which both conditions are true, [Urastberied copying a veuse sry part of page eget USE THIS SPACE FOR SCRATCHWORK. GO ONTO THE NEXT PAGE > 197) 2 2 2 2 2 2 2 MATHEMATICS LEVEL 2 TEST—Continued 28.1 f(x) = f(s) for all weal numbers x and if (6.8) is @point on the graph of f, which of the following points must also be on the graph of 2 (A) (8.3) ®) (3.-8) © (3.8) (D) (3-8) (E) (8,3) If'x = y, then x 29. If x and y are real numbers, which of the following CANNOT be inferred from the statement above?” (A) In order for x? to be equal to sufficient that 2 be equal (0 (B) A necessary condition for x t0 be equal toy is that x° be equal to y" (©) x is equal to y implies that x° is equal toy”. (D) If? is not equal to y°, then x is not equal toy. (E) If? is equal to y*, then x is equal to y. 30. In how many different orders ean 9 students arrange themselves in a straight line? ayo (B) 81 (©) 181.440 (D) 362,880 (E) 387,420,489 USE THIS SPACE FOR SCRATCHWORK. GO ONTO THE NEXT PAGE 2 2 2 2 2 2 2 MATHEMATICS LEVEL 2 TEST—Continued USE THIS S PACE FOR SCRATCHWORK. 31. What vale dos os wo (2) oa 1 (D)2 EE) oes not approach a unique ve approach as hes 1? 32.18 f(s [5 — 3x} then (2) = (A) £2) @) FD © so) © £(5) © #(3) 33. What is the period of the graph of y= 2uanGar +4)? «py (B) [Urautberied copying aroun oT GO ONTO THE NEXT PAGE > sry part of page eget 199) 2 2 2 2 2 2 2 MATHEMATICS LEVEL 2 TEST—Continued East Road 34. The figure above shows a ear that has broken down on East Road. A tow truck leaves a garage ‘on North Road at point B. The straight-line distance between points A and B is 50 miles. Ifthe tow truck travels at an average speed of 45 miles per hour along North and East Roads, how long will it take the tow truck to get to the car? (A) 27 minutes (B) | hourand 7 minutes (C) hour and 28 minutes (D) | hour and 33 minutes (E) | hour and 46 minutes USE THIS SPACE FOR SCRATCHWORK. GO ONTO THE NEXT PAGE 2 2 2 2 2 2 2 MATHEMATICS LEVEL 2 TEST—Continued F(x) 35. If / is a polynomial of degree 3, four of whose values are shown in the table above, then f(x) could equal ww (x +e)e +06 +2) a(e-4) (©) (+ De = 2) =D (B) (e+ DG = @) @+2(x-z)o-) (B) (e+ 2)(e + D(x 2) 36. The only prime factors of a number n are 2, 5,7, and 17. Which of the following could NOT be a factor of n? (A) 10) 20 (C) 25 (D) 30H) 4 7. 16 0 < x < % and sin x = 3cos.x, what is the 37.105 xs Zand 3.cos x, what is th value of 1°? (A) 0.322 (B) 0.333 «C) 0.340 (D) 1.231 (B) 1.249 [Urautberied copying aroun oT sry part of page eget USE THIS SPACE FOR SCRATCHWORK. GO ONTO THE NEXT PAGE > 201 2 2 2 2 2 2 2 MATHEMATICS LEVEL 2 TEST—Continued 38. If f(x) = SV2x, what is the value of f~"(10)? (A) 0.04 (B) 0.89 © 2.00 (D) 2.23 (&) 2236 39. The Fibon: recursively as sequence can be defined 6, = Oyj yay Torn > 3, What is the 10th term of this sequence? (a) 21 (4 © 55 (D) 89 ©) 144 40. If f(x) = 09 — 4x? — 3x + 2, whieh of the following statements are truc 1. The function f is increasing for x > 3. I, The equation (x) = 0 has two nonreal solutions IML, f(x) = =16 forall x > 0. (A) Lonly (B) Ionly (© Tand I (D) Land UL (©) Mand IIL USE THIS SPACE FOR SCRATCHWORK. GO ONTO THE NEXT PAGE 2 2 2 2 2 2 2 MATHEMATICS LEVEL 2 TEST—Continued USE THIS SPACE FOR SCRATCHWORK. 41. Portions of the graphs of f and g are shown above. Which of the following could be a portion of the graph of fe? “ y @) y 1 1 779] i“ Xoo i * Ht i © y © y 1 =i ‘ iO i7* HI ©) y 1 GO ONTO THE NEXT PAGE > 203 2 2 2 2 2 2 2 MATHEMATICS LEVEL 2 TEST—Continued 42. The set of all real numbers « such that Vi? = =x consists of (A) zero only (B) nonpositive real numbers only (©) positive real numbers only (D) all real numbers (E) no real numbers 43. In the triangle shown above, sin a = “ B) © ©) © 44. The length, width, and height of a rectangular solid are 8, 4, and 1, respectively. What is the length of the longest line segment whose end points are two vertices of this solid? (A) v5. Bo © 3vi0 ) 10 &) 12 USE THIS SPACE FOR SCRATCHWORK. GO ONTO THE NEXT PAGE 2 2 2 2 2 2 2 MATHEMATICS LEVEL 2 TEST—Continued USE THIS SPACE FOR SCRATCHWORK. 45. If log,3 = x and log, 5 = y, then log, 45 (A) 2x + y @B v+y © (D)xey (B) 9x4 y 46. If sin @ = 1, then, for all in the interval 0<0< tnd = wot vier ) — vir 47. Which of the following shifts of the graph of y y = a7 — 2x +k, where & isa constant ‘greater than 2? x? would result in the graph of (A) Left 2 units and up & units (B) Left 1 unitand up & + 1 units (C) Right 1 unit and up & + 1 units (D) Lett 1 unit and up & ~ Lunits (E) Right 1 unit and up ~ 1 units GO ONTO THE NEXT PAGE > 205) 2 2 2 2 2 2 2 MATHEMATICS LEVEL 2 TEST—Continued 48. IF the height of a right circular cone is decreased by 8 percent, by what percent must the radius of the base be decreased so that the volume of the ccone is decreased by 15 percent? (A) 4% (B) 7% ©) 8% (D) 30% (E) 45% 49. LE matrix A has dimensions m Xm and matrix B has dimensions n x p, where m,n, and p are distinct positive integers, which of the following statements must be true? 1. The product BA does not exist. UL The product AB exists an mx IIL, The product AB exists and has dimensions nxn, has dimensions (A) Lonly: (B) Honly © Illonly (D) Land I (©) Vand USE THIS SPACE FOR SCRATCHWORK. GO ONTO THE NEXT PAGE 2 2 2 2 2 2 2 MATHEMATICS LEVEL 2 TEST—Continued USE THIS SPACE FOR SCRATCHWORK. 50. If w is the complex number shown in the figure above, which of the following points could be HA BB OC OD GE STOP IF YOU FINISH BEFORE TIME IS CALLED, YOU MAY CHECK YOUR WORK ON THIS TEST ONLY. DO NOT TURN TO ANY OTHER TEST IN THIS BOOK. 207 ‘The Official Study Guide for All SAT Subject Tests How to Score the SAT Subject Test in Mathematics Level 2 ‘When you take an actual SAT Subject Test in Mathematics Level 2, your answer sheet will be “read” by a scanning machine that will record your responses to each question. ‘Then a computer will compare your answers with the correct answers and produce your raw score. You get one point for each correct answer. For each wrong answer, you lose one-fourth of a point. Questions you omit (and any for which you mark more than one answer) are not counted. This raw score is converted to a scaled score that is reported to you and to the colleges you specify. Worksheet 1. Finding Your Raw Test Score STEP 1: Table A lists the correct answers for all the questions on the Subject Test in Mathematics Level 2 that is reproduced in this book. It also serves as a worksheet for you to calculate your raw score. + Compare your answers with those given in the table. + Puta check in the column marked “Right” if your answer is correct. «+ Puta check in the column marked “Wrong” if your answer is incorrect. + Leave both columns blank if you omitted the question. STEP 2: Count the number of right answers. Enter the total here: STEP 3: Count the number of wrong answers. Enter the total here: STEP 4: Multiply the number of wrong answers by .250. Enter the product here: STEP 5: Subtract the result obtained in Step 4 from the total you obtained in Step 2. Enter the result here; _____ STEP 6: Round the number obtained in Step 5 to the nearest whole number. Enter the result here: The number you obtained in Step 6 is your raw score. > 208 Table A Answers to the Subject Test in Mathematics Level 2, Form 3YBC, and Percentage of Students Answering Each Question Correctly Peery Perens enon erie Crete Ciegetretsna ceteris Number | Answer | Right 88 6 D 85 c 31 a B 0 0 0 28 c 65 A 7 Fz) E a c 0 30 D B E 54 a c 54 c 2 2 D n ° 2 3 D 2 0 85 34 c 6 10 8 89 6 8 5r 1 E cy 36 D 51 2 c 54 a E 63 8 8 a 38 c 52 4 o % 39 c 52 5 A 88 40 o 48 16 a a a a 2 " 8 62 2 B 3 8 A 70 4B c 63 18 ° 6 4 B 54 20 o n 45 A 46 2 c 2 16 B 6 2 c a a E “4 2B c 10 48 A 35 a 8 66 49 D 2B 3 E 60 50 A 26 * These percentages are based on an analysis ofthe answer shects ofa representative sample of 15,855 students ‘who took the original form of this test in May 2002, and whose mean score was 652. They may be used as an indication ofthe relative difficulty of a particular question. Each percentage may also be used to predict the likelihood that a typical SAT Subject Test in Mathematics Level 2 candidate will answer that question correctly ‘on this edition of the test. ‘The Official Study Guide for All SAT Subject Tests Finding Your Scaled Score When you take SAT Subject Tests, the scores sent to the colleges you specify are reported on the College Board scale, which ranges from 200-800. You can convert your practice test score to a scaled score by using Table B, To find your scaled score, locate your raw score in the left-hand column of Table B; the corresponding score in the right-hand column is your scaled score, For example, a raw score of 26 on this particular edition of the Subject Test in Mathematics Level 2 corresponds to a scaled score of 620. Raw scores are converted to scaled scores to ensure that a score earned on any one edition of a particular Subject Test is comparable to the same scaled score earned on any other edition of the same Subject Test. Because some editions of the tests may be slightly easier or more difficult than others, College Board scaled scores are adjusted so that they indicate the same level of performance regardless of the edition of the test taken and the ability of the group that takes it. Thus, for example, a score of 400 on one edition of a test taken ata particular administration indicates the same level of achievement as a score of 400 on a different edition of the test taken at a different administration, When you take the SAT Subject Tests during a national administration, your scores are likely to differ somewhat from the scores you obtain on the tests in this book. People perform at different levels at different times for reasons unrelated to the tests themselves. ‘The precision of any test is also limited because it represents only a sample of all the possible questions that could be asked. D210 Mathematics Table B Scaled Score Conversion Table Subject Test in Mathematics Level 2 (Form 3YBC) 50 800 8 630 6 470 4g 800 a 630 5 460 48 00 %6 620 4 450 a7 800 B 610 a 440 46 00 mo 600 2 430 6 800 B 600 1 420 44 800 2 590 0 410 43 790 a 580 4 400 a2 780 0 580 2 390 u 770 19 570 3 370 0 760 18 560 4 360 39 750 7 560 4 360 38 740 16 550 4 340 37 730 15 540 7 340 36 710 4 530 4 330 3 700 13 530 9 330 4 690 12 520 40 320 33 680 " 510 i 310 32 670 10 500 12 300 31 660 8 490 30 650 8 490 2 640 1 480 amp ‘The Official Study Guide for All SAT Subject Tests How Did You Do on the Subject Test in Mathematics Level 2? After you score your test and analyze your performance, think about the following question Did you run out of time before reaching the end of the test? If'so, you may need to pace yourself better. For example, maybe you spent too much time on one or two hard questions. A better approach might be to skip the ones you can’t answer right away and try answering all the questions that remain on the test. Then if there’s time, go back to the questions you skipped. Did you take a long time reading the directions? You will save time when you take the test by learning the directions to the Subject Test in Mathematics Level 2 ahead of time. Each minute you spend reading directions during the test is a minute that you could use to answer questions. How did you handle questions you were unsure of? If you were able to eliminate one or more of the answer choices as wrong and guess from the remaining ones, your approach probably worked to your advantage. On the other hand, making haphazard guesses or omitting questions without trying to eliminate choices could cost you valuable points. How difficult were the questions for you compared with other students who took the test? Table A shows you how difficult the multiple-choice questions were for the group of students who took this test during its national administration. The right-hand column. gives the percentage of students that answered each question correctly. A question answered correctly by almost everyone in the group is obviously an easier question, For example, 93 percent of the students answered question 8 correctly, But only 23 percent answered question 33 correctly. Keep in mind that these percentages are based on just one group of students. They would probably be different with another group of students taking the test. If you missed several easier questions, go back and try to find out why: Did the questions cover material you haven't yet reviewed? Did you misunderstand the directions? D212 Chapter 5 Biology E/M Purpose ‘The Subject Test in Biology E/M measures the knowledge students would be expected to have after successfully completing a college-preparatory course in high school. The test is designed to be independent of whichever textbook you used or the instructional approach of the biology course you have taken, The Biology E/M Test is for students taking a biology course that has placed particular emphasis on either ecological or molecular biology, with the understanding that evolution is inherent in both. The test lets you choose the area in biology for which you feel best prepared. If you are unsure of the emphasis in your biology course, consult your teacher. Format ‘The Subject Test in Biology E/M with either ecological (Biology-E) or molecular (Biology- M) emphasis has a common core of 60 questions, followed by 20 questions in each specialized section (Biology-E or Biology-M). Each test-taker answers 80 questions. Content ‘The content covered in the Subject Test in Biology E/M and descriptions of the topics are shown in the chart on page 214, 2130 ‘The Official Study Guide for All SAT Subject Tests Biology E/M Test Topics organisms (with emphasis on plants and animals), animal behavior selection, speciation, patterns of evolution, classification and diversity of organisms Approximate Percentage of E Test ‘Approximate Percentage of M Test How to Prepare Before you take the Biology E/M Test, you should have completed a one-year course not only in biology but also in algebra so that you can understand simple algebraic concepts (including ratios and direct and inverse proportions) and apply such concepts to solving word problems. Success in high school biology courses typically requires good reasoning, and mathematical skills. Your preparation in biology should have enabled you to develop these and other skills that are important to the study of biology. Familiarize yourself with directions in advance. The directions in this book are identical to those that appear on the test. D2 Biology-E and Biology-M Approximate Skills Specifications Percentage of Test remembering specific facts, demonstrating straightforward knowledge of information and familiarity with terminology ee understanding concepts and refarmulating information into other equivalent forms; applying knowledge to unfamiliar and/or practical situations, solving problems using mathematical relationships ee inferring and deducing from qualitative and quantitative data and integrating information to form conclusions; recognizing unstated assumptions You should be able to recall and understand the major concepts of biology and to apply the principles you have learned to solve specific problems in biology. You should also be able to organize and interpret results obtained by observation and experimentation and to draw conclusions or make inferences from experimental data, including data presented in graphic and/or tabular form. Laboratory experience is a significant factor in developing reasoning and problem-solving skills. Although testing of laboratory skills in a multiple-choice test is necessarily limited, reasonable experience in the laboratory will help you prepare for the test. Notes: (1) You will not be allowed to use a calculator during the Biology E/M Test. (2) Numerical calculations are limited to simple arithmetic. 3) The metric system is used in these tests. How to Choose Biology-E or Biology-M + Take Biology-E if you feel more comfortable answering questions pertaining to biological communities, populations, and energy flow. + Take Biology-M if you feel more comfortable answering questions pertaining to biochemistry, cellular structure and processes, such as respiration and photosynthesis. + Indicate choi of Biology-E or Biology-M on your answer sheet on test day. You can decide whether you want to take Biology-E or Biology-M on the test day by gridding the appropriate code for the test you have chosen on your answer sheet. Only questions pertaining to the test code that is gridded on your answer sheet will be scored. 25) ‘The Official Study Guide for All SAT Subject Tests Note: Because there isa common core of questions, you are not allowed to take Biology-E and Biology-M on the same test date. You can take them on two different test dates. Score ‘The total score for each test reported on the 200-to-800 scale. Sample Questions Classification Questions Each set of classification questions has five lettered choices in the heading that are used. in answering all of the questions in the set. The choices may be statements that refer to concepts, principles, organisms, substances, or observable phenomena; or they may be graphs, pictures, equations, formulas, or experimental settings or situations. Because the same five choices are applicable to several questions, classification questions usually require less reading than other types of multiple-choice questions. Answering a question correctly depends largely on the sophistication of the set of questions. One set may test recall; another may ask you to apply your knowledge to a specific situation or to translate information from one form to another (descriptive, graphical, mathematical). ‘The directions for this type of question specifically state that you should not eliminate a choice simply because it is the correct answer to a previous question, ‘The following are directions for and an example of a classification set. Core Section of Biology E/M Directions: Each set of lettered choices below refers to the numbered statements immediately following it, Select the one lettered choice that best fits each statement and. then fill in the corresponding circle on the answer sheet. A choice may be used once, more than once, or not at all in each set. Questions 1-3 refer to the following pairs of organisms. (A) Monerans and protists (B)_Angiosperms and gymnosperms (© Algae and fungi (D)_ Ferns and mosses (6) Monocots and dicots 1. Distinguished from each other by the presence or absence of a nuclear envelope 2. Distinguished from each other by the presence or absence of flowers D216 Biology 3. Distinguished from each other by the presence or absence of vascular tissue ‘The questions in this group are based on biological diversity and refer, in particular, to identification of distinguishing characteristics among certain groups of organisms that have arisen during evolutionary history. Choice (A) is the correct answer to question 1. This question asks you to recognize that the absence of a nuclear envelope in cells separates prokaryotic cells from all other cells. that do have a nuclear envelope, namely the eukaryotes. This characteristic is significant enough to place prokaryotes (including bacteria) in a separate taxonomic group. Choice (B) is the correct answer to question 2. This question asks you to recognize that the presence or absence of flowers depends on whether seed plants produce seeds that are not enclosed in specialized structures or whether they are contained in specialized complex reproductive structures called ovaries. The former are called gymnosperms and appeared about 200 million years before the emergence of the flowering plants or angiosperms. Choice (D) is the correct answer to question 3. This question is based on the recognition that the development of vascular tissue (phloem and xylem) was a major adaptation in the long evolution of photosynthetic organisms. Mosses were among the first autotrophs to display evolutionary adaptations to land existence, but they are usually less than 20 centimeters tall because they lack the woody tissue required to support tall plants on land, The evolutionary development of vascular tissue made possible the transporting of water and minerals and food between leaves and roots. Ferns are examples of vascular plants. Five-Choice Questions The five-choice question is written either as an incomplete statement or as a question. It is appropriate when: (1) the problem presented is clearly delineated by the wording of the question so that you are asked to choose not a universal solution but the best of the solutions offered; (2) the problem is such that you are required to evaluate the relevance of five plausible, or even scientifically accurate, options and to select the one most pertinent; (3) the problem has several pertinent solutions and you are required to select the one inappropriate solution that is presented, Such questions normally contain a word in capital letters such as NOT, LEAST, or EXCEPT. A special type of five-choice question is used in some tests, including the SAT Subject ‘Test in Biology E/M, to allow for the possibility of multiple correct answers. For these questions, you must evaluate each response independently of the others in order to select the most appropriate combination. In questions of this type several (usually three or four) statements labeled by Roman numerals are given with the question. One or more of these statements may correctly answer the question. You must select from among the 270 ‘The Official Study Guide for All SAT Subject Tests five lettered choices that follow the one combination of statements that best answers the question. In the test, questions of this type are mixed in with the more standard five- choice questions. (Question 5 is an example of this type of question.) In five-choice questions, you may be asked to convert the information given in a word problem into graphical form or to select and apply the mathematical relationship necessary to solve the scientific problem. Alternatively, you may be asked to interpret experimental data, graphical stimulus, or mathematical expressions. When the experimental data or other scientific problems to be analyzed are comparatively extensive, itis often convenient to organize several five-choice questions into sets, that is, to direct each question in a set to the same material, This practice allows you to answer several questions based on the same material. In no case, however, is the answer to one question necessary for answering a subsequent question correctly. Each question in a set is independent of the others but refers to the same material given for the entire set, ‘The following are directions for and examples of five-choice questions. Directions: Each of the questions or incomplete statements below is followed by five suggested answers or completions. Some questions pertain to a set that refers to a laboratory or experimental situation. For each question, select the one choice that is the best answer to the question and then fill in the corresponding circle on the answer sheet. 4, All of the following are population characteristics EXCEPT (A) number of i (B) phenotype (© sex ratio (D) age distribution (©) death rate duals Choice (B) is the correct answer to question 4. This question is a question on population ecology and asks you to consider what constitutes a population. An investigator necessarily has to define the limits of the population, but once those parameters are set, itis possible to study the variations in the time and space in the size and density of the population thus defined. A population can be characterized by the number of individuals present, the age distribution, the death rate within the population, and the sex ratio among the ividuals. However, the phenotype is a characteristic of an organism and is observed at the level of the individual rather than at the level of a population. Daze 5. ATP is produced during which of the following processes? 1. Photosynthesis II. Aerobic respiration IIL, Fermentation (A) only (®) Holy (©) Land Il only (D) [and III only (2) LILand til Choice (E) is the correct answer to question 5, This is a question on cellular and molecular biology that asks you to consider whether ATP is produced by more than one metabolic pathway. Each of the processes designated by a Roman numeral must be evaluated independently. In photosynthesis, solar energy captured by chlorophyll-containing plants creates a flow of electrons that results in the synthesis of ATP. Thus I is correct. Aerobic respiration, the process by which glucose is broken down to CO, and H,O in the presence of O,, is the most efficient mechanism by which cells produce the ATP they need to carry on their other metabolic activities. Thus I] is also correct. Fermentation also involves the breakdown of glucose but without ©,. Under these conditions, substances such as lactic acid or ethyl alcohol and CO, are produced, together with limited quantities of ATP. Although the carbon-containing end products of fermentation still have much of the energy contained in the original glucose, fermentation permits a cell to produce some ATP under anaerobic conditions. Questions 6-8 In a breeding experiment using gray and white mice of unknown genotypes, the following results were obtained. ‘Parents Offspring 1 Gray x White 82 78 \ Gay K ce we % 1 wi xk vii 0 0 WY seay k wit 1 ° 6. Heterozygous gray female parents occur in (A) cross I only (B) cross II only (© cross TV only (D) crosses I and II only (E) crosses II and IV only 29) ‘The Official Study Guide for All SAT Subject Tests 7. If two gray progeny of cross IV mate with each other, what is the probability that any individual offspring will be gray? (A) 100% (B) 75% (©) 50% (D) 25% ©) 0% 8, If the gray female from cross IV were mated with the gray male from cross IT, then which of the following would most likely be true? (A) _Allof the offspring would be gray. (B)Allof the offspring would be white. (©) Half of the offspring would be gray. (D) One-quarter of the offspring would be gray. (E) One-quarter of the offspring would be white. Questions 6-8 are on heredity. They refer to the experiment described in the introductory material. You are asked to draw conclusions from the results of the experiment and to predict the results of further experimentation on the basis of the information obtained. Choice (D) is the correct answer to question 6. This question asks you to determine which gray female parents were heterozygous. First you must realize from the ratio of offspring obtained in all the crosses that gray coat color is dominant over white in these mice. Next, you should note that no white offspring were obtained in cross IV. Thus, the gray female in this cross was homozygous gray. In cross I, approximately 50 percent of the offspring were gray. Therefore, the gray female, mated with a white male, must have been heterozygous. In cross II, a gray female was mated with a gray male, and a 3:1 ratio of gray to white offspring was obtained. Therefore, both gray female and gray male parents were heterozygous. Thus heterozygous females occurred only in crosses I and II. Choice (B) is the correct answer to question 7. This question proposes a hypothetical mating between two gray progeny of cross IV. Since these progeny resulted from a cross between a gray female and a white male and no white offspring were produced, you can conclude that the female parent was homozygous gray and that all the offspring are heterozygous gray. Therefore, the mating of the gray progeny of cross IV will produce offspring in the ratio of 3 gray to 1 white. The probability, therefore, of an offspring of this cross being gray is 75 percent. Choice (A) is the correct answer to question 8. This question asks you to predict the results of a cross between the gray female from cross IV and the gray male from cross II. Erom the data given, you can determine that the gray female in cross IV is homozygous, and the male in cross II is heterozygous. Thus you could expect that all of the offspring from such a mating would be gray. »220 Biology Questions 9-11 ‘Three students added equal volumes of pond water to each of four beakers (I-IV) and placed each in a different constant-temperature bath. ‘The baths were maintained at 5°C, 15°C, 25°C, and 35°C, respectively. The students then added 6 water fleas, Daphnia pulez, to each of the four beakers. After | hour, the students removed 3 Daphnia pulex from each beaker and each student immediately observed one Daphnia pulex under low-power magnification of a light microscope. (The transparent body of the Daphnia pulex can be seen easily under a light microscope.) Heart rates were recorded as beats per minute. The results of the experiment are summarized below. Heartbeat Time per Minute Time Daphnia (average of 3 Beaker ‘Temperature | Daphnia Added | Removed Daphnia) I 5c 2.00 pm. 3.00 pm 41 1 15°C 2:10 pm 3:10 p.m 119 ti 2c 220m 320 pm. 202 W 35°C 230 pm, 3:30 pam 281 9. ‘The independent variable in this experiment is the (A) amount of light (B) number of water fleas (©) pH of the water (D) temperature of the water (B) average heart rate 2p ‘The Official Study Guide for All SAT Subject Tests 10. Ifa graph is constructed using the data given in the table, it will most closely resemble which of the following? o Teper "Taopertue 11, The data obtained in this experiment lend support to which of the following, hypotheses? (A) At 45°C the heart rate of Daphnia would be 320 beats/minute. (B) Daphnia swim more slowly at high temperature. (©) Metabolic rate in Daphnia is directly proportional to water temperature. (D) Heart rate in Daphnia is inversely proportional to water temperature. (E) Between 0°C and 5°C, the heart rate of Daphnia would remain constant. Questions 9-11 describe an experiment that seeks to determine how the metabolism of water fleas is affected by temperature. The experimental setup states that equal volumes of pond water were added to each of four beakers and the same number of fleas were added to each beaker. Choice (D) is the correct answer to question 9. In this question, choices (B) and (C) are incorrect because both remained constant. Choice (A) is irrelevant in the case of water fleas, and choice (E) is the result the experiment seeks to measure. The only variable that changed during the course of the experiment was the temperature. Choice (B) is the correct answer to question 10. This question requires examination of the data in the table, The results show that the average heartbeat per minute of these water fleas increased by about 80 heartbeats per every 10°C increase in temperature. ‘This represents a linear increase of heartbeat with temperature, and only one of the five graphs given shows this. Choice (C) is the correct answer to question 11. This question asks students to evaluate which of the five choices given is a hypothesis that is supported by the data. Choice (A) is incorrect because, although there are no data for the heart rate of the fleas at 45°C, a reasonable inference would be that the heart rate should increase about 80 heartbeats D222 Biology above that at 35°C, to about 360. Choice (B) is not a reasonable hypothesis since the water fleas are likely to move more rapidly at high temperatures when the heart rate is higher. Choice (D) is incorrect because it directly contradicts the data, and choice (E) is also incorrect because there is no reason to infer from the data, that the heart rate would remain constant at lower temperatures. However, since heart rate increases linearly with temperature, data support the hypothesis that metabolic rate is also directly proportional to heart rate. Biology-E Section 12. Which of the following individuals is most fit in evolutionary terms? (A) A child who does not become infected with any of the usual childhood diseases, such as measles or chicken pox A woman of 40 with seven adult offspring, ®) (© A woman of 80 who has one adult offspring (D) A 10-year-old man with no offspring. (6) A childless man who can run a mile in less than five minutes Choice (B) is the correct answer to question 12. For this question you must know the premises upon which Darwin based his explanation of evolutionary change in terms of natural selection, To be fit in evolutionary terms means not only that organisms possessing favorable variations will be able to survive better than those with less favorable variations, but also that the most fit organism will have a higher ability to leave more viable offspring in the next generation. Thus a child who is resistant to certain diseases has not yet demonstrated fitness. Therefore, choice (A) is not the answer to this question. Similarly a person with no offspring has not demonstrated fitness, whether or not she or he is actively exercising. Thus, choices (D) and (E) are incorrect. Finally a woman with numerous surviving offspring is more fit in the evolutionary sense than a woman with one surviving offspring, regardless of the life span of the woman, Thus choice (B) is a better answer than choice (C) 223 ‘The Official Study Guide for All SAT Subject Tests Questions 13-15 Known numbers of seeds from two species (X and Y) of annual plants are mixed together in different proportions and planted in five small plots of soil in the spring, The plants grow, flower, and produce seeds. It is found that the percentage of seeds of species X and species Y in the harvest is usually different from the proportion that was planted, although the total number of seeds produced is the same as the number of seeds planted. ‘The data are plotted on the graph below. z Z 100% 2 sp.xX = 75% 3 px =F 3B S 50% % sp.X 5 25% E sp.X 2 g Ke E& sp.X 0% — 25% 50% 75% 100% é sp.X sp.X sp.X sp.X sp.X Input: Percentage of Seeds Planted 13, What mixture of seeds was harvested in the plot that was planted with 25 percent species X and 75 percent species Y? x Y (A) 25% 75% (B) 40% 60% ©) 50% — 50% (D) 60% 40% (B) 75% 25% 14, What do the data indicate about the ecological relationship between species X and species Y? (A) They are mutualistic for low percentages of X seeds. (B) They are mutualistic for high percentages of X seeds. (C) Xand ¥ compete when both X and Y seeds are present. (D) ¥ competes successfully against X at all percentages of X and Y seeds. (E) Xs parasite of Y when Y is rare. D224 Biology 15, If you started out with 25 percent species X seeds and 75 percent species Y seeds and replanted a plot year after year with the seeds produced each autumn, what pattern would you expect to see in the mixture of the two species over the years of the experiment? (A) Species X would increase to 100% while species Y would decrease to 0%. (B) Species ¥ would increase to 100% while species X would decrease to 0%. (©) One of the species would increase to 100% but which one depends on the initial mixture used to start the experiment. (D)_ The mixture of seeds would eventually stabilize at 75% X and 25% Y. (E) None of the patterns above is consistent with the data. Questions 13-15 test the ecological concept of competition among species. The graph presented with the introductory material shows that the survival of either of two species depends on the relative abundance of each species at the time of seed planting. Choice (C) is the correct answer to question 13. This question is a straightforward graph- reading question: when 25% of the seeds planted were species X and therefore 75% were species ¥, the graph shows that 50% of the seeds harvested were species X and thus 50% were species ¥. Choice (©) is the correct answer to question 14. This question asks you to draw a conclusion about the ecological relationship described in this experiment. Ina mutualistic relationship, both species benefit but the data do not show that this is true at either low or high percentages of species X. Thus choices (A) and (B) can be eliminated, There is no evidence for parasitism so choice (B) can be eliminated. Competition is occurring but not in a manner such that one species is successful over the other no matter what the percentage of seeds of each species planted. Thus choice (D) is incorrect. Choice (D) is the correct answer to question 15. In this question, you are asked to predict the results of a proposed experiment. If you started by planting 25% species X seeds and 75% species Y seeds, you would recover 50% species X seeds and 50% species Y seeds at harvest time. If these seeds were replanted the following year, the graph shows about 70% species X and 30% species Y seeds would be harvested. Replanting these results year after year would increase the percentage of species X seeds harvested to 75% but there will be no further change in percentage of seeds planted and harvested. The percentage of seeds of each species would stabilize when 75% species X and 25% species Y are planted. Neither species would reach either 100% or 0% given the percentage of seeds from each species originally planted. Thus choices (A), (B), and (C) can be eliminated, Choice (E) can be eliminated because the pattern in choice (D) is consistent with the data, 225 ‘The Official Study Guide for All SAT Subject Tests Biology-M Section 16, Which of the following most accurately reveals common ancestry among many different species of organisms? (A) The amino acid sequence of their cytochrome C (B) Their ability to synthesize hemoglobin (©) The percentage of their body weight that is fat (D)_ The percentage of their body surface that is used in gas exchange ()_ The mechanism of their mode of locomotion Choice (A) is the correct answer to question 16. ‘To assess common ancestry, or evolutionary relationship, among organisms, it is necessary to examine the similarities and differences among species for one or more structures that are homologous. For homologous structures—whether complex structures such as limbs or less complex structure such as a single gene product—the differences arise through the accumulation. of mutations over time. Great similarity reflects a shorter time of divergence from a ‘common ancestor. By this reasoning, only choice (A), examination of an enzyme of identical function in various organisms, represents a comparison of a homologous structure. Choice (B) can be ruled out because organisms either possess or lack the ability to synthesize hemoglobin; thus, this character allows one to sort organisms only into two groups, without providing information on relationships within those groups. Choice (C) can be ruled out because the amount of body fat is controlled physiologically, and varies within a single species. Choice (D) can be ruled out because gas exchange does not occur through the surface of some organisms that are only distantly related (eg., mammals vs. insects) or occurs through the entire surface of many organisms that vary tremendously in relationship (eg, all unicellular organisms). Choice (E) can be ruled out because many unrelated organisms do not move at all (eg., plants vs. fungi), or derived their mode of locomotion independently (eg, bats vs. birds vs. flying insects). 2226 Biology Questions 17-19 Thymine is used by animal cells primarily for the synthesis of DNA. A group of sea urchin eggs was fertilized in sea water containing radioactive thymine, Following fertilization samples of embryos were removed at regular intervals and the radioactivity in the embryos’ nucleic acid was measured in counts per minute. The results obtained are shown in the figure below. 400 300 200 100 Radioactive Counts (thousands per minute) 0 50 100 150 200 250 300 ‘Time After Fertilization (minutes) 17, The increase in radioactivity of the embryos with time probably results from (A) _ synthesis of new proteins by the developing embryos (B) synthesis of radioactive thymine by the developing embryos (©) oxidation of radioactive thymine (D) incorporation of radioactive thymine in new cell membranes (E) incorporation of radioactive thymine in new DNA during replication 18. The time required for a complete cell division cycle in the sea urchin embryos studied in the experiment is approximately (A) 25 minutes (B) 50 minutes (© 75 minutes (D) 100 minutes (E) 200 minutes 227 ‘The Official Study Guide for All SAT Subject Tests 19. An appropriate control to show that this experiment measures DNA synthesis and not RNA synthesis would be to perform the same procedures but (A) _ not fertilize the eggs (B) sample the embryos at longer time intervals (©) add radioactive uracil instead of radioactive thymine (D) fertilize the eggs in sea water that does not contain radioactive thymine (&) count the number of cells in the embryos at the beginning and at the end of the experiment Questions 17-19 describe an experiment that asks you to recognize that cell division occurs rapidly after fertilization and that DNA is synthesized when cells replicate. The introductory material tells you that animal cells use thymine primarily for DNA synthesis (thymine is one of the four bases contained in DNA). Choice (E) is the correct answer to question 17. To answer this question, you need to realize that the use of radioactive thymine in the experimental design is needed as a ‘means of measuring its uptake in the embryos, Radioactive thymine is not incorporated in embryonic cell membranes nor is it oxidized or synthesized by the embryos. Thus choices (B), (C), and (D) are all incorrect. The developing embryos do synthesize proteins but choice (A) is incorrect because thymine is not incorporated into proteins. Choice (B) is the correct answer to question 18. To answer this question, you need to know that DNA is synthesized most rapidly during replication so that the largest increase in uptake of radioactive thymine would occur at that time, The graph indicates that the steepest jumps in radioactivity occur every 50 minutes so that would represent a complete cell division cycle. Choice (C) is the correct answer to question 19. To answer this question, you need to know that uracil is contained in RNA but thymine is not, Thus using radioactive thymine measures DNA synthesis and not RNA synthesis. Use of radioactive uracil would yield data that measures RNA synthesis. Without the use of radioactive thymine in the sea water, the experimenter would have no mechanism for measuring any DNA synthesis at all, so choice (D) would not be an appropriate control. Collecting samples either at longer time intervals or only at the start and end of the experiment would provide less data and could not provide any appropriate control. Thus choices (B) and (B) are incorrect. Not fertilizing the eggs would provide no cell division whatsoever and so would provide no supporting evidence to show that the experiment measures DNA synthesis and not RNA synthesis. Thus choice (A) is also incorrect. D228 Biology E/M Test Practice Helps The test that follows is an actual, recently administered SAT Subject Test in Biology E/M, To get an idea of what it's like to take this test, practice under conditions that are much like those of an actual test administration Set aside an hour when you can take the test uninterrupted. Make sure you complete the test in one sitting. Sit at a desk or table with no other books or papers. Dictionaries, other books, or notes are not allowed in the test room. Tear out an answer sheet from the back of this book and fill it in just as you would on the day of the test. One answer sheet can be used for up to three Subject Tests. Read the instructions that precede the practice test. During the actual administration you will be asked to read them before answering test questions. Time yourself by placing a clock or kitchen timer in front of you. After you finish the practice test, read the sections “How to Score the SAT Subject Test in Ecological Biology” or “How to Score the SAT Subject Test in Molecular Biology” and “How Did You Do on the Subject Test in Ecological Biology?” or “How Did You Do on the Subject Test in Molecular Biology?” The appearance of the answer sheet in this book may differ from the answer sheet you see on test day. 229 ‘You MUST decide now whether you want to take a Biology Test with Ecological Emphasis (BIOLOGY-| BIOLOGY-1 TEST or BIOLOGY-M TEST Molecular Emphasis (BIOLOGY-M). The top portion of the section of the answer sheet that you will use in taking the Biology Test you have selected must be filled in exactly as shown in one of the illustrations below. Note carefully that you have to do all of the following on your answer sheet. Print BIOLOGY-E or BIOLOGY-M on the line under the words “Subject Test (print).” In the shaded box labeled “Test Code” fill in four circles as follows: Por BIOLOGY-E For BIOLOGY-M — Fill in cirele 1 in the row labeled V. — Fill in circle 5 in the row labeled V. — Fill in cirele 9 in the row labeled W. —Fill in cirele 7 in the row labeled W. IL in cirele 4 in the row labeled X. — I in cirele B in the row labeled Y. _ ill in circle 5 in the row labeled X. ill in circle C in the row labeled Y. Tent age Tent aae ¥ @2990000090 ¥ O9990@0900 w DODO9OO008 ™ ODOOVODOeDO 2009000 +6 9 KOODOS WOODO 2 ODO9O O00 9 [EO MOMOMOMOMOMOMOMO} ‘Subject Tat prin) ‘Subject Test (pri) BIOLOGY - E BIOLOGY -M 3 Question | Question It Question IIT Question IV Please answer the questions below by fi answer sheet. The information you score on the test. in the appropriate circles in the row labeled Q on the ovide is for statistical purposes only and will not affect your How many semesters of biology have you taken in high school? (If you are taking biology this semester, count it as a full semester.) Fill in only o1 © One semester or less ‘© Two semesters ‘© Three semesters or more © General Biology © Biology with emphasis on ecology * Biology with emp! sis on molecular biology Which of the following best describes your background in algebra? (If you are taking an algebra course this semester, count it as a full semester.) Fill in only one circle of citeles 7-8, © One ‘© Two semesters or more — mester oF less _ Are you currently (aking Advanced Placement Biology? If you ‘When the supervisor gives the signal, turn the page and begin the Biology Test. There are 100 numbered circles oon the answer sheet. There are 60 questions in the core Biology Test, 20 questions in the Biology-E sectio 20 questions in the Biology-M section. Therefore use ONLY ci ind s 1-80 (for Biology-B) OR cireles 1-60 plus 81-100 (for Biology-M) for recording your answers. [Unwaried coping ruse ot sry br of this peo onal »230 © BIOLOGY E/M TEST FOR BOTH BIOLOGY-| AND BIOLOGY-M, ANSWER QUESTIONS 1-60 Directions: Each set of lettered choices below refers to the numbered qu elect the one lettered choice that best answers each question or best fits each statement and then fill in the corre- it jons or staten (3 immediately following sponding circle on the answer sheet. A choice may be used once, more than once, or not at all in each set Questions 1-4 refer to the following plant cell types. (A) Tracheids and vessel elements, (B) Guard cells (C) Parenchyma cells, (D) Sieve tube members and companion cells (E) Sclerenchyma cells 1, Chains of these nonliving cells form continuous. tubes for the transport of water in vascular plants. ‘These cells take up potassium ions and water when sunlight and low concentrations of carbon dioxide are present, which causes them to become rigid. 3. These versatile cells serve as storage sites for sugars and starches in stems and roots. 4. These cells form a living tissue which transports sugar from one part of a vascular plant to another. Questions 5-6 refer to the following. Ay 2 @ 4 © 16 (D) 25 ©) 50 5. The expected percentage of offspring with the recessive phenotype from a cross between two individuals heterozygous for a particular trait 6. The number of different phenotypes possible for the progeny of the cross AaBb x AaBh, where A and B exhibit simple dominance 3YAC Questions 7-10 (A) Monera (B) Protista (©) Fungi (D) Plantae (E) Animalia ontail all the protozoa and most of the algae 8. Contains multicellular heterotrophic organisms that reproduce asexually by spores 9. Contains organisms without membrane-bound, organelles such as nuclei 10. Contains autotrophie organisms with cells that are organized into tissues and organs (GO ONTO THE NEXT PAGE 2310 BIOLOGY-E/M TEST—Continued Questions 11-14 refer to the following illustration of protein synthesis in a mammalian cell. 1. A strand of mRNA being translated 12. A polypeptide being synthesized 13. A barrier to diffusion of large proteins from nucleus to cytoplasm 14, A structure that contains a lipid bilayer Travia Sopng ros GO ONTO THE NEXT PAGE > ‘ny pe ofthis poge sega D232 BIOLOGY-E/M TEST—Continued Questions 15-17 15, Itis secreted by the pituitary gland, (A) Insulin 16, It directly controls metabolic rate. (B) Growth hormone (C) Progesterone 17. ts concentration in the blood rises when the (D) Thyroxin corpus Iuteum develops. (B) Seeretin (GO ONTO THE NEXT PAGE 233) ® BIOLOGY-E/M TEST—Continued Directions: Each of the questions or incomplete statements below is followed by five suggested answers or completions. Some question, select the one cho ior the answer sheet. 18. 19. ‘The ribosomes of a cell are of primary importance for (A) DNA replication (B) transcription (©) translation (D) translocation (E) repression Ifa couple has two boys and one girl, what is the probability that the next child born to this couple will be a girl? a @) ee ae © wo) «e) 3 3 a pertain to a sot that refers to a laboratory of experimental si ~e that is the best answer to the question and then fill in the corresponding circle on jon. For each, 20. Eggs fertilized by two sperm instead of one some~ 2 times form a mitotic spindle with three poles, After mitosis the daughter cells will probably (A) be indistinguishable from normal cells (B) eliminate the chromosomes contributed by the second sperm (C) eliminate the chromosomes contributed by the egg (D) display an abnormal number of chromosomes (E) stop protein synthesis immediately In higher plant cells, a pigment important in the ‘manufacture of carbohydrates from CO, and H,0 is contained in the (A) nucleus (B) vacuole (©) cytoplasm (D) chloroplast (E) centrosome (GO ONTO THE NEXT PAGE > BIOLOGY-E/M TEST—Continued ® 23. Which of the following statements is true for red Ay blood cells that have been added to a flask of x 1 saturated NaCI solution?” (A) The cells will undergo mitosis, (B) The cells will increase in volume. (C) The cells will lose water. (D) The cells are hypertonic relative to the surrounding medium, (E) The concentration of NaCl is lower outside the cells than inside. 24. Today's worldwide human population can best be described as (A) oscillating (B) declining (C) fluctuating near equilibrium (D) growing arithmetically (B) growing exponentially 25. The base of the food web of the open ocean is, provided by (A) phytoplankton (B) zooplankton (C) kelp (D) fish (©) whales 26. Nitrogen fixation is the conversion of atmospheric ygen into 2. In the diagram of the human skeleton above, Which ofthe following isa ball-and-socket joint? (A) ammonia (B) protein (ay (©) urea (B) 2 (D) carbon dioxide «3 (BE) DNA we 5 Unauthorized opving ot outs oi GO ONTO THE NEXT PAGE [an pat of tis poe ege 2350 27. In the fruit fly, the allele for normal wings (W) is dominant over the allele for vestigial wings (w). A cross of two normal-winged flies produced 76 normal-winged and 23 vestigial-winged off- spring. Tt can be concluded that the genotypes of the two parent flies were which of the following? (A) WW and vow (B) WW and Wow (©) Ww and ww (D) Www and Wwe (&) WW and WW 28, Factors that have been known to result in the nation of a species in a particular area include which of the following? L. Use of insecticides. IL. Hunting of the species’ prey UL Habitat destruction (A) Lonly (B) ILonly (© Land Ht only (D) and IH only () I, and ut BIOLOGY-E/M TEST—Continued © S43 ne 08 29. According to the partial Karyotype of a mammal shown above, which of the following must be cue? (A) The organism has a single gene defect. (B) The organism is a male. (C) The organism is a homozygote. (D) The organism is a human: (E) The alleles on both chromosomes labeled 3 are identical. 30. An organism is examined and is found to be multicellular and heterotrophic and to have cell walls made of a substance other than cellulo: ‘The organism belongs to which of the following kingdoms? (A) Monera (B) Protista (©) Fungi (D) Plantae (B) Animalia (GO ONTO THE NEXT PAGE > 31 33 34. Which of the following statements is correct? (A) Heritable variation allows for evolution. (B) Adaptive radiation allows for mutation. (C) Crossing-over allows for mitosis. (D) Translocation allows for DNA replication, (E) Cellular differentiation allows for meiosis. Behavior that remains unaffected by environmental changes is most likely (A) territorial (B) learned (©) innate (D) stereotyped (®) conditioned Aman who has hemophilia and a woman who does. not have hemophilia have a daughter who has hemophilia. Hemophilia is a recessive condition, and the gene is located on the X chromosome. Which of the following ean be concluded? (A) The mother is a cartier for hemophilia (B) Hemophilia is not a sex-linked trait (C) Crossing-over has occurred. (D) All subsequent daughters of this couple will have hemophilia (E) All sons of this couple will have hemophilia All of the following are measures useful in describing a given population's growth rate EXCEPT (A) fertility (B) mortality (C) survivorship (D) age structure (E) habitat Unauthorized opving ot outs oi [an pat of tis poe ege BIOLOGY-E/M TEST—Continued 35, 36. 37. ® Which of the following is LEAST consistent with the fossil record?” (A) Bony fish evolved from amphibians. (B) Mammals evolved from reptiles. (C) Birds evolved from reptiles (D) Reptiles evolved from amphibians. (E) Cartilaginous fish evolved from jawless fish. Which of the following is NOT true of enzymes?” (A) Enzyme activity is affected by changes in ‘temperature. (B) Enzymes change the rate at which biochemical reactions proceed. (©) Enzyme activity is affected by large shifts in pH (D) Enzymes often require the presence of cofactors or coenzymes to become active. (B) Enzymes are assembled from vitamin ‘subunits. ‘The gene for a particular trait that is passed only from fathers to sons is most likely (A) autosomal recessive (B) autosomal dominant (©) codominant (D) Yelinked (B) Xlinked (GO ONTO THE NEXT PAGE 237 ® BIOLOGY-E/M TEST—Continued © Equus Tippavion Nannipps My Calippas 5 [Blxeottaperion 28 Hippidion 2 |B latecanipoas i EE] symone Fe ich Merychippus a Parahippus Anchitherir ER Miohippus Ze eS Mesohippus: 5 28 Orohippus iE ley 38. The diagram above illustrates a proposed phylogeny for horses. Which of the following genera is currently represented by live animals? (A) Epihippus (B) Equus (©) Hippidion (D) Hyracotherium (B) Nannippus Travia Sopng ros GO ONTO THE NEXT PAGE ‘ny pe ofthis poge sega D238 BIOLOGY-E/M TEST—Continued 39. Which of the following organelles in human UL sperm provides the energy needed by the sperm? ut (A) Flagellum 1 (B) Mitochondion (C) Y chromosome "i (D) Centriole (©) Nucleus 40. Which of the following organs secretes the @) hormone responsible for the “Fight-or-flight” v reaction in mammals? a (A) Liver Telophase Metaphase (B) Kidney (C) Pancreas (D) Cowper's gland (E) Adrenal gland 41. Most replication of DNA takes place during which of the following stages of the cell cycle? wl (B) Hand tt (© Wonly (D) IV and V © VI Grsriaed spying wea GO ONTO THE NEXT PAGE [a pat of ts page nega 239) SY BIOLOGY-E/M TEST—Continued 42. Which of the following is a biotic factor that can make a major contribution to the regulation of a population in a given community’? (A) The annual pattern of rainfall (B) The average ratio of O; to CO, (©) The annual pattern of daily temperature ranges (D) The rate of weathering of rocks into soil (E) The number of predators and competitors. 43. Characteristics of adult echinoderms such as sea stars (starfish) include which of the following? L. Tube feet IL Bilateral symmetry TIL. Water vascular system (A) Lonly (B) ILonly (©) and HI only (D) Hand Il only (©) 1, and Ut 4, If in an adult organism the genes A and B occur on one chromosome and their alleles a and b ‘occur on its homologue, which of the following explains a combination of Ab or aB occurring in the gametes? a) Jinkage (B) Lack of dominance (©) Nondisjunetion (D) Crossing-over (&) Blending 45. Which of the following is NOT a major function oof the mammalian kidney? (A) Elimination of urea and other nitrogenous ‘wastes (B) Maintenance of water balance (©) Manufacture of antibodies (D) Regulation of salt excretion (B) Formation of urine from glomerular filtrate 46. An ecologically sound reason for conserving tropical rain forests is that they (A) supply most of the oxygen that humans breathe (B) occupy four-fifths of Earth's surface (C) are the major producers of atmospheric nitrogen (D) are crucial to migratory ungulates like sand wildebet (B) are an important reservoir of biodiversity (GO ONTO THE NEXT PAGE > Questions 47-48 BIOLOGY-E/M TEST—Continued A population study of plants was done in an abandoned field, Each year for 3 years the vegetation was sampled. ‘The chart below indicates the results of the study. Year Number of Plants per Acre Sandspur Ragweed | Timothy Grass | Goldenrod | Wire Grass 1 3,800 4,900 600 0 42 2 1,500 2.209 1185 15 6 3 752 180 2204 790 1,643 47. According to the data, which of the following are initially most successful in the succession taking place in the field described above?” (A) Sandspur and ragweed (B) Sandspur and timothy grass (C) Ragweed and timothy grass (D) Ragweed and wire grass (E) Sandspur and goldenrod an pat tts pee ean 48, The data above suggest that (A) fires cause the changes in the populations (B) floods cause the changes in the populations (C) the plants in the population have similar li spans (D) plant populations are replacing one another (B) the reproductive capacity of plants changes with time GO ONTO THE NEXT PAGE 2a1d Questions 49.51 ‘The figure below represents the increase in prevalence of both keratoses (thickened pigmented patches on the skin) and skin cancers in males of Irish descent in several geographic areas. 60 (Australia & 2 & 30 40 50 60 70 Age (years) — Keratoses ‘Skin Cancers 49. For which of the following groups can 20 percent of the male population be expected to have the indicated condition? xs in Australia at age 50 n Australia at age 30 (©) Keratoses in Australia at age 70 (D) Keratoses in Ireland at age 40 (B) Keratoses in Ireland at age 80 BIOLOGY-E/M TEST—Continued 50. Which of the following can be inferred from these data? (A) Skin cancers develop from keratose (B) Keratoses develop from skin cancers. (C) The majority of males with kerato have skin cancer. (D) The environment in Australia is more likely {o cause keratoses than is the environment in Ireland. (E) The intensity of sunlight is the primary factor causing the development of SI. If the study were conducted as a function of the age of the female population in the same geographic arcas, which of the following results ‘would be most likely?” (A) The data would show a higher percentage of females with the diseases at all ages. (B) The data would show a lower incidence of the diseases, because females have higher levels of estrogen. (©) The data would be the same as for males in Australia and Ireland, but no predictions can be made for Texas. (D) The data would be the same as for males with regard to keratoses but not for skin cancers (B) No accurate predictions can be made from the data because the sample populations would be different. (GO ONTO THE NEXT PAGE > NO TEST MATERIAL ON THIS PAGE 203 Questions 52-55 BIOLOGY-E/M TEST—Continued Charles Darwin and his son Francis performed a series of experiments on phototropism (growth toward fight) of the coleoptile (the cap that covers the below Treat I Coleoptile untreated. IL Tip of coleoptile cut off. Ill Opaque cap placed over coleoptile tip. IV Coleoptite cut halfway through. V_ Transparent cap placed over coleoptile tip. VI Opaque sleeve placed over base of coleaptile. 52. Comparison of treatments I and I shows which of the following? (A) Growth is promoted by cutting off the tip. (B) The tip is the site of sensing light, (C) The tip is the site of auxin synthesis, (D) The tip is necessary for the response to light (E) There is a range of response (o a single treatment, lay part of us page nepal D2aa t leaves of new scedlings of grass). The treatments they used are described Growth Toward Light Allowed Prevented Prevented Allowed Allowed Allowed 53. The fact tha the effect of eutting off the tip (trcatment I) isnot simply duc to wounding of the plant is demonstrated by comparison of which of the following treatments? (A) IV and V (B) I, and Mit (C) I. and 1V (D) II, II, and 1V (B) IV, V, and VI GO ONTO THE NEXT PAGE > 54. Comparison of treatments Ill, V, and VI shows that (A) the tip plays a role in sensing the light (B) the base plays a role in sensing the light (C) confinement of the tip inhibits the response to light (D) confinement of the base facilitates the response to Tight (E) confinement reverses the response to light Q BIOLOGY-F/M TEST—Comtinued 58, To test the hypothesis that the response to light involves differential cell elongation, an experimenter could (A) measure the distance betwe the seedling after it has bent (B) count the number of cells visible in a cross section of the coleoptile (©) compare the length of cells on the stem toward and away from the light (D) determine whether mitosis is affected by light (B) repeat the experiment using light of a different wavelength marks made on of the Questions 56-60 During normal development of the sea urchin, the egg divides once to give two cells. BIOLOGY-E/M TEST—Continued Bach of these cells divides again. The cells continue to divide and, eventually, a sea-urchin larva is formed. It is possible to separate the cells of ‘a young sea-urchin embryo and allow them to develop independently. The results of several such experiments are shown below. Normal Development O-O-—@ -§-—2e “ Po ae fertilized two-cell four-cell cell sea-urchin cee embryo embryo, nbryo larva Experiments: Separate the cells and allow development to proceed. 96. Experiment I suggests that (A) sea urchins would be better adapted if they had smaller egg (B) embryo cells are committed to different developmental fates (C) different cells of an embryo can have equal potential for development (D) 4 particular cell of an embryo always develops into the same structure (E) cell division ensures that both cells will develop ide lay part of us page nepal D246 57. Experiments I and II suggest that (A) se adult (B) larva size is determined by the amount of ‘material in the embryo (C) development must always occur the same way in every embryo (D) embryo cells do not interact with each other (E) natural selection favors the formation of small larvae -chin embryos often grow to full-size GO ONTO THE NEXT PAGE > 58. Experiments III and IV together suggest that (A) there is a difference between separating cells ‘along the vertical axis and the horizontal axis of an cight-cell embryo (B) embryo cells cannot be separated without damaging development (C) material at the top of the embryo is the same as material at the bottom (D) cells divide correctly only when they are verti (E) embryo cells do not differ umtl gastrulation 59. The different results in experiments II and IV probably are caused by (A) failure of mitosis to 0 third cell div (B) loss of chromosomes by the top four cells (C) fertilization of the top and bottom of the egg by two different sperm (D) different genes being expressed in the top four cells than in the bottom four cells ‘ur normally at the (E) some genes in the left half of the embryo that ‘are different from those in the right half of the embryo BIOLOGY-E/M TEST—Continued 60. Which of the following questions is NOT addressed by this series of experiments? (A) When do the cells of an embryo become different from each other? (B) Can cells of an embryo survive when separated from each other? (©) Can smaller larvae be produced by experimental manipulation? (D) When are components in the fertilized eg activated? (B) Can the cells of an embryo be made to develop abnormally? If you are taking the Biology-E test, continue with questions 61-80. If you are taking the Biology-M test, go to question 81 now. Directions: Each of the questions or incomplete statem BIOLOGY-E TEST below is followed by five suggested answers or completions. Some questions pertain to a set that refers to a laboratory or experimental situation. For each question, select the one choice that is the best answer to the question and then fill in the correspon the answer sheet 61. Stream and river ecosystems differ from other aquatic ecosystems because streams and rivers (A) move continuously in one direction and have a nutrient content that is dependent ‘on location (B) support a greater diversity of aquatic plants, (C) have highly variable salinity (D) include the greatest biodiversity of all ecosystems because of the fluctuating water levels (E) support the largest stationary plankton communities 62. A trophic level within an ecosystem is best characterized by the (A) size of food eaten at that level (B) nutrient source of the organisms in each level (C) stages in ecological succession (D) habitats of the organisms within that level (B) elevation above sea level, [Unautheried copying rouse of [ny pat of is poge Mega D248 63. ot. According to most scientific theories of the origin Of life, the first organisms were (A) eukaryotic (B) parasi (C) symbiotie (D) anaerobic (B) pathogenic ‘The global eycles of nitrogen and phosphorus differ in that (A) nitrogen is recycled whereas phosphorus not (B) animals get most of their nitrogen from the water they drink whereas they 2% phosphorus from the food they (C) nitrogen occurs primarily in deep sediments. whereas phosphorus occurs primarily in the atmosphere (D) nitrogen is lost to the oceans where phorus is not (B) nitrogen has a gaseous phase whereas phos phorus does not phos- (GO ONTO THE NEXT PAGE > Sparrow Lo Ladybug Spider Aphid Rosebush 65. In the food web shown above, in which the arrows. indicate the direction of energy flow, the ladybug is considered to be a (A) herbivore (B) primary consumer (C) decomposer (D) producer (E) camivore 66. A stream is free of pollutants within a few miles downstream of a point at which a small amount of sewage is being dumped into it. This is most likely the result of (A) succession (B) biological magnification (C) evaporation (D) photosynthesis (E) decomposition aod coping pao tie page tga BIOLOGY-E TEST—Continued 67. 68, 9. ‘The term “adaptive radiation” refers to the (A) ability of one sp niche (B) ability of a species to adapt itself to rapidly changing conditions (©) evolution from a single ancestral species into several species adapted to various environments (D) ability of a species to adjust its temperature by radiating heat (B) advantages of radial symmetry to a stations species to adapt to only one Which of the following does NOT refer primarily (0 a relationship between members of dif sp (A) Mutualism (B) Hibernation (C) Parasitism (D) Commensatism (B) Predation Plant seeds following? L Wind IL. Water IL. Birds an be dispersed by which of the (A) Lonly (B) Momly (©) Land Il only (D) [and III only (©) I Ul,and tt Questions 70-72 BIOLOGY-E TEST—Continued ‘Two types of barnacles, Chthamalus and Balanus, grow on rocks along the North Atlantic coastline. Both grow on rock surfaces exposed at ow tide and covered at high tide. At the beginning of a study of competition between these baracles, a researcher removed selected Balanus from a region and followed the changes in distribution of both species for 12 months. The distribution of Chrhamalus and Balanus are shown in Figures 1, 2, and Figure 1 Figure 2 Figure 3 Beginning of study 3 months after 12. months after (Shaded individuals selected Balanus removed selected Balanus removed to be removed) Gy Balas & Chihamatus Coastal mpgs maa] GO ONTO THE NEXT PAGE amy part of tus page fs Weae »250 1 ‘Since both species of bamacles have free- swimming larvae that settle on hard surfaces, the change in the distribution of Chthamatus observed 3 months after removal of the larger Batanus individuals could best be explained by which of the following? (A) Balanus feeds on Chthamatus larvae. (B) Balanus does not reproduce as quickly as Chihamatus. (©) Balanus has less tolerance for wet conditions. (D) Balanus adults are mo! (E) Balanus is less susceptible to predators. The distribution of the two species at 3 and 12 months suggests all of the following EXCEPT: (A) Balanus sometimes dominates over the smaller Chihamalus. (B) Chthamalus ean tolerate more drying than Balanus. (©) Balanus adults are swept away more often than Chthamatus. (D) Balanus and Chuhamatus larvae can settle in the same are (B) Balanus is larger and thus needs more feeding time in the water. aod coping pao tie page tga BIOLOGY-E TEST—Continued ‘72, Based on this study, on rocks with tops below the idtide line, it can be predicted that (A) more of the rock surface would be covered by Chthamatus (B) the two barnacle populations would be equal (C) there would be few, if any, Balanus (D) there would be few, if any, Chthamalus (B) Balanus individuals would become smaller BIOLOGY-E TEST—Continued ‘Questions 73-75 refer to the following experiment in which an agar petri dish was prepared as shown below. Using aseptic wchniques, an experimenter spread E. coli bacteria on the agar uniformly throughout the dish. The dish was then incubated at 31°C for 24 hours. Nutrient Agar Nutrient Agar with Penicillin [Unautheried copying rouse of [ny pat of is poge Mega D252 73. Which of the following distributions of bacterial colonies is most likely to be observed on completion of the experiment? (Dots represent bacterial coloni “ ®B) I © L ©) I to) (GO ONTO THE NEXT PAGE > 174, The cells that survived exposure to penicillin were most likely able to do so because they (A) mutated as a result of the exposure (B) had a more rapid metabolism than the other cells (C) lacked cell walls (D) already possessed penicillin resistance (B) formed spores BIOLOGY-E TEST—Continued 75. In the experiment, penicillin causes a stress that, in an ecosystem, would promote (A) natural selection (B) Lamarckian evolution (C) competition (D) mutation (E) parasitism ® BIOLOGY-E TEST—Continued Questions 76-80 A scientist studied a field that had been burned in a brushfire ten years before. She identified seven different species and produced the table below. Scientific Name [Gross Form [Microscopic | Stem Form Color Reproduction Form Platismartia | sheetlike ‘eukaryotic and | not applicable | green on top none evident glauca prokaryotic white below filaments Funaria cushionlike | eukaryotic not applicable | green throughout | spore-producing ‘americana multicellular structures on stalks Dryopteris roots, under- | cukaryotie fleshy ‘green leaves, white | spores on ‘Spinulosa | ground stems, | multicellular stem and roots | underside of ‘compound leaves. leaves Picea roots, stems, erect, woody | green leaves, brown | cones rubens needlelike leaves ‘stem and roots. Smilax roots, vine, erect, herbaceous | green leaves and | flowers herbacea | broad leaves _| multicellular stem, white roots, Smilax roots, woody | eukaryotic erect ‘green leaves, brown | none evident rowundifotia | vine, broad multicellular ‘stem and roots leaves Monotropa roots, stems, | eukaryotic erect white throughout | fruits uniflora broad leaves | multicellular 76. The organism that evolutionarily is most closely 77. Symbiosis is best illustrated by which of the related to Smilax herbacea is following organisms? (A) Platismatia glauca (A) Platismatia glauca (B) Dryopteris spinulosa B) Dryopteris spinulosa (C) Picea rubens (C) Picea rubens (D) Smilax rowndifotia (D) Smilax herbacea (EB) Monotropa uniftora ©) Smilax romndifotia Grariaed spying wera GO ONTO THE NEXT PAGE [ay pat of tis page oga D254 78. Which of the following shows the simplest level of physical organization? (A) Funaria americana (B) Monotropa uniftora (©) Drvopteris spinulosa (D) Picea rubens (E) Smilax roundifotia 79. W pl (A) Platismatia glauca (B) Funaria americana (©) Dryopteris spinulosa (D) Picea rubens (B) Monotropa uniftora ich of the followi most likely a flowering BIOLOGY-E TEST—Continued 80, The appearance of these plants in the burned area is an example of what biological process? (A) Evolution by natural selection (D) Eutrophication (E) Recombination STOP IF YOU FINISH BEFORE TIME IS CALLED, YOU MAY CHECK YOUR WORK ON THE ENTIRE BIOLOGY-E TEST. sry par of hs poge hege 2550 BIOLOGY-M SECTION © If you are taking the Biology-M tes Be sure to start this section of the test by continue with questions 81-100. ing in circle 81 on your answer sheet. Directions: Each of the questions or incomplete statements below is followed by five suggested answers or completions, Some questions pertain to a set that refers to a laboratory or experimental situation, For each question, select the one choice that is the best answer to the question and then fill in the corresponding circle on the answer sheet i NH} ° o-c. 5 AAR doo Poo o=c | 7 \o~e | oO tpmeee eee o ° I 81. Which of the following is correct about the phospholipid shown above? (A) Only I would be found in the middle of the lipid bilayer. (B) Only I would be found in the middle of the lipid bilayer. (C) Both [and I would be found in the middle of the lipid bilayer, (D) Il is hydrophilic. (B) [ and II are hydrophobic. nautoriaed sopying a outs oi GO ONTO THE NEXT PAGE ny pet ofthis pag Mega D256 82 83 Products of the light reactions of photosynthesis that later participate in the dark reactions of photosynthesis inelude which of the following? I. Reduced NADP (NADPH) WL ATP MIL. 0; (A) Lonly (B) Honly (©) Monty (D) [and I only (E) [and iT ‘The way in which an enzyme and its specific substrate interact is best described by the (A) fluid-mosaic model (B) induced-fit model (C) Oparin hypothe: (D) Lyon hypothesis (B) competitive ision principle If a somatic callin a diploid organism contains ten pairs of chromosomes, what is the total ‘number of chromatids that are present in the cell after the DNA has replicated but before mitosis, has taken place? (A) 10 (B) 20 ©) 30 (D) 40 (E) 80 BIOLOGY-M SECTION—Continued 85. Which of the following atmospheric gases shows a net release during photosynthesis in plants? (A) Carbon dioxide (B) Oxygen (©) Methane (D) Hydrogen (B) Nitrogen 86. Which of the following are the final products of fermentation? (A) Carbon and oxygen (B) Glucose and alcohol (C) Carbon dioxide and oxygen (D) Carbon dioxide and alechol (B) Oxygen and water 87. A function of transfer RNA is to (A) receive the genetic information from ‘nuclear DNA (B) store the genetic information in the nucleus (C) store RNA in the ribosomes (D) transfer the genetic information from the rnucleus to the cytoplasm (B) position amino acids for protein synthesis by pairing with codons in messenger RNA BIOLOGY-M SECTION—Continued 88. Which of the following statements most accurately describes a basic difference between mitosis and (A) Homologous chromosomes form tetrads in mitosis but not in meiosis. (B) Homologous chromosomes form tetrads in meiosis but not in mitosis. (©) The nuclear membrane disappears in mitosis ‘but not in meiosis. (D) A spindle forms in mitosis but not in meiosis. (B) A spindle forms in meiosis but not in mitosis. 89. The BF protein produced in the bacterium, Bacillus thuringiensis, kills corn earworms that ingest the Bacillus. Ifthe Br gene were transferred to corn so that corn could express the Br protein, which of the following would be cted to occur When corn earworms eat the 1. Corn earworms that eat the Bf corn would be Killed. UL, Bacillus bacteria that infect the Bt com would be killed IIL. The cor earworms would incorporat the Br gene into their chromosomes, (A) Lonly (B) only (©) Monly (D) Land II only (B) Mand II only [Unautheried copying rouse of [ny pat of is poge Mega D258 90. The wavelengths of light absorbed by chlorophyll are similar to the wavelengths of light that are associated with the greatest amount of oxygen release by plants, Based on these observations which of the following is a reasonable hypothesis about the function of chlorophyll? (A) Itplaysa role in cell respiration. (B) It plays a role in the light reactions of photosynthesis. (C) Ittakes part in HO release. (D) Ittakes part in CO, fixation. (E) It generates energy. 91. Cellular respiration shares which of the following characteristics with the light-dependent reactions (B) Production of AMP (C) Production of GIP (D) Production of oxygen (EB) Use of carbon dioxide in synthetic reactions 92. The position of a mutation in a gene and the location of an altered amino acid sequence in the corresponding protein are (A) not related (B) inversely related (C) related in bacteria but not in mammals (D) species-dependent (B) in the same relative position 93. In order for an animal that was cloned from mother to grow and develop normally, it must have received (A) half of its mother’s DNA sequences (B) half of its father’s RNA sequences. (C) all of its mother’s RNA sequences (D) all of its father's DNA sequences (E) all of its mother’s DNA sequences (GO ONTO THE NEXT PAGE > BIOLOGY-M SECTION—Continued © Questions 94.97 Bowls and 7 — water only Bowls 2 and 8 — water + 20 water plants Bowls 3 and 9 — water + 40 water plants Bowls4 and 10—water + 2 goldfish Bowls 5 and 11 —water + 4 goldfish Bowls 6 and 12 — water + 20 water plants + 2 goldfish A biologist set up 12 bowls as described above. She exposed bowls | to 6 to light for 24 hours and placed bowls 7 10 12 im the dark for 24 hours. She determined the CO, content of the water in micromoles per liter for each bow! at the end of the 24 hours. The results are indicated below. Experimental Results Light Dark Bowl# — [COs] Bowl# — [C03] 1 7 102 2 8 10 3 9 169 4 ott 5 ul 9 6 R192 94. The process responsible forthe relatively Tow 96, The difference in COs concentrations for bowls concentrations of CO, in bowls 2 and 3 is 2 and 6 can best be explained by (A) respiration (A) photosynthesis caried out by water plants (B) fermentation (B) respiration carried out by water plants (C) photosynthesis (C) respiration carried out by goldfish (D) photoperiodism (D) competition between water plants and goldish () transpiration (E) experimental error 95, The main controls for bow 4 are 97. Which of the following is the best explanation for (A) Land 3 the fact that the CO; concentration of bow! 4 is (@) Land almost the same as that of bow! 10 and the CO, (C) Land 10 concentration of bowl 5 is almost the same as that (D) Zand 5 of bowl 11? (©) and 8 (A) Photosynthesis does not occur in the light, (B) Photosynthesis does not occur in the dark. (©) Respiration and photosynthesis occur at the ‘same rate in the light. (D) Respiration is not affected by either light or dark. (B) Goldfish are more active in the absence than in the presence of plants. Tons coping Toe GO ONTO THE NEXT PAGE any par ofthe ge neg 259) BIOLOGY-M SECTION—Continued ‘Questions 98-100 refer to the following experimental procedure. A protein is purified from a frog embryo. The protein sample is divided into five fractions. One fraction is not treated. The other fractions are partially digested by using enzymes that act on specific amino acid sequences. La every case, the digestions are carried out at the appropriate temperature and pH. The samples are then separated by elecirophoresis as shown below. Unaltered Protein Protein Protein Protein Digested by Digested by _ Digested by EnzymeX — EnzymeY Enzyme Z Gmanraagsopine ear (GO ONTO THE NEXT PAGE » 260 98, 99, In the electrophoresis experiment described, the distance moved by a fragment within the electric field is influenced by which of the following? 1. The number of amino acids in the fragment I. The amount of electric current used in the apparatus ILL. The porosity of the gel matrix (A) only (B) Monly (©) Land ILonly (D) 1 and 111 only (©) 1.11, and Itt Which of the following techniques could have been used as an alternative to electrophoresis to separate the products of digestion with enzyme Z? 1. Translation II, Chromatography TIL Serial dilution (A) Tonly (B) Ionly (©) Mronly (D) I and If only () 1 tl,and IL STOP BIOLOGY-M SECTION—Continued 100. OF the two fragments resu ® from the digestion of the protein with enzyme Z, one is larger and. the other is smaller than either of the fragments resulting from the digestion with enzyme Y ‘The most logical explanation for this is that (A) the protein fragments produced by enzy have the same molecular weights as those produced by enzyme Z (B) proteins are produced by ribosomes (C) enzymes Y and Z have different amino acid sequences, (D) electric current is divided into diserete units (B) the protein is cut at different amino acid sequences by enzymes Y and Z IF YOU FINISH BEFORE TIME IS CALLED, YOU MAY CHECK YOUR WORK ON THE ENTIRE BIOLOGY-M TEST. [Unsutveried copying or rouse oi any prt ofthis poge' Mepa 261) ‘The Official Study Guide for All SAT Subject Tests How to Score the SAT Subject Test in Ecological Biology When you take an actual SAT Subject Test in Ecological Biology, your answer sheet will be “read” by a scanning machine that will record your responses to each question. ‘Then a computer will compare your answers with the correct answers and produce your raw score. You get one point for each correct answer. For each wrong answer, you lose ‘one-fourth ofa point. Questions you omit (and any for which you mark more than one answer) are not counted. This raw score is converted to a scaled score that is reported to you and to the colleges you specify. Worksheet 1. Finding Your Raw Test Score STEP 1: Table A lists the correct answers for all the questions on the Subject Test in Ecological Biology that is reproduced in this book. It also serves as a worksheet for you to calculate your raw score. + Compare your answers with those given in the table. + Puta check in the column marked “Right” if your answer is correct. + Put a check in the column marked “Wrong” if your answer is incorrect. + Leave both columns blank if you omitted the question. STEP 2: Count the number of right answers. Enter the total here: STEP 3: Count the number of wrong answers. Enter the total here: STEP 4: Multiply the number of wrong answers by .250. Enter the product here: STEP 5; Subtract the result obtained in Step 4 from the total you obtained in Step 2. Enter the result here: STEP 6: Round the number obtained in Step 5 to the nearest whole number. Enter the result here: The number you obtained in Step 6 is your raw score. » 262 Table A Answers to the Subject Test in Ecological Biology, Form 3¥AC, and Percentage of Students Answering Each Question Correctly a vs Brace fend a Ri fers erlemmoremecaclacocals>>mcleraaeclscoay Table A continued on next page 2630 ‘The Official Study Guide for All SAT Subject Tests Table A continued from previous page a perry Right | Wrong end * These percentages are based on an analysis ofthe answer sheets of a representative sample of 3,130 students who took the original form of this test in May 2002, and whose mean score was 601. They may'be used as an Indication ofthe relative difficulty of particular question, Each percentage may also be used to predict the likelihood that a typical SAT Subject Test in Ecological Biology candidate will answer that question correctly on this edition of the test, D264 Biology Finding Your Scaled Score When you take SAT Subject Tests, the scores sent to the colleges you specify are reported on the College Board scale, which ranges from 200-800. You can convert your practice test score to a scaled score by using Table B. To find your scaled score, locate your raw score in the left-hand column of Table B; the corresponding score in the right-hand column is your scaled score. For example, a raw score of 21 on this particular edition of the Subject Test in Ecological Biology corresponds to a scaled score of 450. Raw scores are converted to scaled scores to ensure that a score earned on any one edition of a particular Subject Test is comparable to the same scaled score earned on any other edition of the same Subject Test. Because some editions of the tests may be slightly easier or more difficult than others, College Board scaled scores are adjusted so that they indicate the same level of performance regardless of the edition of the test taken and the ability of the group that takes it, Thus, for example, a score of 400 on one edition of a test taken at a particular administration indicates the same level of achievement as a score of 400 on a different edition of the test taken at a different administration, When you take the SAT Subject Tests during a national administration, your scores are likely to differ somewhat from the scores you obtain on the tests in this book. People perform at different levels at different times for reasons unrelated to the tests themselves. The precision of any test is also limited because it represents only a sample of all the possible questions that could be asked. 2650 ‘The Official Study Guide for All SAT Subject Tests Table B Scaled Score Conversion Table Subject Test in Ecological Biology (Form 3¥AC) 610 2 380 19 800 45 610 " 380 B 800 4 600 10 370 n 800 a 600 9 370 18 720 a2 590 8 360 6 780 a 590 7 350 1% 780 40 580 6 350 B 7m 39 570 5 340 n 770 38 510 4 340 n 760 37 560 3 330 70 750 36 560 2 330 6 750 35 590 1 320 Cy 740 34 540 0 320 6 730 3 540 4 310 6 730, 32 530 2 310) 6 720 aI 520 3 300 64 720 30 520 4 300 63 m0 Ea 510 5 290 62 m0 28 500 6 290 61 700 a 500 7 230 Cy 630 26 430 8 280 59 630 % 490 8 270 58 680 By 480 a0 270 37 620 a 470 oT 270 56 670 2 460 a2 260 55 670 a 450 a3 260 54 660 a 450 4 260 53 650 19 440 415 250 82 650 8 430 46 250 51 640 ” 420 a 240 50 640 6 420 78 240 8 630 5 a0 19 230 48 630 “ 400 20 220 a 620 8 400 » 266 Biology How Did You Do on the Subject Test in Ecological Biology? After you score your test and analyze your performance, think about the following questions: Did you run out of time before reaching the end of the test? If so, you may need to pace yourself better. For example, maybe you spent too much time on one or two hard questions. A better approach might be to skip the ones you can't answer right away and try answering all the questions that remain on the test. Then if there's time, go back to the questions you skipped. Did you take a long time reading the directions? You will save time when you take the test by learning the directions to the Subject Test in Ecological Biology ahead of time. Each minute you spend reading directions during the test is a minute that you could use to answer questions, How did you handle questions you were unsure of? Ifyou were able to eliminate one or more of the answer choices as wrong and guess from the remaining ones, your approach probably worked to your advantage. On the other hand, making haphazard guesses or omitting questions without trying to eliminate choices could cost you valuable points. How difficult were the questions for you compared with other students who took the test? Table A shows you how difficult the multiple-choice questions were for the group of students who took this test during its national administration. ‘The right-hand column gives the percentage of students that answered each question correctly. A question answered correctly by almost everyone in the group is obviously an easier question. For example, 85 percent of the students answered question 21 correctly. But only 32 percent answered question 77 correctly. Keep in mind that these percentages are based on just one group of students. They would probably be different with another group of students taking the test. If you missed several easier questions, go back and try to find out why: Did the questions cover material you haven't yet reviewed? Did you misunderstand the directions? 267 ‘The Official Study Guide for All SAT Subject Tests How to Score the SAT Subject Test in Molecular Biology When you take an actual SAT Subject Test in Molecular Biology, your answer sheet will be “read” by a scanning machine that will record your responses to each question. ‘Then a computer will compare your answers with the correct answers and produce your raw score. You get one point for each correct answer. For each wrong answer, you lose ‘one-fourth ofa point. Questions you omit (and any for which you mark more than one answer) are not counted. This raw score is converted to a scaled score that is reported to you and to the colleges you specify. Worksheet 1. Finding Your Raw Test Score STEP 1: Table A lists the correct answers for all the questions on the Subject Test in Molecular Biology that is reproduced in this book. It also serves as a worksheet for you to calculate your raw score. + Compare your answers with those given in the table. + Puta check in the column marked “Right” if your answer is correct. + Put a check in the column marked “Wrong” if your answer is incorrect. + Leave both columns blank if you omitted the question. STEP 2: Count the number of right answers. Enter the total here: STEP 3: Count the number of wrong answers. Enter the total here: STEP 4: Multiply the number of wrong answers by .250. Enter the product here: STEP 5; Subtract the result obtained in Step 4 from the total you obtained in Step 2. Enter the result here: STEP 6: Round the number obtained in Step 5 to the nearest whole number. Enter the result here: The number you obtained in Step 6 is your raw score. » 268 Table A Answers to the Subject Test in Molecular Biology, Form 3YAC, and Percentage of Students Answering Each Question Correctly eer pacers erry es Right | Wrong Table A continued on next page 269 ‘The Official Study Guide for All SAT Subject Tests Table A continued from previous page 5 8 @ E 6 D 6 4 c 7 e 69 % c 64 88 8 6 % c 79 89 A a a o % Ey 8 7 98 E 3 31 A 7 99 8 3% 2 £ a 100 E 18 * These percentages are based on an analysis ofthe answer sheets of a representative sample of 3.964 students who took the original form of this test in May 2002, andl whose mean score was 640. They may be used as an indication ofthe relative dificult of a particular question. Fach percentage may also be used to predict the likelihood that a typical SAT Subject Test in Molecular Biology Subject candidate sell answer that question correctly on this edition ofthe test D270 Biology Finding Your Scaled Score When you take SAT Subject Tests, the scores sent to the colleges you specify are reported on the College Board scale, which ranges from 200-800. You can convert your practice test score to a scaled score by using Table B. To find your scaled score, locate your raw score in the left-hand column of Table B; the corresponding score in the right-hand column is your scaled score. For example, a raw score of 21 on this particular edition of the Subject Test in Molecular Biology corresponds to a scaled score of 470. Raw scores are converted to scaled scores to ensure that a score earned on any one edition of a particular Subject Test is comparable to the same scaled score earned on any other edition of the same Subject Test. Because some editions of the tests may be slightly easier or more difficult than others, College Board scaled scores are adjusted so that they indicate the same level of performance regardless of the edition of the test taken and the ability of the group that takes it, Thus, for example, a score of 400 on one edition of a test taken at a particular administration indicates the same level of achievement as a score of 400 on a different edition of the test taken at a different administration, When you take the SAT Subject Tests during a national administration, your scores are likely to differ somewhat from the scores you obtain on the tests in this book. People perform at different levels at different times for reasons unrelated to the tests themselves The precision of any test is also limited because it represents only a sample of all the possible questions that could be asked. and ‘The Official Study Guide for All SAT Subject Tests p22 Table B Scaled Score Conversion Table Subject Test in Molecular Biology (Form 3¥AC) 80 800 46 620 78 800 45 620 8 00 4 610 7 800 a 610 78 790 2 600 75 730 a 590 a 720 40 590 B 780 38 580 2 m0 38 580 n 780 a7 570 70 760 36 560 cy 750 5 580 Cy 750 34 580 o 70 33 580 Cy 730 2 540 6 730 31 530 ea m0 30 530 63 720 29 520 2 70 2 510 ét 70 a 510 0 700 2 500 59 690 B 490 58 630 a 490 7 680 a 480 56 680 2 470 6 70 a 470 sa 670 2 460 53 660 19 450 52 650 18 440 51 650 v7 440 50 640 16 430 49 640 15 420 48 620 “ 420 a 630 8 410 Biology How Did You Do on the Subject Test in Molecular Biology? After you score your test and analyze your performance, think about the following questions: Did you run out of time before reaching the end of the test? If so, you may need to pace yourself better. For example, maybe you spent too much time on one or two hard questions. A better approach might be to skip the ones you can't answer right away and try answering all the questions that remain on the test. Then if there's time, go back to the questions you skipped. Did you take a long time reading the directions? You will save time when you take the test by learning the directions to the Subject Test in Molecular Biology ahead of time. Each minute you spend reading directions during the test is a minute that you could use to answer questions, How did you handle questions you were unsure of? Ifyou were able to eliminate one or more of the answer choices as wrong and guess from the remaining ones, your approach probably worked to your advantage. On the other hand, making haphazard guesses or omitting questions without trying to eliminate choices could cost you valuable points. How difficult were the questions for you compared with other students who took the test? Table A shows you how difficult the multiple-choice questions were for the group of students who took this test during its national administration. ‘The right-hand column gives the percentage of students that answered each question correctly. A question answered correctly by almost everyone in the group is obviously an easier question. For example, 87 percent of the students answered question 5 correctly. But only 35 percent answered question 53 correctly. Keep in mind that these percentages are based on just one group of students. They would probably be different with another group of students taking the test. If you missed several easier questions, go back and try to find out why: Did the questions cover material you haven't yet reviewed? Did you misunderstand the directions? 2730 Chapter 6 Chemistry Purpose ‘The Subject Test in Chemistry measures the understanding of chemistry you would be expected to have after successfully completing a college-preparatory course in high school and is designed to be independent of the particular textbook or instructional approach used. Format This is a one-hour test with 85 multiple-choice questions. Content The test covers the topics listed in the chart on the next page. Different aspects of these topics are stressed from year to year. However, because high school courses differ, both in the amount of time devoted to cach major topic and in the specific subtopics covered, it is likely that most students will encounter some questions on topics with which they are not familiar. Every edition of the test contains approximately five questions on equation balancing and/or predicting products of chemical reactions; these are distributed among the various content categories. 275) ‘The Official Study Guide for All SAT Subject Tests Topics Covered ‘Atomie Structure, including experimental evidance of atomic structure, quantum numbers and energy levels (orbital), electron configurations periodic trends fusture including Lewis structures, three-dimensional molecular shapes, polatity Bonding, including ionic, covalent, and metalic band, relationship of bonding to properties and struc tures; intermolecular forces such as hydrogen boning, dipole-dipole forces, dispersion (London) forces Pees and Bases, including Bronstod-Lowry theory, strong and weak acid and bases, pH, titrations, indicators Oxidation-Reduction, including recognition of exidation-edvction reactions, combustion, oxidation rumbers, use of ecitvty series Preci including basic solubilty rules Mole Concept, including molar mass, Avogadeo's numb, empirical and molecular formulas Chemical Equations, including the balancing of equations stoichiometric calculations, percent yield, limiting reactants Equilibrium and Equilibrium Systems, including factors affecting positon of equilibrium (LeChatelie’s principe in g2senus and aqueous systems, euilrium constants, equilbium expressions ar Rates of Reactions, including factors affecting reaction rates, potential energy diagrams, activation energies es Including conservation of energy, calorimetry and specific heat, enthalpy (het| changes associated with phase changes and chica reactions, heating and cooing curves, randomness (entropy) ey 7 Including knowledge of laboratory equipment, measurements, pracedures, observation, safety, calculation, deta analysis, imerpretaton of graphical dat, drawing conclusions from observations ané data ‘Approximate Percentage of Test Skills Specifications Ser Remembering fundamental concepts and specific information: demonstrating familaity with terminology Application of K ‘Applying a single principle to unfair and/oe practical situations: o obtain a qualitative result or salve @ ‘quantitative problem LL wledge Inferring and deducing from qualitative data and/or quantitative data; integrating two or more relationships ta draw conclusions or solve prblems ‘Approximate Percentage of Test D276 Chemistry How to Prepare + Take a one-year introductory chemistry course at the college-preparatory level. + Laboratory experience is a significant factor in developing reasoning and problem- solving skills and should help in test preparation even though laboratory skills can be tested only in a limited way in a multiple-choice test. + Mathematics preparation that enables handling simple algebraic relationships and applying these to solving word problems will help. + Familiarize yourself with the concepts of ratio and direct and inverse proportions, exponents, and scientific notation, + Familiarize yourself with directions in advance. The directions in this book are identical to those that appear on the test. You should have the ability to + recall and understand the major concepts of chemistry and to apply the principles to solve specific problems in chemistry. + organize and interpret results obtained by observation and experimentation and to draw conclusions or make inferences from experimental data, including data presented in graphic and/or tabular form Notes: (I) A periodic table indicating the atomic numbers and masses of elements is provided for al test administrations (2) Calculators aren't allowed to be used during the test. ) Problem solving requires simple numerical calculations. (4) The metric system of units is used. Score ‘The total score is reported on the 200-to-800 scale. Sample Questions ‘Three types of questions are used in the Chemistry Subject Test: classification questions, relationship analysis questions, and five-choice completion questions. Note: For all questions involving solutions, assume that the solvent is water unless otherwise noted. 277 ‘The Official Study Guide for All SAT Subject Tests Classification Questions Each set of classification questions has, in the heading, five lettered choices that you will use to answer all of the questions in the set. The choices may be statements that refer to concepts, principles, substances, or observable phenomena; or they may be graphs, pictures, equations, numbers, or experimental settings or situations. Because the same five choices are applicable to several questions, the classification questions usually require less reading than other types of multiple-choice questions. Answering a question correctly depends on the sophistication of the set of questions. One set may test your ability to recall information: ay ask you to apply information to a specific situation or to translate information from one form to another (descriptive, graphical, mathematical). The directions for this type of question specifically state that you should not eliminate a choice simply because it is the correct answer to a previous question. Following are the directions for and an example of a classification set. Directions: Each set of lettered choices below refers to the numbered statements immediately following it. Select the one lettered choice that best fits each statement or answers each question and then fill in the corresponding circle on the answer sheet. A choice may be used once, more than once, or not at all in each set. Questions 1-3 refer to the following aqueous solutions: (A) 0.1 MHCL (B) 0.1 MNaCl (©) 0.1 MHC,H,O. (D) 0.1 MCH,OH (E) 0.1 MKOH 1. Is weakly acidic 2. Has the highest pH 3. Reacts with an equal volume of 0.05 M Ba(OH), to form a solution with pH = 7 ‘These three questions belong to the topic category of acids and bases and require you to apply knowledge in this area to the particular solutions specified in the five choices Choice (C) is the correct answer to question 1. To answer the first question, you must recognize which of the choices above are acid solutions. Only choices (A) and (C) satisfy this requirement, Choice (B) refers to a neutral salt solution, choice (D) isa solution of an alcohol, and choice (B) is a basic solution. Both choices (A) and (C) are acidic solutions, but choice (A) is a strong acid that is completely ionized in aqueous solution, while choice (C) is only partially ionized in aqueous solution. Since the concentrations of all the solutions are the same, you do not need to consider this factor. The hydrogen ion concentration of a 0.1-molar acetic acid solution is considerably smaller than 0.1-molar. ‘The hydrogen ion concentration in choice (A) is equal to 0.1-molar. Thus, choice (C) is a weakly acidic solution and is the correct answer. D278 Chemistry Choice (E) is the correct answer to question 2. To answer the second question, you need to understand the pH scale, which is a measure of the hydrogen ion concentration in solution and is defined as pH =~log {H']. The higher the pH, the lower the hydrogen ion concentration and the more basic the solution, Among the choices given above, choice (B) is the most basic solution. Choice (A) is the correct answer to question 3. To answer the third question, you need to know that acids react with bases to form salts and water. Since the question refers to equal volumes of each solution, assume | liter of each solution is available. Barium hydroxide solution is a strong base, ie., is completely ionized in water, and 1 liter of 0.05 M Ba(OH), provides 0.1 mole of OH ions in solution. When 1 liter of this solution is added to 1 liter of either 0.1 M NaCl, 0.1 M CH,OH, or 0.1 M KOH no reactions occur and the resulting, solutions remain basic, i.e, the pH will be greater than 7 in each case. When 0.1 mole OH ions reacts with 0.1 mole of acetic acid, the resulting solution will also be basic and have a pH greater than 7 because acetic acid is a weak acid, ice, is incompletely ionized in water. The acetic acid reacts with the OH. ions as follows: HC,H,O, + OH # CHO, +H,0 ‘The acetate salt formed hydrolyzes in water yielding a solution containing more OH ions than H’ ions. When 1 liter of 0.05 M Ba(OH), reacts with 1 liter of 0.1 M HCL, there is a reaction between 0.1 mole OH ions and 0.1 mole H’ to form 0.1 mole H,O. The resulting, solution contains Ba®” ions and Cl ions and equal concentrations of OH and H ions. The solution formed is neutral and the pH is 7. Relationship Analysis Questions ‘This type of question consists of a specific statement or assertion (Statement! followed by an explanation of the assertion (Statement II). The question is answered by determining if the assertion and the explanation are each true statements and, if so, whether the explanation (or reason) provided does in fact properly explain the statement given in the assertion. This type of question tests your ability to identify proper cause-and-effect relationships. It probes whether you can assess the correctness of the original assertion and then evaluate the truth of the “reason” proposed to justify it. The analysis required by this type of question provides you with an opportunity to demonstrate developed reasoning skills, and the scope of your understanding of a particular topic. On the actual Chemistry Test, the fllowing type of question must be answered on a special section (labeled “Chemistry at the lower left-hand comer of your answer sheet. These questions will be numbered beginning with 101 and must be answered according tothe following directions. cE 1 i 101 |@® ©|O O|O SAMPLE ANSWER GRID 279 ‘The Official Study Guide for All SAT Subject Tests Directions: Each question below consists of two statements, | in the left-hand column and IJ in the right-hand column, For each question, determine whether statement | is true or false and whether statement II is true or false and fill in the corresponding T or F circles on your answer sheet. Fill in circle CE only if statement Il is a correct explanation of the true statement I, EXAMPLES: EX. H,S0, isa strong acid BECAUSE H.S0, contains sulfur EX2__ Anatom of oxygen is BECAUSE an oxygen atom contains an equal number electrically neutral of protons and electrons T]_a [oe SAMPLE ANSWERS 11 @ ©) @ O|O e218 C1} O|@ I 0 4, The electrolysis of a concentrated BECAUSE _sodiumchlorideisacovalent solution of sodium chloride compound. produces chlorine ‘The above question has several components. Statement I, the assertion, has to do with an oxidation-reduction reaction, more specifically, an electrochemical reaction, This statement is true because the electrolysis of a concentrated sodium chloride solution yields chlorine gas at the anode (oxidation) and hydrogen gas at the cathode (reduction). ‘The electrolytic solution gradually becomes alkaline with the accumulation of hydroxide ions (i.., OH" ions) as the reaction proceeds. Statement II, the reason, is false because the type of chemical bonding in sodium chloride ionic. According to the directions for answering this question type, you should fill in the corresponding T and F circles on your answer sheet. I u 5. Atoms of different elements can BECAUSE atoms of each element have have the same mass number a characteristic number of protons in the nucleus. This is a question on atomic structure. The sum of the number of protons plus the number of neutrons contained in the nucleus of an atom is the mass number. However, atoms of the same element may have different numbers of neutrons in their nuclei and thus have different masses. Such atoms, which have the same number of protons but different numbers of neutrons, are called isotopes of an element ('C and ''C, for example). The existence of isotopes makes it possible for atoms of different elements, »280 Chemistry that is, with different numbers of protons, to have the same total mass or mass number (2C and "'N, for example). Thus Statement I is true. Statement I! is also true because the number of protons in the nucleus of an atom is a characteristic feature that identifies each element. But it is not the reason that explains the existence of isotopes and so does not properly explain Statement I. Thus, to answer this question, you should fill in both T circles for this question, but not the CE circle. I I 6. When the system CO(g) + BECAUSE an increase of pressure on C1(g) = COCI,(g) is at equilibrium, a system will be relieved and the pressure on the system is when the system shifts to increased by decreasing the volume a smaller total number of at constant temperature, more moles of gas. COC,(g) will be produced Statement I is true because whenever stress is applied to a system at equilibrium the system will tend to shift to relieve the stress (Le Chatelier’s principle). In the system described, the stress is caused by an increase in pressure resulting from a decrease in the volume and will be relieved by the reaction of some CO and Cl, to form more COCI,. The new equilibrium that will be established will contain a smaller total number of moles of gas, thereby reducing the pressure stress. This is the explanation given in Statement, II, which is not only true but also correctly explains the phenomenon described in Statement I. Thus, to answer this question correctly you should fill in both T' circles as well as the CE circle. Five-Choice Completion Questions ‘The five-choice completion question is written either as an incomplete statement or as a question, It is appropriate when: (1) the problem presented is clearly delineated by the wording of the question so that you are asked to choose not a universal solution but the best of the solutions offered; (2) the problem is such that you are required to evaluate the relevance of five plausible, or even scientifically accurate, options and to select the one most pertinent; (3) the problem has several pertinent solutions and you are required to select the one inappropriate solution that is presented. Such questions normally contain a word in capital letters such as NOT, LEAST, or EXCEPT. A special type of five-choice completion question is used in some tests, including the SAT Subject Test in Chemistry, to allow for the possibility of multiple correct answers. For these questions, you must evaluate each response independently of the others in order to select the most appropriate combination, In questions of this type several (usually three or four) statements labeled by Roman numerals are given with the question. One or more of these statements may correctly answer the question, You must select, from among the five lettered choices that follow, the one combination of statements that best answers the question. In the test, questions of this type are intermixed among the more standard five- choice completion questions. (Question 8 is an example of this type of question.) 281) ‘The Official Study Guide for All SAT Subject Tests In five-choice completion questions, you may be asked to convert the information given in a word problem into graphical form or to select and apply the mathematical relationship necessary to solve the scientific problem, Alternatively, you may be asked to interpret experimental data, graphical stimuli, or mathematical expressions. ‘When the experimental data or other scientific problems to be analyzed are comparatively extensive, itis often convenient to organize several five-choice completion questions into sets, that is, direct each question in a set to the same material. This practice allows you to answer several questions based on the same material. In no case, however, is the answer to ‘one question necessary for answering a subsequent question correctly. Each question in a set is independent of the others and refers only to the material given for the entire set. Directions: Each of the questions or incomplete statements below is followed by five suggested answers or completions. Select the one that is best in each case and then fill i the corresponding circle on the answer sheet. ‘The hydrogen ion concentration of a solution prepared by diluting 50 milliliters of 0.100-molar HNO, with water to 500 milliliters of solution is (A) 0.0010 M (B) 0.0050 M (© 0.010M (D) 0.050 M (©) 10M Choice (C) is the correct answer to question 7. This is a question that concerns solution concentrations. One way to solve the problem is through the use of ratios. In this question, a solution of nitric acid is diluted 10-fold; therefore, the concentration of the solution will decrease by a factor of 10, that is, from 0.100-molar to 0.010-molar. Alternatively, you could calculate the number of moles of H ions present and divide this value by 0.50 liter: (0.100 X 0,050)/0.5 = M of the diluted solution. D282 Chemistry Liquid Mercury 8. The bulb of the open-end manometer shown above contains a gas. True statements about this system include which of the following? 1. Only atmospheric pressure is exerted on the exposed mercury surface in the right side of the tube. II, The gas pressure is greater than atmospheric pressure. IIL, The difference in the height, h, of mercury levels is equal to the pressure of the gas. (A) Tonly (B) [ll only (©) [and II only (D) [and Il only (BE) 1 Ul, and tt Choice (C) is the correct answer to question 8. This is a laboratory-oriented question pertaining to the measurement of gas pressures. It demands higher-level analytical skills that involve drawing conclusions from results obtained in an experiment. To answer this question correctly, you must first understand that, in an open type of manometer, the air exerts pressure on the column of liquid in the open side of the U-tube and the gas being studied exerts pressure on the other side of the U-tube. It is clear then that Statement I is true since the data given show that the manometer is open-ended and its right side is exposed to the atmosphere. Statement II is also a true statement because the level of liquid mercury is higher in the right side, which is exposed to the atmosphere, than in the left side, which is exposed to the gas. Thus the gas pressure is greater than atmospheric pressure. Statement III is not a correct statement because the pressure of the gas in the bulb, expressed in millimeters of mercury, is equal to the difference in height, hr of the two mercury levels, plus the atmospheric pressure. Thus only Statements I and Mare correct. 283 ‘The Official Study Guide for All SAT Subject Tests 9, A thermometer is placed in a test tube containing a melted pure substance. As slow cooling occurs, the thermometer is read at regular intervals until well after the sample has solidified. Which of the following types of graphs is obtained by plotting temperature versus time for this experiment? wes eg 2 2 NN E a 5 é é Time eg (DE é é Time Time) @e+ Z E \ 3 “Time Choice (B) is the correct answer to question 9. This is a question on states of matter. You must convert the description of the physical phenomenon given in the question to graphical form, When a liquid is cooled slowly, its temperature will decrease with time. ‘Thus the first portion of a graph depicting this phenomenon must show a decrease when temperature is plotted against time. When a pure liquid substance reaches its fusion (melting) point, continued cooling will release heat with time as the substance solidifies. During this period there is no drop in temperature. After the substance has completely solidified, further cooling will cause an additional drop in temperature. The only graph shown that accurately depicts the events described is (B), which is the answer. Cu*(aq) +... (ag) >... Cul(s) +... 1,9) 10, When the equation above is balanced and all coefficients are reduced to lowest whole- number terms, the coefficient for I (aq) is “ (8) © (@) (E) D284 Chemistry Choice (D) is the correct answer to question 10. This question pertains to the balancing of chemical equations. In order to answer this question correctly, you need to recognize that both mass and charge must be conserved in any chemical equation. With this in mind, the chemical equation is correctly written as 2.Cur(aq) +41 (aq) > 2 Cul) + 1,0) ‘The coefficient for I (aq) is 4. 11. From their electron configurations, one can predict that the geometric configuration for which of the following molecules is NOT correct. (A) PF, trigonal planar (B) CF, tetrahedral (© CHCL, irregular tetrahedron (D)_ OF, bent (v-shaped) (E) HF linear Choice (A) is the correct answer to question 11. This is a question on chemical bonding and requires you to apply the principles of molecular bonding. Each of the molecules given is correctly paired with the term describing its molecular geometry except choice (A). The geometry of PF, is not trigonal planar, but trigonal pyramidal, because this geometry corresponds to 2 maximum possible separation of the electron pairs around the central atom, phosphorus, and therefore yields the most stable configuration; the central atom of the molecule is surrounded by three single bonds and one unshared electron pair. Thus, the correct answer is choice (A). Note that this is the type of question that asks you to identify the one solution to the problem that is inappropriate. ++ $0,(g) +...0,(—) >? 12, According to the reaction above, how many moles of $O,(g) are required to react completely with 1 mole of O,(g)? (A) 0.5 mole (B) 1 mole (© 2 moles (D) 3 moles (E) 4 moles Choice (C) is the correct answer to question 12. ‘This is a question on descriptive chemistry that also tests your ability to balance chemical equations. The correct answer to this question depends first on your knowing that the combustion of sulfur dioxide, $0,(g), produces sulfur trioxide, SO,, The stoichiometry of the correctly balanced equation indicates that 2 moles of SO,(g) are needed to react completely with 1 mole of ,(g) to form 2 moles of SO, 285) ‘The Official Study Guide for All SAT Subject Tests 13, Analysis by mass of a certain compound shows that it contains 14.4 percent hydrogen and 85.6 percent carbon. Which of the following is the most informative statement that can properly be made about the compound on the basis of these data? (A) Itis a hydrocarbon. (B) Its empirical formula is CH, (©) Its molecular formula is C,H. (D) Its molar mass is 28 grams. (2) Itcontains a triple bond. Choice (B) is the correct answer to question 13. This is a question on stoichiometry that tests the important skill of scientific reasoning based on experimental evidence. The question states that 100 percent of the composition of the compound analyzed can be accounted for with the elements hydrogen and carbon, Thus, this compound is a hydrocarbon and choice (A) is a correct statement. It is not the correct answer to the question, however, because you can deduce more specific conclusions about this compound from the information given. The relative percentage composition provides evidence that the atomic ratio of carbon to hydrogen in the compound must be 85.6/12.0 : 14.4/1.0 or 1:2. Therefore, you can conclude that the empirical formula for the compound is CH,,a hydrocarbon. Thus choice (B) isa better answer than choice (A). Since you do not Know the total number of moles of the compound used for analysis, you cannot calculate the molar mass or derive the molecular formula for this compound. Thus choices (C) and (D) cannot be determined from the information given and so they are not correct answers to the question, It is known, however, that a substance with an empirical formula of CH, cannot have a triple bond. Therefore, choice (E) is incorrect. > 286 Chemistry Test Practice Helps The test that follows is an actual, recently administered SAT Subject Test in Chemistry, To get an idea of what it's like to take this test, practice under conditions that are much like those of an actual test administration Set aside an hour when you can take the test uninterrupted. Make sure you complete the test in one sitting. Sit at a desk or table with no other books or papers. Dictionaries, other books, or notes are not allowed in the test room. Tear out an answer sheet from the back of this book and fill it in just as you would on the day of the test. One answer sheet can be used for up to three Subject Tests. Read the instructions that precede the practice test. During the actual administration you will be asked to read them before answering test questions. Time yourself by placing a clock or kitchen timer in front of you. After you finish the practice test, read the sections “How to Score the SAT Subject Test in Chemistry” and “How Did You Do on the Subject Test in Chemistry?” The appearance of the answer sheet in this book may differ from the answer sheet you see on test day. 287 CHEMISTRY TEST ‘The top portion of the section of the answer sheet that you will use in taking the Chemistry Test must be filled in exactly as shown in the illustration below. Note carefully that you have to do all of the following on your answer sheet 1. Print CHEMISTRY on the line under the words “Subject Test (print).” 2. In the shaded box labeled “Test Code” fill in four circles: —Fill in circle 2 in the row labeled V. —Fill in circle 2 in the row labeled W. —Fill in circle 4 in the row labeled X. —Fill in circle D in the row labeled Y. LS ‘Subject Test (print) ¥ QDeDOOOOOD w DQD@DOOOOO9D CHEMISTRY xDDD WOOO oe DODO DOOOD 3, Please answer the questions below by filling in the appropriate circles in the row labeled Q on the answer sh The information you provide is for statistical purposes only and will not affect your Question | How many semesters of chemistry have you taken in high school? (If you are taking chemistry this Semester, count it as a full semester.) Fill in only one circle of circles 1-3. © One semester or less. Fill in circle 1 # Two semesters, —Fill in circle 2, ‘© Three semesters or more Fill in cirete 3, juestion IL How recently have you studied chemistry’? © Lam currently enrolled in or have just completed a chemistry course. ‘© Thave not studied chemistry for 6 months or more. Fill in cirete 5, Fill in cirele 4. Question IIL Which of the following best describes your preparation in algebra? (If you are taking an algebra course this semester, count it as a full semester.) Fil in only one circle of circles 6-8. © One semester or less Fill in cirele 6, * Two semesters —Fillin cirele 7, ‘© Three semesters or more Fill in circle 8. Question LV ‘Are you currently taking Advanced Placement Chemistry? If you are, fill in cirele 9. ‘When the supervisor gives the signal, turn the page and begin the Chemistry Test. There is total of 85 questions in the Chemistry Test (1-70 plus questions 101-115 that must be answered on the special section at the lower left-hand commer of the answer sheet) Unautried copying trou] amy art ofthis page ta iega D288 289) = SLNANATA FAL AO ATAVL O1GOMAd : “MOO WOU HOVLAG LON OG 2) Gee | CT] eR | CT] ET] RY] HRT | HD | woLRe | oa | OTE] eT 1 wy aq ow) wy ng dN) o souag aprunoy | 2 sot oor us| 96 | so | v6 | o6 | v6 3 Tort Tower ac aT TOAST] STUST] LOIS] FOST | rN | eR a wy sa QL) PD) Ma) ws wa) PN pure _ 69 89. $9 +9 £9 wo 19 09 & TET MELT] HT | HET] SUT | TEOCT YEH | CT] MOET | RTE eOURE] ETT a 2 a 8) 8] 8 jaw) sa) ya] 3s) aa | gr) ev) er aa nH Z cut ot | sor | cor_| oo | sor| ror | os | ss | us Bl 2 Pee PTD] over nome | eae RET HF ME | Co oOT Teor] aor | 1e9eT | HEAT GERET | To eT] EE LeT] ToceT Ss jf MW 3H ny ar) so.) am | JH eT. | ef | 89 ron og_| se wo | of |u| | | u_| us | _o | cs Be fmisifioser WHIT] WeeIT| GeumtPerooT | Toror| TrOr | Tao) Hose | TTS | cers | Toe | GOWN | Ors a a x I ul po | BV] Pa} ua) MM OL | OW) GN | 3z A AS | aa a Fe ts | es oy | ar | ow | o» | sp | ow | ce | cm | wm | o | oe | ae | oe = fare Power WO] EH | I] HH] CO] HT [MOS] OOS | FOOE | OGLE | DOPE | ROOF | OTe m2 ay | sa ep uz a9) Nw] od) ad mW] 49] A | EL | og | 8D O38 ve _| se ie | oe | oe | we | oz | oe | ce | ve | ee | ce | i | oz 2 wos eeree 39 wre 9 av Wv aw é 8L £1 al 2 fare Tsar z10 a aN a ag e oL 6 s b £ 2 er Cr z Hn H 3 z a < EXAMINATION. CHEMISTRY TEST Note: For all questions involving solutions, assume that the solvent is water unless otherwise stated. Throughout the test the Following s enthalpy molar ‘number of moles, pressure molar gas constant entropy = temperature volume sNomtsen bols have the definitions specified unless otherwise noted, atm = atmosphere(s) 2 = gram(s) J = joule(s) KJ = kilojoutets) L = texts) mL = millilicers) mm = millimeter(s) mol = mole(s) Vo = volt(s) Directions: Each set of lettered choices below refers to the numbered statements or questions immediately fol- lowing it, Select the one lettered choice that best fits each statement or answers each qu swer sheet. A choice may be used corresponding circle on the at ‘Questions 1-3 refer to the following pieces of Iaboratory equipment. (A) Condenser (B) Funnel © Pipet (D) Balance (E) Barometer 1, Commonly used to transfer an exact volume of liquid from one container to another 2. Commonly ced in a distillation setup 3. Commonly used in a filtration setup 3YAC2 tion and then fill in the ice, more than once, oF not at all in eacl Questions 4-6 refer to the following information. Na,Cr0,, a soluble yellow solid PbCrO4, an insoluble yellow solid NaNO 3, soluble white solid PD(NO,)>,a soluble white solid (A) Yellow solid and colorless solution (B) Yellow solid and yellow solution (C) White solid and colorless solution (D) No solid and yellow solution (B) No solid and colorless solution 4, Observed when 1.0 mol of Na,CrO, and 2.0 mol of Po(NOs)s are mixed with I L of water 5. Observed when 3.0 mol of NasCrOq and 1.0 mol of Pb(NO,), are mixed with IL of water 6. Observed when 1.0 mol of NaNO, and 1.0 mol of PO(NOs): are mixed with IL of water (GO ONTO THE NEXT PAGE Q CHEMISTRY TEST—Continued Questions 7-9 refer to the following. (A) Reduction potential (B) Ionization energy (ionization potential) (C) Blectronegativity (D) Heat of formation (E) Activation energy 7. Is the energy change accompanying the synthesis ‘of a compound from its elements in their standard states 8. Isthe energy needed to remove an electron from a ‘gaseous alom in its ground state 9. Is the minimum energy needed for molecules to ct and form products Questions 10-13 refer to the following pairs of substances. (A) NH and N3Hy, «B) 60 and "0 (C) NH,CI and NH,NO3 (D) CH,OCH, and CH,CH,OH () 0, and O5 10. Are isotopes IL. Have both ionic and covalent bonds 12, Are allotropes 13, Are strong electrolytes in aqueous solution Questions 14-17 refer to the following subshells. (A) Is (B) 2s (©) 3s (D) 3p (E) 3a 14. Contains up to ten electrons 15, Contains one pair of electrons in the ground-state electron configuration of the lithium atom 16, Is exactly one-half filled in the ground-state electron configuration of the phosphorus atom 17. Contains the valence electrons in the ground-state electron configuration of the magnesium atom Questions 18-20 refer to the following gases. (A) 0, @) 0, © co ©) Ch, (©) SO; 18, Contributes to acid rain 19. In the stratosphere, sereens out a large fraction of | ultraviolet rays from the Sun 20. Is a product of the incomplete combustion of hydrocarbons (GO ONTO THE NEXT PAGE 291 CHEMISTRY TEST—Continued Questions 21-24 refer to the lettered solutions in the laboratory schemes represented below. 0.1. MHCI 0.1. M NaOH Hydrochloric Acid Sodium Hydroxide © Mix equal volumes 0.1 M NaOH m Hydroxide 21. Has a hydroxide ion concentration of 10°7 M at 298 K 22. Has the highest pH at 298 K 23. Has a pH greater than 7, but less than 13 at 298 K 24, Has a pH greater than 2, but less than 7 at 298 K (seorronseneAT mas SNTOTHE NENT Pace > D292 wy CHEMISTRY TEST—Continued PLEASE GO TO THE SPECIAL SECTION LABELED CHEMISTRY AT THE LOWER LE! HAND CORNER OF THE PAGE OF THE ANSWER SHEET YOU ARE WORKING ON AND. ANSWER QUESTIONS 101-115 ACCORDING TO THE FOLLOWING DIRECTIONS. Part B Directions: Each question below consists of two statements, In the left-hand column and IL in the right-hand column. For each question, determine whether statement [is true or false and whether statement Iis true or false and fill in the corresponding Tor F circles on your answer sheet. Fill in citcle CE only if statement Iisa comect explanation of the true statement I EXAMPLES: 1 u EX 1, H,S0, isastrong acid BECAUSE —H,SO, contains sulfur. EX 2, Anatomof oxygen is BECAUSE an oxygen atom contains an equal electrically neutral number of protons and electrons. cies cE] PLE ANSWERS lex O lexa e 1 101. CyHy and CH have the same chemical and BECAUSE CyH and C,H, have the same percentages physical properties bby mass of hydrogen, 102. The melting of ice is an exothermic process BECAUSE water has a relatively high specific heat capacity 103. A2g sample of nitrogen and a2 g sample of | BECAUSE equal masses of gaseous substances contain ‘oxygen contain the same number of molecules the same number of molecules. 104, When an atom absorbs a photon of visible BECAUSI light, one of its electrons is promoted to a higher energy state an electron has a negative charge. 105. ‘The alkali metals are very good reducing BECAUSE the alkali metals are easily oxidized agents 106. A 1.0 g sample of calcium citrate BECAUSE. there are more Ca atoms in 1.0 mol of Cay(CgHsO)> (molar mass 498 g/mol), calcium carbonate than in 1.0 mol of calcium contains more Ca than a 1.0 g sample of citrate. calcium carbonate, CaCO, (molar mass 100 g/mol), 107. The water molecule is polar BECAUSE the radius of an oxygen atom is greater than that of a hydrogen atom. (GO ONTOTHE NEXT PAGE > 293 any ar of hs pose Hegel ® All indicators are colorless in neutral solution 109. A 1M sucrose solution and a 1M NaCl solution have the same freezing point 110, The average kinetic energy of gas molecules increases as the temperature increases IIL. When a cone should be added ated acid is diluted, the acid lowly to the water 112. Methane, CH,, is very soluble in water 113. A I mol sample of electrons is required to reduce 0.5 mol of chlorine gas to chloride ions 114. In 0.1 M acetie acid, [H*] is smaller than [H*] is in 0.1 M hydrochloric acid 113. A fluoride ion, F> and an oxide ion, O*-, hhave the same diameter RETURN TO THE SECTION OF YOUR AN! Unauthorised copying of ruse of ofthe page ga D294 CHEMISTRY TEST—Continued BECAUSE. BECAUS BECAUSE. BECAUS BECAUSE BECAUSE ‘AUSE, BECAUSE. u \dicators develop color only in the presence of a strong acid of a strong base. 1A sucrose solution and a 1M NaCl solution contain the same number of solute particles per liter of solutio the average speed of gas molecules decreases as the temperature increases. if water is added to a concentrated acid, violent splattering might occur. water molecules form hydrogen bonds with methane molecules. chlorine molecules are diatomic and the charge on the chloride ion is —1 ‘a molecule of acetic acid contains more atoms than docs a molecule of hydrogen chloride the fluoride ion, F~, and the oxide ion, 0, have the same number of electrons. VER SHEET YOU STARTED FOR CHEMISTRY AND ‘GO ONTOTHE NEXT PAGE ) CHEMISTRY TEST—Continued ® Part C Directions: Each of the questions or incomplete statements below is followed by five suggested answers or completios H,S(g) +. .O,(g) -.-H,O(g) +... 80,¢2) sof 25. When 2 mol of H3S(g) react with an exe oxygen according to the equation above, how ‘much H,0(g) is produced? (Equation balanced.) (A) L mol (B) 2 mol (C) 3 mol (D) 4 mol () 6 mol 26. Increasing the temperature of a gas in a rigid closed. container increases Which of the following? I. The pressure of the gas I, The average speed of the gas molecules IIL The mass of the gas (A) Lonly (B) Honly (C) Land Il only () Hand Hl only ©) LM, and 27. The number of electrons in '3§Sn?* is 2 (B) 48 © 50 (Dy 52 68 Select the one that is best in each case and then fill in the corresponding circle on the answer sheet. 28. When two colorless liquid reagents are mi Which of the following observations would suggest that a chemical reaction has occurred? 1, Formation of a precipitate IL. A color change TIL. Appearance of gas bubbles (A) Lonly (B) ULonly (©) Land Il only (D) Mand HI omy (©) Land ttt (A) FB: F (B) :F:P:E E Os PE F; OBE a (E) BBE FE (GO ONTO THE NEXT PAGE 295) 30. Which of the following is a transition element? (A) Iron (B) Carbon (©) Potassium (D) Tin () Radium 31. When 50. mL of 1.5 M NaCl(ag) is diluted with pure water to a final volume of 150. mL. wi the molarity of the resulting solution? (A) 0.10. M (B) 050M 15M (asm ©) 50M 32. A 40.0 g sample of a hydrated salt was heated Until all the water was driven off. The mass of the solid remaining was 32.0 g. What was the percent cof water by mass in the original sample? (A) 13.0% (B) 20.0% (©) 25.0% (D) 75.0% (E) 80.0% 33. A solution that has pH of 6.0 is (A) strongly basic (B) slightly basic (©) neutral (D) slightly acidie (E) strongly acidic 34. Which of the following molecules is a saturated hydrocarbon’? (A) CH (B) CoH, ©) CHACI (p) Cc, (B) CO, Unauthorised copying of ruse of ofthe page ga D296 CHEMISTRY TEST—Continued -2+Fej04(9) +. CO(g) > «.. Fels) +. COXg) 35. When the equation above is balanced and all the coefficients are reduced to lowest whole-number terms, what is the coefficient for Fe;03(s) ? wl (By 2 3 w)4 5 36. In which of the following compounds does. nitrogen have an oxidation number of +5 ? (A) HNO, (B) Ny (©) NO, (D) N,0 (£) NHOW 37. If both NaOH and KOH were the same price per kilogram, it would be cheaper to use NaOH. to neutralize a quantity of acid because NaOH. (A) weighs less per mole than KOH (B) weighs more per mole than KOH. (C) neutralizes more acid per mole than KOH (D) neutralizes less acid per mole than KOH (B) is less dense than KOH 38. When a given amount of Ca(OH), is completely neutralized with H,SO,, which of the following is the mole ratio of Ca(OH), t0 H,SO, in this reaction? Aid (B) 1:2 tl (B) 2:1 (B) 41 ‘GO ONTOTHE NEXT PAGE ) CHEMISTRY TEST—Continued 39. Factors that influence whether or not two Vapor Pressure of colliding molecules will react include which of ‘Temperature °C) Ethyl Alcohol (mm H; the following “o 350 I. The energy of the collision 0 538 IL The orientation of the molecules 80 813 IIL. The size difference between the reactant and 90 1,182 product molecules 100 1,698 (A) Lonly 42. The barometric pressure on Pikes Peak (B) Monty (14,109 fect) in Colorado averages 455 mm Hg. (©) Land I only From the table above, one can conclude that the (D) [and MI only boiling point of ethyl alcohol at this altitude (©) Ul, and I would be (a) 100°C 2SOAg) + Og) @ 280K) (B) between 90°C and 100°C (©) between 80°C and 90°C 40. What is the expression for the equilibriut (D) between 70°C and 80°C constant, K,,, for the reaction represented (B) between 60°C and 70°C above? seZn(s) + Hag) > 43. When the equation for the reaetion represented above is completed and balanced and all eoeffi- B0.}[0.) cients are reduced to lowest whole-number terms, the cocfficient for H"(ag) is [so.] + [02] (a2 {so,] (B)3 4 (D5 (©) 6 O Ky 44. Which of the following statements is true concerning a saturated solution of a salt at a constant temperature? 41. A solution contains 1.00 mol of glucose, (A) The concentrations of salt and solvent are S ne NH ek usually equal Cette, and 2.00 mol of ura, (NH2),C0. (B) The amount of dissolved salt is constant. in 7.00 mol of water, What is the mole Fraetion (C) Addition of solid salt shifts the equilibrium, of glucose in the solution? ‘which results in an inerease in the amount of dissolved salt & oss (D) The solution is unstable and sudden erystal- ©) 0.200 lization could occur. (D) 0.333 (E) At the same temperature, a saturated solution (©) 0.500 (of any other salt has the same concentration, ‘GO ONTOTHE NEXT PAGE ) 297 2CO(g) + OXg) + 2 COQ) 45. According to the reaction represented above, 1.00 mol of CO(g) reacts at O°C and 1 atm to consume how much O;(«)? (A) 32.08, (B) 12 (©) 2.4L (D) 1.00 mot (E) 2.00 mol 46. Species that in water can either accept or donate protons include which of the following?” 1 CH, UL. HCO~ UL HPO, (A) Lonly (B) ILonly (©) only (D) Mand IM only (&) 1. and MT 47. The ionization energies of Li and Hare 520 kimol and 1,312 ki/mol, respectively. ‘The ionization energy of He is (A) 496 ki/mol (B) 656 ki/mol (©) 899 ki/mot (D) 1,086 1J/mot (B) 2.372 kJ/mol 448, An active ingredient in common houschold bleach solutions is most likely to be which of the following? (A) NaCl (B) Naclo, (©) NaHCO, (D) Na,So, (E) HCHO, Unauthorised copying of ruse of ofthe page ga D298 S) CHEMISTRY TEST—Continued ana bint 49, The missing product in the nuclear reaction represented above is «tH (B) jie (©) fe (D) 4Li (BE) SLi HK) + HOW > H,O%ag) + Cag) 50. All of the following statements are correct for the reaction represented by the equation above EXCEPT: (A) 1,0 is the conjugate acid of HO. (B) CI isthe conjugate base of HCI. (©) HO is behaving as a Bronsted-Lowry base. (D) HCL is a weaker Bronsted-Lowry acid than #0. (E) The reaction proceeds essentially to completion ‘GO ONTOTHE NEXT PAGE ) CHEMISTRY TEST—Continued Pam) | 2 1 os | 04 vay) | 100 | 200 | 400 | 500 Tix) | 200 | 200 | 200 | 200 SI. The data given in the table above describe the behavior of a sample of gas. Which of the following empirical laws does the data illustrate? (& is a constant.) AT at constant V (A) P= Pi + Py + Py +... at constant V and 7 © P=E scott? S = KT at constant P > " for (number of moles) at constant Vand T 52. Of the following, which is an example of an oxidation- reduction reaction? (A) Fe(s) + Snag) — Snis) + Fe*(ag) (B) HCOy (aq) + OHMag) > CO; ag) + HOW (©) Po?(ag) + 21ag) > PbIyGs) (D) HCI) + NHY@) > NH,CIG) (©) Ba?*(ag) + Mn0,2°(aq) > BaMnO,(s) Ng) + 3H) @ ZNHYe) + heat 53. Which of the following statements about the reaction represented above is true? (A) The forward reaction is endothermic. (B) A 28 g sample of Nx(g) reacts completely with a 3 g sample of H3(«), (©) NHy(g) will dissociate into equal masses of No(g) and ints occupy a smaller volume than the products e temperature and pressure. (E) The equilibrium concentration of ammonia is affected by a change in temperature, ahora copying or ease GO ONTO THE NEXT PAGE sry pr ofthis page Me 299 ® ‘54. The element carbon is the chief constituent of all of the following EXCEPT (A) coal (B) glass. (©) diamond (D) charcoal (E) graphite CHE! 55. ALO®C and 1.0 atm, the density of approximately (A) 0.80 gi &) Log (C) 13 ef () 25 yt. (&) 28 aft 56. Which of the following contains a weak organic id? (A) Vinegar (B) Hydrogen peroxide (C) Baking soda (D) Freon gas (©) Ammonia =:P\Ojo(s) +. HOW) > ... HyPO(ag) 57. When | mol of P,O,9(s) reacts completely with water to produce H,PO,(ag) according to the reaction represented by the unbalanced equation above, the number of moles of H,0()) consumed is (A) I mol (B) 3mol (©) 4mot (D) 6 mol () 12 mol 58. Increased randomness results under which of the following conditions? 1. ALL sample of He(g) and a 1 L sample of Ne(g) are mixed in a 2 L flask. I. Tee melts. IIL. CaQ(s) reacts with CO,(g) to form CaCOy(9. (A) Lonly (B) Ionly (© Land Il only (D) I and iff only. (©) 1M and UL »300 TRY TEST —Continued 58, 60. 61 62. ® CH) + 8 O,g) > SCO) + 6H,01) According to the balanced equation above, when 4mol of Ox(g) react completely with CsH) (0, which of the following is true? (A) L mol of CsH)a()) must (B) 2mol of CsHj,(l) must react. (©) 3mol of HOM) must be formed. (D) 12 mol of H,0(0) must be formed, (E)_ 5 mol of COs(g) must be formed. ‘Truc statements about transition metals include which of the following? 1, Most can exhibit more than one stable oxidation state IL. Their compounds are often colores IL. Their ions have partially filled p-orbitals (A) Lonly (B) Ios (© Tand Tl only (D) I and If only © 1,0 and mt ‘The molarity of solution X is to be determined by 4 tration procedure. To carry out this procedure, all of the following must be known EXCEPT the (A) equation for the chemical reaction that occurs during the titration (B) volume of solution X that is used (C) mass of solution X that is used (D) volume of the solution that reacts with X (B) molarity of the solution that reacts with X ‘The primary intermolecular attraction that makes it possible (o liquefy hydrogen gas is called (A) London dispe (B) dipole-dipole attraction (C) covalent bonding. (D) ionic bonding (B) hydrogen bonding GO ONTOTHE NEXT PAGE > CHEMISTRY TEST—Continued © ‘Questions 63.65 Mgi) + 2H* > Mg?" + Hye) ‘A student performed an experi the amount of hydrogen gas released in a re ‘The studemt produced the hydrogen gas by reacting hydrochloric acid and a strip of magnesium metal according to the equation above. All of the magnesium ‘metal was consumed and the hydrogen gas was collected by displacement of water in an inverted bottle. The student's data contain the following information. Mass of Mg on Volume of gas collected over water, Water temperature, 22.0°C Room temperature... 22.0°C Atmospheric pressure. 749.8 mm Hg ‘Vapor pressure of water at 19.8 mm Hg. 63. What number of moles of magnesium was used’? (A) 5.8 107! mol (B) 3.0 107 mol (©) 24107 mol (D) 14107 mol (8) 1.0107 mot any pat of page Megat 64. Why is it essential 10 know the water temperature in this experiment? 1, To find the vapor pressure of the water UL. To control the rate of reaction IML. To make sure that the reaction goes «0 completion (A) Lonly (B) Honly (©) Land Ill only (D) Mand Hl only (P) I, Mand ith 65. The volume of the dry hydrogen gas at | atm and room temperature would be (ay) 25200498 +198) (B) C52) 760 = 19.8) mL 0) ORE ® (760 — 19.8) L (749.8252) ™ GO ONTO THE NEXT PAGE > 301) ® CHEMISTRY TEST—Continued H,(g) + Fy(g) 3 2 HP(@) + 537.6 KI 66. £0.10 mol of HF(g) is formed according to the reaction represented above, approximately how ‘much heat is evolved? A) BK (B) 27K © sa (D) 110K (©) 20K 67. A chemical reaction is used to separate a mixture into separate substances in which of the following situations? (A) Pure water is obtained from ocean water by evaporating the water and condensing it (B) Iron filings are separated from sand by the use of a magnet. (C) Iron metal is produced from ore containing iron(IIP) oxide. (D) Plant pigments in a solution are separated by the use of paper chromatography (E) Sand is obtained from a sand-sugar mixture by adding water to dissolve the sugar. 68. If a compound has an empirical formula of CH, and a molar mass of 70 g/mol, which of the following is most likely to be its mol formula? (A) CoH, (B) CAH, © Cy (D) CsHs ©) Cy PCI(g) + energy 2 PCIYg) + Cly(2) 69. The system above is at equilibrium in a closed ‘container. Which of the following would inerease the amount of PCI, in the system? (A) Decreasing the pressure of the system at constant temperature (B) Lowering the temperature at constant pressure (C) Adding a catalyst (D) Adding some C1,(g) to the reaction vessel (E) Removing some PCIs(g) from the reaction vessel 70. Which of the following terms gives a qualitative rather than a quantitative description of the concentration of a solution? (A) Molality (B) Mass percentage (©) Dilute (D) Mole fraction (B) Molarity STOP IF YOU FINISH BEFORE TIME IS CALLED, YOU MAY CHECK YOUR WORK ON THIS TEST ONLY. DO NOT TURN TO ANY OTHER TEST IN THIS BOOK. Unauthorised copying of ruse of ofthe page ga » 302 Chemistry How to Score the SAT Subject Test in Chemistry When you take an actual SAT Subject Test in Chemistry, your answer sheet will be “read” bya scanning machine that will record your responses to each question. Then a computer will compare your answers with the correct answers and produce your raw score. You get one point for each correct answer. For each wrong answer, you lose one-fourth of a point, Questions you omit (and any for which you mark more than one answer) are not counted. This raw score is converted to a scaled score that is reported to you and to the colleges you specify. Worksheet 1. Finding Your Raw Test Score STEP 1: Table A lists the correct answers for all the questions on the Subject Test in Chemistry that is reproduced in this book. It also serves as a worksheet for you to calculate your raw score. + Compare your answers with those given in the table, + Puta check in the column marked “Right” if your answer is correct. + Puta check in the column marked “Wrong” if your answer is incorrect. + Leave both columns blank if you omitted the question. STEP 2: Count the number of right answers. Enter the total here: STEP 3: Count the number of wrong answers. Enter the total here: STEP 4: Multiply the number of wrong answers by .250. Enter the product here: STEP 5: Subtract the result obtained in Step 4 from the total you obtained in Step 2. Enter the result here: STEP 6: Round the number obtained in Step 5 to the nearest whole number. Enter the result here: The number you obtained in Step 6 is your raw score. 303 ‘The Official Study Guide for All SAT Subject Tests Table A Answers to the Subject Test in Chemistry, Form 3YAC2, and Percentage of Students Answering Each Question Correctly ed eed aes cers pores ee 2 A 1 34 A 88 3 8 6 8 A 60 4 A 8 36 A 6 5 8 31 37 A 54 6 E 59 36 c 68 7 Do a 39 c 55 a 8 6 40 8 66 9 E 7 u A 60 10 8 a a E 13 it c 52 3 A 58 2 E 38 4a 8 a4 3 c 48 45 8 50 0 E 68 46 D 55 6 A St a E 42 16 D 7 48 8 33 v c cy 43 c 68 8 E 6 50 0 53 18 A a 51 c u 20 c we 52 A 53 2 C 8 53 E 8 2 8 48 5a 8 n 23 E 40 55 c 2 4 0 5 56 A 63 25 8 85 57 D m0 26 c 8a 58 c me n 8 a 59 c 16 28 E eB 60 c a 2 E 68 6 c 45 30 A 8 62 A B 3 8 Cy 3 E 52 2 8 eB 64 A a »304 Table A continued on next page Chemistry Table A continued from previous page poetry eee preter Question | Correct the Question | Question reget rete peer ees Right | wrong 65 2B 106 FF 2 66 8 ww 1 81 87 81 108 FF 98 68 7 109 FF 7 69 37 0 ik ai 70 8 wn | Tce @ 101 40 12 a 102 a 113 45 103 6a 114 % 104 58 115 36 105 a + These percentages are based on an analysis ofthe answer sheets ofa representative sample of 5,571 students ‘who took the original form of this test in November 2002, and whose mean score was 593. They may be used as aan indication of the relative difficulty of « particular question. Each percentage may also be used to predict the .lihood that a typical SAT Subject Test in Chemistry candidate will answer that question correctly on this edition ofthe test 305 ‘The Official Study Guide for All SAT Subject Tests Finding Your Scaled Score When you take SAT Subject Tests, the scores sent to the colleges you specify are reported on the College Board scale, which ranges from 200-800. You can convert your practice test score to a scaled score by using Table B. To find your scaled score, locate your raw score in the left-hand column of Table B; the corresponding score in the right-hand column is your scaled score, For example, a raw score of 39 on this particular edition of the Subject Test in Chemistry corresponds to a scaled score of 590. Raw scores are converted to scaled scores to ensure that a score earned on any one edition of a particular Subject Test is comparable to the same scaled score earned on any other edition of the same Subject Test. Because some editions of the tests may be slightly easier or more difficult than others, College Board scaled scores are adjusted so that they indicate the same level of performance regardless of the edition of the test taken and the ability of the group that takes it. Thus, for example, a score of 400 on one edition of a test, taken ata particular administration indicates the same level of achievement as a score of 400 on a different edition of the test taken at a different administration, When you take the SAT Subject Tests during a national administration, your scores are likely to differ somewhat from the scores you obtain on the tests in this book. People perform at different levels at different times for reasons unrelated to the tests themselves. ‘The precision of any test is also limited because it represents only a sample of all the possible questions that could be asked. » 306 Chemistry Table B Scaled Score Conversion Table Subject Test in Chemistry (Form 3YAC2) Pc a 8 800 43 640 84 800 8 630 88 800 a7 630 82 800 48 620 81 790 45 620 80 730 4 10 19 780 43 610 78 780 2 600 1 70 a 600 16 0 40 590 5 760 ww 590 m 70 38 580 3B 750 a 580 2 750 36 570 n 740, Fa 570 70 740 34 560 69 730 2 560 68 730 22 550 8 720 au 550 66 720 20 540 70 23 540 4 m0 28 530 63 m0 a 530 62 700 6 520 6 700 25 520 0 690 2 510 59 690 2 500 58 680 n 500 8 630 2 420 56 670 20 480 55 610 19 480 54 680 8 480 53 650 a 470 52 650 6 470 51 650 5 460 50 640 4 450 307 ‘The Official Study Guide for All SAT Subject Tests How Did You Do on the Subject Test in Chemistry? After you score your test and analyze your performance, think about the following question Did you run out of time before reaching the end of the test? If'so, you may need to pace yourself better. For example, maybe you spent too much time on one or two hard questions. A better approach might be to skip the ones you can't answer right away and try answering all the questions that remain on the test. Then if there’s time, go back to the questions you skipped. Did you take a long time reading the directions? You will save time when you take the test by learning the directions to the Subject Test in Chemistry ahead of time. Each minute you spend reading directions during the testis. a minute that you could use to answer questions, How did you handle questions you were unsure of? Ifyou were able to eliminate one or more of the answer choices as wrong and guess from the remaining ones, your approach probably worked to your advantage. On the other hand, making haphazard guesses or omitting questions without trying to eliminate choices could cost you valuable points. How difficult were the questions for you compared with other students who took the test? Table A shows you how difficult the multiple-choice questions were for the group of students who took this test during its national administration. The right-hand column gives the percentage of students that answered each question correctly. A question answered correctly by almost everyone in the group is obviously an easier question. For example, 79 percent of the students answered question 10 correctly. But only 40 percent answered question 23 correctly. Keep in mind that these percentages are based on just one group of students. They would probably be different with another group of students taking the test. If you missed several easier questions, go back and try to find out why: Did the questions cover material you haven't yet reviewed? Did you misunderstand the directions? » 308 Chapter 7 Physics Purpose ‘The Subject Test in Physics measures the knowledge you would be expected to have after successfully completing a college-preparatory course in high school. The testis not based on any one textbook or instructional approach, but concentrates on the common core of material found in most texts. Format This one-hour test consists of 75 multiple-choice questions. Topics that are covered in ‘most high school courses are emphasized. Because high school courses differ, both in percentage of time devoted to each major topic and in the specific subtopics covered, most students will find that there are some questions on topics with which they are not familiar. Content This test covers topics listed in the chart on the next page. 309) ‘The Official Study Guide for All SAT Subject Tests Topics Covered = Circular Motion, such as uniform circular mation and centripetal force: ‘Simple Harmonic Motion, such as mass on a spring, and the pendulum } such as the law of gravitation, orbits, Kepler's Laws Electric Fields, Forces, and Potentials, such as Coulomb's law, induced charge, nce, such as parallel-plate capacitors and transients ‘Sloman ond DUC uo oesiors nbs, ses pale networks, Ohms law, Joule’s law ‘Magnetism, such 2s permanent magnet, fields caused by currents, particles in magnetic fel, Faraday’s law, Lenz's law General Wave Properties, such as wave speed, frequency, wavelength, superposition, standing waves, Doppler effect Reflection and Refraction, such as Snel's law, changes in wavelength and speed Ray Opties, such as image formation using pinholes, micros, and lenses Physical Optics, such as single-sit diffraction, double-sit interference, polarization, calor thermal expansion Laws of Thermodynamics, such as first and second laws, internal energy, entropy, heat engine efficiency nomena, such as photons, photoelectic effect ‘Atomic, such as the Rutherford and Bohr models, atomic energy levels, atomic spectra ‘Nuclear and Particle Physics, such as radioactivity, nuclear reactions, fundamental particles Relativity, such as time dilation, length contraction, mass-energy equivalence General, such as history of physics and general questions that overlap several major Analytical Skills, such as graphical analysis, measurement, and math skills. Contemporary Physics, such as astrophysics, superconductivity, and chaos theory Approximate Percentage of Test rn cn D310 Physics How to Prepare ‘The test is intended for students who have completed a one-year introductory physics course at the college-preparatory level. You should be able to: + recall and understand the major concepts of physics and to apply these physical principles you have learned to solve specific problems + understand simple algebraic, trigonometric, and graphical relationships, and the concepts of ratio and proportion and apply these to physics problems. Laboratory experience is a significant factor in developing reasoning and problem- solving skills, This multiple-choice test can measure laboratory skills only in a limited way, such as data analysis. Familiarize yourself with directions in advance. The directions in this book are identical to those that appear on the test. Approximate Skills Specification srcentage of Test Recall Generally involves remembering and understanding concepts or information Recall and use of a single physical relationship In each of the six major content topics, some questions may deal oncept Prob Notes: (1) This test assumes that the direction of any current is the direction of flow of positive charge (conventional current). (2) Calculator use is not allowed during the test. (3) Numerical calculations are not emphasized and are limited to simple arithmetic. (4) This test predominantly uses the metric system. Score ‘The total score for each test is reported on the 200-to-800 scale. 3p ‘The Official Study Guide for All SAT Subject Tests Sample Questions ‘Two types of questions are used in the Subject ‘Test in Physics and are shown in the following samples. All questions in the test are multiple-choice questions in which you must choose the BEST response from the five choices offered. Classification Questions Each set of classification questions includes five lettered choices that you will use to answer all of the questions in the set (see sample questions 1-4). These choices appear before the questions in the set. In addition, there may be descriptive material that is relevant in answering the questions in the set. The choices may take various forms, such as words, phrases, sentences, graphs, pictures, equations, or data. The numbered questions themselves may also take such forms, or they may be given in the question format directly. To answer each question, select the lettered choice that provides the most appropriate response. You should consider all of the lettered choices before answering a question, The directions for this type of question state specifically that a choice cannot be eliminated just because itis the correct answer to a previous question. Because the same five choices are applicable to several questions, the classification questions usually require less reading than other types of multiple-choice questions. Therefore, classification questions provide a quick means, in terms of testing time, of determining how well you have mastered the topics represented. The set of questions may ask you to recall appropriate information, or the set may ask you to apply information to a specific situation or to translate information between different forms (descriptive, graphical, mathematical). Thus, different types of abilities can be tested by this type of question. Directions: Each set of lettered choices below refers to the numbered questions immediately following it. Select the one lettered choice that best answers each question and then fill in the corresponding circle on the answer sheet. A choice may be used once, more than once, or not at all in each set. D312 Physics Questions 1-2 Q Q ~\ Extreme Extreme Position Position A small sphere attached to the end of a string swings as a simple pendulum, The sphere moves along the arc shown above. Consider the following properties of the sphere. (A) Acceleration (B) Kinetic energy (© Mass (D) Potential energy (B) Velocity 1. Which property remains constant throughout the motion of the sphere? 2. Which property goes to zero and changes direction at each extreme position Q Choice (C) is the correct answer to question 1, To answer this question, you may know that in classical mechanics mass is a fundamental property of an object that does not depend on the position or velocity of the object. Alternately, you may realize that, since a pendulum during its motion repeatedly speeds up, slows down, and changes direction, the sphere’s velocity, kinetic energy, and acceleration must also change. Also, since the height of the sphere varies, so must its potential energy. Thus you can also obtain the answer by the process of elimination. Choice (B) is the correct answer to question 2. To answer this question, you must know some specific details about the motion of the pendulum, At each extreme position Q, the velocity and the kinetic energy (which is proportional to the square of the speed) are both zero, but kinetic energy has magnitude only and thus no direction to change. Velocity does have direction, and in this case the velocity of the sphere is directed away from the center, or equilibrium position, just before the sphere reaches Q, but directed toward the center just after leaving Q. The velocity changes direction at each point Q. The only other choice that has direction is acceleration, but acceleration has its maximum magnitude at each point Q and is directed toward the center, both shortly before and shortly after the sphere is at Q. 313) ‘The Official Study Guide for All SAT Subject Tests Questions 3-4 ‘The following graphs show the net force F on an object versus time f, for the object in straight-line motion in different situations. (A) F (B) F For each of the following speed v versus time t graphs for the object, choose the graph above with which it is consistent. 3. y olL_+r Questions 3 and 4 test the application of physical principles to information presented in graphical form. In each of these questions two concepts are involved. From Newton’s second law we know that the net force on an object is equal to the object's acceleration multiplied by the object’s mass, a constant. Thus graphs of acceleration versus time must, have the same shape as the graphs of force versus time that are given in the options. We must also know that at a particular time the acceleration of an object in its direction of, motion is equal to the rate of change of its speed, as determined by the slope of the speed v versus time f graph at that particular time, a Physics Choice (C) is the answer to question 3. The slope of the graph continually increases with increasing f; therefore, the object’s acceleration and consequently the net force on the object must also increase continually. The only graph that shows this relationship is graph (C). Choice (A) is the correct answer to question 4. In this question, the graph initially shows a constant speed, implying an acceleration and net force of zero. Then the curve sharply increases for a brief time, implying a large positive acceleration and large net force. Finally the curve returns to constant speed, implying a return to a zero net force. Graph (A) is the only choice that shows a force that varies in this manner. Five-Choice Completion Questions ‘The five-choice completion question is written either as an incomplete statement or as a question, In its simplest application, it poses a problem that intrinsically has a unique solution. It is also appropriate when: (1) the problem presented is clearly delineated by the wording of the question so that you choose not a universal solution but the best of the five offered solutions; (2) the problem is such that you are required to evaluate the relevance of five plausible, or scientifically accurate, choices and to select the one most pertinent; or 3) the problem has several pertinent solutions and you are required to select the one that is inappropriate or not correct from among the five choices presented. Questions of this, latter type (see sample question 6) will normally contain a word in capital letters such as NOT, EXCEPT, or LEAST. A special type of five-choice completion question is used in some tests to allow for the possibility of more than one correct answer. Unlike many quantitative problems that must by their nature have one unique solution, situations do arise in which there may be more than one correct answer. In such situations, you should evaluate each answer independently of the others in order to select the most appropriate combination (see sample question 7). In questions of this type, several (usually three) statements labeled by Roman numerals are given with the question. One or more of these statements may correctly answer the question. The statements are followed by five lettered choices, with each choice consisting of some combination of the Roman numerals that label the statements, You must select from among the five lettered choices the one that gives the combination of statements that best answers the question. In the test, questions of this type are intermixed among the more standard five-choice completion questions. The five-choice completion question also tests problem-solving skills. With this type of question, you may be asked to convert the information given in a word problem into graphical forms or to select and apply the mathematical relationship necessary to solve the scientific problem, Alternatively, you may be asked to interpret experimental data, graphs, or mathematical expressions. Thus, the five-choice completion question can be adapted to test several kinds of abilities. When the experimental data or other scientific problems to be analyzed are comparatively long, it is often convenient to organize several five-choice completion questions into sets, with each question in the set relating to the same common material that precedes the set (see sample questions 8-9). This practice allows you to answer several questions based on. 315) ‘The Official Study Guide for All SAT Subject Tests information that may otherwise take considerable testing time to read and comprehend. Such sets also test how thorough your understanding is of a particular situation. Although the questions in a set may be related, you do not have to know the answer to ‘one question ina set to answer a subsequent question correctly. Each question in a set can be answered directly from the common material given for the entire set. Directions: Each of the questions or incomplete statements below is followed by five suggested answers or completions, Select the one that is best in each case and then fill in the corresponding circle on the answer sheet. 40 ohms 120volts 20 ohms 5. If the internal resistance of the 120-volt battery in the circuit shown above is negligible, the current in the wire is (A) OA ®) 2A © 3A () 6A (©) 9A Choice (B) is the correct answer to question 5. In this question, you must apply two concepts to solve the problem. First, you must recognize that the two resistors are connected in series and thus are equivalent to a single resistor whose resistance is 60 ‘ohms, the sum of the two component resistances. Next, applying Ohm’s law, you will find that the current is given by the potential difference divided by this equivalent resistance. ‘Thus, the answer is 120-Yolts, which equals 2 amperes. 6. Alll ofthe following are vector quantities EXCEPT (A) force (8) velocity (©) acceleration (0) power (© momentum Choice (D) is the correct answer to question 6. This question isa straightforward question that tests your knowledge of vector and scalar quantities, A vector quantity is one that has both magnitude and direction, All five quantities have a magnitude associated with D316 Physics them, but only quantities (A), (B), (C), and (E) also have a direction. Power, a rate of change of energy, is not a vector quantity, so the correct answer is choice (D). 7. A ball is thrown upward. Air resistance is negligible. After leaving the hand, the acceleration of the ball is downward under which of the following conditions? 1. Onthe way up I, On the way down IIL, At the top of its rise (A) Tonly (B) Ulonly (© Tand I only (D) and IIT only () 1 U,and iL Choice (E) is the correct answer to question 7. In this question, one or several of the phrases represented by the Roman numerals may be correct answers to the question. One must evaluate each in turn, When the ball is on the way up, its speed is decreasing so the acceleration of the ball must be directed in the direction opposite to the ball’s velocity. Since the velocity is upward, the acceleration must be downward, making I correct. When the ball is on the way down, its speed is increasing, so its acceleration must be directed in the same direction as its velocity, which is downward. So II is also correct Finally, at the top of the rise, the ball has an instantaneous speed of zero, but its velocity is changing from upward to downward, implying a downward acceleration and making II correct also. A simpler analysis would be to realize that in all three cases, the ball is acted on by the downward force of gravity and no other forces. By Newton’s second law, the acceleration must be in the direction of the net force, so it must be downward in all. three cases. Since the phrases in I, II, and III are each correct answers to the question, the correct answer is choice (E). 317) ‘The Official Study Guide for All SAT Subject Tests Questions 8-9 In the following graph, the speed of a small object as it moves along a horizontal straight line is plotted against time. mod Speed (m/s) P33 45 6 Time(s) 8, ‘The magnitude of the acceleration of the object during the first 3 seconds is (A) 3 mis (8) 4mist (C) 6 mis? (D) 12 mis (8) 36 mis 9, The average speed of the object during the first 4 seconds is, (A) 1.9 mis (B) 3.0 m/s (© 4.0 mis (D) 6.0 mis (E) 7.5 m/s Questions 8 and 9 are a set of questions, both based on the graph provided. Choice (B) is the correct answer to question 8. To answer this question, you need to know that the magnitude of the acceleration is equal to the magnitude of the slope of a graph of speed versus time. In this situation, from time = 0 to time = 3 seconds, the graph has 12 mis 38 correct answer is choice (B). a constant slope of = 4m/s’, which is the magnitude of the acceleration. So the Choice (E) is the correct answer to question 9, The average speed of an object during a certain time is equal to the total distance traveled by the object during that time divided by the time. In question 9, the total distance traveled by the object during the first 4 seconds is equal to the area under the graph from time=0 to time = 4 seconds. This area is 5-(38)(12m/s) + (1s)(12m/s) = 18m+12m = 30m. The average speed is therefore 30m ‘. Wm =7.5 mis. pate Physics Test Practice Helps The test that follows is an actual, recently administered SAT Subject Test in Physics To get an idea of what it's like to take this test, practice under conditions that are much like those of an actual test administration. © Set aside an hour when you can take the test uninterrupted. Make sure you complete the test in one sitting. © Sitata desk or table with no other books or papers. Dictionaries, other books, or notes are not allowed in the test room. © Do not use a calculator. Calculators are not allowed for the Subject Test in Physics. © — Tear out an answer sheet from the back of this book and fill it in just as you would on the day of the test. One answer sheet can be used for up to three Subject Tests. © Read the instructions that precede the practice test. During the actual administration you will be asked to read them before answering test questions © Time yourself by placing a clock or kitchen timer in front of you. © After you finish the practice test, read the sections “How to Score the SAT Subject Test in Physics” and “How Did You Do on the Subject Test in Physics?” © The appearance of the answer sheet in this book may differ from the answer sheet you see on test day. 319) PHYSICS TEST ‘The top portion of the section of the answer sheet that you will use in taking the Physics Test must be filled in exactly as shown in the illustration below. Note carefully that you have to do all of the following on your answer sheet. 1. Print PHYSICS on the line under the words “Subject Test (print).” 2. In the shaded box labeled “Test Code” fill in four circles: —Fill in circle 2 im the row labeled V. —Fill in circle 3 in the row labeled W. —Fill in circle 3 in the row labeled X. the row labeled Y. aca ‘ea Test ri . 5090000 eee “ O00000 ° DOOO09 3. Please answer the three questions below by filling in the appropriate circles in the row labeled Q on the answer sheet. The information you provide is for statistical purposes only and will not affect your score ‘on the test Question | How many semesters of physies have you taken in high school, including any semester in which you are currently enrolled? (Count as two semesters any case in which a full year's course is taught in a one-semester {half-year) compressed schedule.) Fill in only one circle of cireles 1-3 ‘© One semester or less Fill in circle 1. © Two semesters © Three semesters or more Question 2 About how often did you do lab work in your first physics course? (Include any times when you may have watched a film or a demonstration by your teacher and then discussed or analyzed data.) Fill in only one circle of circles 4-7. ‘* Less than once a week Fill in circle 4. # About once a week Fill in circle 5, = A few times a week Fill in circle 6. ‘© Almost every day —Fill in circle 7. 3 If you have taken or are currently taking an Adyanced Placement (AP) Physies course, which of the following describes the course? Fill in both circles if applicable. (If you have never had AP Physics, leave circles 8 and 9 blank.) ‘© A-course that uses algeba and trigonometry but NOT calculus (Physics B) —Fill in circle 8. ‘© A-course that uses calculus (Physies C) II in circle 9 When the supervisor gives the signal, turn the page and begin the Physics Test. There are 100 numbered circles on the answer sheet and 75 questions in the Physics Test. Therefore, use only circles 1 to 75 for recording your answers. ® PHYSICS TEST Note: To simplify calculations, you may use g = 10 m/s” in all problems. Directions: E: th set of lettered choices below refers to the numbered questions immediately follo it. Select the one lettered choice that best answers each question, and then fill in the corresponding circle on the answer sheet ‘A choice may be used once, more than once, or not at all in Questions 1-3 (A) Coefficient of linear expansion (B) Latent heat of fusion (C) Latent heat of vaporization (D) Specific heat (E) Coefficient of thermal conductivity Select the quantity above that should be used in the calculation of each of the following. 1, The amount of heat required to change 100 grams, of ice at 0° C into water at 0° C 2. ‘The temperature at which a 0.5-centimeter gap between 1.0-meter concrete slabs in a sidewalk will close up completely 3. The time required for 100 joules of heat to pass. through a copper tod of length 2 meters and cross- sectional area 0.5 square meter that connects two ‘objects at different temperatures Questions 4-5 relate to the following particles. (A) Electron (B) Neutron (©) Proton (D) Neutrino (E) Photon 4, Which particle constitutes the nucleus of an ordinary hydrogen atom? 5. Which charged particle in the list is the feast th set. Questions 6-7 An automobile starts from rest and moves along a straight road, In the graph below, the distance x of the automobile from its starting point is given as a funetion of time & ance Di o Time (A) Interval A (B) Interval B (©) Interval © (D) Interval D (&) Interval E 6, During which interval is the automobile stationary and farthest from its starting position? 7. During which interval does the speed of the automobile have its maximum value? GO ONTO THE NEXT PAGE 321) © PHYSICS TEST — Continued © Questions 8.9 relate to the field lines that are shown in the following diagrams. We AN IS «© Nz ©) Aaa 7s ToL Tet tt 8. Which diagram best represents the electtie field produced by two oppositely charged particles? “ 9. Which diagram best represents an electric field produced by oppositely charged parallel plates that are large and close together? any prt ofthis og sepa D322 Questions 10-12 relate to calculations or explanations based on the following principles. (A) Conservation of energy alone (B) Conservation of momentum alone (C) Conservation of both energy and momentum (D) Conservation of charge (E) Mechanical equivalence of heat 10. Used to calculate the velocity of two moving freight cars, after they couple and move together, given the initial masses and velocities of the freight 11, Used to calculate the speed of a lump of clay that hits and sticks to a block of wood suspended as a pendulum, given the height to which the block swings and the masses of the block and the clay 12, Used to calculate the speed of a pendulum bob at the bottom of its swing given the height from which the bob is released from rest GO ONTO THE NEXT PAGE © PHYSICS TEST — Continued Directions: Each of the questions or incomplete statements below is followed by five suggested answers or completions. Select the one that is best in each case and ther 13. A skydiver has been in the air long enough to be falling at a constant terminal speed of 50 meters per second. How much farther will the skydiver fallin the next 2.00 seconds? (A) 19.6m (B) 50m ©) 98m, (D) 100 m (E) 120m 14, Tttakes about 1,0 second for an object to fall ‘5 meters vertically. If this same object is thrown. horizontally with a speed of 30 meters per second from a roof-top 5 meters above ground, about how many meters from the base of the building will the object land? (A) 30m, (B) 30V2 m (©) 30¥3 m (D) 60m © 90m fill in the corresponding circle on the answer sh ” ko 15, Assume that every projectile Fired by the toy cannon shown above experiences a constant net force F along the entire length of the barrel. Ifa projectile of mass m leaves the barrel of the cannon with a speed v, at what speed will a projectile of mass 2m leave the barrel? » we » py OF Ov ©) 20 ©) 4 GO ONTO THE NEXT PAGE 323 ® Questions 16-17 ‘The following diagram shows @ permanent magnet and a coil of copper Wire that is part of a closed circuit. 16. What happens as the north pole of the magnet is moved al constant speed into the coil? (A) The magnet gains potential energy. (B) The magnet attracts the coil. (©) The coil attracts the magnet. (D) A current flows in the coil, producing a Magnetic field, (E) The magnet loses kinetic energy 17. Which of the following would be different if the magnet were turned around so the south pole ‘moved into the coil at the same speed as before? (A) The direction of the forces on the magnet (B) The direction of the energy transfer (C) The direction of the current in the coil (D) The magnitude of the current in the coil (E) The sign of the charges moving in the coil any prt ofthis og sepa D324 PHYSICS TEST — Continued 18, 20. ® An electric current in a copper wire is the result of the motion of which of the following? (A) Copper atoms (B) Copper oxide molecules (C) Protons (D) Electrons (B) Neutrons Eyeglasses, magnifying glasses, and optical icroscopes depend for their operation primarily fon the phenomenon of (A) reflection (B) refraction (C) interference (D) dispersion (B) diffraction 40! 60. In the circuit shown above, the current through the battery will be greatest when the switches are in which of the following positions? (A) [and IIT (B) Land IV (C) Wand IIL (D) Hand IV (E) The current will be the same regardless of how the switches are positioned. GO ONTO THE NEXT PAGE © PHYSICS TEST — Continued Questions 21-23, ‘The diagram above shows a pendulum that swings, to.a maximum height /+ above its lowest point ¥. ‘The mass of the pendulum bob is 0.05 kilogram. At point Y, the bob has a speed of 3.0 meters per second. 21. The momentum of the pendulum bob as it passes through point ¥ is most nearly (A) 0.05 kgems (B) 0.15 kgems. (C) 0.23 kgemws (D) 0.45 kgems (E) 0.50 kgem/s. 22, The height fis most nearly (A) 0.15 m (B) 030m (©) 0.45 m. (D) 0.60 m (&) 0.90 m 23. Ifthe potential energy of the pendulum bob is, zevo al point Y, the total energy (kinetic plus potential) of the pendulum bob is most nearly (A) 0.05 J (B) 0.151 (©) 0.231 (D) 0.451 () 0.501 my part of his page sega Questions 24.25 Two masses m, and m, are hung from the ceiling by two ropes as shown above. The tension in the ipper rope is T, and the tension in the lower rope 24, Which of the following is correct? (A) 1; is always greater than T), (B) T, is always less than T, © is always equal to (D) 7, is greater than T, only if m, is greater than my () 1, is greater than T, only if’ m, is greater than m, 25. Which of the following best represents the forces acting on my? 47, @) 4r T | + ms me a mye. © ©) Tapeh t TH mye mye © nya 1 ms GO ONTO THE NEXT PAGE 3250 © PHYSICS TEST — Continued 26. A boat that can move at 5 kilometers per hour in still water is crossing a river whose current is 2 kilometers per hour. The problem isto steer the boat so that it will land directly across the river from where it started. The solution to the problem is best represented by which of the following sketches in which the river is flowing to the right? E “j wt 7 4 © 27. One harmonic of a note produced by a flute has a wavelength and an associated frequency f. If the wavelength of another harmonic ofthis note is, 2A, what is its associated frequency? w if ®) $F or 2h © 46 any prt ofthis og sepa »326 28, Sound waves can exhibit which of the following. HL. Refraction (A) Lonly (B) Honly (©) Land I only (D) Hand III only () 1,1, and Ut Unpotarized Ws ¢ 29. Polarizing sheets X and Y shown above are oriented so that none of the unpolarized light shining on X is transmitted through Y. Axis ( is perpendicular to both sheets. Which of the following will result in the transmission of light through ¥? (A) Rotation of ¥ by 90° about axis f (B) Rotation of ¥ by 180° about axis ¢ (C) Rotation of ¥ by 360° about axis ¢ (D) Placement of a third polarizer between X and ¥, with its polarizing axis oriented. the same way as X (E) Placement of a third polarizer between X and ¥, with its polarizing axis oriented the same way as Y GO ONTO THE NEXT PAGE Questions 30.32 230) ep in \ 120 150 180 210 240 Time (s) 30 60 90 An automobile with a mass of 1.5 10° kilograms is traveling on a flat, level road. The above graph shows the automobile’s speed as a function of time, 30. The automobile’s acceleration at the end of (60 seconds is (A) 0.25 m (B) 2.5 mis? ©) 4mis? (D) 15. més () 60. mis? 31, The constant braking force applied to stop the (A) LN (B) SON (©) 15x 10°N (D) 45x 108 N (©) 14x 105N 32. The speed of the automobile 10 seconds after the brakes are applied is A) Lmis (B) 10 mis. ©) 15 mis (D) 20 mis. (E) 30 mis. my part of his page sega PHYSICS TEST — Continued 33, 34. attached (0 a vertically ‘mounted spring that has spring constant k. The object is displaced from its equilibrium position and allowed to oscillate. Assume that air resistance and friction are negligible. To increase the frequency of the motion, one could An object of mass m (A) increase the amplitude of the motion (B) change to a spring with a greater spring ‘constant (©) mount the spring horizontally (D) attach an object of greater mass (E) attach an object of the same mass but greater density ‘Two blocks of identical mass are connected by a light string as shown above. The surface is frictionless and the pulley is massless and frictionless. The acceleration of the two-block system is most nearly (A) 20 mis? (B) 15 mis? ©) 10 mis? () Sms? (©) 25s GO ONTO THE NEXT PAGE 327) © PHYSICS TEST — Continued © 35. Consider the following four forces involvi ‘object at rest on a tabletop. 1. The gravitational force on the object due to the Earth IL The gravitational force on the Barth due to the object IIL. The force on the tabletop due to the object IV. The force on the object due to the tabletop Which, if any, of these forces are action-re: pairs in accordance with Newton's third law? (A) Pair | and II only (B) Pair | and 1V only (C) Pair | and 11, and pair Il and 1V (D) Pair Tand IV, and pair TT and TL (E) There are no action-reaction pairs among these forces. Questions 36-37 A heat engine operates between two reservoirs, one at a temperature of 300 K and the other at 200 K. In fone eycle, the engine absorbs 600 joules of heat and, does 150 joules of work. 36. How much heat is exhausted by the engi cycle? (A) 1501 (B) 4501 (©) 5501 (D) 6004 (E) 7501 37. The actual effi (A) 75% (B) 67% (©) 50% ©) &) 25% ncy of the engine is most nearly any prt ofthis og sepa D328 Nijeo 38. A magnet, whose poles are shown in the figure above, moves with velocity v toward a small object of charge Q initially at rest. Which of the following is a correct statement about the for the object due to the magnet as the obj encounters the field? (A) Its zero, (B) Itis perpendicular to the pags (C) Ibis directed parallel to the magnetic field (D) Itisin the same direction as. v (B) Itisim the direction opposite to ¥. GO ONTO THE NEXT PAGE x x x x x Oy 3m x x x x Ti, B= 6teslas 39. A loop of wire shaped into a tangle, shown above, ‘carries a current of 2 amperes in a clockwise dirce- tion. A magnetic field of 6 teslas is directed into the paper. What are the magnitude and direction. Of the force applied by the magnetic field to the S-meter edge of the triangle? Magnitude Direction wo | (C) 48N \ (E) 36N \ 40. If two electrically charged particles repel each other with Forces of equal magnitude, then the ‘charges must (A) have different magnitudes (B) have the same magnitude (©) have different signs (D) have the same sign (E) be separated by unit distance lay pr ofthis page seg PHYSICS TEST — Continued © “s/ Qu 41. A battery and three identical lightbulbs are con- nected as shown in the figure above. With the switeh § closed, the brightness of each lightbulb is noted. When switch 5 is opened, the brightness of which of the lightbulbs will change? (A) Lonly (8) Honly (C) Land Il only (D) Hand Il only (©) 1.1, and Ut 42. All of the following scientists made significant contributions to the field of nuclear physics EXCEPT (A) Galileo. (B) Rutherford (C) Becquerel (D) Curie (8) Fermi 43. Which of the following distinguishes an atom, of one isotope of an clement from an atom of a different isotope of the same element? (A) The addition or loss of a beta particle (B) The addition or loss of an alpha particle (C) The amount of nuclear charge (D) The number of orbital clectrons (8) The amount of nuclear mas GO ONTO THE NEXT PAGE > 329) 44. A hydrogen atom, originally in its ground state, absorbs a photon and goes into an excited state. ‘The atom will then most likely (A) be ionized (B) emit a photon (C) emit an electron (D) always be in that excited state (B) undergo nuclear fission 45. A worker hits a metal pipe with a hammer. The ratio of the intensity of loudness as heard by people standing 100 meters away from the ‘worker fo the intensity as heard by people standing 200 meters away from the worker is (A) al (B) 21 © bl (Dy 82 (Bb 46. A light ray is incident from air upon a semicircular piece of glass as shown above. Which of the labeled rays best represents the subsequent path of the light? (yA (B) B wc (yD (HE any part this ge og »330 PHYSICS TEST — Continued a7. 48. 49. ® Huygens’ principle states that every point on a ‘wave front is the source of a new wave front ‘To which of the following types of waves does. Huygens’ principle apply’? 1. Water waves IL. Sound waves IIL. Electromagnetic waves (A) Lonly (B) Honly (©) Monly (D) Hand Ill only (®) LU, and It ‘When coal burns, it produces heat in the amount of 2.5 x 10° joules per gram. About 4,000 joules of heat is required (o raise the temperature of one Kilogram of water by one degree. The amount of, coal required {6 heat 5 kilograms of water from 10°C to 60°C is most nearly (A) 10 grams. (B) 40 grams (©) 100 grams (D) 400 grams (E) 1,600 grams ‘When a person touches the metal part of a bicycle handlebar on a cold day, th colder than the plastic handgrip, even though both are at the same temperature, This phenomenon is due primarily to which of the following? ‘metal seems much (A) The thermal conductivity of the metal is, ‘greater than that of the plastic. (B) The thermal conductivity of th than that of the plastic (©) The density of the metal is greater than that of the plastic (D) The density of the metal is less than that of the plastic. (E) The latent heat of fusion of the metal is greater than that of the plastic. ctal is less GO ONTO THE NEXT PAGE > PHYSICS TEST — Continued 50. An object of mass m rests on horizontal __ Satellite frictiontess surface. A force F- making an angle Se with the horizontal is then applied to the object to ‘ move it along the surface. The acceleration ofthe + object is we FE (B) 55 F eos A satelite moves ina circular orbit of radius r co Fone around a planet of mass Mand radius R, as shown above. The speed of the satellite would be (wo) Esine greater if M and r were changed in which of the m following ways? (e) Fane M © (A) Decreased No change (B) Decreased Increased Questions 51-52 (©) Nochange —Nochange (D) Nochange Increased {A person is standing on a seale thats located on (E) Increased Nocchange ‘platform at the surface of Earth. The platform is supported by a machine that can move the platform 54. A circuit consists of a batt of voltage V and a up and down at various accelerations while keeping it resistor of resistance R. The current through the level. circuit is ZIP the battery is changed to one of voltage 2V and the resistor to one with resistance 51. At what acceleration of the platform does the 4R, the current through the ci Iachine have vo exert the LEAST force on the war toa (21 (B) 4.9 m/s? down or (©) 9.8mis? up wet (D) 9.8 m/s? down 2 ©) 196 m/s up «4 52. Ifthe person’s weight has apparently doubled. according to the reading on the scale, what is the acceleration of the platform? (A) About 9.8 m/s? up (B) About 9.8 m/s? down (©) About 19.6 m (D) About 19.6 m, (B) It cannot be determined without knowing the ‘mass of the person. ing rou GO ONTO THE NEXT PAGE > 331) 59. An electrically charged, insulated metal rod is observed to attract a neutral pith ball and, after contact is made, to repel the ball. Which of the following can be concluded about the rod? (A) The rod had a positive charge before contact and a negative charge after contact, (B) The rod had a negative charge before contact, and a positive charge after contact. (C) The rod’s charge before and after contact had the same sign, (D) The rod had a charge before contact, but no charge after contact ‘The rod had less charge before contact than after contact. q 4q ® @ 56. Two positive charges of magnitudes q and 4q are 6 centimeters apart, as shown above. If the Aric field is zero at a point P (not shown) located on the line segment joining the charges, ‘what is the distance of point P from the charge of magnitude q? (A) bem (B) 2em (©) 3em (D) dem ©) 5 57. A beam of light traveling through the air strikes the surface of a material in which the speed of light is different from what itis in the air, Which Of the following is true of the Tight as it passes into the new medium? (A) The frequency changes but the wavelength stays the same, (B) The wavelength changes but the frequeney says the same (©) Neither the frequency nor the wavelength change. (D) Both the frequency and the wavelength change. () Since the speed of light isa universal constant, the speed in the new material isthe same as it was in air. fay prt this page egal D332 PHYSICS TEST — Continued © ° wane te} [L F 58. A convex lens is used as a magnifier when a real object 0 is placed inside the focus F, as shown above. The image produced is (A) real and inverted (B) real and upright (C) virtual and inverted (D) virtual and upright (B) none of the above 59. The separation of white light into colors by lass prism is a result of (A) interferenc: (B) diffraction (C) (otal internal reflection (D) variation of absorption with wavelength (B) variation of index of refraction with frequency (60. An object with a mass of 5 kilograms is placed at est on an imaginary planet where the gravitational field is 4 newtons per kilogram. One can be certain that the object on this planet, as compared to the object when itis on Earth, will (A) require a greater force to accelerate it on a horizontal surface at 1 m/s? (B) have less weight (C) have less mass (D) have greater mass. (B) have greater acceleration during free fall GO ONTO THE NEXT PAGE > 61. A cartravels around a circular track that has a radius of 1 kilometer. If the car completes 3 tips around the track in 5 minutes, which of the follow- ing expressions gives the average speed of the car in kilometers per hour? Bx) *S(/60) cw SuonexIey “ Ben 5(60) G)2x\) (B)(60) Gr) 5(V/60) © ©) «) ap —- — fe (62. Two identical, human-looking robots are standing and facing forward in separate spaceships. Both ships are moving at 0.8¢ but are traveling in ‘opposite directions, as shown above. If a person on the spaceship with robot A could mak: surements on both robots, which of the following would the person observe to be different? (A) The robots’ heights (B) The length of the robots’ feet from toe to heel (©) The width of the robots’ faces (D) The length of the robots’ legs (E) The width of the robots’ shoulders lay pr ofthis page seg PHYSICS TEST — Continued © 63. The experimental study of the photoelectric effect and its analysis by Einstein confirmed the assumption of the (A) photon aspect of light (B) crystal structure of materials (C) discrete charge on the electron (D) energy-mass relationship of special relativity (B) uncertainty principle of position and ‘momentum 64. The radius of the first Bohr orbit of an electron in hydrogen atom is about 10~!! meter. The radius of the nucleus is about 10“ meter. If'a model of the hydrogen atom were built with the diameter of the electron orbit equal to the width of a classroom, (about 10 meters), which of the following would _most closely represent the size of the nucleus? (A) The chair you are si (B) Your head (C) The eraser on the end of a new pencil (D) The point of a ball point pen (B) A.red blood cell 65. Which of the following is true of any material in ‘a superconducting state that carries a current? (A) Ithas a large internal magnetic field. (B) Ithas no external magnet field (C) thas no resistance. (D) Ithas a temperature of absolute zero. (B) Ithas a very high temperature. GO ONTO THE NEXT PAGE > 333) 66. The graphs below represent velocity as a function of time 1 for five different particles, each movi along a straight line. Which particle experiences. the greatest displacement between 1= 0 and 1=1 second? 1 N oO. i as) “ (B) Velocity (m/s) Ri oh >) T -1 (©) Velocity (m/s) jo of + —1(s) (D) Velocity (m/s) (B) Velocity (m/s) 1 > ol tw 67. The density of a certain material is 3 grams per cubic centimeter. What is the density of the ‘material expressed in kilograms per cubie m (A) 0.3 kg/m" (B)—3kg/m? ©) 30 kg/m? (D) 300 kg/m’ 3.000 ke/m? [Urautharaed copying of ruse of ary ar lho pe soon. D334 PHYSICS TEST — Continued 68 69. 70, ‘A system consists of two pucks moving without friction on a horizontal surface. If the pucks collide elastically, properties of the system that are the same before and after the collision include which of the following? 1. Momentum II. Kinetic enerey IIL Total energy (A) Lonly (B) Monly (C) Land If only (D) Mand Ill only (©) I, Hand tt IC akes an amount of work W to stretch a spring a distance x beyond its natural length. If the spring obeys Hooke"s law, how much work is required to, stretch the spring a distance 2x beyond its natural length? a) Ww (B) 2 «© 3W (b) 4 ® ow A child on a swing can greatly inerease the amplitude of the swing’s motion by “pumping” at the natural frequeney of the swing. This is an example of which of the following? (A) Conservation of momentum (B) Newton's first law of motion (©) Newton’s third law of motion (D) Resonance (B) Interference GO ONTO THE NEXT PAGE > FSFE SFT TF] r 71. A negatively charged oil drop is maintained at rest between charged parallel plates, as shown above, by balancing the downward gravitational force F, on the drop with an upward electric force F,. If the mass of the oil drop is: 1x 10°® kilogeam and the electric field strength between the plat 10 newtons per coulomb, then the charge on the oil drop is most nearly (ay P1086 (B) Ix 104C © 1x10 C () 1x 10°C © 1x10" Cc PHYSICS TEST — Continued 72. An object 0 is just outside the focal point F of | concave mirror, as shown in the diagram above. As the object is moved away from the mirror, the image will do which of the following? (A) Decrease in size and move close tothe (0) Decrease n size and move farther fom the (©) increase in size and move coset (D) Increase in size and move fre rom the (B) Iteannot be determined without knowing the exact focal length, GO ONTO THE NEXT PAGE > 335) PHYSICS TEST — Continued R RK x y R R 73. Which of the following occurs when li reflected from a smooth flat glass surface, as shown above? 75. Four resistors of equal resistance R are connected as shown above. What is the total resistance (A) The light is somewhat intensified, between points X and Y? (B) The light is somewhat polarized. k (©) The light is focused. wt (D) The velocity of the light is reduced. (E) The color of the light is shifted toward the wk blue end of the spectrum, 2 OR (D) 2R (E) aR 74, Light of wavelength 7. is incident from the let on 4 pair of narrow slits, as shown above. If point P isa bright spot (maximum intensity) on a distant een, one ean be certain that the difference between distances and y is () cero @) 42 (C) nA, where n is an integer © (n+ 4), where mina ing (©) (0-4) aware nian integer STOP h before time is called, you may check your work on this test only. Do not turn to any other test in this book. Physics How to Score the SAT Subject Test in Physics When you take an actual SAT Subject Test in Physics, your answer sheet will be “read” by a scanning machine that will record your responses to each question, Then a computer will compare your answers with the correct answers and produce your raw score. You get one point for each correct answer. For each wrong answer, you lose one-fourth of a point, Questions you omit (and any for which you mark more than one answer) are not counted. This raw score is converted to a scaled score that is reported to you and to the colleges you specify. Worksheet 1. Finding Your Raw Test Score STEP 1: Table A lists the correct answers for all the questions on the Subject ‘Test in Physics that is reproduced in this book. It also serves as a worksheet for you to calculate your raw score. + Compare your answers with those given in the table, + Puta check in the column marked “Right” if your answer is correct. + Puta check in the column marked “Wrong” if your answer is incorrect. + Leave both columns blank if you omitted the question. STEP 2: Count the number of right answers. Enter the total here: STEP 3: Count the number of wrong answers. Enter the total here: STEP 4: Multiply the number of wrong answers by .250. Enter the product here: STEP 5: Subtract the result obtained in Step 4 from the total you obtained in Step 2. Enter the result here: STEP 6: Round the number obtained in Step 5 to the nearest whole number. Enter the result here: The number you obtained in Step 6 is your raw score. 337) ‘The Official Study Guide for All SAT Subject Tests Table A Answers to the Subject Test in Physics, Form K-3XAC, and Percentage of Students Answering Each Question Correctly Pree ere Question frereteee fetter amber | Answer | Right | Wrong Sots 54 33 8 57 6 34 D Ey 80 35 c 66 n 36 8 59 63 a E 6 89 38 8 8 55 39 A 30 % 40 D 65 0 a E 40 70 a2 A 64 05 a E 58 69 4a 8 33 83 45 A 40 87 6 8 58 4 a E 8 80 8 8 8 69 49 A n a7 50 c n n 51 D 65 81 s2 A 46 a 33 E a7 49 sa D 6 58 58 c 35 16 56 8 39 81 7 8 3 0 58 D a7 8 59 E 2 4 60 8 65 2 6 A 48 78 62 8 4 66 6 a 41 a 64 D 2 Table A continued on nest page »338 Physics Table A continued from previous page ones Cre Seer aed cee Number Number | Answer | Right | V ed * These percentages are based on an analysis ofthe answer shects ofa representative sample of 2410 students who took this test in January 2003, and whose mean score was 628. They may be used as an indication ofthe relative difficulty of« particular question. Each percentage may also be used to predict the likelihood that a typical SAT ‘Subject Test in Physics candidate will answer that question correctly om this edition of the test 339) ‘The Official Study Guide for All SAT Subject Tests Finding Your Scaled Score When you take SAT Subject Tests, the scores sent to the colleges you specify are reported on the College Board scale, which ranges from 200-800. You can convert your practice test score to a scaled score by using Table B. To find your scaled score, locate your raw score in the left-hand column of Table B; the corresponding score in the right-hand column is your scaled score, For example, a raw score of 41 on this particular edition of the Subject Test in Physics corresponds to a scaled score of 670. Raw scores are converted to scaled scores to ensure that a score earned on any one edition of a particular Subject Test is comparable to the same scaled score earned on any other edition of the same Subject Test. Because some editions of the tests may be slightly easier or more difficult than others, College Board scaled scores are adjusted so that they indicate the same level of performance regardless of the edition of the test taken and the ability of the group that takes it. Thus, for example, a score of 400 on one edition of a test, taken ata particular administration indicates the same level of achievement as a score of 400 on a different edition of the test taken at a different administration, When you take the SAT Subject Tests during a national administration, your scores are likely to differ somewhat from the scores you obtain on the tests in this book. People perform at different levels at different times for reasons unrelated to the tests themselves. ‘The precision of any test is also limited because it represents only a sample of all the possible questions that could be asked. D340 Physics Table B Scaled Score Conversion Table (Form K-3XAC) 660 a 650 2 400 640 1 400 640 0 390 630 4 380 620 2 380 620 3 370 610 4 360 600 5 360 600 4 350 590 a 340 580 4 330 580 4 330 570 a0 320 560 n 310 560 12 310 550 13 300 58 790 2 540 a4 290 5) 730 a 530 18 290 56 780 20 520 16 280 55, m0 13 520 a0 280 5h 760 18 510 18 270 53 750 7 510 19 270 2 780 16 500 st 40 6 490 50 730 4 490 19 730 13 480 48 720 2 470 "7 m0 " 470 46 m0 10 460 5 700 8 450 au 690 8 450 0 690 7 440 2 680 6 430 a 570 5 420 40 570 4 420 341) ‘The Official Study Guide for All SAT Subject Tests How Did You Do on the Subject Test in Physics? After you score your test and analyze your performance, think about the following questions: Did you run out of time before reaching the end of the test? If so, you may need to pace yourself better. For example, maybe you spent too much time on one or two hard questions. A better approach might be to skip the ones you can't answer right away and try answering all the questions that remain on the test. Then if, there's time, go back to the questions you skipped. Did you take a long time reading the directions? You will save time when you take the test by learning the directions to the Subject Test in Physics ahead of time. Each minute you spend reading directions during the test is a minute that you could use to answer questions. How did you handle questions you were unsure of? Ifyou were able to eliminate one or more of the answer choices as wrong and guess from the remaining ones, your approach probably worked to your advantage. On the other hand, making haphazard guesses or omitting questions without trying to eliminate choices could cost you valuable points. How difficult were the questions for you compared with other students who took the test? Table A shows you how difficult the multiple-choice questions were for the group of students who took this test during its national administration, The right-hand column, gives the percentage of students that answered each question correctly. A question answered correctly by almost everyone in the group is obviously an easier question, For example, 89 percent of the students answered question 6 correctly. But only M4 percent answered question 15 correctly. Keep in mind that these percentages are based on just one group of students. They would probably be different with another group of students taking the test. If you missed several easier questions, go back and try to find out why: Did the questions cover material you haven't yet reviewed? Did you misunderstand the directions? p3a2z Chapter 8 Chinese with Listening Purpose ‘The Subject Test in Chinese with Listening measures your understanding of Mandarin Chinese in the context of contemporary Chinese culture. The questions on the test are written to reflect general trends in high school curricula and are independent of particular textbooks or methods of instruction. Format This isa one-hour test with about 20 minutes of listening comprehension and 40 minutes of usage and reading comprehension. ‘There are 85 multiple-choice questions in three sections, Content Listening Comprehension questions test the ability to understand the spoken language and are based on short, spoken dialogues and narratives primarily about everyday topics. There are two different kinds of listening comprehension questions: (A) a spoken statement, question, or exchange, followed by a choice of three possible responses (also spoken); (B) a spoken dialogue or monologue with a printed question or questions (in English) about what was said. Usage questions ask you to select the answer that best completes a Chinese sentence in a way that is structurally and logically correct. Questions are written to reflect instructional practices of the curriculum. This section of the test is therefore presented in four columns across two pages of the test book to allow each question and its answer choices to be shown in four different ways of representing Chinese: traditional and simplified Chinese characters on the left page, and phonetic transcriptions in Pinyin romanization and the Chinese phonetic alphabet (Bopomofo) on the right page. You should choose the writing form you are most familiar with and read only from that column, Reading Comprehension questions test your understanding of such points as main and supporting ideas, themes, and the setting of passages. Some of the passages are based 343 ‘The Official Study Guide for All SAT Subject Tests on real-life materials such as timetables, forms, advertisements, notes, letters, diaries, and newspaper articles. All passages are printed in both traditional and simplified Chinese characters, While most questions deal with understanding of literal meaning, some inference questions may also be included. All reading comprehension questions are in English. se with Listening oe Listening Comprehension Usage Reading Comprehension CD Players Using CD Players for Language Tests with Listening ‘Take an acceptable CD player to the test center. Your CD player must be in good working order, so insert fresh batteries on the day before the test. You may bring additional batteries and a backup player to the test center. CD players cannot be shared with other test-takers. ‘Test center staff won't have batteries, CD players, or earphones for your use, so your CD player must be: + equipped with earphones + portable (hand-held) + battery operated You are not allowed to use a CD player with recording or duplicating capabilities. Note If the volume on your CD player disturbs other test-takers, the test center supervisor may ask you to move to another seat. What to do if your CD player malfunctions: + Raise your hand and tell the test supervisor. + Switch to backup equipment if you have it and continue the test. If you don’t have backup equipment, your score on the Chinese with Listening Test will be canceled. But scores on other Subject Tests you take that day will still be counted. ‘What if you receive a defective CD on test day? Raise your hand and ask the supervisor for a replacement. Daa Chinese with Listening How to Prepare ‘The best preparation is gradual development of competence in Chinese over a period of years. The test is appropriate for students who have studied Mandarin Chinese as a second or foreign language for two to four years in high school, or the equivalent. A practice CD with different sample questions can be obtained, along with a copy of the SAT Subject Tests Preparation Booklet from your school counselor, or you can access the listening files at www.collegeboard.com. You should also take the practice test included with this book. Familiarize yourself with the test directions in advance. The directions in this book are identical to those that appear on the test. Note ‘The SAT Subject Test in Chinese with Listening is offered only at designated test centers on designated test dates. To take the test, you MUST bring an acceptable CD player with earphones to the test center, Scores The total score is reported on the 200-to-800 scale. Listening, usage, and reading subscores are reported on the 20-to-80 scale. Sample Questions Following are some samples for each section of the SAT Subject Test in Chinese with Listening. All questions are multiple choice. You must choose the best response from the three or four choices offered for each question. In an actual test administration, all spoken Chinese will be presented as recorded audio Text that appears in this section in brackets ({ ]) will be recorded in an actual test and it will not be printed in your test book. Spoken text appears in printed form here because a recorded version is not available. 345 ‘The Official Study Guide for All SAT Subject Tests Sample Listening Questions Please note that the CD does not start here. Begin using the CD when you start the actual practice test on page 355. PartA Directions: In this part of the test, you will hear short questions, statements, or commands in Mandarin Chinese followed by three responses in Mandarin Chinese designated (A), (B), and (C). You will hear the questions or statements, as well as the responses, just one time, and they are not printed in your test book. Therefore, you must listen very carefully. Select the best choice and fill in the corresponding circle on your answer sheet Question 1 (Narrator) [Number 1 (Woman) PE I) AE ZEAE SL? (Man) (A) SAE LEAT. 8) REMRERS. © Betkeia. | (5 seconds) Choice (C) is the correct answer because it responds to the question “where is the library?” Choice (A) is incorrect because it tells when the library opens, and choice (B) is incorrect because it tells what’s inside the library. Question 2 (Narrator) [Number 2 (Man) AERA? (Woman) AH, BARA. (Man) (A) SAT? @B) $ yak? © #521 (5 seconds) Choice (B) is the correct answer because it asks how much the book costs. The conversation concerns the price of a book. The man asks if the book is expensive, and D346 Chinese with Listening the woman replies that it is neither expensive nor cheap. Choice (A) is incorrect because it asks about length of time. Choice (C) is incorrect because it asks if there are many. Part B Directions: You will now hear a series of short selections. You will hear them only once, and they are not printed in your test book. After each selection, you will be asked to answer one or more questions about what you have just heard, These questions, each with four possible answers, are printed in your test book. Select the best answer to each question from among the four choices printed and fill in the corresponding circle on your answer sheet. You will have fifteen seconds to answer each question. Questions 3-4 (Narrator) [Questions 3 and 4, Listen to find out what the woman will do next summer. (Woman) 4-238 ARG? (Man) BEB, TARWFRLARA ADP MSR PERERA (Woman) BFRARAHAKMELEM A, MAAR URL. (Narrator) Now answer questions 3 and 4.] (30 seconds) 3. Where will the woman spend the summer next year? (A) InChina (B) In Japan (©) In Hong Kong (D)_ In the United States Choice (D) is the correct answer because the woman states in the conversation that she will stay in the United States next summer, Choices (A), (B), and (C) are the places where the man will go next summer. 347) ‘The Official Study Guide for All SAT Subject Tests 4, What will the woman do? (A) Visit friends (B) Go to school (©) Look for a job (D) Travel abroad Choice (B) is the correct answer because the woman states in the conversation that she will go to summer school. None of the other answer choices are mentioned by the woman in the conversation. Sample Usage Questions Directions: This section consists of a number of incomplete sentences, each of which has four possible completions, Select the word or phrase that best completes the sentence structurally and logically and fill in the corresponding circle on your answer sheet. ‘This section of the testis presented in four columns to allow each question to be shown in four different ways of representing Chinese: traditional characters, simplified characters, Pinyin romanization, and the Chinese phonetic alphabet (Bopomofo). TO SAVE TIME, IT RECOMMENDED THAT YOU CHOOSE THE WRITING FORM WITH WHICH YOU ARE MOST FAMILIAR AND READ ONLY FROM THAT COLUMN: AS YOU WORK THROUGH THIS SECTION OF THE TEST. D348 Chinese with Listening qAOU SHU OF] ATTeOY |, THOUIDyEIs BuApod01d amp 01 papuadde uonsonb v se 2nok ynoqe Mop}, Suytreaut papuayut axp sdaation ju unouoad se[Nuts uossad-puoras axp) SuLAo[}o} du (Q) asioyp Afuo “‘SamIoyP sMSUR ano} AY JO “g UONSANb oF ZaMSUE 29209 ayp S| (C1) P10 “sapped [euy-22uDyU9s Jo 9sn ayR SIs94 HoRSOND STU, = @ ou @) “% @ % @ zs © rq (O) Qa (9) (0) 5 @ vu (@) @) % @ + Ww) * wy) Aw) Aw) eg oN FE Ty Bos Fukuerpng guyz uenyreusyom “SRB IER YR UW “SOE Bale HWE “s g wonsenp 349 ‘The Official Study Guide for All SAT Subject Tests ae samIoyp zaMsuE JayIO ayp Jo aUON, ,"uOyE ~ 1 quoape Supfur-Surpuessoj a]qeaouruiow ayy asneoaq 9 uonsanb oy saMsuE 3291509 auf, ‘suoHINAS “yxoquo> sty Uy PoaL09 se uoos sv,, Surueat ayy Aat109 o 180] 20 Aqqeamonais if yates pasted st (q) 22104 canb StU, @ ys uns. (q) x @ Yh Wie © oO ‘Supaye “wang (O) Ho ee CO) We ee @ ae Kg) eK @ id w) nop w) ae w) wee" (yy “TH ow 3 Bo ‘F yo YR ws? g worsen D350 Chinese with Listening Sample Reading Questions Directions: Read the following texts carefully for comprehension, Each is followed by ‘one or more questions or incomplete statements. Select the answer or completion that is best according to the text and fill in the corresponding circle on the answer sheet. This section of the test is presented in two writing systems: traditional characters and simplified characters. IT Is RECOMMENDED THAT YOU CHOOSE THE WRITING SYSTEM WITH WHICH YOU ARE MORE FAMILIAR AND READ ONLY THAT VERSION AS YOU WORK THROUGH THIS SECTION OF THE TEST. Questions 7-8 RA GRARARERA Fi "8 tk He HE Baste Ae Se AoE RS — FRG ZSE e ‘Eee; 2(02)321-8400 AAG em RASH HK Bk ae ee DAGhwwers tak fe FIELDS 38: 28(02)321-8400 7. What is this? (A) A business card (B) A thank-you note (©) Areturn envelope (D) A concert ticket Choice (A) is the correct answer to question 7. The text contains a person’s name, workplace, profession, office address, and telephone number, all arranged in the standard format for an individual's business card. 351) ‘The Official Study Guide for All SAT Subject Tests 8, The person named is a (A) professor (B) singer (© conductor (D) journalist Choice (A) is the correct answer to question 8. The text of the business card shows that the person works in the music department of a university and gives the person’s professional title as “professor and composer.” D352 Chinese with Listening Test Practice Helps The test that follows is an actual, recently administered SAT Subject Test in Chinese with Listening, To get an idea of what it's like to take this test, practice under conditions that are much like those of an actual test administration. Set aside an hour when you can take the test uninterrupted. Make sure you complete the test in one sitting. Sit at a desk or table with no other books or papers. Dictionaries, other books, or notes are not allowed in the test room. Tear out an answer sheet from the back of this book and fill it in just as you would on the day of the test. One answer sheet can be used for up to three Subject Tests. Read the instructions that precede the practice test. During the actual administration you will be asked to read them before answering test questions. Time yourself by placing a clock or kitchen timer in front of you. After you finish the practice test, read the sections “How to Score the SAT Subject Test in Chinese with Listening” and “How Did You Do on the Subject Test in Chinese with Listening?” The appearance of the answer sheet in this book may differ from the answer sheet you see on test day. 353) NO TEST MATERIAL ON THIS PAGE D354 CHINESE TEST WITH LISTENING ‘The top portion of the section of the answer shect that you will use in taking the Chinese Test with Listening must be filled in exactly as shown in the illustration below. Note carefully that you have to do all of the following on your answer sheet. 1, Print CHINESE WITH LISTENING on the line under the words “Subject Test (print) 2. In the shaded box labeled “Test Code” fill in four circles: —Fill in circle 3 in the row labeled V. —Fill in circle 5 in the row labeled W, Lin circle 4 in the row labeled X. Fill in circle D in the row labeled Y. Please answer Part land Part I{ below by filling in the appropriate circles in the row labeled Q on. your answer sheet, Select the answers that correspond to your Chinese language experience and to the Chinese language classes you have taken or are taking at present. The information that you provide is for statistical purposes only and will not affect vour score on the test. PARTI Which ofthe following i the primar. souree of your knowledge of the Chinese language? —Circle 1 — Living in a place and/or a home in which Mandarin Chinese is used —Circle 2. — Living in a place and/or a home in which Chinese other than Mandarin is used —Cirele 3. — Studying Chinese in an extracurricular (after-school. weekend, summer, and/or study abroad) program (e.g,, Chinese language school) —Cirele 4 — Studying Chinese in classes at your regular elementary and/or middle school (grades K through 8) —Circle 5 — Studying Chinese in classes at your regular high school (grades 9 through 12) PART II How long have you studied Chinese in your regular high school (grades 9 through 12)? (Choose only one.) Circle 6 — 0 (vero) to 2 years —Cirele 7 — 210 2% years —Cirele 8 — 3 (0 3% years —Cirele 9 — 4 years When the supervisor tells you to do so, turn the page and begin the Chinese Test with Listening. There are 100 numbered circles on your answer sheet. Use only circles 1 to 85 to record your answers fo the 85 questions in the Chinese Test with Listening, 355) ® CHINESE TEST WITH LISTENING ® PLEASE NOTE THAT YOUR ANSWER SHEET HAS FIVE ANSWER POSITIONS, MARKED A,B, C.D, AND E, WHILE THE QUESTIONS THROUGHOUT THIS TEST CONTAIN EITHER THREE OR FOUR ANSWER CHOICES. BE SURE NOT TO MAKE ANY MARKS IN COLUMN E, AND DO NOT MAKE ANY MARKS IN COLUMN D IF THERE ARE ONLY THREE CHOICES GIVEN. SECTION I LISTENING Approximate time—20 minutes Questions 1-30 Part A Directions: In this part ofthe test, you will hea short questions, statements, or commands in Mandarin Chinese, followed by three responses in Mandarin Chinese, designated (A), (B). and (C). You will hear the questions or statements, as well a the responses, just one time, and they are not printed in your test booklet. ‘Therefore, you mus listen very carefully. Select the bes response and fill in the corresponding circle on your answer sheet. Now listen to the following example, but do not mark the answer on your answer sheet ‘You will hear: You will also hear: ‘The answer that most logically responds to the question is (C). Therefore, you should choose answer (C).. Now listen to the first exchange. 1. Mark your answer on your answer sheet 11, Mark your answer on your answer sheet. 2. Mark your answer on your answer sheet. 12, Mark your answer on your answer sheet, 3. Mark your answer on your answer she. 13, Mark your answer on your answicr sheet 4. Mark your answer on your answer shect. 14, Mark your answer on your answer sheet. 5. Mark your answer on your answer sheet 15, Mark your answer on your answer sheet. 6. Mark your answer on your answer sheet. 16, Mark your answer on your answver sheet. 7. Mark your answer on your answer sheet. 17. Mark your answer on your answer sheet. 8. Mark your answer on your answer sheet. 18, Mark your answer on your answer sheet. 9. Mark your answer on your answer sheet. 19, Mark your answer on your answer sheet. 10, Mark your answer on your answer sheet. 3ZLC END OF PART A. GO ON To PART B. Jane Bert of this peue egal GO ONTO THE NEXT PAGE > D356 SE TEST WITH LISTENING—Continued © Part B Directions: You will now hear a series of short selections. You will hear them only once, and they are not printed in your test booklet. After each selection, you will he asked to answer one or more questions about what you have just heard. These questions, each with four possible answers, are printed in your test booklet. Select the best answer to each question from among the four choices printed and fill in the corresponding, cirele on your answer sheet. You will have fifteen seconds to answer each question. Now listen to the following example, but do not mark the answer on your answer sheet. You will hear: You will see: ‘What are the (wo people talking about? (A) Food (B) Homework (C) History (D) Language ‘The best answer to the question is (D), “Language.” Therefore, you should choose answer (D). Now listen to the first selection, (Questions 20-21 Question 23 20. What is the man’s native language? 23. Which of the following activities is NOT (a) French mentioned by the speaker? (B) Chinese (A) Watching television (© English (B) Singing son; (D) German (C) Riding bicycles (D) Playing tennis 21. How did the man's son learn German? (A) His father taught him, (B) He took lessons from the woman. (C) He studied it in college. (D) He grew up in Germany. Question 24 24. What happened to the man? (A) He forgot to return all the books. (B) He missed the train. Question 22 (C) He lost his football tickets. (D) He took the wrong bus. 22. What is the speaker's problem? (A) He spent too long debating an issue. (B) He got up too late for work. (C) His sleeping pills are all gone. (D) He has a headache and can’t concentrate, GO ONTO THE NEXT PAGE > 357) ® CHINESE TEST WITH LISTENING—Continued Questions 25.26 25. What color shoes did Xiaohua originally want? (A) White (B) Yellow (©) Brown (D) Black 26. What does Xiaohua dislike about the pair of brown shoes? (A) The price (B) The size (©) The style (D) The quality Question 27 27. What information is provided in the announcement? (A) A change of location (B) Additional show times (C) A revision to the program (D) The deadline for purchasing tickets Questions 28-29 28. On which day of the week is the event being held? (A) Monday (B) Wednesday (C) Saturday (D) Sunday 29. Which of the following statements about the male speaker is truc? (A) He is living in a dormitory. (B) He is the woman’s classmate (©) He is a Chinese history major. (D) He is the organizer of the event. Question 30 30, Which of the following items is the man NOT required to bring? (A) Photos: (B) Transcript (C) Medical report (D) ID card END OF SECTION I. DO NOT GO ON TO SECTION II UNTIL YOU ARE TOLD TO DO SO. lay par of his poe seal D358 ® CHINESE TEST WITH LISTENING—Continued © TIME FOR SECTIONS If AND III -40 minutes. SECTION II USAGE Suggested time—15 minutes Questions 31-55, Directions: This section consists of a number of incomplete statements, each of which has four possible completions. Select the word of phrase that best completes the sentence structurally and logically and fill in the corresponding circle on your answer sheet. This section of the testis presented in four columns across two pages to allow each question to be shown in four different ways of representing Chinese: traditional characters, simplified characters, Pinyin romanization, and the Chinese phonetic alphabet (Bopomofo). TO SAVE TIME, IT IS RECOMMENDED THAT YOU CHOOSE THE WRITING FORM WITH WHICH YOU ARE MOST FAMILIAR AND_ READ ONLY FROM THAT COLUMN AS YOU WORK THROUGH THIS SECTION OF THE TEST. Example: AE __Aiht © 42 __$AGH. TE__youhing shan bint (A) tat we (A) fda co 1B) th med 8) Sinai (0) adh OAS (0) cfg 80 O) Bath D Ad D) SH ade The best completion is answer (B). Therefore, you should choose answer (B) and fill in the correspon c on your answer sheet, Remember to work with one column only and start by filling in one of the next to number 31 on your answer sheet. [Unauthorised copying oy ruse of Shy er ofthis ge tea GO ONTO THE NEXT PAGE > 359) 33. xs CHINESE TEST WITH LISTENING—Continued S RRR > * (A) & fal (By Ra (Cc) ete T (D) Fe PREP . (A) AR ABR AE (By Ae ARAB AAR, (CRAP AAR (D) WEARS RAR RT—RKBHE- ___ oe (A) KETHRE (By ER THAE ( RT AR OA (D) KEL T ROR ERT Belt iit « (A) Fre (B) 45> (©) Rae (D) teat MBG PLR to a) # (By ak (c) & (Dy »360 a 33, 34, 35, RRL th ae (ay AEs (B) # RE (©) RT (D) FE SREP (A) ABR He A (B) AAEM, (0) RAP RAR (D) WARS AAR ATR (A) KETHRE (B) BRT AK (©) RTRRGA (D) KLAR T WR ERT REAR (A) Bret (By wh sh ©) Bie (D) ose oy (a) BRR a (B) R # « (vy GO ONTO THE NEXT PAGE > 31. Xitozhiing shus de hua, nt 31 diu___ ma? (A) Kin de dio (B) na bu chi (C) tingdéngle (D) fing xiagu 32, Fintiin shingwi i _ 32. (A) én de win gilai (B) gilai de hén win © 1 de win gi hn (Dwain de qi héw Iai 33. Zudle yi tim de shi, 33, (A) dajia te dow Ii yé (B)_yé lle dou dij (C) Bile aajia ye dou (D) adja y le 34. Ta de chézi hui le, M4, yo w6 qh jie (a) 308 (8) iva (©) bitin (D) ici 35, Ti chingg@ ching de bi who 36. (a) ai (6) iv (©) ang () bing inwtrarined copying oy wuss oT ofthis peg siege wd @) Rane ae ae La gn aie as es a ae ar GH OS CHINESE TEST WITH LISTENING—Continued g 2% ry boy eh a4 4 Be 28 Po 1 “ 4 4 g 4 4, bp BaF bras. B2RS brag BRE antag Be eR 2 he ei he 54 ree a ¢ a. Leak r, Lean oe GO ONTOTHE NEXT PAGE > 361) 36. 2 38 38, 40. CHINESE TEST WITH LISTENING—Continued ® RINT RT KBP? (A) 84K (By AFAR (C) Bab D) #ER (A) HARKS ROS (B) RF HRAM CRS) © AMERHN RT 0) CeRRFIAA REMI (A) RRKRAE (B) ARRA RM (C) ARR BAA (D) PARE AM Me AR fe AE SE BD AEB + LR RAPHE © (A) Ra (B) 2k (© #B (D) #R ARIE AAR 1 OIE > (A) Be (By LE (C) RR (D) BH (Unauorsed copy any part of hs page sega » 362 36. MAT RT 37 38 39, 40. Oh (A) (B) © w) BL? BR BPA wh a AR BLK te HT. (A) B) © ) HADRF HOR RESO ACS SACLAY RT Cee AF SAR HE BEY I . (A) (B) © () RRABAK RA ER REBRBK HRERKE SRMARD RH (A) B) © (D) WEKAHR © Re ee ah ca AAEM ETE fe nbs hfe Ala RR “ 1B) © () GO ONTO THE NEXT PAGE > (CHINESE TEST WITH LISTENING—Continued © 36, Women ké bi Ke we ER REO ELS i? (A) chang qilai a 4 (8) mad wali » » (©) ting ehulai © 3 (D) kan shanglai © 3 37, Ti___qile ve &t (A) dao gdngyndn haizi di yijing AREY ER RoE (B)izi di e6ngyusn yijing dio Ores erg gp (©) gongyusin yijing dai dio haizk OF POL REEF (D)_yijing dai hai do goneywsin Ore Ree eg y 38. Wa shiingbin de difang eA Rg gE (A) bi yudn 8H iw fia Wee RA AS (B) ci bd yun we jia BRRREEX (C)_ wa jis Wha yuan wi ORS RRHE () Kowa jaa bs wi yun ORAS PAW 30, Suisin ti 23i Vinggus zhiguo, wk EERT EERE tab tht shud Yingwén, PRERRETR (A) sinhou we & (B) dinshi ® Br © er One (D) yuishi © uP de shihou, ti__shuijit a Ad (A) ahangzai we R B) singed ® < % (©) céngli ORE (D) inhow wh Tata copying a Tee pata GO ONTO THE NEXT PAGE 363) 4 2. 48. 44 45 CHINESE TEST WITH LISTENING—Continued S . KPA ERSAR? A) ne BK. Hl (Be (D) ae HB BEAM AMA © a) 4 BR & (D) FF BT 24h aMAMA_ Bt FM A (A) #31 (B) Hee (C) ae (D) ae Rime kT (A) 8? 48 B) FAFSA (©) Bed ae (D) FR al KB SEAS BERS? 43. 44 4s a @) ite o me (a) (By ic () ae t5 ie a) (By c () ia) Re a) (By © () a a) (By « (D) FD oe Bevo Hi fe Bl Bn HM MA ° fe Hee a mh aw & RAR tk He al Weak “a abe he kT 2d ELE oh LB ae A Ae BLE La mE eG GO ONTO THE NEXT PAGE > ® 4 44, 4s. CHINESE TEST WITH LISTENING—Continued ® ___husehazhin zdul al ana ssh yo dua ji? aE ” (A) Xiding we zai (A) (B) Céng dio (B) Be (©) Wang eo téng oe (D) You se yin 3 2. Wainshang wimen ___ qit kin ge péngydu. ee PaR BREED (A) xiang. (A ® (B) céng B) 2 (©) tng © & (D) dei (Dy 2 iin i _ ste sin feng ae F Beet xin géi ws. EGE (A) eng wa 8) méi wo (C) song () # (D) hui oO & Zhdi jn faingjidn shi __ wei nt MSS ES FP ed vibai de. ae (A) bie wee (B) bijido (By 2 Ee (C) xidngdang cE? (D) feiching meh Gi fen baogio gue. 45.2 5 RE & 2 (A) phi ido jid tongsti A& RARER (B) _jido téngshi Behe re & (C) ji téngshi jido géi Oheres {D) ngshi Or RK KR GO ONTOTHE NEXT PAGE 365) S S CHINESE TEST WITH LISTENING—Continued 3 46, SRAMREFRR TR EREER AI > (a) 59h (By) AR (C) rhe ) HF 47, fee Bw BAM Lat © a € (By (©) ae (Dy 3 48, MARR RBA TR E> ae Fi © # (Dy 49, HRRRA PARRA T > (A) sha (By) Ma (©) wae (D) 2% 50, HAMA RR WR LERAT © (A) #& © + (Dy) a of ths ago Hes » 366 46. 47. 48 50, ERKMR ETM TE EMER ROME (A) 8 9r £8 (A (D) MRE BAWLA © A) = (B) ak ( (D) te BR RB ——_ fk: a) @ 8) Al wo & w) # RELA PLR LIB © (A) abe (B) AF © oF (D) £F HAWMARAI AAs WR RARMNT © a) ® (B) oO + wo) R GO ONTO THE NEXT PAGE CHINESE TEST WITH LISTENING—Continued 46, Nusfei digi yo singin kudi ln, Késhi WES RK RE SRS DE cchT gin zhii ing bit biokud +8 RRREE “w Wad B) Aimi B22 (©) yishing On % (D) qizhing Oss 49. Taz athuan ER LG bigren de méobing, BRR DE (A) Ww (B) fang ws © wo OR (D) tise ome 48. Suindn ni bit xing qi, késhi cr a a a a di ga. aS ww wo (®) ging 6 © ¥ © (D) tin o 49. Taniing dijibin___Zhdnggu6 shi desho. 4.8 YO ee (A) duiys W Rou (8) guinys @ gw (© youyi Ouw ©) avi Ow 50. Qidatidn 13 ging c6ng Yinggus hus WSS EL RTL ENED, ingeus le, a bar et as aan a ®) ai ® (© ci oe (D) you OZ (GO ONTO THE NEXT PAGE 367) CHINESE TEST WITH LISTENING—Continued si AUK RH si AK __R_ oR WhETs HEAT © (A) Roe (A) ab ne A 8) an ot 1B) ane A (oe (0) ae (D) Boe HB D) BB so aR AL A A + 5 RE LI BEART API FELAT OPE > (A) 84% (A) fia (B) 3p (B) EA (RH () aa (D) Aka (D) 3 FAB A RIN 53. SRE A Ba mk et! (a) (ay (By & wy & oO ® ok wy iD) ® So KRREREEARME 7 SH MG RRREE ARE as ws (B) ve (B) co o (D) ¥# (D) # 85. th EAT 58. th tosh jhtr Ea © Foie tet bis ° (A) Mat (Ay &F @) AH B) Ae (Osa ©) 5% ) ae (D) eit inautiavid copying vous ot ny ar of hs age eg GO ONTO THE NEXT PAGE > > 368 5s. CHINESE TEST WITH LISTENING—Continued Bioshi___feng___ chu do sw dishang qi le. ne (A) ing i we (B) wei dou @) & (©) dK oom ye © % io hai @) g Lin Hiosht__ Fang chide jb, 52.2. »370 © CHINESE TEST WITH LISTENING—Continued Question 57 57. On what day can the coupon be used? (A) Tuesday (B) Friday (© Saturday (D) Sunday Questions 58-59 LAHEE (-) KSB THT ’ ART HK SHie ReR RLOH 400 Hs TR ae © 58. Where would this message most likely be found? 59, How often should this product be used? (A) On a bulletin board: (A) Twice cach month (B) On a medicine label (B) Every three weeks (©) Inan appliance manual (©) Once aday (D) Ina cookbook (D) Every twelve hours any ar of tis pede to ese GO ONTOTHE NEXT PAGE > 371d ® CHINESE TEST WITH LISTENING—Continued Questions 60-61 RHREMAT— Hefei he HAE aR Biba RARE LE AH, RENEBLAT HW, RALWRAAM. RPL R ARE EMD 5] Lie FH PRS ba FFFHAR, FATS SOR. AA LA SLUT, Pra ay kan HL FEES, Baths (AA RASH ARRKG. RRRERKAE-—DA Rte be BE STA Bib F RE RRL Att. ROA wELAL WM, RALBRAT HM. RPEOBA REDE FS) LE PRE Lote FFT Bs, RATS HBR. DA LAA PRR, Aree ay RAR A FREE, AA RAMHFARAK. 160, What docs Professor Huang like about the place where he lives? (A) Fresh air (B) Convenient location (©) Lake-front view (D) Friendly neighbors Question 62 SAAT AS NB + RCE « TBE REBUPEMETTEIS + SEEDERS «IEE PTUEAS RCA TTAT PRENSA RATHER « 61. According to the passage, which of the following is true about Professor Huang’? (A) He lives with his family. (B) He takes the bus to work every day (C) He likes to go swimming in the lake. (D) He walks to school in the morning CMRSIN] SHERRI ALL HASAN + PARR MEN « TBO ReTRETIS * REO PE LNA « BREE ATARI + SISTEMA SRT 62. The passage recommends boarding a bus in front of the (A) sports arena (B) city hall (C) hotel parking lot (D) airport GO ONTO THE NEXT PAGE > Question 63 CHINESE TEST WITH LISTENING—Continued & BE RACER Be RR & KS b RAGA Hs KR 63. What is this advertisement about? (A) A grand opening Questions 64-66 64. This excerpt RBLT, ARRKAA RARAN. Asai WRI. EMD REO Kk, MLAS. PP: bien, alba Ao Bhs BAe RE RA Rap hib a BRA: A AoA, HE FP Ae RELE, RAIA, ® BAKA, ARPM—-ER Theo 33 KR K, GAB He: toe, WA Fro Bee Roly Wh AB] ERA Re WRAL: Ae Hepa, FE Ss (A) fast-food advert (B) restaurant review (C) newspaper food column (D) room-service menu ‘The featured item is a (A) beverage (B) vegetable dish (©) main course (D) dessert any ar of ts pege ool. . Whiel the following is true of the described in the excerpt? (A) It contains various herbs and spices. (B) Ieis qui . (C) Ittastes best when served hot. (D) Iisa simple, easily prepared dish. ® GO ONTO THE NEXT PAGE > 373 ® CHINESE TEST WITH LISTENING—Continued Question 67 AA AS 67. What information is given in the sign? (A) An exchange rate (B) A dosage (© Adiscount (D) A time period Question 68, AG RAAT Sb KBR BE AR aA RM ATES EB BE EE ERB AEM bidp ty SF Lip kates Pe 68, Who sent this letter? (A) A student (B) A manager ident of Shanghai ident of Beijing GO ONTO THE NEXT PAGE > Questions 69-71 CHINESE TEST WITH LISTENING—Continued ALA RAH A W695 LT — RR Aa FB eR AE RRR BAAR DEAE BRM. weet 7 Raa EE BRK BY HIE BR. OR RE RBA APRA AE FRBLASAM Ri. ey FRR RAR. eA AS AER OE, te BR da ASI OEE, He Bak SL AH RL ER, TRAMR AG. MARI EM, TRARRAG, MAH RA, RASARRA, RRA RA, RARB A, RERAK Bh. Scan WME A IR th a ALA ER RH BM AS BEAR HE FARE RR Bo (69. What does Honghong’s brother like to do? 71. What ean be concluded about Honghong? (A) Take pictures of scenery ays (B) Design new hairstyles (B) She wants to become a hair designer. (C) Teach Honghong to read (C) She does not consider her brother an artist, (D) Repair neighbors’ appliances (D) She does not like to have her picture taken. e very much enjoys nature photography 70. What is the occupation of Honghong’s father? (A) Hairdresser (B) Photographer (C) Technician (D) Teacher Question 72 LPR GFSHAP ARIE RAE EF R56 iM 5 BE 04 Op ASS Hah -A St aARe AAR RRC AR MTHS Ate MAES AMIR AZ SASL AE ° LER EERE HR HABEAS IEE BE HS Ree si t-Astame AMAL ERR SK ARH Bas MHAFRS ARMAS FAIRE Ro 72. This is an invitation to a (A) spring festival (B) welcoming reception (C) class reunion (D) student orientation Srp of is eget GO ONTO THE NEXT PAGE 375) ® CHINESE TEST WITH LISTENING—Continued Question 73, BR Ab aE BR Hay 8 Maem: gmasema Fae: EMER HA EPAME FER PAAR TRIM tee mk rors OE ARO A: RAFF 2m sa Racecar: a 73. When is this place closed”? (A) Satunay aftemoon (B) Every Monday (©) During lunchtime (D) From 3 P.M. to4 PM. Questions 74-75 CRAKS B) RH FPRARERG-EK Ww. -h2ZAS+—-A, A-RHRRABA, BM BR. AA RT RR TLK fo Me API TT SAM, 517 BH. RR te Fo — HM Re RATGORMER, ER LENE. BMANT Fy KRG, SS HT CRAASS) BH Re (HAS B) Rif SPRAMSR GH - BR ROMA, AST BR BB Rte to — A He Wy RA RATGERTR, ER ERA. BHAMT By KAM, ORS AT CRASS) RA Re 74, What inspired Mr. Zhang? (A) Watching a traditional Chinese dance (B) Hearing a famous singer (C) Seeing the view from a boat (D) Reading a work by a young poet Ursus spying oy ea] pate ts nage eae 2376 How long did the two people take to complete their work? (A) Three to four hours (B) Five to six days (C) Two to three weeks (D) One to two months GO ONTO THE NEXT PAGE > Questions 76-77 CHINESE TEST WITH LISTENING—Continued RAR AMP ap Gi A Ae Mie EE 8 ADAM ERR ES WE hE 555-7729 BARE AMP AB aE AY 8a eB sk te FAS BAT RA IE AR A Ay & at ihe We 955-7729 76. What position is being advertised? (A) Salesperson (B) Research librarian (C) Restaurant manager () Dietitian Questions 78-79 Which of the following is required of the applicant? (A) Be a college graduate (B) Be experienced (C) Be willing to work long hours (D) Be fluent in Japanese ALRARRH EE. DH DERE SHO ARE, RX AERARR ML GRY LG RA RB ABR. NLR REARS, Mr BBR, HEREEE. TRPG IK O WR, MRAM, BRAS Bo MAKARRTMA, RR ARMAGH, TER PAP RT. MAMA TT Pee RERUA, FMEA, AK BARK, AURRLRE EMR PMA-KATALHRAHA. RNAREKEHSEH. AH MRE EM RAEI, RPPRERRL. FREES BMG AT RMR. HAA SHUM, MAR WBI, KUMGSH. TAP PTEY WR, BRAM, MA TLAE Wo MAKRAGTAG, BR ARUBA OR, TERR PARA AT. MRR S SEP RAULE, RARE, ARI. HORE PUR NERA OMEGA PA. 78. Why is Liu Li especially fond of his grandfather? (A) He taught Liu Li how to read, (B) He was a kung fu master. (C) He knew many folk songs. (D) He took care of Liu Li as a child. any ar of ts pege ool. afternoon? (A) Watch television (B) Read the newspaper (C) Do stretching exercises (D) Go for a walk . What did Liu Li's grandfather usually do in the GO ONTO THE NEXT PAGE > 377) ® CHINESE TEST WITH LISTENING—Continued Question 80 BART AREA FERRARA —. SRRRAN EE RAFRo . FEOHARM RMR. MARRERO FR Ww. BALM, FAR. BAT AREA TAR AM RA —. SRKARNS TR, RBM RH Re =. FEWRARARAR ai Ap At RARE HY OR Fie RS wy. WHEE, SRR. 80. According to the passage, which of the following is supposed to predict fair weather? (A) A west wind blowing in the summer (B) Smoke rising straight up into the sky (C) Snow melting when it touches the ground (D) Mountains appearing to be closer than they RERTRM LEH PREM’ 0 ARB RR AAA GA © O iis +H As: RR BR RM AEH LRA SRSA RS BA RMA HRA really are Questions 81-82 KAP MRR KA PARES PEAEMTE Lis: O BRIE RE bite RECS AFA fo BiGM © TAB A BE ERRA GB © O Pa eis +iR ds * Rew Rs RAs trwis > AE. AT KRG EA» RAMA PROM > 81. What does this flyer advertise? (A) A class in business English (B) A new method for teaching conversational English (©) A video for beginning English learners (D) An audioeassctte series for learning English Unauthorized copying or ease o amy par tive nage ee D378 '82. Which of the following is NOT included? (A) Pronunciation drills (B) Sentence pattems (C) Writing exercises (D) A vocabulary list GO ONTO THE NEXT PAGE > © CHINESE TEST WITH LISTENING—Continued © Questions 83-85, AASB ARERR YP a EMER. ARAM RH SAE seam: ANAS +—A AKA ES EPR PAT WRB ARS GLa AM HAE SA Hr eit: AA+LAS=+—B 83. For which of the following is the largest discount 85. How long does the sale last? offered? (A) One day (A) Books (B) Three days (B) Gift items (©) Aweek (C) Computers (D) A month (D) Stationery 84. What is the occasion for the sale? (A) A grand opening (B) An anniversary (C) Ahioliday (D) A closeout END OF SECTION III, STOP IF YOU FINISH BEFORE TIME IS CALLED, YOU MAY CHECK YOUR WORK ON SECTIONS II AND II. DO NOT TURN TO ANY OTHER TEST IN THIS BOOK. sry pertot he pa tegnt 379) ‘The Official Study Guide for All SAT Subject Tests How to Score the SAT Subject Test in Chinese with Listening ‘When you take an actual SAT Subject Test in Chinese with Listening, you receive an overall composite score as well as three subscores: one for the listening section, one for the reading section, and one for the usage section, ‘The listening, reading, and usage scores are reported on the College Board's 20-80 scale. However the composite score, which is the most significant of the scores reported to the colleges you specify, isin the form of the College Board’s 200-800 scale. Worksheet 1. Finding Your Raw Listening Subscore STEP 1: Table A lists the correct answers for all the questions on the Subject Test in Chinese with Listening that is reproduced in this book. It also serves as a worksheet for you to calculate your raw Listening subscore. + Compare your answers with those given in the table. + Puta check in the column marked “Right” if your answer is correct. + Puta check in the column marked “Wrong” if your answer is incorrect. + Leave both columns blank if you omitted the question. STEP 2: Count the number of right answers for questions 1-19. Enter the total here: STEP 3: Count the number of wrong answers for questions 1-19. Enter the total here: STEP 4: Multiply the number of wrong answers from Step 3 by .500. Enter the product here: STEP 5: Subtract the result obtained it Step 4 from the total you obtained in Step 2. Enter the result here: STEP 6: Count the number of right answers for questions 20-30. Enter the total here: STEP 7: Count the number of wrong answers for questions 20-30. Enter the total here: — STEP 8: Multiply the number of wrong answers from Step 7 by .333. Enter the product here: STEP 9: Subtract the result obtained in Step 8 from the total you obtained in Step 6. Enter the result here: »380 STEP 10: Add the result obtained in Step 5 to the result obtained in Step 9. Enter the sum here: STEP 11: Round the number obtained in Step 10 to the nearest whole number. Enter the result here: The number you obtained in Step 11 is your raw Listening subscore. 381) ‘The Official Study Guide for All SAT Subject Tests Worksheet 2. Finding Your Raw Reading Subscore STEP 1: Table A lists the correct answers for all the questions on the Subject Test in Chinese with Listening that is reproduced in this book. It also serves as a worksheet for you to calculate your raw Reading subscore. STEP 2: Count the number of right answers for questions 56-85. Enter the total here: STEP 3: Count the number of wrong answers for questions 56-85. Enter the total here: STEP 4: Multiply the number of wrong answers by .333, Enter the product here: STEP 5: Subtract the result obtained in Step 4 from the total you obtained in Step 2. Enter the result her STEP 6: Round the number obtained in Step 5 to the nearest whole number. Enter the result here: The number you obtained in Step 6 is your raw Reading subscore. D382 Worksheet 3. Finding Your Raw Usage Subscore STEP 1: Table A lists the correct answers for all the questions on the Subject Test in Chinese with Listening that is reproduced in this book. It also serves as a worksheet for you to calculate your raw Usage subscore. STEP : Count the number of right answers for questions 31-55. Enter the total here: STEP Enter the total here: : Count the number of wrong answers for questions 31-55. STEP 4: Multiply the number of wrong answers by .333. Enter the product here: STEP 5: Subtract the result obtained in Step 4 from the total you obtained in Step 2 Enter the result here: STEP 6: Round the number obtained in Step 5 to the nearest whole number. Enter the result here: The number you obtained in Step 6 is your raw Usage subscore. 383) ‘The Official Study Guide for All SAT Subject Tests Worksheet 4. Finding Your Raw Composite Score STEP 4: Enter your unrounded raw Listening subscore from Step 10 of Worksheet 1. Enter the result here: STEP 2: Enter your unrounded raw Reading subscore from Step 5 of Worksheet 2 Enter the result here: STEP 3: Enter your unrounded raw Usage subscore from Step 5 of Worksheet 3. Enter the result here: STEP 4: Add the results obtained in Steps 1, 2, and 3 Enter the sum here: STEP 5: Round the number obtained n Step 4 to the nearest whole number. Enter the result here: ‘The number you obtained in Step 5 is your raw composite score. D384 Chinese with Listening Table A Answers to the Subject Test in Chinese with Listening, Form 3ZLC, and Percentage of Students Answering Each Question Correctly Brace Eng 8 o c A 8 c c 96 c B 9 A o 8 c 0 a A B 94 c A 31 c B % c o 19 iT A % 8 A 8 2 8 9 a A oa 8 A 8 45 B 0 4 c 7 46 B 69 5 c 34 a o 0 16 A 98 8 c 7% v 8 9% 49 B 8 18 A cl 50 D 79 18 A 2 st A 2 a 8 2 52 A a2 2 D s 53 B 70 2 D a 5a c 65 B c 99 5 c a7 a D % 56 D 2 B A 8 5 A 9 6 c a 58 E 2 a 8 6 59 D a B c Co) 0 A a a A 64 8 e 4 30 D a 62 A 6 31 C 8 63 a 0 32 8 % Cy c 6 Table A continued on nest page 385) ‘The Official Study Guide for All SAT Subject Tests Table A continued from previous page ers pete ies 6 8 5 16 c 13 66 D n n 8 n 6 c 10 18 D 8 68 c 64 19 8 19 68 A 92 80 8 0 70 5 82 8 D 55 n c 68 82 c 68 2 8 45 83 D 59 B 8 70 8h c 8 m c 1B 85 8 9 15 8 Ey > These percentages are based on an analysis ofthe answer sheets of a representative sample of 1,363 students who took the original form of this test in November 2003, and whose mean composite score was 723. They may be used as an indication ofthe relative dificult of particular question, Each percentage may also be used to predict the likelihood that atypical SAT Subject Test in Chinese with Listening candidate will answer that {question correctly on this edition ofthe test. D386 Chinese with Listening Finding Your Scaled Score When you take SAT Subject Tests, the scores sent to the colleges you specify are reported on the College Board scale, which ranges from 200-800. Subscores are reported on a scale which ranges from 20-80. You can convert your practice test scores to scaled scores by using Tables B, C, D and E. To find your scaled score, locate your raw score in the left-hand column of the table; the corresponding score in the righthand column is your scaled score. For example, a raw score of 59 on this particular edition of the Subject Test in Chinese with Listening corresponds to a scaled composite score of 690. Raw scores are converted to scaled scores to ensure that a score earned on any one edition of a particular Subject Test is comparable to the same scaled score earned on any other edition of the same Subject Test. Because some editions of the tests may be slightly easier or more difficult than others, College Board scaled scores are adjusted so that they indicate the same level of performance regardless of the edition of the test taken and the ability of the group that takes it, Thus, for example, a score of 400 on one edition of a test taken at a particular administration indicates the same level of achievement as a score of 400 on a different edition of the test taken at a different administration. When you take the SAT Subject Tests during a national administration, your scores are likely to differ somewhat from the scores you obtain on the tests in this book. People perform at different levels at different times for reasons unrelated to the tests themselves. The precision of any test is also limited because it represents only a sample of all the possible questions that could be asked. Your scaled composite score from Table B is . Your scaled listening score from Table C is . Your scaled reading score from Table D is . Your scaled usage score from Table E is . 387) ‘The Official Study Guide for All SAT Subject Tests Table B Scaled Score Conversion Table Subject Test in Chinese with Listening Composite Score (Form 3ZLC) 85 ‘800 47 630 9 430 84 800 46 620 8 420 a wo | 45 za 7 x0 a 00 4 an 6 0 a #00 43 ono 5 0 30 #00 a 600 4 400 13 7 4 590 3 400 78 790 40 520 2 a0 n 70 P sen 1 30 36 7e0 38 se a 30 7 770 T 570 1 30 4 0 26 570 2 m0 73 760 % se 3 370 2 750 3 se 4 30 n 250 3 $0 5 30 70 70 2 sen $ 360 69 40 ci 540 7 350 8 70 0 san 8 340 @7 m | 2 520 2 240 66, 720 28 530 10 330. 65 720 an 520 aT) 330 64 m0 26 520 M2 320 8 70 ra 510 3 x0 m 700 4 510 4 a0 a 700 z san 45 10 0 om Zz S00 “16 300 58 ew | 2 4s0 a7 230 8 a0 20 4s0 “3 20 7 a0 8 40 “3 20 56 m | 40 20 20 Ea 70 7 770 ZI 770 st ) 6 70 2 770 58 0 5 4 23 20 PI 650 ri 40 24 20 a 650 8 40 25 250 0 640 12 0 26 20 43 «40 1 40 2 20 48 620 0 30 28 40 29 230 30 20 31 220 D388 Chinese with Listening Table C Scaled Score Conversion Table Subject Test in Chinese with Listening Listening Subscore (Form 3Z1.C) 30 15 a 0 7B 14 53 4 41 8 ™ 13 53 2 40 a n 2 52 3 39 6 69 1" 51 4 38 B 6 10 51 3 37 2 64 9 50 6 35 a cy 8 49 q 4 a 6 1 49 8 33 a oo 6 48 9 32 20 Ey 5 a 10 3 9 58 4 46 1 30 8 ST 3 45 12 2 7 56 2 a 8 28 6 55 1 a8 389) ‘The Official Study Guide for All SAT Subject Tests Table D Scaled Score Conversion Table Subject Test in Chinese with Listening Reading Subscore (Form 3ZLC) 2 80 5 68 1 a7 8 19 “ 87 0 45 a a B 88 4 44 2B n 2 85 2 2 B 76 1 64 3 a 24 8 10 63 4 a a B 9 82 4 40 2 4 a 81 6 40 a 3 7 59 a 40 20 72 é 58 Fy 8 n 5 56 4 39 8 10 4 54 a0 38 wv 69 3 52 »390 Chinese with Listening Table E Scaled Score Conversion Table Subject Test in Chinese with Listening Usage Subscore (Form 3ZLC) as) Raw Score| Score _|Raw Score| 25 80 4 4 80 1B 23 79 2 2 n " 2 78 10 20 4 a 19 73 8 18 n 7 7 10 6 16 68 5 5 a 4 80 7 4 8 32 3910 ‘The Official Study Guide for All SAT Subject Tests How Did You Do on the Subject Test in Chinese with Listening? After you score your test and analyze your performance, think about the following question Did you run out of time before reaching the end of the test? If'so, you may need to pace yourself better. For example, maybe you spent too much time on one or two hard questions. A better approach might be to skip the ones you can't answer right away and try answering all the questions that remain on the test. Then if there’s time, go back to the questions you skipped. Did you take a long time reading the directions? You will save time when you take the test by learning the directions to the Subject Test in Chinese with Listening ahead of time, Each minute you spend reading directions during the test is a minute that you could use to answer questions. How did you handle questions you were unsure of? Ifyou were able to eliminate one or more of the answer choices as wrong and guess from the remaining ones, your approach probably worked to your advantage. On the other hand, making haphazard guesses or omitting questions without trying to eliminate choices could cost you valuable points. How difficult were the questions for you compared with other students who took the test? Table A shows you how difficult the multiple-choice questions were for the group of students who took this test during its national administration. The right-hand column gives the percentage of students that answered each question correctly. A question answered correctly by almost everyone in the group is obviously an easier question. For example, 98 percent of the students answered question 4 correctly, But only 37 percent answered question 55 correctly. Keep in mind that these percentages are based on just one group of students. They would probably be different with another group of students taking the test. If you missed several easier questions, go back and try to find out why: Did the questions cover material you haven't yet reviewed? Did you misunderstand the directions? D392 Chapter 9 French Purpose There are two Subject Tests in French: French and French with Listening, Both tests evaluate your reading skills through precision of vocabulary, structure use, and comprehension of a variety of texts. The Subject Test in French with Listening measures your ability to understand spoken as well as written French, Format +The Subject Test in French takes one hour and includes 85 multiple-choice questions. + The Subject Test in French with Listening also takes one hour, with about 20 minutes for listening questions and 40 minutes for reading questions. There are 85 to 90 multiple-choice listening and reading questions. Content Both tests evaluate your reading ability in three areas through a variety of questions requiring a wide-ranging knowledge of French: Precision of Vocabulary questions test knowledge of words representing different parts of speech and some basic idioms within culturally authentic contexts Structure questions measure your ability to select an appropriate word or expression that is grammatically correct within a sentence. One part of the test contains vocabulary and structure questions embedded in longer paragraphs. Reading comprehension questions test your understanding of such points as main and supporting ideas, themes, and setting of a passage. Selections are drawn from fiction, essays, historical works, newspaper and magazine articles, or everyday materials such as advertisements, timetables, forms, and tickets. French red ees Vocabulary in Context Structure 30-40 Reading Comprehension 30-40 393) ‘The Official Study Guide for All SAT Subject Tests In addition to these reading questions, the Subject Test in French with Listening also measures your ability to understand the spoken language with three types of listening questions: ‘Type one asks you to identify the sentence that most accurately describes what is presented in a picture or a photograph or what someone in the picture or photograph might say. ‘Type two tests your ability to answer general content questions based on short dialogues or monologues. ‘Type three requires you to answer more specific questions based on longer dialogues or monologues. French with Listening Listening Section (20 Minutes) 35 Pictures 8-12 questions Short dialogues: 6-12 questions Long dialogue 10-15 questions Reading Section (40 minutes) 65 Vocabulary 16-20 questions Structure 16-20 questions Reading Comprehension: 20-25 questions How to Prepare Both tests are written to reflect general trends in high school curricula and are independent of particular textbooks or methods of instruction, The French ‘Tests are appropriate for you if you have studied the language for three or four years in high school, or the equivalent; however, if you have two years of strong preparation in French, you are also encouraged to take the tests. Your best preparation for the tests is a gradual development of competence in French over a period of years. Familiarize yourself with the directions in advance. The directions in this book are identical to those that appear on the test, French with Listening A practice audio CD is included with this book. A practice CD with different sample questions can be obtained, along with a copy of the SAT Subject Tests Preparation Booklet, D394 French from your school counselor, or you can access the listening files at www.collegeboard, com, You should also take the practice test included with this book CD Players Using CD Players for Language Tests with Listening ‘Take an acceptable CD player to the test center. Your CD player must be in good working order, so insert fresh batteries on the day before the test. You may bring additional batteries and a backup player to the test center. CD players cannot be shared with other test-takers, ‘Test center staff won't have batteries, CD players, or earphones for your use, so your CD player must be: + equipped with earphones + portable (hand-held) + battery operated You are not allowed to use a CD player with recording or duplicating capabilities, Note Ifthe volume on your CD player disturbs other test-takers, the test center supervisor may ask you to move to another seat, ‘What to do if your CD player malfunctions: + Raise your hand and tell the test supervisor. + Switch to backup equipment if you have it and continue the test. If you don’t have backup equipment, your score on the Subject ‘Test in French with Listening will be canceled. But scores on other Subject Tests you take that day will still be counted, ‘What if you receive a defective CD on test day? Raise your hand and ask the supervisor for a replacement. Scores For both tests, the total score is reported on the 200-to-800 scale, For the listening test, listening and reading subscores are reported on the 20-to-80 scale. 395) ‘The Official Study Guide for All SAT Subject Tests Sample Reading Questions Four types of reading questions are used in the French Tests. All questions in the tests are multiple-choice questions in which you must choose the BEST response from the four choices offered. ‘Your answer sheet has five answer positions marked A, B, C, D, and E, while the questions throughout this test contain only four choices. Be sure NOT to make any marks in column E. PartA Directions: This part consists of a number of incomplete statements, each having four suggested completions. Select the most appropriate completion and fill in the corresponding circle on the answer sheet. 1, fai perdu mon argent parce qu’il y avait un trou dans la ... de mon pantalon. (A) manche (8) jambe © poche (D) ceinture Choice (C) is the correct answer because pants have pockets in which people keep money. ‘This question tests vocabulary. You are asked to choose the appropriate noun from the four answer choices. Choices (A) and (D) are not normally used to carry money, and choice (B) refers to a part of the body. 2. Charles avait tant mangé qu'il ne pouvait plus ... une bouchée. (A) soutenir (8) emporter (© avaler (D) évaluer Choice (C) is the correct answer to question 2. In this question, you are asked to find the appropriate verb from the four answer choices. The verb avaler is the only option that can be used correctly in connection with une bouchée. Choices (A), (B), and (D) are incorrect. D396 French Part B Directions: Each of the following sentences contains a blank. From the four choices given, select the one that can be inserted in the blank to form a grammatically correct sentence and fill in the corresponding circle on the answer sheet, Choice (A) may consist of dashes that indicate that no insertion is required to form a grammatically correct, sentence. 3. Dans sa cuisine, il fallait toujours que tout___impeccable et reluisant. (A) est (B) soit © était (D) serait Choice (B) is the correct answer to question 3 because from the four answer choices soit is the correct form of the verb étre, You need to know that il fallait que in the sentence is the past tense of il faut que, an impersonal expression that is followed by a verb in the subjunctive, Choices (A), (C), and (D) are forms of étre in the indicative and are therefore incorrect. 4, _____ est le meilleur joueur de cette équipe? (A) Qu (B) Quelle ©) Quest-ce qu’ (D) Qui Choice (D) is the correct answer to question 4. In this question you are asked to choose the appropriate pronoun from the four answer choices. The question mark tells you that the missing pronoun is interrogative and the verb est tells you that itis the subject of the sentence. Qui is an interrogative pronoun and the subject. Choices (A), (B), and (C) are incorrect because choice (A) cannot be used as a subject, choice (B) is an interrogative adjective, and choice (C) is an interrogative pronoun used as a direct object. 397) ‘The Official Study Guide for All SAT Subject Tests Part C Directions: The paragraphs below contain blank spaces indicating omissions in the text. For some blanks it is necessary to choose the completion that is most appropriate to the meaning of the passage; for other blanks, to choose the one completion that forms a grammatically correct sentence, In some instances, choice (A) may consist of dashes that indicate that no insertion is required to form a grammatically correct sentence. In each case, indicate your answer by filling in the corresponding circle on the answer sheet. Be sure to read the paragraph completely before answering the questions related to it, Das que vous _5_ le temps de prendre contact avec elle, donnez-_6 _ un coup de téléphone. II faut Pavertir que tout soit arrangé et que jarriverai _7_ ving. 5. (A) auriez (B) ayer (©) aurez (D) aviez 6 (A) lui ®) alle © vous ©) la 7. (A) le (B) au © surle (D) dansle 5. Choice (D) is the correct answer to question 5; aurez is the future tense of the verb avoir. Expressions such as quand and des que are followed by the future in French when the verb in the main clause is in the present tense, as it is here with the present imperative donnez, Choice (A) auriez is the conditional, choice (B) ayez is the present subjunctive, and choice (D) aviez is the imperfect. 6. Choice (A) is the correct answer to question 6. What is missing in this part of the sentence is an indirect object pronoun that refers back to elle (in avec elle). The indirect object indicates the person to whom the coup de téléphone should be given, ‘The correct pronoun form in question 6 is lui. Choice (B) elle is not correct because it is used for the subject of a sentence or after a preposition, choice (C) vous is incorrect because something should be given to the woman designated by elle, not the person spoken to, and choice (D) la is incorrect because itis the direct object pronoun, not the indirect object pronoun. D398 French 7. Choice (A) is the correct answer to question 7. When giving arrival and departure dates in French (the sentence here provides an arrival date), the date is preceded by le without a preposition. The other suggested answers contain prepositions and are therefore incorrect. Part D Directions: Read the following selections carefully for comprehension. Each selection is followed by a number of questions or incomplete statements. Select the completion or answer that is BEST according to the selection and fill in the corresponding circle on the answer sheet. «image Center» est I’histoire d’une passion, Hésitant entre Vart et la science, Sylvie Magnus, 24 ans, passe deux ans & Ecole des Beaux Arts et complete sa formation 4 Londres, Lugne oi elle apprend les applications de informatique sur Vimage. Et cest le déclic, peindre avec la lumiére, créer des décors magiques pour des défilés de mode, ou des effets spéciaux pour le cinéma, tout la fascine, Une étude de marché lui apprend qu'il nexiste pas d’agence spécialisée dans la conception de ces images. Sylvie décide 10 donc de combler lespace: elle crée, grace 4 un prét de famille et & des subventions, la premiére agence européenne conseil en image de synthése: «Image Center». 8. Quiest-ce que Sylvie Magnus a étudié aprés ses deux ans &l’Ecole des Beaux Arts? (A) Les arts décoratifs (B) La cinématographie (© Les nouvelles technologies (D) La médicine Choice (©) is the correct answer to qustion 8. The text tells you that Sylvie Magnus studied computer graphics after finishing her fine arts education (Sylvie Magnus passe deux ans ii I'Bcole des Beaux Arts et complete sa formation a Londres, oit elle apprend les applications de 'informatique sur image, lines 2-5). The other choices are incorrect. 399 ‘The Official Study Guide for All SAT Subject Tests 9, Ala ligne 10, «combler l'espace» veut dire (A). répondre a un besoin (B) over un bureau (©) faire des recherches scientifiques (D) faire des subventions Choice (A) is the correct answer to question 9. The text states that Sylvie Magnus has learned that there was no agency that specialized in computer graphics and decided to create one. The expression does not mean that she rented an office (B), did s research (C), or subsidized anything (D). On the contrary, she received a subsidy (line 1). 10. Comment est-ce que Sylvie Magnus a trouvé argent pour lancer «Image Center»? (A)_ Elle a travaillé dans un hépital, (B) Elle en a gagné pendant la révolution. (©) Elle a organisé des défilés de mode. (D) Elle en a emprunté a ses parents. Choice (D) is the correct answer to question 10. The text tells you that grice d un prét de famille (lines 10-11) Sylvie Magnns was able to create her agency. She did not obt: the money by working in a hospital (A), earning it during a revolution (B), or organizing fashion shows (C). Sondage Vous, amateurs de télé— Question 1 Utilisez-vous personnellement une télécommande? Question 2 Vous-méme, quand vous utilisez cette télécommande, vous vous en servez pour: couper le son et faire autre chose? changer de chaine des que le programme ne vous plait pas? suivre plusieurs émissions en méme temps? chercher une émission particuligre? éviter la publicité? rechercher la publicité? » 400 Résultat: jon des Frangais Agés de 16 ans et plus Question 1: qui utilisent personnellement une télécommande Que: ‘en servent pour —couper le son pour Faire autre chose 6 —changer da chaine ds que le programme ne leur plait pas 4B —suive plusieurs émissions en mame temps 2 cchorcher une émission particule a —éviter la publics a —rechercher la publicits 2 11, Quest-ce qu'une télécommande? (A) Une sorte de téléviseur (B) Une émission de télévision (©) Une sorte de publicité (D) Un apparel électronique Choice (D) is the correct answer to question 11. What a télécommande is must be inferred because it is not stated directly in the survey. The text tells you that, among other things, a télécommande can be used to change television channels and to cut the sound of a program. It is a remote control. It is not a kind of television set (A), nor a television program (B), nor publicity (C). 12, Selon ce sondage, on se sert le plus souvent d'une télécommande pour (A) acheter quelque chose (B) trouver une émission plus intéressante (©)_ pouvoir regarder deux émissions & la fois (D)_ vérifier le bon fonctionnement de son téléviseur Choice (B) is the correct answer to question 12. This question asks you what the remote control is most frequently used for, according to the survey results. You must select the use in the chart that was selected by the most respondents and has the highest percentage This use is changer de chaine dés que le programme ne leur plait pas, or “change the channel as soon as they no longer like the program,” The other choices were selected by a lower percentage of respondents. 401) French Test Practice Helps The test that follows is an actual, recently administered SAT Subject Test in French. To get an idea of what it’s like to take this test, practice under conditions that are much like those of an actual test administration. © Set aside an hour when you can take the test uninterrupted. Make sure you complete the test in one sitting. © Sit ata desk or table with no other books or papers. Dictionaries, other books, or notes are not allowed in the test room. © — Tear out an answer sheet from the back of this book and fill it in just as you would on the day of the test. One answer sheet can be used for up to three Subject Tests. © Read the instructions that precede the practice test. During the actual administration you will be asked to read them before answering test questions. © Time yourself by placing a clock or kitchen timer in front of you © After you finish the practice test, read the sections "How to Score the SAT Subject Test in French” and “How Did You Do on the Subject Test in French?” The appearance of the answer sheet in this book may differ from the answer sheet you see on test day. D402 FRENCH TEST The top portion of the section of the answer sheet that you will use in taking the French Test must be filled in exactly as shown in the illustration below. Note carefully that you have to do all of the following on your answer sheet. 1. Print FRENCH on the line under the words “Subject Test (print) 2. In the shaded box labeled “Test Code" fill in four circles: —Fill in circle in the row labeled V. in the row labeled W —Fill in circle | in the row labeled X. Je B in the row labeled Y. —Fill in circle Sano siet Tat on * 0980000000 FRENCH 5®@ 9909 10000 @ OOOO OO OO Please answer either Part [ or Part II by filling in the specific circle in row Q. You are to fill in ONE and ONLY ONE, , as described below, to indicate how you obtained your knowledge of French. The information you provide is for statistical purposes only and will not influence your score on the test PartI If your knowledge of French docs not come primarily from courses taken in grades 9 through 12, fill in cirele 9 and leave the remaining circles blank, regardless of how long you studied the subject in school For example, you are (0 fill in cirele 9 if your knowledge of French comes primarily from any of the following sources: study prior to the ninth grade, courses taken at a college, or special study, livin home in which Prench is the principal language spoken, or extensive residence abroad that includes significant experience in the French language. ina Part I If your knowledge of French does come primarily from courses taken in secondary school, fill im the circle that indicates the level of the French course in which you are currently enrolled. If you are not now enrolled in a French course, fill in the circle that indicates the level of the most advanced course in French that you have completed. © First year: first or second half —Fill in circle 1 © Second year: first half —Fill in circle 2, second half —Fill in circle 3, © Third year: first half —Fill in circle 4. second half —Fill in cirele 5, ‘© Fourth year: first half —Fillin circle 6, second half —Fill in circle 7, © Advanced Placement course or a course at a level higher than fourth year, second half high school course work plus a minimum of four weeks of study abroad Fill in circle 8, ‘When the supervisor gives the signal, turn the page and begin the French Test. There are 100 numbered circles on the answer sheet and 85 questions in the French Test. Therefore, use only circles 1 to 85 for recording your answers. Drauthoriaed copying arouse al] [any prof ts pee os 403) ® PL , WHILE THE QUI TO MAKE ANY MARKS IN COLUMN E. Directions: This part consists of a number of incomplete statements, each ha in the corresponding cirele on the answer sheet. Select the most appropriate completion and fi 1. Blle ne peut pas se le payer. Cest beaucoup trop pour elle, (A) cher (B) fort (©) gentil (D) plein 2. Ce matin je n'ai pas eu le temps de lire le - (A) paquet (B) nouveau (©) journal (D) magasin 3. Le chef du personnel a ordoni portes des bureaux soien! aque toutes les cing heures. (A) sernées (B) fumées. (©) levees 4, Tuas vraiment mauva (A) oublies (B) obliges (©) perds (D) poses. 5. Attendez! Je ne peux pas vous comprendre si vous parlez tous (A) alla fois (B) aPheure (©) aPoceasi n (D) ala prochaine axac parm FRENCH TEST 6 10. ® EASE NOTE THAT YOUR ANSWER SHEET HAS FIVE ANSWER POSITIONS MARKED A, B,C, D, and STIONS THROUGHOUT THIS TEST CONTAIN ONLY FOUR CHOIC . BE SURE NOT ng four suggested completions. Nous sommes en retard. Il faut nous (A) endormir (B) dépécher (C) promener (D) moucher Suzanne est punie parce qu'elle a (A) pris (B) donné (C) preté (D) vole 11 pourra quitter Phopital ds qu'il n'aura plus de (A) cofire (B) fievre (©) drogue (©) confort Claudette s'est servie d'une - Tes mains. pour s'essuyer (A) assiete (B) fourchette (C) allumette (D) serviette Denise a 616 -- affectée par le vol de sa voiture. (A) brillamment (B) cordialement (C) sagement (D) fortement GO ONTOTHE NEXT PAGE > ® Directions: FRENCH TEST—Continued ® Each of the following sentences contains a blank. From the four choices given, select the one that can be inserted in the blank to form a grammatically correct sentence and fill in the corresponding ‘on the answer sheet. Choice (A) may consist of dashes that indicate that no insertion is required to form a grammatically correct, sentence. 23. On me demande toujours c vais faire de ma vie. Je n’en sais rien! (A) ee dont (B) quoi (©) ce que (D) quests qui 24. Je vois que vous aimer les livres. Vous - avez beaucoup dans votre bibliotheque. (A) tui (B) leur (©) en Dy 25. David et jel sont sortis de (A) courant (B) ayant couru (C) courir (D) avoir coura 26. 11 est important de ------- soigner, Madame. Prenez.un cachet d’aspitine et t (A) vous (B) toi © tui Dt 27. Je qu'il devait me tléphoner. (A) regrette (B) pense (©) doutais (D) eraignais amv pettus paoe ates as 28. 30. 31 ‘Apres accident, on a transporté Sandrine au hopital de la ville (A) nouvel B) nee (C) beau (D) moderne Les agents ont suggéné que tu addresses & pour te renseigner. (A) leur (B) eux (©) soi (D) ceux (A) sienne (B) autre (C) leurs. (D) blew 1 me semble que la Tour d' Argent est un des restaurants de France; la cuisine y est excellente (A) meilleurs (B) mieux (C) supérieurs (D) bien V’ai rendu visite a Philippe en (A) juillet (B) Paris (C) printemps, (D) dimanche GO ONTOTHE NEXT PAGE > ® 33, o4. 3s. 36, Heureusement, le roman n’était pas cennuyeux que le film, (A) assez (B) tant ©) pew (D) aussi Nous invitons - (A) plusieurs (B) beaucoup (©) un peu () un peu? Méme si tu n’as pas beaucoup de temps pour visiter la ville, va ~ (A) moins, (B) moins de (©) le moindre (D) au moins D'un seul geste, elles ont toutes -—---- la main. A) le (B) leves (C) leve (D) le a FRENCH TEST—Continued 37. 38, 39, Quel e peinture préférez-vous, cette peinture-ci ou qui est la-bas? (A) laquelle (B) Pune (©) cee (D) celle ‘A ma grande surprise, ce film ~ beaucoup plu. (A) lui (Br (©) en (D) les Cette année, les es kilo. (A) par (B) un (© Ie (D) pour GO ONTO THE NEXT PAGE > 407) © FRENCH TEST—Continued ® Part C Directions: ‘The paragraphs below contain blank spaces indicating omissions in the text. For some blanks, itis necessary to choose the completion that is most appropriate to the meaning of the passage; for other blanks, to choose the one completion that forms a grammatically correct sentence. In some instances, choice (A) may consist of dashes that indicate that no insertion is required to form a grammatically correct sentence. In cach case, indicate your answer by filling in the corresponding circle on the answer sheet. Be sure to read the paragraph completely before answering the questions related to it. C’Gtait en général (40) la fin de Papres-midi que 40.(A) vers (B) par (41) alla porte de Pimmeuble. Je montais vite (C) autour de (D) atravers jusqu’a I'appartement. Jacques m'accueillait avec un 41. (A) je sonnais, sourire empressé. «Je ne_(42)__dérange pas?» «Tu ne (B) fouvrais (C) je sontais me déranges_(43)_ .» «Comment ga va» «Ga va (D) je quittais toujours tres bien _(44) je te vois.» Sa gentillesse me 42. (A) tui (B) les touchait et me _(45) le coeur. (C) me te 43. (A) jamais (B) tien (C) aucun (D) personne 44.(A) quand, (B) mais. (C) done (D) pourtant 45. (A) réchauffait (B) batait (C) langait (D) rendait [amv pettus pao ras al] GO ONTOTHE NEXT PAGE > as ® FRENCH TEST—Continued Le juge d’instruection pensait au début qu'il '46)_arriver & une décision la semaine suivante. Pourtant, (47) entendu _(48)_disaient les il a changé 49) 46, 47. 48. 49, (A) apu (B) pourrait (©) pourra (D) puisse A) apres (B) bien (©) ayant (D) sans, (A) ce qui (B) ee que (©) quoi (D) aquot (A) @avis (B) opinion (©) samémoire (D) de esprit GO ONTO THE NEXT PAGE 409 [—Continued © Part D Directions: Read the followi incomplete statements. Select th circle on the answer sheet xts carefully for comprehension. Each is followed by a number of questions or swer Or completion that is best according to the text and fill in the corresponding Orages en fuite, chaleur & la baisse, vents d’ouest et nuages arrivent, ee 58, Demain il y aura moins de 59, Quel temps fera-til demain a Lyon? (A) chaleur (A) Il pleuvra, (B) vent (B) I fera beau. (©) nuages (©) Mneigera. (D) fraicheur (D) Ily aura des orages. [Tnauioriaed spying op reuse oT GO ONTO THE NEXT PAGE > aud ® Peter Schulz avait soixante-quinze ans. I était de santé délicate, et age ne I’avait pas épargné. Sa vie avait été pauvre en événements. Il était seul depuis des années, Sa femme était morte, Il en conservait un souvenir attendti. ILy avait vingtcing ans qu'il l'avait perdue: et, pas un soir depuis, il ne s'était endormi, ‘sans un petit entretien mental, triste et tendre, avec 1 Fassociait A chacune de ses journées. Il n’avait pas eu d’enfants: ¢°étail le grand regret de sa vie. avait reporté son besoin d'affection sur ses auxquels il était attaché, comme un pere A ses fils. avait trouvé peu de retour. Un vieux coeur peut se semtir ts pres d'un jeune coeur, et presque du méme fige: i sait combien sont breves les années qui I'en séparent. Mais le jeune homme ne s'en doute point: le vieillard est pour Iui un homme dune autre époque. Le vieux Schulz avait rencontré parfois quelque reconnaissance chez des éleves, touchés par I'intérét Vif et frais qu'il prenait & tout ce qui leur arrivait «heurcux ou de malheureux: ils venaient le voir de temps en temps: ils lui écrivaient, pour le remereier, quand ils quittatent Pu certains lui écrivaient encore, une ou deux fois, les années suivantes. Puis, le vieux Schulz n’entendait plus parler deux, sinon p: les journaux, qui lui faisaient connaitre avancement de tel ou tel: et il se réjouissait de leurs succes, comme sie étaient les siens. II ne leur en voulait pas de leur il y trouvait mille excuses; il ne doutait point de leur affection, et prétait aux plus égoistes les sentiments qu'il avait pour eux. versité; 60. Le grand regret de la vie de Schulz était (A) @avoir perdu sa femme (B) avoir des éleves ingrats (©) eétre sans enfants, (D) de ne pas avoir une bonne santé ol attitude de Schulz (A) Il s'intéresse beaucoup & leurs vies. (B) Iles trouve égoistes, (C) Mest jaloux de leur su (D) Ine ies trouve pas du tout reconnaissants. FRENCH TEST—Continued ® 62. Quelle a 64¢ Ia profession de Peter Schulz? A) M (B) Professeur (C) Journaliste (D) Psychologue 63. Selon Pauteur, qu’est-ce qui décrit le mieux les rapporis entre les jeunes et les Vieux’? (A) Is se respectent mutuellement. (B) Ils partagent les mémes idées. (C) Les vieux se sentent plus proches des jeunes que les jeunes des vieux. (D) Les jeunes estiment plus les views quand ils ‘ont eux-mémes vi (B) Quand ils étaient heureux (C) Quand ils avaient fini leurs études universitaires (D) Quand ils avaient trouvé du travail 65. Comment Schulz apprend-il les nouvelles de ses. anciens éteves?” (A) Ses anciens é1eves lui éerivent souvent. (B) Il leur écrit pour demander des nouvelles. (C) Hit les journaux, (D) L’université lui envoie des nouvel 66. Selon I'auteur, Peter Schulz a eu une vie (A) tres matheure (B) active et mouvementée (C) bien appréciée et respectée (D) consacrée au succes des autres GO ONTO THE NEXT PAGE > ® ‘Cela devient malheureusement une banal de constater le déclin de la natalité dans notre pays et, plus particuligrement, dans les grandes villes. Pour des parents, avoir des enfants et les élever représente, en milieu urbain, des contraintes beaucoup plus fortes qu’ ailleurs, notamment lorsque le pore et la mere travaillent. Une contrainte se situe souvent au niveau du et el, dans les logements neuls, "es sont souvent réduites au minimum, les plans sont mal congus pour des familles hombreuses. La venue dun enfant supplémentaire conduit bien souvent 2 un changement d’appartemen ce qui pose des problémes financiers difficiles. pr [any pa of ts poge eos FRENCH TEST—Continued 67. 68. 09. Le sujet principal que traite Pauteur est (A) Ia population des grandes villes, (B) le manque de logements en milieu urbain (©) le nombre croissant de familles nombreuses (D) Ia baisse du nombre de naissances L’auteur exprime son point de vue sur les faits qu'il présente surtout par le mot (A) malheureusement (ligne 1) (B) notamment (ligne 6) (C) logement (ligne 9) (D) minimum Gigne 11) Le “changement d’appartement” (ligne 13) est ausé par (A) Varrivée d’un nouveau-né (B) une femme qui travaille (C) des problémes financiers (D) Ia peur des grandes villes GO ONTO THE NEXT PAGE 43) 70. Cette publicité a pour titre “Bonjour, vois suggérer qu'on (A) doit dire (©) habite tous la terre (D) veut tous se reneontrer Yonjour” aux voisins (B) veut vendre des tentures aux voisins FRENCH TEST—Continued ‘Dans la gande communauté humaine, nous sommes tus visas. Et Zue un bon wisi, ces aus apprendre sinter dns a etre locale de la cormmunaané (Chee Komats, nou prousons do machines intl et des engin de consrction els que excavate, obs, ase t smachines-outile. Cesta die des instruments permecant Apbrenir une meileure qualité de vie. De méme, now nous efogaas doeoet our leben deb cammunaut ex délopant Tncallabraton locale dn le monde des afi ds ntepeses ales des hangs commercsux et des serves. En un ma, 3 ‘copéter pour un mone mei ehkomarsu 72. Selon cette publicité, la philosophie de Komatsu est (A) de construire des machines moins chéres (D) de prévoir 71. Komatsu est une compagnie qui fabrique des ‘machines pour (A) Vindustrie d°équipements (B) les petits commerces (C) les besoins ménagers (D) aérospatiale [any part ofthis page egal Data (B) de eréer un monde plus harmonicux (©) dencourager Martisanat le monde & venir GO ONTOTHE NEXT PAGE > 78. Le Comité du Tourisme a choisi ces chateaux parce 79. Parmi les “Routes de Beauté quills (A) intéressent les spi (B) appa jennent tous a IE (C) sont ouverts au public de temps en temps (D) sont un bel exemple de larchitecture FRENCH TEST—Continued ® Les Routes &™ ROI RENE Les Routes Roi René, qui furent aussi celles des Plantagenéts,regroupent, ‘otarsment,Iessentie! des chaueaux de I Anjou ouverts au public; pour Ia plupar: habits par lurs propriaires, ils offent une image vivante du Patimoine historique et architectural angevin, Elles permettent également de découvrir de nombreux sites et monuments et ‘de comprendre |'tme du "pays": 'Artde vivre en Anjou se perpétue et fait du ‘Maine-et-Loire d'aujourd'hui uno wre prvi dela Valle dela Loire, Le Roi Reng, cousin du Roi de France, fu 'insigateur de la ve artistique et itéraire de 1” Anjou. Quel plus bel erabléme pour ces itinérares? [Les Routes Rol René font pure des Routes de Beauté. parce segment les “Routes Roi Reni écialistes, (A) qui parcourt une région oi le déplacement t cst pratique on y trouve des monuments de toute yoque | honore un protecteur des lettres et des (D) qu’ est embleme des Rois de France sry part of ie page gat Drauthoraed spying or uso oi GO ONTOTHE NEXT PAGE > Da6 ® FRENCH TEST—Continued C’était dans Vhiver de 1792. La disette régnait a Strasbourg. La maison de Dietrich, le maire, était pauvre, la table frugale, mais hospitaliére pour Rouget de Accablé dune inspiration sublime, le jeune officier s’endormit a (éte sur son clavecin et ne s'éveilla qu’au jour. Les chants de Ia nuit, impressions dun réve, lui remonterent avec peine dans la mémoire. Il les écrivit, Jes nota, et courut chez Dietrich. La femme et les filles| dlu vieux patriote n’étaiont pas encore levées. Dietrich les éveilla, il appela quelques amis, tous passionnés comme lui pour la musique. Sa fille ainge accompagna, Rouget chanta. A Ia premiére strophe les visages palirent; a la seconde, les larmes coulerent; aux derniéres, le délire de enthousiasme éclata. L"hymine de la patrie était trouvé! Le nouveau chant, exéeuté quelques jours apres & Strasbourg, vola de ville en ville sur tous les orchestres populaires. Marseille I'adopta pour étre chanté au ‘commencement et ila fin des séances de ses clubs. Les Marseillais le répandirent en France, en le chantant sur leur route. De la lui vint le nom de “Marseillaise”, 80. Ol est-ce que la “Marseillaise” a 6té composée? (A) A Marseille (B) A Strasbourg © ALitle (D) Sur la route 81. L’inspiration pour la musique est née (A) dun reve B) d'un p (C) d'un repas (D) Cune bataille 82. Quelle profession Rouget de Lisle exerg ‘moment-la? (A) Troubadour (B) Maire (©) Chef dorchestre (D) Soldat 83. Qui a interprété la *Marseillaise” pour la premiére fois? (A) Le maire et sa femme (B) Rouget de Lisle et la femme du maire (C) Rouget de Lisle et la fille du maire (D) Le maire et sa fille 84, D'apris le texte, quel a été le destin du nouveau chant? (A) On ne I’a plus jou. (B) Les autres villes I'ont volé. (C) Ona beaucoup modifié. (D) On !’a chanté un peu partout. 85. Pourquoi "hymne national frangais sappelle-Cil la “Marseillaise”? (A) Un Marseillais I'a composé. (B) Les Marseillais ont rendu populaire (C) Les Marseillais étaient les plus patriotiques. (D) Les Marseillais étaient les meilleurs chanteurs. STOP IF YOU FINISH BEFORE TIME IS CALLED, YOU MAY CHECK YOUR WORK ON THIS TEST ONLY. DO NOT TURN TO ANY OTHER TEST IN THIS BOOK. a a7) ‘The Official Study Guide for All SAT Subject Tests How to Score the SAT Subject Test in French When you take an actual SAT Subject Test in French, your answer sheet will be “read” by a scanning machine that will record your responses to each question, Then a computer will compare your answers with the correct answers and produce your raw score. You get one point for each correct answer. For each wrong answer, you lose one-third of a point. Questions you omit (and any for which you mark more than one answer) are not counted. This raw score is converted to a scaled score that is reported to you and to the colleges you specify. Worksheet 1. Finding Your Raw Test Score STEP 1: Table A lists the correct answers for all the questions on the Subject Test in French that is reproduced in this book. It also serves as a worksheet for you to calculate your raw score. + Compare your answers with those given in the table. + Puta check in the column marked “Right” if your answer is correct. + Puta check in the column marked “Wrong” if your answer is incorrect. + Leave both columns blank if you omitted the question STEP 2: Count the number of right answers, Enter the total here: STEP 3: Count the number of wrong answers. Enter the total here: STEP 4: Multiply the number of wrong answers by .333, Enter the product here: STEP 5: Subtract the result obtained in Step 4 from the total you obtained in Step 2. Enter the result here: STEP 6: Round the number obtained in Step 5 to the nearest whole number. Enter the result here: The number you obtained in Step 6 is your raw score. Date Table A Answers to the Subject Test in French, Form 3XAC, and erry Percentage of Students Answering Each Question Correctly Percentage Batt Perens ty" A 45 c A 5 D o 58 A A 51 A Oo 49 8 a B 8 c 23 8 A 87 D A 31 10 Oo i 42 o a6 1 A 3 3 A 73 2 A 66 “4 A 9 3 8 36 45 A 39 “ oD B 46 B 53 8 A 8 a c 6 16 D 34 48 B 9 v Oo 4a 49 A 49 18 A 2 50 c o 8 Oo 2 51 B a4 2 c 52 52 c 49 a t) 35 53 B 7 2 oD a 5A B 48 2B c e 55 c 86 a c 6 50 0 16 % A a 57 o 1B 26 A 88 58 A 39 a 8 60 59 B 89 28 A a a0 c 93 2 8 68 8 A 79 30 A 66 82 B 88 3 A 70 83 c 53 32 A 81 fa c 61 Table A continued on nest page a9) ‘The Official Study Guide for All SAT Subject Tests Table A continued from previous page c 6 D 5 7 6 a D 34 18 18 cy A 69 19 49 6 A 63 80 83 70 c 30 et 8 Tn A 83 82 6 2 8 5 83 60 B c 38 Cy 39 a 8 6 85 70 18 D 57 * These percentages are based on an analysis ofthe answer sheets of a representative sample of 3,607 students who took the original form ofthis test in June 2001, and whose mean score was 600. They may be used as an indication ofthe relative difficslty of a particular question, Each percentage may also be used to predict the likelihood that a typical SAT Subject Test in French candidate will answer that question correctly on this edit ofthe test D420 French Finding Your Scaled Score When you take SAT Subject Tests, the scores sent to the colleges you specify are reported on the College Board scale, which ranges from 200-800. You can convert your practice test score to a scaled score by using Table B. To find your scaled score, locate your raw score in the left-hand column of Table B; the corresponding score in the right-hand column is your scaled score. For example, a raw score of 37 on this particular edition of the Subject Test in French corresponds to a scaled score of 570. Raw scores are converted to scaled scores to ensure that a score earned on any one edition of a particular Subject Test is comparable to the same scaled score earned on any other edition of the same Subject Test. Because some editions of the tests may be slightly easier or more difficult than others, College Board scaled scores are adjusted so that they indicate the same level of performance regardless of the edition of the test taken and the ability of the group that takes it. Thus, for example, a score of 400 on one edition of a test taken at a particular administration indicates the same level of achievement as a score of 400 on a different edition of the test taken at a different administration, When you take the SAT Subject Tests during a national administration, your scores are likely to differ somewhat from the scores you obtain on the tests in this book. People perform at different levels at different times for reasons unrelated to the tests themselves The precision of any test is also limited because it represents only a sample of all the possible questions that could be asked. az French How Did You Do on the Subject Test in French? After you score your test and analyze your performance, think about the following questions: Did you run out of time before reaching the end of the test? If so, you may need to pace yourself better. For example, maybe you spent too much time on one or two hard questions. A better approach might be to skip the ones you can't answer right away and try answering all the questions that remain on the test. Then if there's time, go back to the questions you skipped. Did you take a long time reading the directions? You will save time when you take the test by learning the directions to the Subject Test in French ahead of time. Each minute you spend reading directions during the test is a ‘minute that you could use to answer questions, How did you handle questions you were unsure of? Ifyou were able to eliminate one or more of the answer choices as wrong and guess from the remaining ones, your approach probably worked to your advantage. On the other hand, making haphazard guesses or omitting questions without trying to eliminate choices could cost you valuable points. How difficult were the questions for you compared with other students who took the test? Table A shows you how difficult the multiple-choice questions were for the group of students who took this test during its national administration, The right-hand column gives the percentage of students that answered each question correctly. A question answered correctly by almost everyone in the group is obviously an easier question. For example, 95 percent of the students answered question 4 correctly. But only 22 percent answered question 19 correctly. Keep in mind that these percentages are based on just one group of students. They would probably be different with another group of students taking the test. If you missed several easier questions, go back and try to find out why: Did the questions cover material you haven't yet reviewed? Did you misunderstand the directions? 4230 French with Listening You see: ‘You hear: [ (A) Quelle joie d’étre seul! (B) Que cest agréable de faire du vélo! (©) Le moteur fait trop de bruit! (D)_ Nous adorons la course & pied.) Choice (B) is the correct answer. Statement (B), “Que cest agréable de faire du vélo!” best reflects what you see in the picture or what someone in the picture might say. 4250 ‘The Official Study Guide for All SAT Subject Tests 1, You see: You hear: [Numéro 1 (Woman) (A) Elle porte toujours un bonnet de bain. (B) Bille a toujours peur de Feau, (©) Le ski nautique lui plait beaucoup. (D) Elle est en train de plonger dans Veau,] {7 seconds) Choice (D) is the correct answer to question 1. In this question you see a drawing that shows a slow motion scene of a gir! diving into a pool. Choice (D) best reflects what can be seen in the picture. Choices (A), (B), and (©) are incorrect because the girl in the picture is not wearing a cap, does not seem to be afraid of water, and is not shown on. water skis. D426 ‘The Official Study Guide for All SAT Subject Tests Questions 2-3 You hear: {(Tone) (Man) Votre passeport, madame. (Woman) Voila (Man) Et quest-ce que vous ferez. au Canada? (Woman) _Je vais passer les vacances avec ma famille. (Man) ‘Trds bien, madame. Je vous souhaite un bon séjour] (6 seconds) 2. [Numéro 2 (Man) Qui parle a cette femme? a Un professeur. @) Un douanier. oO Un chauffeur] (seconds) Choice (B) is the correct answer to question 2, Un dowanier, because the woman is crossing the border into Canada. In this question, you choose the person who is talking to the woman in the dialogue. Un professeur or ut chauffeur would not ask to see the woman's passport. Therefore, choices (A) and (C) are incorrect. 3. [Numéro 3 (Man) Queest-ce que la femme va faire? (a) Obtenir un passeport. ®) Chercher sa famille, oO Entrer au Canada] (7 seconds) Choice (C) is the correct answer to question 3. This question asks what the woman is about to do, Choice (C) is the only logical answer according to the dialogue. After crossing the border, the woman will enter Canada. Choice (A) is incorrect because she already has a passport, and choice (B) is incorrect because she is vacationing with her family, not looking for them. Daze ‘The Official Study Guide for All SAT Subject Tests incorrect because she did not see the movie (A) and does not mention that she intends to go to Boston (D). 5. (Man) [Qu’est-ce que Maude avoue & son amie?] (12 seconds) Qurest-ce que Maude avoue a son amie? (A) Elle alu Ie livre. (B) Elle nla pas encore vu le film. (©) Elle n'a pas compris la critique. (D) Elle a écouté la cassette. Choice (B) is the correct answer to question 5, When Maude answers her friend, she tells her at the end: Il parait qu'il est trés trés bien, le film ... je compte aller le voir la semaine prochaine. Choice (A) is incorrect because Maude states the opposite (le livre, je ne le connais pas). According to the dialogue, Maude admits neither choice (C) nor choice (D); both choices are therefore incorrect. 6. (Man) [Dans cette discussion, quiest-ce qu'on peut dire des deux amies?] {12 seconds) Dans cette discussion, qu’est-ce qu’on peut dire des deux amies? (A) Elles se font des compliments. (B) Biles s'ignoren (©) Elles s‘inquidtent (D) Elles partagent les mémes goiits. Choice (D) is the correct answer to question 6, It can be inferred from the dialogue that, the two friends share the same interests, at least with regard to movies. Choices (A), (B), and (C) cannot be said about the two friends, They do not exchange compliments (A), do know each other (B), and do not become anxious or upset (C). D430 French with Listening Test Practice Helps The test that follows is an actual, recently administered SAT Subject Test in French with Listening. To get an idea of what it's like to take this test, practice under conditions that are much like those of an actual test administration. Set aside an hour when you can take the test uninterrupted. Make sure you complete the test in one sitting. Sit at a desk or table with no other books or papers. Dictionaries, other books, or notes are not allowed in the test room. Tear out an answer sheet from the back of this book and fill it in just as you would on the day of the test. One answer sheet can be used for up to three Subject Tests. Read the instructions that precede the practice test. During the actual administration you will be asked to read them before answering test questions. Time yourself by placing a clock or kitchen timer in front of you. After you finish the practice test, read the sections “How to Score the SAT Subject Test in French with Listening” and “How Did You Do on the Subject Test in French with Listening?” The appearance of the answer sheet in this book may differ from the answer sheet you see on test day. 431d FRENCH TEST WITH LISTE) SECTION I LISTENING Approximate time—20 minutes Questions 1-27 NOTE THAT YOUR ANSWER SHEET HAS FIVE ANSWER POSITIONS, MARKED A, B, AND E, WHILE THE QUESTIONS THROUGHOUT THIS PART CONTAIN ONLY FOUR CHOICES. BE SURE NOT TO MAKE ANY MARKS IN COLUMN E, Part A Directions: For each question in this part, you will hear four sentences, designated (A), (B), (C), and (D), They will not be printed in your test booklet. As you listen, Took at the pieture in your test booklet and select the choice that best reflects what you see in the picture or what someone in the picture might say ‘Then fill in the corresponding circle on the answer sheet. You will hear the choices only once. Now look at the following example. You see: You hear: Statement (B), “Que c'est agréable de faire du vélo,” best reflects what you see in the picture or what some- fone in the picture might say. Therefore, you should choose answer (B). 3ZLC [Unautherined copying rouse al GO ONTO THE NEXT PAGE > 433) FRENCH TEST WITH LISTENING— Continued TUnmatriasd coping ox ruse al GO ONTO THE NEXT PAGE > lay pr of hs ge eg 435) vied coping fins pe eae FRENCH TEST WITH LISTENING— Continued ® GO ONTO THE NEXT PAGE > 437 ® FRENCH TEST WITH LISTENING— Continued ® PLEASE NOTE THAT YOUR ANSWER SHEET HAS FIVE ANSWER POSITIONS, MARKED A, B,C, D, AND E, WHILE THE QUESTIONS THROUGHOUT THIS PART CONTAIN ONLY FOUR CHOICES. BE SURE NOT'TO MAKE ANY MARKS IN COLUMN E. Part C Directions: You will now hear some extended dialogues or monologues. You will hear each only once. Afier each dialogue or monologue, you will be asked several questions about what you have just heard. These questions are also printed in your test booklet. Select the best answer to each question from among the four choices printed in your test booklet and fill in the corresponding circle on the answer sheet. There is no sample question for this par. Dialogue numéro 1 Dialogue numéro 2 17. Quel genre de film vontils voir? 21. Quel est Pun des jours oit Philippe va a école (A) Une histoire de pirates. seulement le matin? (B) Un film daventures, (A) Le samedi. (©) Un film e’espionnage. (B) Le lund (D) Un sujet eactuatite, (©) Le mardi. (D) Le jeu. 18, Qu'est-ce que Bernard et Sophie doivent faire & la {in de leur discussion? 22. Que fait Philippe d2s qu'il rentre chez lui? (A) Is doivent se séparer (A) 11 prend un godter (B) Ils doivent se dépécher. (B) Mregarde la tele (C) Is doivent rentrer chez eux. (C) I léphone a ses amis. (D) Ils doivent aller diner. (D) Hit un roman 19. Quelle impression donne le gargon? 23. A part le tennis. quelle autre activité sportive (A) Hest concilian Philippe préfere-til? (B) I n'a pase’ opinions (A) Le foot (©) Mestent (B) La course (D) Hest comique (©) La mare (D) Le velo 20. Pourquoi ont-ils cho (A) Paree que la jeune fille n’aime (B) Paree que le gargon en a entendu parler. (C) Pare que la jeune problemes sociaux, (D) Parce que le garcon a envie de s’'amuser ce soir-la, apying arouse of GO ONTO THE NEXT PAGE lay per ofthis ge ete 439) 25. FRENCH TEST WITH LISTENING— Continued Dialogue Numéro 3 Qu’est-ce qu'on peut dire du travail de Marie~ France? (A) ILarest pas exigeant. (B) C'est un emploi temporaire. (©) Moblige Marie-France a se déplacer. (D) Il w'intéresse pas Matie- Qu’est-ce que Marie-France pense des ordinateurs portatifs? (A) Elle les trouve intimidants (B) Ele eroit qu ©) Elle trouve qu’ils codtent trop cher (D) Elle ne s'en occupe pas. Is déshumanisent la vie. Qu’est-ce que Marie-France craint? (A) Que ses enfants ne se servent trop de son ordinateur portatif. (B) Qu'elle n’efface des lignes importantes dans un dossier. (©) Que Pordinateur portatif ne provoque des disputes. (D) Que ordinateur portatif n’apporte la vie du travail a Ja maison, Comment peut-on caractériser latitude de Caroline sur l'emploi des ordinatours portatifs? (A) Elle est neutre. (B) Elle est désintéressée, (© Elieest sarcastique. (D) Elle est encourageante. END OF SECTION I. DO NOT GO ON TO SECTION II UNTIL YOU ARE TOLD TO DO SO. [sry pat ot tus poe eg Daa ® FRENCH TEST WITH LISTENING— Continued Direction Each of the following sentences contains a blank. From the four choices given, select the one that can be inserted in the blank (o form a grammatically correct sentence and fill in the correspor jing circle on the answer sheet. Choice (A) may consist of dashes that indicate that no insertion is required to form a grammatically correct sentence. 40. Déun (A) lev (B) leve (C) levés (D) levées mmun accord, elles se sont toutes —~ 41. L’été prochain je voudrais voyager ——-—- Mexique. (A) dans (B) en © ala (D) aw 42. Les jeunes mariés se sont installés dans leur appartement, (A) grande (B) beau (C) propre (D) moderne 43. On voudrait bien savoir tableaux. A) qu (B) auquel ©) qu 44, Fermez la porte, afin que la pidce fraiche, rester (A) peut (B) pouvait (C) pouvez (D) puisse 45. On partira A) si (B) des que (C) sans que (D) avant que vous serez. prats, [Unauthorised copying o vous ol ies 46. 47, 48, 49, 50. Je voudrais y aller avee (A) elles Bw (©) leur (D) ils Croyant qu’ Henri sei fait de souci. 1a, Jeannette ne s'est (A) souvent (B) encore ©) nut (D) guere I plaisent. Montréal depuis trois ans et ils s'y (A) seraient (B) ont été (C) étaient (D) sont Ces toujours. tun événement ------- je me souviendrai (A) que (B) qui (C) auquel (D) dont Mon avocat s*occupe bien ---—- mes affaires. (A) des (B) de Oa (D) en GO ONTO THE NEXT PAGE > 443) © FRENCH TEST WITH LISTENING — Continued ® Part C ‘The paragraphs below contain blank spaces indicating omissions in the text. For some blanks necessary to choose the completion that is most appropriate to the meaning of the passage: for other blanks, to ‘choose the one completion that forms a grammatically correct sentenee. In some instances, choice (A) may consist ‘of dashes that indicate that no insertion is required to form a grammatically correct sentence. In each case, indicate your answer by filling in the corresponding circle on the answer sheet, Be sure to read the paragraph completely before answering the questions related to it, Hier soir, comme nous n’ _(S1)_ rien de mieux 51. (A) avons. (B) avions faire, nous nous sommes promenés dans le parc (C) aurions: (D) ayons pendant deux heures et _(52)_avant de rentrer nous: 52. (A) midi coucher. Mon copain, _(53)_rien ne géne, s'est tout (B) trente (C) demie de suite endormi _(54)_que moi, je n'ai pas pu fermer (D) le quart 55)_. J'avais toujours devant moi _(56)_de ce pauyre 53. (A) que (B) quel homme _(57)_ que nous avions vu assis sur_(58)_ pre's (©) celui (D) quelqu’un de la porte d’entrée du pare. Sk tan 54. (A) tandis (B) de sorte (C) sans (D) autant 55. (A) Voreiller (B) lett (C) Poeil (D) la couverture 56.(A) @) (©) lacopie (D) le signe 57. (A) file (B) fume (C) enterré (D) affame 58. (A) une banlieue B) un bane (C) um banquet (D) une banque [hauthoraed spying ot euss ol GO ONTOTHE NEXT PAGE > ® Hier, nous avons fait une balade en montagne. Nous, sommes partis tr’s tt (59), et c'est mon pore _(60)_ a conduit I1_(61)_ encore tres frais, mais le soleil brillait B1, bien sGr, tout le monde était _(62)_! Quand nous sommes arrivés dans _(63)_ magnifique, nous avons, a pour _(66)_reposer. Vers midi nous avons pique-niqué (67)_ d'un torrent. Nous avions tous trés faim, et les sandwichs ont vite disparu, neuthorized copying or ease a] any prt fis page seg (64)_mareher, en nous_(65)_de temps en temps FRENCH TEST WITH LISTENING— Continued 59. 61 02 63. 65. 67. ® (A) le matin (B) matin (© du matin (D) dans le matin A) ~ (B) il ©) qui (D) qw’ (A) faisait (B) avait (©) montait (D) était (A) Ala rigueur (B) de bonne heure (C) dans le vent (D) de bonne humeur (A) un endroit (B) un lac (©) une place (D) une altitude A)» (B) pour © de (Da Oy) (B) (©) arrétés (D) arrétamt “ — (B) se © me (D) nous (A) au bord (B) abord (©) autour (D) aw fond GO ONTO THE NEXT PAGE > 445) © FRENCH TEST WITH LIS’ incomplete statements. circle on the answer sheet. Directions: Read the following texts carefully for comprehension. Each is followed by a number of questions or Jeet the answer or completion that is best according to the text and fill in the corresponding (Ce passage a été écrit en 1856.) Que faites-vous, Gaston’? Quand viendrez-vous? Vous aviez. pourtant promis de nous rejoindre, ‘Comment avez-vous pu rester dix grands jours sans. ig me voir? Quand nous étions ensemble dans notre 5 cher Arlange, vous ne saviez pas me quitter pour tune heure. Diew! que les heures sont longues & Paris! ‘Maman me parle & chaque instant contre vous, mais a votre nom seul il se fait dans mon coeur un tapage qui m’empéche d'entendre, Elle me dit que vous m’avez 10 abandonnée: vous devinez que je n’en erois rien. ‘Vous n’étes pas homme a fermer un si bon livre Ia premigre page. Moi, depuis que je ne vous ai plus je suis tout hébstée et toute languissante. Imaginez- ‘vous que par moments je erois que je ne Suis pas 5 votre femme, et que cette belle cérémonie de léglise, ‘etce bal ol nous étions si heureux, sont un réve quit a trop 161 fini, Vous n'imaginerez jamais combien vous me manquez. Quand je sors avec maman, je dans les rues: tout ce que j'ai vu 2 présent, e”est que vous n'y étes pas, Le soir, 'embrouille réguliérement votre nom dans, ‘mes priéres; le matin, en m’éveillant, je regard vous n’étes point autour de moi. Est-il possible que je pense tant A vous et que vous mayez oublige? 25 Peut-étre m'en voulez-vous de Vous avoir quitté si brusquement et sans vous dire adieu. Si vous saviez! Ce n'est pas moi qui m’acnlevée. 68, De quoi la narratrice de (A) De Pabsence de Gaston (B) Du bruit de ta ville (C) De la mere de Gaston (D) De sa maladie récente 69, Oit Ia narratrice de ce passage se trouve-t-elle' (A) Ala campagne (B) Pres de la mer (©) Dans son village natal (D) Dans une grande ville [any part of hia page To egal Daas 70. Quelle est l'attitude de la mere de la narratrice envers Gaston? (A) Bille le trouve assez sympathique. (B) Elle éprouve de I'indifférence envers lui (C) Elle montre une certaine hostlité & son égard. (D) Bille aime presqu’autant que sa fille 71. A qui la narratrice éerit-elle? (A) A son fibre (B) A son amant (C) A son fiancé (D) A son mari 72. La phrase, “Vous n’étes pas homme a fermer un si bon livre la premitre page” (lignes 11-12) laisse entendre que (A) la narratrice a épousé Gaston récemment (B) la narratrice est bibliothécaire (C) Gaston se méfie de la narratrice (D) Gaston aime pas les livres courts 73. D’aprds le texte, que fait a narratrice quand elle se proméne avec sa mere? (A) Elle fait des prigres pour Gaston, (B) Elle achote des cadeaux pour Gaston, (C) Eille cherche Gaston partout (D) Elle parle de Gaston a sa mere. 74, Selon la narratr esprit de Gaston’ quel pourrait tre [état (A) Ila peurde lui dire adic (B) est triste quand il y a du brouillard. (C) Mest plutot religieux. (D) lest en colere contre elle 75, La narratrice a quitté Gaston parce qu’elle (A) préfere Paris la campagne (B) a 616 obligée de le faire (C) a besoin de soins médi (D) veut se faire religi GO ONTO THE NEXT PAGE © FRENCH TEST WITH LISTENING— Continued LA CARTE INTEGRALE (carte orange annuelle) La carte Intégrale, c'est : Un coupon unique, valable toute l'année Ses avantages "Chaque mois, elle vous simplifie la vie !", ‘une seule démarche lors du premier abonnement. ~Vous riavez plus defile dattente enfin de mois, Pour tous vos déplacements en Ile de France. Un abonnement personnel et permanent, utilisable a volonté sur les RER, Bus, Métros et trains d'lle de France, en . fonction des zones choisies (mémes zones au choix que la carte orange). 76. La carte Intégrale vous permettra (A) de louer une voiture & tarif réduit (B) Cutiliser tous les transports publics (C) de régler toutes vos factures mensuelles ala fois (D) d’obtenir les hebdomadaires les plus récents neuthorized copying or ease a] any prt fis page seg Vous niavee méme plus besoin dy penser. Vous avez. deja votre coupon. "Vous choisissez, vous changez davis, votre abonnement c'est ‘comme vous en avez envie !" 77. Le coupon est valide pendant (A) toute la vie (B) un mois (C) les vacances (D) douze mois GO ONTO THE NEXT PAGE > 447) © FRENCH TEST WITH LIS’ Oh! ces journées de neige, quelle transformation 80. Quel effet la neige a subite elles opéraient en nous, autour de nous d2s les premiers flocons! La lumicre se retirait. Tout igne devenait terme: le platre des fagades prenait une 5 couleur grise, fange, les arbres paraissaicnt plus noirs. Dehors, quand nous levions la téte, ¢” était presque tune ivresse de recevoir sur la figure, sans savoir od elles se poscraient, ces mille petites abcilles blanches dont Ie froid nous piquait Ie visage, avee une si (A) une maison ING—Continued le pour Pauteur? (A) Bille détruit le monde qui Pentoure, (B) Elle erée un nouveau décor. (C) Elle attriste Pobservateur. (D) Elle enlaidit le paysage. 81. Dans le deuxidme paragraphe, la neige envahit 10 furtive, une si délicate précision qu’elles semblaient (B) un jardin avoir ehoisi, tout en tourbillonnant, la place oi elles (©) une place nous atteindraient. Le ciel n’était plus gris; il était (D) une ville roux, opaque. Bt peu a peu, les grilles du college, les branches, les banes, les tots, devenaient d'autres 82, Dans ce passage, la neige est 15 prilles, d'autres branches, d'autres bans, d'autres () cattivees toits “ Mais le vent cessait, Alors elle tombait plus vite (B) balayée (C) personni ‘et recouvrait tout, uniformément, de sa blancheur () enlevee duveteuse comme si elle avait profité de 20 dinattention pour s'installer, en dominatrice, pour *autcur attribue & la neige un pouvoir infiltrer jusqu’entre les fentes des persiennes, sous, 83. L’auteur attibue a la neige un po moment Tes tuiles, et méme dans les recoins du grenier en (A) fortifiant passant par un carreau eassé (B) magique (C) chimique 78, Les enfants semblent accueillir la neige avec (D) exotique (A) plaisir (B) dégoat ©) tristesse (D) crainte 79, Les flocons de neige sont comparés & des (A) fleurs (B) pierres (©) oiseaux (D) insectes [Unmhorisd copying vous oT GO ONTO THE NEXT PAGE sry par ofthis ge ooo Daas ® FRENCH TEST WITH LI Lé& VIE C'EST COMME LA PLUS BELLE AVENUE DU MONDE UN RELAIS, Gbe GE ACCUEILLE LE PLUS BEAU TAANEMET RELAIS DU MONDE ENING—Continued Aujourathui plus de 6000 malades ‘attendent. Attendent la greffe b J. — cripouraitleursowerta vie. bs - y attendent et espérent. y 7 Cequilsattendent,ce cuits espérent ce ne sont pas les progiés dans le traitement : celui-ci est parfaitement au point. Ce quils ottendent, ce quis esperent ‘ce nest pas quiune équipe médicale soit disponible: elles sont nombreuses en France & maitiser cette thérapeutique. lls cttendent et espérent le rein, le coour. le poumon. le foie irémédiablernent perdus. FAITES PASSER LA VIE Dimanche 29 marssurlesChomps + ysées en avon! premiére du Marathon eh de Pats, 3 6n10précies,Téqupe —— frangite rls 4x 100m posse relat dof vi. 84. Qu’est-ce que cette publicité veut encourager? 86. Qu’est-ce qui do (A) La course & pied (A) L’entrainement des jeunes sportfs (B) La recherche médicale (B) L*éducation des futurs médecins (©) Des dons d’argent (C) Les techniques médicales (D) Des dons d’organes (D) Les chances de survie 85. Qui va participer & I’événement annoneé’? (A) Des médecins (B) Des malades (©) Des sportits (D) Des chercheurs, END OF SECTION II STOP IF YOU FINISH BEFORE TIME IS CALLED, YOU MAY CHECK YOUR WORK ON SECTION Il OF THIS TEST. DO NOT TURN TO ANY OTHER TEST IN THIS BOOK. lane part of ts page a iigal. 449) ‘The Official Study Guide for All SAT Subject Tests How to Score the SAT Subject Test in French with Listening When you take an actual SAT Subject Test in French with Listening, you receive an overall composite score as well as two subscores: one for the reading section, one for the listening section, ‘The reading and listening scores are reported on the College Board’s 20-80 scale. However the composite score, which is the most significant of the scores reported to the colleges you specify, isin the form of the College Board’s 200-800 scale. D450 Worksheet 1. Finding Your Raw Listening Subscore STEP Table A lists the correct answers for all the questions on the Subject Test in French with Listening that is reproduced in this book. It also serves as a worksheet for you to calculate your raw Listening subscore. + Compare your answers with those given in the table. + Puta check in the column marked “Right” if your answer is correct. + Puta check in the column marked “Wrong” if your answer is incorrect. + Leave both columns blank if you omitted the question. STEP 2: Count the number of right answers for questions 1-8 and 17-27. Enter the total here: STEP 3: Count the number of wrong answers for questions 1-8 and 17-27, Enter the total here: STEP 4: Multiply the number of wrong answers by .333. Enter the product here: STEP 5: Subtract the result obtained in Step 4 from the total you obtained in Step 2. Enter the result here: STEP 6: Count the number of right answers for questions 9-16. Enter the total here: STEP 7: Count the number of wrong answers for questions 9-16. Enter the total here: STEP : Multiply the number of wrong answers for step 7 by .500. Enter the product here: STEP 9: Subtract the result obtained in Step 8 from the total you obtained in Step 6. Enter the result here: — STEP 10: Add the result obtained in Step 5 to the result obtained in Step 9. Enter the result here: STEP 11: Round the number obtained in Step 10 to the nearest whole number. Enter the result here: The number you obtained in Step 11 is your raw Listening subscore. 451) ‘The Official Study Guide for All SAT Subject Tests Worksheet 2. Finding Your Raw Reading Subscore STEP 1: Table A lists the correct answers for all the questions on the Subject Test in French with Listening that is reproduced in this book. It also serves as a worksheet for you to calculate your raw Reading subscore. STEP 2: unt the number of right answers for questions 28-86. Enter the total here: STEP 3: Count the number of wrong answers for questions 28-86, Enter the total here: STEP 4: Multiply the number of wrong answers by .333, Enter the product here: STEP 5: Subtract the result obtained in Step 4 from the total you obtained in Step 2. Enter the result here: STEP 6; Round the number obtained in Step 5 to the nearest whole number. Enter the result here: The number you obtained in Step 6 is your raw Reading subscore. Das Worksheet 3. Finding Your Raw Composite Score STEP 1: Enter your unrounded raw Reading subscore from Step 5 of Worksheet 2. Enter the result here: STEP 2: Enter your unrounded raw Listening subscore from Step 10 of Worksheet 1. Enter the result here: STEP 3: Add the result obtained in Step 1 to the result obtained in Step 2. Enter the result here: STEP Enter the result here: : Round the number obtained in Step 3 to the nearest whole number. The number you obtained in Step 4 is your raw composite score. 453 ‘The Official Study Guide for All SAT Subject Tests Table A Peery Answers to the Subject Test in French with Listening, Form 3ZLC, and Percentage of Students Answering Each Question Correctly a een of Students Bac ony co Soeur the Question es ght | Wrong} Correctly 34 38 c 87 56 34 A 63 90 cy c % 10 38 A 43 36 3 c 6 49 38 8 33 15 39 A 38 Ey 40 D 7 4 a D 59 82 (7 c 37 oT 43 C 8 82 4 D 6 5 5 8 4 2 4% A 55 30 a D 2 a 48 0 35 2 49 D 36 69 50 B 60 51 st B n 88 52 c 85 5 53 A 38 6 54 A 45 B 55 c 2 Ey 58 8 3 26 5 o a 50 58 B a 6 58 A oy 92 60 c 8 86 a A 69 90 62 o n a7 63 a 5 92 64 c Gy Dasa Table A continued on nest page French with Listening Table A continued from previous page ere Revd neces Era 65 66 D 87 A a 68 A 34 6 oD 50 70 C 28 n D 80 n A 54 2 c 31 m4 D 63 i 8 2 ‘These percentages are based on an analysis ofthe answer sheets of a representative sample of 1,519 students ‘who took the original form ofthis test in November 2003, and whose mean composite score was 602. They may ‘be used as an indication of the relative difficulty ofa particular question. Each percentage may also be used to predict the likelihood that atypical SAT Subject Test in French with Listening candidate will answer that {question correctly on this edition ofthe test, 455 ‘The Official Study Guide for All SAT Subject Tests Finding Your Scaled Score When you take SAT Subject Tests, the scores sent to the colleges you specify are reported on the College Board scale, which ranges from 200-800, Subscores are reported on a scale which ranges from 20-80. You can convert your practice test scores to scaled scores by using Tables B, C, and D. To find your scaled score, locate your raw score in the left- hand column of the table; the corresponding score in the right-hand column is your scaled score. For example, a raw score of 47 on this particular edition of the Subject Test in French with Listening corresponds to a scaled composite score of 610. Raw scores are converted to scaled scores to ensure that a score earned on any one edition of a particular Subject Test is comparable to the same scaled score earned on any other edition of the same Subject Test. Because some editions of the tests may be slightly easier or more difficult than others, College Board scaled scores are adjusted so that they indicate the same level of performance regardless of the edition of the test taken and the ability of the group that takes it. Thus, for example, a score of 400 on one edition of a test taken ata particular administration indicates the same level of achievement as a score of 400 on a different edition of the test taken at a different administration, When you take the SAT Subject Tests during a national administration, your scores are likely to differ somewhat from the scores you obtain on the tests in this book. People perform at different levels at different times for reasons unrelated to the tests themselves. The precision of any test is also limited because it represents only a sample of all the possible questions that could be asked. ‘Your scaled composite score from Table B is : ‘Your scaled listening score from Table C is : Your scaled reading score from Table D is : D456 French with Listening Table B Scaled Score Conversion Table Subject Test in French with Listening Composite Score (Form 321C) 400 390 390 380 370 370 360 350 340 340 330 320 320 310 300 290 200 20 20 270 270 260 260 280 260 250 240 230 230 220 220 220 220 210 210 210 210 200 200 200 200 457) French with Listening Table D Scaled Score Conversion Table Subject Test in French with Listening Reading Subscore (Form 3ZLC) 40 39 38 37 38 35 33 52 79 25 55 2 32 51 78 2 55 a 31 50 7 23 54 4 30 9 6 2 8 6 2 48 78 a 82 6 28 47 8 2 52 7 2B 48 1” 9 5 8 a 6 73 18 50 3 26 ao 72 7 19 0 Fy 43 n 6 49 a % 2 7 6 48 12 2 4“ 69 4 "7 43 2 40 68 8 46 14 2 39 7 12 46 8 2 38 or " 45 16 a 37 66 10 44 0 a 36 6 3 43 18 20 35 Cy 8 2 8 20 34 68 7 a 20 20 33 62 6 40 459 ‘The Official Study Guide for All SAT Subject Tests How Did You Do on the Subject Test in French with Listening? After you score your test and analyze your performance, think about the following question Did you run out of time before reaching the end of the test? If'so, you may need to pace yourself better. For example, maybe you spent too much time on one or two hard questions. A better approach might be to skip the ones you can't answer right away and try answering all the questions that remain on the test. Then if there’s time, go back to the questions you skipped. Did you take a long time reading the directions? You will save time when you take the test by learning the directions to the Subject Test in French with Listening ahead of time. Each minute you spend reading directions during the test is a minute that you could use to answer questions. How did you handle questions you were unsure of? Ifyou were able to eliminate one or more of the answer choices as wrong and guess from the remaining ones, your approach probably worked to your advantage. On the other hand, making haphazard guesses or omitting questions without trying to eliminate choices could cost you valuable points. How difficult were the questions for you compared with other students who took the test? Table A shows you how difficult the multiple-choice questions were for the group of students who took this test during its national administration. The right-hand column gives the percentage of students that answered each question correctly. A question answered correctly by almost everyone in the group is obviously an easier question. For example, 95 percent of the students answered question 13 correctly. But only 22 percent answered question 47 correctly. Keep in mind that these percentages are based on just one group of students. They would probably be different with another group of students taking the test. If you missed several easier questions, go back and try to find out why: Did the questions cover material you haven't yet reviewed? Did you misunderstand the directions? D460 Chapter 10 German Purpose There are two Subject ‘Tests in German: German and German with Listening. The reading-only test measures your ability to understand written German, German with Listening measures your ability to understand spoken and written German. Format + The Subject Test in German takes one hour and includes 80 to 85 multiple-choice questions. + The Subject Test in German with Listening also takes one hour and includes 85 to 90 multiple-choice listening and reading questions. Listening questions require answers to questions based on shorter and longer listening selections. + Both tests evaluate your reading ability through a variety of questions requiring a wide-ranging knowledge of German. Content Both tests comply with the German spelling reform (Rechtschreibreform) as much as possible. They evaluate reading ability in these areas: Sentence completion and paragraph completion questions test vocabulary and grammar requiring you to know the meaning of words and idiomatic expressions in context and to identify usage that is structurally correct and appropriate. For each omission, you must select the choice that BEST fits each sentence. Reading comprehension questions test your understanding of the content of various materials taken from sources such as advertisements, timetables, street signs, forms, and tickets. They also examine your ability to read passages representative of various styles and levels of difficulty. Each test edition has several prose passages followed by questions that test your understanding of the passage. The passages, mostly adapted from literary sources and newspapers or magazines, are generally one or two paragraphs in length and test whether you can identify the main idea or comprehend facts or details in the text. 461) ‘The Official Study Guide for All SAT Subject Tests ‘The Subject Test in German with Listening also measures the ability to understand spoken language with two types of listening questions: Type One contains short dialogues/monologues with one or two multiple-choice questions. Dialogues/monologues, questions, and answer choices are recorded. Questions are also printed in the test book. Type Two contains longer dialogues and monologues with several multiple-choice questions. Dialogues/monologues and questions are only recorded and not printed in the test book. Answer choices are not recorded; they appear only in the test book. wa a (grammar) Reading Comprehension—lauthentic stimulus 50 materials and passages) German with Listening een) Er ons oe ener Listening Seotion (20 minutes} 35 ‘Short dialogues/monalogues Long dialogues/monologues Reading Section (40 minutes) 65 Vocabulary in Context Structure in Context {grammar} Reading Comprehension—lauthentic stimulus materials and passages) How to Prepare Both tests assume differences in language preparation; neither is tied to a specific textbook or method of instruction. The German tests are appropriate for students who have completed two, three, or four years of German language study in high school or the equivalent. Your best preparation for these tests is a gradual development of competence in German over a period of years. Familiarize yourself with directions in advance. The rections in this book are identical to those that appear on the test. German with Listening ‘A practice audio CD is included with this book, A practice CD with different sample {questions can be obtained, along with a copy of the SAT Subject Tests Preparation Booklet, D462 German from your school counselor, or you can access the files at www.collegeboard.com. You should also take the practice test included with this book. CD Players Using CD Players for Language Tests with Listening ‘Take an acceptable CD player to the test center. Your CD player must be in good working order, so insert fresh batteries on the day before the test. You may bring additional batteries and a backup player to the test center. CD players cannot be shared with other test-takers, ‘Test center staff won't have batteries, CD players, or earphones for your use, so your CD player must be: + equipped with earphones + portable (hand-held) + battery operated You are not allowed to use a CD player with recording or duplicating capabilities, Note Ifthe volume on your CD player disturbs other test-takers, the test center supervisor may ask you to move to another seat, What to do if your CD player malfunctions: + Raise your hand and tell the test supervisor. + Switch to backup equipment if you have it and continue the test. If you don’t have backup equipment, your score on the Subject ‘Test in German with Listening will be canceled. But scores on other Subject Tests you take that day will still be counted, ‘What if you receive a defective CD on test day? Raise your hand and ask the supervisor for a replacement. Scores For both tests, the total score is reported on the 200-to-800 scale. For the listening test, listening and reading subscores are reported on the 20-to-80 scale. 463 ‘The Official Study Guide for All SAT Subject Tests Sample Reading Questions Your answer sheet has five answer positions marked A, B, C, D, and E, while the questions throughout this test contain only four choices. Be sure NOT to make any marks in column E. PartA Directions: This part consists of a number of incomplete statements, each having four suggested completions. Select the most appropriate completion and fill in the corresponding circle on the answer sheet. 1. Ich glaube, er kommt schon. (A) nachstem (B)_niichster (© niichstes (D)_nachsten -Mittwoch zuriick. Choice (D) is the correct answer to question 1, This question tests your knowledge of the correct weak adjective ending following a presupposed dative-preposition that would answer to the question “when” (“wann”), You need to know that the gender of “Mittwoch” is masculine and that the correct preposition (eliminated here) would be “an.” The entire prepositional phrase would be: an dem (am) niichsten Mittwoch; however, “am” or “an dem” is eliminated, an ellipsis very commonly used in temporal phrases. Choices (A), (B), and (C) cannot structurally be preceded by “am” or “an dem.” Daca German 2. Diesen Sommer konnten die Touristen in Europa gar nicht iiber das Wetter....... (A) sagen (B)_kennen (©) Klagen (D) denken Choice (C) is the correct answer to question 2. This question tests your knowledge of verbs in combination with a negation and a preposition, You are asked to choose the verb that fits best. Given the context (tourists could not complain about the weather in Europe this year), “klagen” is the only possible option that not only fits contextually but also structurally. 3. Annie ist die jtingste Tochter der Familie, bei haben. (A) dem (B) denen (©) der (D) die wir diesen Sommer gewohnt Choice (C) is the correct answer to question 3. This question asks you to choose the correct form of the relative pronoun “die” following the preposition “bei.” You should know that “bei” asks for the dative and that the gender of “Familie,” to which the relative pronoun refers, is feminine. Choice (C) is therefore the only possible answer, since “der” is the dative form of “die.” Part B Directions: In each of the following paragraphs, there are numbered blanks indicating that words or phrases have been omitted. For each numbered blank, four completions are provided. First read through the entire paragraph. Then, for each numbered blank, choose the completion that is most appropriate and fill in the corresponding circle on the answer sheet. Ich verabschiede mich jetzt, weil ich morgen 4. (A) gern (B) frah (©) schon (D) langsam 4650 ‘The Official Study Guide for All SAT Subject Tests aufstehen muss, um Berlin zu einer wichtigen 5. (A) auf (B) an (© nach © w Konferenz 6. (A) fart (8) fahren (© gefahren (D) zu fahren Choice (B) is the correct answer to question 4. This question isa vocabulary question that tests your knowledge of adverbs. You are asked to choose the adverb that fits best. Given the context (the person has to go on a business trip), “fridh” is the most appropriate of the four choices to complement the verb “aufstehen.” Choice (C) is the correct answer to question 5, In this question, you ate asked to choose the correct preposition, which, here is part of an idiomatic expression. You need to know that of the four choices only the preposition “nach” is appropriate in connection with a motion verb (fahren) and the name of a city (Berlin). Choice (D) is the correct answer to question 6. In this question, you are asked to choose the correct form of the verb “fahren.” The infinitive form of “fahren” with “zu” is required because the clause is introduced by “um.” The other choices—(A) third person singular present tense, (B) infinitive without “zu,” (C) past participle—are therefore not, appropriate to form a grammatically correct sentence. D466 German Part C Directions: Read the following texts carefully for comprehension. Each is followed by a number of questions or incomplete statements. Select the answer or completion that is best according to the text and fill in the corresponding circle on the answer sheet. Betreten der Baustelle verboten Eltern haften fir ihre Kinder! 7. Wo findet man dieses S ld? (A) Auf einem Kinderspielplatz (B) An einem Gefiingnis (©) Vor einer Baumschule (D) Auf einem Bauplatz Choice (D) is the correct answer to question 7, This question asks you where you would see such a sign, This sign tells you that you are not to enter the construction site. It continues that parents are responsible for their children's actions. “Bauplatz” in choice (D) isa synonym for “Baustelle.” Both nouns, translated into English, mean construction site, Choice (A) refers to a playground (Kinderspielplatz), choice (B) to a prison (Gefiingnis), and choice (C) to a nursery (Baumschule) 467 ‘The Official Study Guide for All SAT Subject Tests Der Frankfurter Sinkkasten ist ein Verein, der von drei jungen Leuten—Aina, Wolfgang und Werner—in einem Kellergewélbe am Main gegriindet wurde, nachdem sie sich eines Tages entschlossen hatten, ihren Feierabend nicht weiter in Kneipen zu verbringen. Der Sinkkasten verlangt einen Mitgliedsbeitrag von drei Euro monatlich, obgleich es ihm gar nicht um Gewinne geht. Hier konnen aber endlich jeden Abend Jugendliche zusammenkommen und frdhlich sein, Im Sinkkasten treten auBerdem viele prominente Musiker und Gruppen auf. Dazu kommen dann noch interessante Theaterauffiihrungen. Oft werden den Gsten auch sehr gute Filme gezeigt. Junge Maler kénnen hier ihre ersten Werke ausstellen, und regelmaig diirfen die jungen Gaste selbst auch mal Kiinstler spiclen: sie kénnen beim freien Malen ihre bisher verborgenen Talente entdecken. Die schnsten Werke werden anschlieBend ausgestellt. Das Programm ersetzt den Jugendlichen Theater, Kino und Kneipe zugleich. Deshalb kommen sie auch in Scharent Langst hat es sich herumgesprochen, dass man im Sinkkasten ganz nette Leute kennenlernen kann, Die Stadiverwaltung von Frankfurt am Main hat inzwischen den Sinkkasten schitzen gelernt: seit Anfang 1995 wird der Klub vom Kulturamt mit Geld unterstiitzt. 8, Was konnen die Gaste in diesem Klub tun? (A) Ihre eigenen Schépfungen ausstellen (B) Endlich ihre Kochkunst zeigen (©) Ohne monatlichen Beitrag alles mitmachen (D) Die taglichen Hausaufgaben erledigen_ Choice (A) is the correct answer to question 8. In this question, you are asked what club members and guests can do when visiting the “Sinkkasten.” To answer this question, you have to read the second and third paragraphs carefully. Nothing is mentioned with respect to choices (B) “Kochkunst” and (D) “Hausaufgaben.” “Monatlicher Beitrag” in choice (C) is mentioned in the second paragraph (*Mitgliedsbeitrag ... monatlich”), but, itis stated here that each member of the “Sinkkasten” has to contribute 3,-E per month, while choice (C) describes exactly the opposite. Choice (A) Ihre eigenen Schépfungen ausstellen is the only correct answer to the question and is supported by “regelmafiig diirfen die jungen Gaste selbst auch mal Kiinstler spiclen:” ... up to ... “Die schénsten Werke werden anschlieBend ausgestellt.” Dace German 9. Was kann man im allgemeinen tiber den Klub sagen? (A) Er ist das Kulturzentrum der Stadt Frankfurt (B) _Erist finanzieller Mittelpunkt fiir die Stadtvater. (© Erist Anziehungspunkt fir viele junge Leute. (D)_Erist als kultureller Treffpunkt nicht erfolgreich. Choice (C) is the correct answer to question 9, This question asks what can be said in general about this club (“Der Sinkkasten”). The entire reading passage includes information about how and where young people used to spend their free time and how “der Sinkkasten” has changed their habits and what the club means to them, Choice (C) Er ist Anziehungspunkt fiir viele junge Leute summarizes in one sentence this passage and is therefore the only correct answer. Choice (A) describes the club as the cultural center (‘Kulturzentrum”) of the city of Frankfurt, which is obviously never mentioned in the text. Choice (B) refers wrongly to the club as a financial center for representatives of the city government, and choice (D) claims erroneously that the club is unsuccessful as a cultural meeting place. 469 German Test Pract The t tice Helps ‘est that follows is an actual, recently administered SAT Subject Test in German. To get an idea of what it’s like to take this test, practice under conditions that are much D470 like those of an actual test administration. Set aside an hour when you can teke the test uninterrupted. Make sure you complete the test in one sitting. Sit at a desk or table with no other books or papers. Dictionaries, other books, or notes are not allowed in the test room. Tear out an answer sheet from the back of this book and fill it in just as you would on the day of the test. One answer sheet can be used for up to three Subject Tests. Read the instructions that precede the practice test. During the actual administration you will be asked to read them before answering test questions. Time yourself by placing a clock or kitchen timer in front of you After you finish the practice test, read the sections “How to Score the SAT Subject Test in German” and “How Did You Do on the Subject Test in German?” The appearance of the answer sheet in this book may differ from the answer sheet you see on test day. ® 10, Bei mir zu Hause musste man immer vor (D) sich Jnr spat und ich gehe schlafen. (A) fast (B) schon © erst (D) schon 12. Wir sind nicht Auto auszugeben, (A) zufi (B) fenig (© vollstindig (D) imstande +, $0 viel Geld fiir ein neues 13, Franz ist in der Klche und will braten, (A) einen (B) eine © ein (D) eines 14, Wir sind alle zu ihrem Geburtstag cingeladen (A) worden (B) wurden (©) werden (D) geworden Ursuhoreed spring or ue GERMAN TEST—Continued . Teh sage dir nicht, was dieses Wort ® Wie spit ist es? Meine ~ stchen geblieben, (A) Uhr (B) Zeit (C) Stunde (D) Seite Nach ‘gesund, - Krankheit ist ihr Vater endlich wieder (A) lange (B) langer (©) langes (D) langem (A) verste (B) kennt (C) kann (D) bedeutet 6 (A) Brot (B) von Brot (C) Brote (D) dem Brot mir bitte noch ein St GO ONTO THE NEXT PAGE 473) neues Gesehiift Gestern habe ich noch schnell Blumen ftir meine Freundin _(32)_, denn sie hat heute ihre neue Boutique 33)_. Auf dieses Geschiit ist sehr_G4)_, und ich fert davon. SchlieBlich war es meine bin auch beg. 35)_gewesen so etwas anzufangen, [Urata sopying of reuse of sry pa oft page seg D476 33. 34 2. (A) B) © ) ) B) © ©) “) @B) © ) ” @) © ©) GERMAN TEST—Continued gesehen bestellt besucht gewachsen cerschdpft erwacht rift erwartet stolz steif spit starr Zeit Sorge Idee Kunst GO ONTOTHE NEXT PAGE ® GERMAN TEST—Continued © “ART C Directions: Read the following texts carefully for comprehension. Each is followed by a number of questions or incomplete statements. Select the answer or completion that is best according to the text and fill in the corresponding circle on your answer sheet 44. Dieses Verzeichnis informiert aber den Inhalt (A) eines Romans. (B) einer Zeitung (©) eines Pakets. (D) einer Tasche Unauthorized opving ot ouse a] GO ONTO THE NEXT PAGE D478 ® GERMAN TEST—Continued ® SS MARKENS MOTIVE finer Sondermarken-Dauer Marken. sind der Hirnfor Komponistin Fanny Hensel gewidmet. Cécile Vogt (1864-1962), von Geburt Frangisin, forschte viele Johreehnte gemeinsam mit ihrem Mann, dem Newrolo- zen Oskar Vogt. Unier ande- rem gelang den beiden erst: mals" eine Zuordnug von Reizefekten 479) GERMAN TEST—Continued Notruf Iklappe hochheben und festhalten! Warten bis ‘Autobahnmelsterei Heimsheim sich meldet! 49. Wann wiirde man diesen Apparat benutzen? (A) Bei cinem Kinobesuch (B) Nur spat abends (C) Bei einem Unfall (D) Nur friih morgens. Unauhortied oopying or ea] GO ONTO THE NEXT PAGE [any part otis poe tage Daso Der Engel schwieg Von der Erztihlung Der Zug war pilnkilich, Heinrich Bélls erstem Buch, wurden zuniichst etwa 145 Exemplare verkauft. Der noch unbekannte Autor ethielt daftir im Juni 1950 ein Honorar von nnur 58 Mark. Obwohl das sehr wenig war, schrieb Boll weiter. Sein erster Roman Der Engel schwieg wurde im Jahr 1992, also Jahre nach Bots Tod, aufgearbeitet und soll Teil einer kritischen bekannt, dass weit ib den Jahren 1946 bis 1955 noch nicht publiziert sind, Darunter sind Hérspiele, Dramen, Essays und Erzahlungen Bin Kritiker nennt den Roman Der Engel schwieg cin ,charakteristisches Exempel der Heimkehrer- und ‘Trdmmerliteratur”. Der Roman sei ein , literarisches: Dokument’ aber deutsche Zustinde und. Befindlichkeiten im Mai 1945" ‘Aim Tage der Kapitulation kehrt der Soldat Hans. Schnitzler in seine zerbombte Heimatstadt Kn, die auch Bolls Heimatstadt war, zurick. Dieser Roman, snthallt Viel Autobiographisches, Schnitzler hat, wie Boll, Buchhindler gelernt, beide kehiren ohne gultige Papiere heim, beide sind sie Mors Eindrucksvoll schildert BOIl die Stunde Null, den Hunger, den Gestank, die seclische Verwiistung und natiilich dic robe Liebe. shy part othe page op GERMAN TEST—Continued 50, Was erfahren wir diber die Erzihlung Der Zug war pitnktlich ? (A) Sie war Bolls erster Roman. (B) Sic wurde schnell und gut verkauft, (C) Boll bekam nicht viel Geld dafir (D) Boll hat sie 1958 geschrieben. 51. Was erfahren wir iiber Bills Werke Jahren 1946-1955 ? usden (A) Die meisten sind verloren gegangen, (B) Alle sind im Juni 1950 erschienen. (C) Sie sind noch nicht interpretient worden. (D) Viele sind noch nicht verdffentlicht worden. 52. Die Handlung von Heinrich Balls Roman Der Engel schwieg spielt (A) wihrend des Ersten Weltkriegs (B) am Ende des Zweiten Weltkriegs (©) im den zwanziger Jahren (D) in den sechziger Jahren 53. Wovon handelt der Roman Der Engel schwvieg ? (A) Yon einer Zugfahrt (B) Von cinem kranken Kind (C) Von einem wiehtigen Dokument (D) Von einem Heimkehrer 54. Wie sieht ein Kritiker den Roman Der Enget schwieg ? (A) Als Beschreibung des Lebens in Deutschland, tunmittelbar nach Kriegsende (B) Alsein typisches Beispiel der deutschen Klassischen Literatur (C) Als Kapitulation vor den Problemen des Lebens (D) Als Beschreibung der zerbombten Stidte nach dem Krieg. GO ONTO THE NEXT PAGE > 4g1D Der Waldkauz Hu, Huy, Huuuuh, seufzt der Waldkauz durch die laue Friihlingsnacht. Br lisst sich von einem Ast fallen, breitet seine ein Meter groBen Fligel aus und segelt lautlos tiber den Waldboden. Es ist stockdunkel, aber der Kauz, weicht elegant jedem Hindernis a als hatte er eine Infrarot-Brille auf, Hu, Huwu! Wer nnachts den Ruf des Waldkauzes hirt, denkt an das Jammern schrecklicher Gespenster. Dabei ist sein Ruf nichts anderes als eine Liebeserkkirung. Das Huu, Hauu bedeutet: Hallo, Waldkauzin, wo bist du? Ich liebe dich”: Warum findet sich der Waldkauz in der Nacht so gut 2urecht? Er hat besonders lichtempfindliche Augen und sicht deshalb nachts zehnmal besser als wir Menschen. Er sieht war nicht in Farbe, kann aber bei ein bisschen Mondlicht alles erkennen. Noch viel wichtiger ist sein Geh6r. Der Waldkauz, hat cin groBes Trommelfell, hirt im Umks Meter auch das kleinste es nur ein Windhauch oder gutes Gehér kann er zentimetergenau feststellen, wo eine Maus gerade Iiuft. Lautlos fliegt er Uber sein. Opfer, ergreift es und verspeist es auf einer Lichtung. Waldkiuze sind Eulen, Sie leben in Waildern und Parks und werden bis zu 38 em gro® und 10 Jahre alt. Das , Familicnleben" des Waldkauzes ist bokannt. Mit seinem Hu-Huuu-Rufen lockt er ein Weibchen in sein bis zu 20 Hektar groBes Revier und verjagt damit, gleichzeitig andere Mannchen. Seiner Partnerin bleibt er ein Leben lang treu und fittert sie auch, wahrend sie die Bier in einer Baumhohle ausbritet. GERMAN TEST—Continued 56, Welche Wirkung hat das Rufen des Waldkauzes? (A) Es verjagt Gespenster (B) Es erschreckt Rehe. © Es lockt Weibchen an. (D) Es weekt Mannchen auf, 57. Was erfahren wir diber das Gehr des Waldkauzes? (A) Es ist sehr gut entwickelt, (B) Es funktionient nicht bei Dunketheit, (©) Es wird mit zunchmendem Alter besser. (D) Es nimmt keine Tierlaute wah. 58. Warum kann der Waldkauz nachts so gut sehen? ws Augen sind grofer als die von Menschen. (B) Seine Augen sind sehr dunkel. (©) Seine Augen reagieren auf das geringste Licht. (D) Seine Augen leuchten in der Nacht auf. ‘59, Was frisst der Waldkauz wohl? (A) Beeren (B) Kleint (©) Vogeleier (D) Blatter 60. Was erfahren wir Uber das Verhiltnis des Waldkauzes zu einer Partnerin? (A) Er verbringt sein Leben mit nur einer Partnerin (B) Er lebt nur im Sommer mit einer Partnerin (© Er hat mehrere Partnerinnen gleichzeitig (D) Er tilt eine Partnerin mit anderen Mannchen. ‘GO ONTOTHE NEXT PAGE 483) S GERMAN TEST—Continued © Far Ole GitickwUnscse, BLUMER UNd GESCHERKE ANLASSLICH UNSERER HOCHZEIT SAGER WIR GARZ GERZLICH: »OARKE scAON« ROLF UNO ANGELIKA REeUuMEYER 61. Was lesen wir hier? (A) Eine Hochzeit (B) Eine Danksagu (C) Bine Gltickwunsehkarte (D) Bine Binladung aaamoreed spying a use GO ONTO THE NEXT PAGE Dasa © GERMAN TEST—Continued © ‘Wiener Kammerorchester ve Dirigent Krzysztof Penderecki Fahrausweis Kernzone 100 Grosser Saal Bonuskonzert Freie Platzwahl Preis 8S 70,00 175 SNerSmoer 9s, Il 19.30 Uhr 1'931750'310028" ‘A-1037 Wien « LothringerstraBe 20 - Telefon 71212 11 62. Was filreine K. (A) Eine Eintritiskarte (A) Der Bintrtt ist frei (B) Bine Platzkarte (B) Die Plitze sind nicht reserviert. (©) Eine Tel e (C) Man kann umsonst mit dem Bus fahren (D) Eine Postk (D) Man bekommt einen freien Parkplatz st das? 63. Was bedeutet Freie Platzwahl™? a GO ONTO THE NEXT PAGE 485) Peter Wiegand aus Tegernsee So viel ist wohl noch kein Bayer in der Welt herumgekommen: Peter Wiegand aus Tegemsee ist seit 27 Jahren auf allen Meeren unterwegs. Jetzt Konnte der Seemann in Sydney (Australien) cin tolles Jubilium feiern: 30-mal dic Welt umrundet, 3000 000 Kilometer. Es gibt kaum cin Plitzchen dieser Erde, das der 47-Iihrige nicht kent: OD Alaska oder Gronland, ob dic Bora Bora ~ oder Fidschi-Inseln, ob Leningrad oder New York ~ er ist itallen Wassern gewaschen. Dabei hatte Wiegand eine Bilderbuchkarriere: Als 20-Jahriger begann er als Steward auf dem Luxusschiff Berlin. Damals waren Kreuzfahrten noch weitgehend unbekannt, Seit Jahren ist der =Weltenbummler* Hoteldirektor auf Deutschlands, ns, der MS Europa. Dort ist er nach dem Kapitiin der zweite Mann und Chef von 220 Mitarbeiter. Wenn das Schiff — meist ‘morgens — in einen Hafen einlauft, dann ist Wiegand mit dem Proviantmeister auf den Markten dieser Welt unterwegs, um Frischwaren zu kaufen, Neun Monate pro Jahr dampfi der Seebir durch die Welt, freie Tage gibt’s nicht, daftir aber drei Monate durehgehend Urlaub, Den verbringt Wiegand am liebsten im Sommer auf seiner Terrasse mit herrlichem Blick uber den Tegernsee. In der Freizeit verreist der _-Kilometer-Milliond" kaum, geht hdchstens spazieren lund steht als Hobbykoch mit ,,Vorliebe fur exotisches Essen" hinter dem Herd, Luxusliner Nummer [Urata sopying of reuse of sry pa oft page seg GERMAN TEST—Continued 64, Wiegand war in Australien, weil (A) erdort seinen Urlaub verbrachte (B) sein Schiff dort im Hafen tag (©) er Sydneys 30. Jubikium feiemn wollte (D) erdas Land kennen lernen wollte 65, Warum hat Wiegand den Titel ,Hoteldirektor"? (A) In jeder Hafenstadt muss er Zimmer fur die Passagiere finden, (B) Er bucht nur Zimmer fiir seine Mitarbeiter, (C) Ein groBes Luxusschiff ist eigentlich wie cin Hotel (D) Br wind lieber ,Hoteldirektor" als Weltenbummler" genanat. 66. Auf welchem Schiff arbeitet Wiegand jetzt? (A) Auf der Berlin (B) Auf der Sydney (©) Auf der Tegemsee (D) Auf der Europa 67, In der Freizeit hat Wiegand keine Lust, (A) auf der Terrase seines Hauses zu sitzen (B) Zeit in der Ktiche zu yerbringen (C) ausgedehnte Reisen zu machen (D) den Tegemsee 2u besuchen ‘GO ONTOTHE NEXT PAGE ) © GERMAN TEST—Continued ® Rettung fiir Halbaffen? Sie sind so gro8 wie Katzen, ihre Ohren sehen aus wie die von Fiedermausen, und sie haben Schnauzen wie Ratten, Besonders auffillig ist ihr extrem langer Mittelfinger. Die Fingertiere sind die Primaten, die am chesten vor Aussterben bedroht sind, Fingerticre ‘gehiren zu den 30 Lemurenarten, die noch auf der ‘ostafrikanischen Insel Madagaskar lebes Im Primatenzentrum der Duke University (US-Staat North Carolina) ist nun vor wenigen Wochen erstmals ingertier, das man auf Madagaskar ,Aye-Aye™ ‘nent, in Gefangenschatt geboren worden. Die Primatologen haben bereits insgesamt 400 Tiere aus, 15 verschiedenen Lemurenarten in den Wilde von North Carolina aufgezogen, Sie wollen die ausgewachsenen Halbaffen spiiter einmal in besonderen Reservaten auf Madagaskar aussetzen. L Lemuren vor allem durch die Zerstérung der teopischen Walder auf der ostafrikanischen Insel bedroht. Rund 85 Prozent der Biiume sind bereits abgcholzt oder abgebrannt AuBerdem schen viele, die auf Madagaskar leben, dic Aye-Ayes" als Ungliicksbringer; sie verfolgen und taten sie, ider werden die selte authored sopving or ease of] any part ofc page's meget 70, Warum werden die Lemuren in North Carolina, aufgezogen? (A) Sie sollen vor dem Aussterben gerettet werden, (B) Sie sollen dort eine noue Heimat finden. (©) Die Primatologen durfen in Madagaskar nicht arbeiten. (D) Es gibt dort keine Unghiicksbringer. 71. Welche Gefahr herrseht fir die Halbaffen in ihrer urspeinglichen Heimat? (A) Die Insein werden bevolkert (B) Die Walder werden zerstort. (©) Sie werden von Ratten bedroht (D) Es gibt dort nicht genug Primatologen 72, Was haben die US-Wi (A) Sic haben den Aberglauben der Leute aut Madagaskar bekiimpft. (B) Sie haben den selisamen Mittelfinger dieser Tiere entwickelt (©) Sie haben die Halbaffen auBerhalb ihrer Heimat geztichtet (D) Sie haben den Namen der ,.Aye-Aye* bekannt ‘gemacht. nnschaftler erreicht?” 73, Was wollen dic US-Primatologen eines Tages mit den Halbaffen wn? (A) Sie nach Madagaskar zurickbringen (B) Sie den Eingeborenen schenken (©) Sie in North Carolina aussetzen (D) Sic in einem Primatenzentrum behalten 74, Wie sehen viele Einwohner Madagaskars die Halbaffen? (A) Sie halten sie fur hasslieh. finden sie uninteressant nden sie besonders delikat. ©) Sie (D) Sie fulhlen sich von ihnen bedroht GO ONTO THE NEXT PAGE © GERMAN TEST—Continued © die stachelschweine i: im europa-center BERUNER LTERAAISCHES KABANETT MOH Telefon 2614795 Sy Beainn 19:30unr Eintritt DM 30,— aru Woot pene _pusrz No 51836 3 i 75, Was kann man mit dieser Karte machen (A) In den Zoo gehen (B) Im Europa-Center arbeiten (C) Eine Vorstellung besuchen (D) Eine Berlinrundfahrt buchen [Urauthorzed spying ox ruse at GO ONTO THE NEXT PAGE 489) 76, Worauf macht diese Anzeige den Autofahrer 77. Weshalb ist diese An aufmerksam? CN) GERMAN TEST—Continued Achtung Nebel ! ‘Auf diesen Autobahnabschnitien muss jetzt mit Nebel gerechnet werden Letbite Soongobet, sm _Nobelabscnite pales me ‘utobarn ‘Autobahn in Bau ‘Spletels (A) Eine Wetterlage (B) Entfernungen (C) Eine Umteitung. (D) Bauarbeiten [Unaaorsed oop ico. so wichtig? (A) Sie beschreibt Umleitungen, (B) Sie gibt Entfernungen an, (©) Sie weist auf Tankstellen hin. (D) Sic want vor schlechter Sicht GO ONTO THE NEXT PAGE > ® GERMAN TEST—Continued Seelendusche 78, Wer hat die Zelle entworfen? ‘Wire es nicht wunderbar, wenn man einfach eine (8) NASA ‘Telefonzelle betreten und sich aus einer nervosen, unsicheren Person in einen ruhigen, selbstbewussten Supermenschen verwandeln kéinnte? Frank Italiane ‘glaubt an diese Méglichkeit. Deswegen hat seine Firma, die Environ Corporation, eine computer- zgelenkte Kabine entworfen, in der der Mensch eine stressfreie Umgebung findet Der , Environ" Raum ist dazu gedacht, Menschen bei der Bewaltigung von Schmerzen, Stress und. verschiedenen psychologischen Problemen 7u helfen: und zwar durch gefilterte, ionisierte Luft, wohl riechende Difte, multidimensionale KLinge und eine Beleuehtung, die stindig ihre Farbe, Form und Intensitat wechselt. Beim Design des ,Environ" hat man sich die anthropometrische Technologie der NASA zunutze ‘gemacht, Anthropometric ist die Exforschung der Grien-, Gestalt- und Bewegungscharakteristika des menschlichen Kérpers. Sie hat entscheidende Bedeutung beim Design von Kleidung, Ausriistung und Arbeitsplitzen in den Flugkrpern der NASA. Die am Century City Hospital in Los Angeles und im Headache and Pain Center in Beverly Hills, ‘getestete Kabine ist 2,30 m hoch, 1,80 m lang und 1,20 m breit. Die erste Generation des .Environ" wird an Krankenhiuser, Stiftungen und andere medizinische Betriebe verkauft werden, aber bis zum Jahre 2010 sollen billigere Modelle fur den Hausgebrauch auf dem Markt sein, 79. 80. sl. 83, (B) Century City Hospital (© Headache and Pain C (D) Environ Corporation er Wer oder was steuert die Kabine? (A) Ein Psychologe (B) Ein Krankenhaus © Ein Computer (D) Ein Supermensch Wie kann in der Kabine Stress abgebaut werden? (A) Dureh besondere Lichteffekte (B) Dureh Schocktherapie (©) Dureh intensives Training (D) Dureh Schmerzmittel Wer sollte den ,.Environ™ Raum hauptsiichlich benutzen? (A) Verschiedene Psychologen (B) Leidende Menschen (©) Supermenschen (D) Computerspezialisten . Welche Wirkung soll die Kabine haben?” soll Menschen helfen. soll die Welt verinder. (©) Sie soll Telekommunikation verbessern (D) Sie soll zur Abriistung beitragen. Wo sollen diese neuen Kabinen zuerst eingesetzt werden? (A) In Privatwohnungen (B) In Kasernen (© In Kauthausem (D) In Krankenhause GO ONTO THE NEXT PAGE agi) 84, Woflir ist diese Annonce? (A) Eine Arbeitsstelle (B) Ein Hotel in den Alpen (C) Binen Skiuelaub (D) Freizeitkleidung GERMAN TEST—Continued © Sokrotirin © Sachbearbolterin mit EDV-Kenntnis- ‘sen und Englisch aut Zeit arbeiten. Den Wunsch nach einer abwochslungsreichen ‘Tatigkeit solten Sie schon mitoringon Tel. 089/3330. Leopoldstrase 28a 8000 Munchen 40 85, Wenn man bei ,.TeamWork* angestell ist, (A) bekommt man EDV-Kenn (B) lernt man Englisch (©) hat man se efor (D) hat man ausreichende Freizeit yen eigenen T STOP IF YOU FINISH BEFORE TIME IS CALLED, YOU MAY CHECK YOUR WORK ON THIS TEST ONLY. DO NOT TURN TO ANY OTHER TEST IN THIS BOOK. siege German How to Score the SAT Subject Test in German When you take an actual SAT Subject Test in German, your answer sheet will be “read” bya scanning machine that will record your responses to each question. Then a computer will compare your answers with the correct answers and produce your raw score. You get one point for each correct answer. For each wrong answer, you lose one-third of a point. Questions you omit (and any for which you mark more than one answer) are not counted. This raw score is converted to a scaled score that is reported to you and to the colleges you specify. Worksheet 1. Finding Your Raw Test Score STEP 1: Table A lists the correct answers for all the questions on the SAT Subject ‘Test in German that is reproduced in this book. It also serves as a worksheet for you to calculate your raw score. + Compare your answers with those given in the table. + Puta check in the column marked “Right” if your answer is correct. + Puta check in the column marked “Wrong” if your answer is incorrect. Leave both columns blank if you omitted the question. STEP Count the number of right answers. Enter the total here: STEP 3: Count the number of wrong answers. Enter the total here: STEP 4: Multiply the number of wrong answers by .333. Enter the product here: STEP 5: Subtract the result obtained in Step 4 from the total you obtained in Step 2. Enter the result here: STEP Round the number obtained in Step 5 to the nearest whole number. Enter the result here: The number you obtained in Step 6 is your raw score. 493) German Table A continued from previous page pa ight | wiong | Consectiy® a Peete 65 c 6 76 A 32 66 D 81 7 D 40 7 c 45 78 D 0 68 A CF 79 c 51 69 c 6 80 A 46 70 A 2 a 8 a7 n B 82 82 A 60 n c 38 83 D 64 B A 61 a4 A 83 4 D 39 85 D 57 5 c 69 * These percentages are based on an analysis ofthe answer sheets ofa representative sample of 629 students ‘who took the original form of this test in Jane 2001, and whose mean score was 543, They may be used asan indication ofthe relative difficulty of a particular question. Fach percentage may aso be used to predict the likelihood that a typical SAT Subject Test in German candidate will answer that question correctly on this, edition ofthe test 495 D German Table B Scaled Score Conversion Table Subject Test in German (Form 3XAC) ae 800 a 550 9 350 790 43. 530 5 330. 770 a” 520 3 320 750, 37 430 1 300 740 36 4s0 2 300 740, 35 480 3 300 710 4 460 7 280 690 rc] 450 9 20 660 ro 430 13 250 850 23 420 15 240 600 16 380 22 200 560 10 360 28 200 497) ‘The Official Study Guide for All SAT Subject Tests How Did You Do on the Subject Test in German? After you score your test and analyze your performance, think about the following questions: Did you run out of time before reaching the end of the test? If s0, you may need to pace yourself better. For example, maybe you spent too much time ‘on one or two hard questions. A better approach might be to skip the ones you can't answer right away and try answering all the questions that remain on the test. Then if, there's time, go back to the questions you skipped. Did you take a long time reading the directions? You will save time when you take the test by learning the directions to the Subject Test in German ahead of time. Each minute you spend reading directions during the test is a minute that you could use to answer questions. How did you handle questions you were unsure of? Ifyou were able to eliminate one or more of the answer choices as wrong and guess from the remaining ones, your approach probably worked to your advantage. On the other hand, making haphazard guesses or omitting questions without trying to eliminate choices could cost you valuable points. How difficult were the questions for you compared with other students who took the test? Table A shows you how difficult the multiple-choice questions were for the group of students who took this test during its national administration, The right-hand column, gives the percentage of students that answered each question correctly. A question answered correctly by almost everyone in the group is obviously an easier ‘question, For example, 92 percent of the students answered question 4 correctly. But only 19 percent answered question 36 correctly. Keep in mind that these percentages are based on just one group of students. They would probably be different with another group of students taking the test If you missed several easier questions, go back and try to find out why: Did the questions cover material you haven't yet reviewed? Did you misunderstand the directions? Dass German with Listening ‘The Subject Test in German with Listening is offered once a year only at designated test centers, To take the test you MUST bring an acceptable CD player with earphones to the test center. Sample Listening Questions ‘The text in brackets [ ] is only recorded; it will not appear in your test book. The questions in Part A, however, will be recorded and printed in your test book. Please note that the CD does not start here. Begin using the CD when you start the actual practice test on page 504. Your answer sheet has five answer positions marked A, B, C, D, and E, Because the questions throughout this test contain only three or four choices, do NOT make any marks in column E, and do not make any marks in column D if there are only three choices given. PartA Directions: In this part of the test you will hear several selections. They will not be printed in your test book. You will hear them only once. Therefore, you must listen very carefully. In your test book you will read one or two short questions about what was said. Another speaker will read the questions for you. Each question will be followed by four choices marked (A), (B), (C), and (D). The choices are not printed in your test book. You will hear them once. Select the best answer and fill in the corresponding circle on your answer sheet. (Narrator) [Questions 1 and 2 refer to the following exchange] (Woman) [Kénnten Sie mir bitte dieses Kleid heute noch reinigen, (Man) Das ist leider unméglich. Wir machen in einer Stunde, um neunzehn Uhr, Feierabend. 499) ‘The Official Study Guide for All SAT Subject Tests (Woman) Aber bitte, ich muss dieses Kleid unbedingt heute zum Konzert tragen!] 1. (Man) [Wo findet dieser Dialog wohl statt?] Wo findet dieser Dialog wohl statt? (Woman) —_{(A) In einer Reinigung. (B) In einem Konzertsaal. (©) Inciner Boutique. (D) Auf einem Ball) (8 seconds) Choice (A) is the correct answer to question 1. In this question, you are asked to choose from the four answer choices where the short dialogue you just heard takes place. You have to understand the verb “reinigen” and make the connection between “reinigen” mentioned in the dialogue and the noun “Reinigung” in one of the answer choices. Choices (B), (C), and (D) do not apply to “reinigen” and are therefore incorrect. 2, (Man) [Welche Tageszeit ist est?} Welche Tageszeit ist est? (Woman) {(A) Morgen. ®) Mitta © Abend. (D) Nachtmittag] (5 seconds) Choice (C) is the correct answer to question 2. In this question, you are asked to select from the four choices at which time of day the dialogue is taking place. The man in the dialogue says that they “machen Feierabend” in an hour, at 7 p.m. Even if the German 24-hour clock is not known, the use of the word “Feierabend” can also lead to the correct answer. Choices (A), (B), and (D) are incorrect because there is no reference to morning, noon, or afternoon in the dialogue. »500 ‘The Official Study Guide for All SAT Subject Tests 4, (Man) [Warum glaubt Chris, dass Miguel ein guter Schwimmer ist?] (12 seconds) (A) Er besucht die Highschool. (B) Er hat Auszeichnungen gewonnen. (©) Er wohnt in Kalifornien. (D) Er ist im Fernsehen erschienen. Choice (B) is the correct answer to question 4. This question asks why Chris thinks that Miguel is a good swimmer. Chris states in the dialogue that Miguel has won several medals. The use of “Medaillen” (medals) is a more specific way to describe “Auszeichnungen,” which is a more general term, It cannot be inferred from choices (A) and (C) that Miguel is a good swimmer, and choice (D) is not at all mentioned in the dialogue. 5. (Man) [Was will Chris noch vor seiner Reise tun?] (12 seconds} (A) Studienfiicher auswahlen, (B) Medaillen gewinnen. (© Englisch lernen. (D) Viel schwimmen, Choice (C) is the correct answer to question 5. This question asks what Chris wants to do before he goes on his trip. At the end of the dialogue, Chris says that he needs to practice English in order to understand as much as possible in class and to be able to talk with his new family. Since the activities in the other answer choices are not mentioned as something he wants to do before leaving, choices (A), (B), and (D) are incorrect. »502 GERMAN TEST WITH LISTENING The top portion of the section of the answer sheet that you will use in taking the German Test with Listening must be filled in exactly as shown in the illustration below. Noic carefully that you have to do all of the following on your answer sh 1. Print GERMAN WITH LISTENING on the line under the words “Subject Test (print).”, 2. In the shaded box labeled “1 st Code” fill in four circles: —Fill in citele 5 in the row labeled V. lin circle 4 in the row labeled W. —Fill in circle 3 im the row labeled X. —Fill in circle E in the row labeled Y. Test Coss ‘Subject Test prin » OQ800800089 © 0090006065005 GERMAN WITH LISTENING KOOS#OO W008 CE OMOMOMOMOMOMOMOMO] Please answer either Part I or Part II by filling in the specific circle in row Q. You are to fill in ONE and ONLY ONE circle, as described below, to indicate how you obtained your knowledge of German, The information you provide is not Part II IF your knowledge of German does not come primarily from courses taken in grades 9 through Je-9 and leave the remaining circles blank, regardless of how long you studied the subject in school For example, you are to fill in circle 9 if your knowledge of German comes primarily from any of the following sourees: study prior fo the ninth grade, courses taken at a college, special study, living in a home in which German is the principal language spoken, or extensive residence abroad that includes. significant experience in the German language. If your knowledge of German does come primarily from courses taken in secondary school oF the equiva- lent, fill in the circle that indicates the level of the German course in which you are currently enrolled. If you are not now enrolled in a German course, fill in the circle that indicates the level of the most advanced course in German that you have completed. First year: first or second half Fill in circle 1. # Second year: first half ill in circle 2 second half ill in circle 3. © Third year: first half ill in circle 4. second half ill in circle 5. * Fourth year: first half Fill in circle 6. second half Fill in circle 7, # Advanced Placement course for a course at a level higher than fourth year, second half high school course work plus a minimum of four weeks of study abroad —Fill in circle 8. When the supervisor gives the signal, turn the page and begin the German Test with Listening. There are 100 numbered circles on the answer sheet and 87 questions in the German Test with Listening. Therefore, use only circles 1 to 87 for recording your answers. [Unntoviaed copying oy reuse of sry prof hi page fs oga D504 GERMAN TEST WITH LISTENING PLEASE NOTE THAT YOUR ANSWER SHEET HAS FIVE ANSWER POSITIONS MARKED A, B, E, WHILE THE QUESTIONS THROUGHOUT THIS TO MAKE ANY MARKS IN COLUMN E. Dyand ‘TEST CONTAIN ONLY FOUR CHOICES, BE SURE NOT. SECTION I LISTENING Approximate time—20 min Question 1-27 PARTA Directions: In this part of the test you will hear several selections. They will not be printed in your test book. You ‘will hear them only once. Therefore, you must listen very carefully. In your test book you will read one or two short questions about what was said. Another speaker will read the questions for you. Each question will be followed by four choices marked (A), (B), (C), and (D). 1 of printed in your test book. You will hear them Select the best answer and fill in the corresponding circle on your answer sheet. | 1e choices are Listen to the following example. You will hear You will hear and read: ‘Was schenkt der Mann Lisa zum Geburtstag? | You witl hear: ©e00 ‘The best answer to the question Was schenkt der Mann Lisa zum Geburtstag?* is (B), [Ein Buch." Therefore you should choose option (B), Now listen to the first selection, 1. Wie sind Christa und Julia mit Anna verwandt? 3. Wo spielt sich diese Szene ab? Mark your answer on your answer sheet. Mark your answer on your answer sheet. 2. Warum gehen die beiden ins Altersheim? 4. Was will die Frau nicht? ‘Mark your answer on your answer sheet, ‘Mark your answer on your answer sheet, 3YLC TE 0 ONTOTHE NEXT PAGE eaaneto [sonra ienaxrmce ) 505) 5. Worlber sprechen die beiden? Mark your answer on your answer sheet. 6, Was bemerkt Erich am Anfang? Mark your answer on your answer sh 7. Was macht Tante Mia, als der Telefonanrut kommt? Mark your answer on your answer sheet 8, Warum ruft Sonja ihre Tante an? Mark your answer on your answer sheet. any pet of ts prow atogn | »506 GERMAN TEST WITH LISTENING— Continued 10, Was will dieser Mann? Mark your answer on your answer sheet. Warum wird der Mann seinen Zug nicht verpassen’? Mark your answer on your answer sheet. Was hat der Mann nicht verstanden? ct. Mark your answer on your answer sh Was soll es in der folgenden Woche wieder geben’? Mark your answer on your answer sheet GO ONTO THE NEXT PAGE > Auf der Reise 36. Du bist schon wieder da’? Du musst aber schnell ‘gelaufen ===! (A) sein (B) gewesen © sei (D) warst 37. Morgen missen wis fahren um 6 Uhr ab. aufatehen, denn wir (A) frat (B) spit (©) bald (D) langsam Zu Hause 38, Warum sprichst du nicht einmal mit deinem Lehre (A) dariber (B) damit © darin (D) dafar 39. zwei Wochen wohne ich bei meiner Gro&mutter. (A) Vor (B) Seit © Bis (>) Um D510 GERMAN TEST WITH LISTENING— Continued 40, 41 42, 43, Ich wei nicht. kommt. je mit uns ins Restaurant (A) ob (B) dass, (© als (D) wenn Ich war mit meinen Hausaufgaben heute schnell fertig, denn sie waren sehr ~ (A) freundtich, (B) cinzig (© einfach (D) pankttich (A) der (B) das (©) den (D) die hier kalt. Ist ~~ Heizung an? Ich hatte diese politische Lage falsch gesehen und ‘musste meine Meinung (A) nehmen (B) umdrehen (©) ander (D) ausprobieren GO ONTO THE NEXT PAGE ) Pittsburghs Wandlung Wo man heute in eleganten Boutiquen einkauft und in Feinschmeckerrestaurants speist, rauchten frither die Kamine der Bisen- und Stahlindustie. Der grifite Arbeitgeber in der Stadt ist jetzt die University of Pittsburgh". Dazu kommen nspruchsvolle Jobs in 170 Forschungsinstitutionen ‘und 700 Hochtechnologiefirmen, Nach einem kurzen Spaziergang durch Pittsburghs Downtown”, die nach europaischen MaBstben Sauber und sicher ist, ereicht man das William Penn Hotel in der Grant Street. Der Pitsburgher Kohle- Konig Henry Clay Frick lieB es 1916 bauen, weil er tunbedingt das grit und sehénste Hotel zwischen Chicago und New York besitzen wollte. Heute gilt das licbevoll restaurierte Haus mit 595 Zimmern als nziges Grand Hotel* in Pittsburgh. Wenn man die Hotelhalle betrit, fut man sich wie in einem Sehlossssal mit hohen Fensterbgen, Stuckdecken, Kristallleuchtern und vielen Zimmerpalmen. Zum «fternoon-Tea" klimpert ein Klavierspieler heitere Melos jolden Triangle”, der geschaftigen Innenstadlt Pittsburghs, lohnt sich ein kleiner Abs zum Station Square. Uber die Smithfield Steet Bridge, die den Monongahela-Fluss tiberspannt, kommt man zum Pittsburgh & Lake Brie Railroad" Bahnhof. Er wurde 1901 fertig gestellt. Ende der sechziger Jahre rollte der letzte Passagierzug aus dem Gebiiude. Einige Zeit spiter baute Charles Muir aus Dewoit die tunteren Etagen in cin feines Restaurant um: Das Grand Concourse. In der groBen Wariehalle richte cr den Speisesaal fir 500 Gaste cin sondern auch wegen der Universititen, des Museumskomplexes und der weltberahmten Krankenhduser ist die Stadt Pittsburgh flr Reisende aus Deutschland interessant; und auf dem Flughafen ‘cht es so schnell und einfach, wie man es sich in den Warteschlangen auf den Airports in New York ‘und Los Angeles immer wiinscht [any Bar of his page oe GERMAN TEST WITH LISTENING— Continued 64, Wofir war Pittsburgh frdher bekannt? (A) Pur elegante Boutiquen (B) Fur viele Sehulen (C) Pur groBe Krankenhiiuser (D) Fur Schwerindustrie 65. Wo sind die meisten Einwohner von Pittsburgh, heute angestellt? (A) Auf dem Flughafen (B) An der Universitit (C) Im William Penn Hotel (D) Im Grand Concourse Restaurant 66. Wer war Henry Clay Frick? (A) Ein bekannter Architekt (B) Ein Buirgermeister von Pittsburgh (C) Bin Industriemagnat (D) Ein Restaurantbesitzer 67. Wozu wird heute der ehemalige Pittsburgher Bahnhof benutzt? (A) Als Anschlussbahnhof (B) Als Lagerhalle fir Kohle (C) Als Kongresshalle (D) Als gutes Lokal 68. Was war Charles Muir wohl von Beruf? (A) Lokomotivfihrer (B) Muscumsdirektor (C) Baumeister (D) Reiseftihrer 69. Warum lohint es sic (A) Die Stadt hat viel zu bi (B) Pittsburgh hat ein berhmtes Schloss. (C) Die Leute sind freundlich. (D) Die Hotels sind dort sehr billig. Pittsburgh zu besuchen? GO ONTO THE NEXT PAGE 515) Fachbuchhandlung fur Geisteswissenschaften ‘Siamtliche Semesteriteratur haben wir vorratig, ‘oder wir bemihen uns, diese sofort zu, beschatten. Besteliservice auch telefonisch, BesTellBuch 8324051 ‘Sortiment lunterstutzt ihr Studium: kompetent, freundlich, schnell. ‘Thielaliee 34, Mo. bis Fr. 9-18, Sa.10-19 Uhr D516 GERMAN TEST WITH LISTENING— Continued © 70. Was fiir Biicher kauft man bei Tell? (A) Kinderbiicher (B) Reisebiicher (©) Lehrbiicher (D) Kochbiicher 71. Welchen Service bietet die Fachbuchhandlung an’ (A) Biicher konnen ins Haus geliefert werden. (B) Biicher kinnen billiger beschafft werden (C) Biicher kinnen ausgelichen werden, (D) Bucher konnen telefonisch bestellt werden. GO ONTO THE NEXT PAGE ) Der Kranfiihrer Hans Magnussen sah zehn Minuten nach drei, ,Bald Feierabend™, dachte er. Als Magnussen die Baggerschaufel noch einmal herunterlie8, glaubte er, auf etwas Hartes 2u stotlen, Ex stellte den Bagger” ab und kletterte aus dem Kran, uum zu sehen, was es sein konnte. Magnussen holte inen Spaten und begann zu graben. Da fand er eine relativ grofe Kiste, sich herausstellte ~ 23 200 Gold- und Silbermiinzen aus dem 14. und 15, Jahrhundert enthielt. Es war der bedeutendste Fund in der Geschichte der deutschen Miinzen, Wem gehirte nun dieser Gold- und Silberschatz” Das Land, auf dem Magnussen die Kiste entdeckt hatte, gehOrte dem Land Schleswig-Holstein, und ‘hleswig-Holstein hatte auch die Demolierung des Gebiiudes in der Stadt Liibeck angeordnet, unter dem der Schatz verborgen lag. Nicht nur das Land Schleswig-Holstein, auch die Baggerfirma wollte den ganzen Fund fir sich beanspruchen und behauptete, ‘dass ja der Kranfubrer nur in ihrem Auftrag gehandelt hhabe. Das deutsche Gesetz aber bestimmte Folgendes: Wenn etwas entdeckt wird, was schon so lange unter der Erde liegt, dass man den urspriinglichen igentiimer nicht mehr feststellen kann, dann mu der Fund zwischen dem ,.Enidecker" und dem Landeigenttimer geteilt werden. Und so entschied. auch das Bundesgericht. Das Land Jnleswig-Holstein will jetzt den kulturell und historisch wertvollen Fund behalten und wird dem Kranfilhrer Magnussen fiir seinen Anteil 700 000 Mark zablen. * ogger excavator GERMAN TEST WITH LISTENING— Continued 72. Wann entdeckte der Kranfiihrer Magnussen den Fund? (A) Kurz vor Arbeitsseh (B) Um die Mitagszeit (© Am frien Morgen (D) Mitten in der Nacht 73. Warum stellte Magnussen (A) Er dachte, es sei schon Feierabend. (B) Der Bagger funktionierte nicht richtig. (©) Der Schutt war 2u schwer fir die Schaufel (D) Ex wollte wissen, was unter der Erde war. len Kran ab? 74. Das Land Schleswig-Holstein wollte den ganzen Fund behalten, weil (A) ihm das Land geharte (B) Magnussen in Libeck wohnte (©) Magnussen das Gebaude zerstorte (D) der Eigentiimer im Bundesgericht arbeitete 75. Wie alt waren die Minzen? (A) Fast hundert Jahre (B) Etwas ber tausend Jahre (© Genau fintzig Jahre (D) Mehrere hundest Jahre 76. Das Bundesgericht entschied, dass (A) die Baggerfiema die Munzen behalten durfte (B) Schleswig-Holstein und Magnussen den Fund, teilen sollten (© der Eigentimer des Fundes gesucht werden sollte (D) die Demoticrungsarbeiten abgebrochen werden sollten 77. Wer bekam am Ende den Schatz? (A) Die Stadt Litbeck (B) Die Baggerfirma (©) Das Land Sehleswig-Holstein igliche Bigentiimer GO ONTO THE NEXT PAGE > 517) © GERMAN TEST WITH LISTENING— Continued Nordamerika riickt ein Drittel naher : ‘* Haben Sie Gaschaft Freunde odé i escteparre Jer Angentige in «* Teleforiren Sie of in ce Vererigten Staaten oder nach Kanada? \Wenn Se eine dleser Fragen mit ja beantworten kénnen, dann haben Si einen Grund 2ur Freud: Seit 1. Mai telefonieren Sie 37% ginstiger von Deutschland in die USA und nach Kaneda. ‘86. Was will man mit dieser Anzeige mitteilen? (A) Man soll sich Freunde in Amerika suchen. (B) Deutsche reisen éfter nach Nordamerika. (C) Man soll viel mehr in der ganzen Welt telefonieren. (D) Anrufe nach Nordamerika sind jetzt billiger. [Unauthorised spying of reuse of ‘GO ONTOTHE NEXT PAGE > 521) How to Score the SAT Subject Test in German with Listening ‘When you take an actual SAT Subject Test in German with Listening, you receive an overall composite score as well as two subscores: one for the reading section, one for the listening section. The reading and listening scores are reported on the College Board’s 20-80 scale. However the composite score, which is the most significant of the scores reported to the colleges you specify, is in the form of the College Board’s 200-800 scale. Worksheet 1. Finding Your Raw Listening Subscore STEP 1; Table A lists the correct answers for all the questions on the SAT Subject Test in German with Listening that is reproduced in this book. It also serves as a worksheet for you to calculate your raw Listening subscore. + Compare your answers with those given in the table. + Puta check in the column marked “Right” if your answer is correct. + Puta check in the column marked “Wrong” if your answer is incorrect. + Leave both columns blank if you omitted the question. STEP 2: Count the number of right answers for questions 1-27. Enter the total here: STEP 3: Count the number of wrong answers for questions 1-27. Enter the total here: STEP : Multiply the number of wrong answers by .333. Enter the product here: STEP Enter the result here: 5: Subtract the result obtained in Step 4 from the total you obtained in Step 2. STEP 6: Round the number obtained i Step 5 to the nearest whole number: Enter the result here: The number you obtained in Step 6 is your raw Listening subscore, 523 ‘The Official Study Guide for All SAT Subject Tests Worksheet 2. Finding Your Raw Reading Subscore STEP 1: Table A lists the correct answers for all the questions on the SAT Subject Test in German with Listening that is reproduced in this book, It also serves as a worksheet for you to calculate your raw Reading subscore. STEP 2: Count the number of right answers for questions 28-87, Enter the total here: STEP 3: Count the number of wrong answers for questions 28-87. Enter the total here: STEP 4: Multiply the number of wrong answers by .333, Enter the product here: STEP 5: Subtract the result obtained in Step 4 from the total you obtained in Step 2. Enter the result her STEP 6: Round the number obtained in Step 5 to the nearest whole number. Enter the result here: The number you obtained in Step 6 is your raw Reading subscore. D524 Worksheet 3. Finding Your Raw Composite Score STEP 1: Enter your unrounded raw Reading subscore from Step 5 of Worksheet 2 Enter the result here: STEP 2: Enter your unrounded raw Listening subscore from Step 5 of Worksheet 1 Enter the result here: STEP Add the result obtained in Step 1 to the result obtained in Step 2. Enter the result here: STEP 4: Round the number obtained in Step 3 to the nearest whole number. Enter the result here: The number you obtained in Step 4 is your raw composite score. 5250 ‘The Official Study Guide for All SAT Subject Tests Table A Answers to the Subject Test in German with Listening, Form 3YLC, and Percentage of Students Answering Each Question Correctly errr an Right | Wrong A a7 D 38 A 80 B 64 c 95 A @ 8 62 D n A 2 0 8 64 1 8 oI 2 D 86 8 A 4 4 D 56 5 D 88 16 0 5 ” A 68 1B A 49 18 B 52 2 o 8 2 B 80 2 c 6 2 A 46 a c 55 Ed 8 % 6 a 39 a D 3 a D 64 2 A 95 30 B 1 31 a 2 2 D n Table A continued on next page 2526 German with Listening Table A continued from previous page 6 B 6 7 c 67 6 c 50 78 c 54 oa D 46 73 D 2 68 c B 80 8 a 6 A 90 at A 66 70 c cy a 8 56 n D 5 83 c 54 2 A 7 84 D 82 B o 55 85 c a m A 7 86 D 87 8 D 68 a7 A 50 18 B 59 “These percentages are based on an analysis ofthe answer sheets of representative sample of 483 students who took the original form of this test in November 2002, and whose mean composite score was 589. They may bbe used as an indication of the relative difficulty ofa particular question, Each percentage may also be used to predict the likelihood that a typical SAT Subject Test in German with Listening candidate will answer that question correctly om this edition ofthe tes. 527 ‘The Official Study Guide for All SAT Subject Tests Finding Your Scaled Score When you take SAT Subject Tests, the scores sent to the colleges you specify are reported on the College Board scale, which ranges from 200-800, Subscores are reported on a scale which ranges from 20-80. You can convert your practice test scores to scaled scores by using Tables B, C, and D. To find your scaled score, locate your raw score in the left- hand column of the table; the corresponding score in the right-hand column is your scaled score. For example, a raw score of 47 on this particular edition of the Subject Test in German with Listening corresponds to a scaled composite score of 570. Raw scores are converted to scaled scores to ensure that a score earned on any one edition of a particular Subject Test is comparable to the same scaled score earned on any other edition of the same Subject Test. Because some editions of the tests may be slightly easier or more difficult than others, College Board scaled scores are adjusted so that they indicate the same level of performance regardless of the edition of the test taken and the ability of the group that takes it. Thus, for example, a score of 400 on one edition of a test taken ata particular administration indicates the same level of achievement as a score of 400 on a different edition of the test taken at a different administration, When you take the SAT Subject Tests during a national administration, your scores are likely to differ somewhat from the scores you obtain on the tests in this book. People perform at different levels at different times for reasons unrelated to the tests themselves. The precision of any test is also limited because it represents only a sample of all the possible questions that could be asked. Your scaled composite score from Table B is : ‘Your scaled listening score from Table C is Your scaled reading score from Table D is : p528 German with Listening Table B Scaled Score Conversion Table Subject Test in German with Listening Composite Score (Form 3YLC) 87 ‘800 49 580 1 390 86 780 48 580 10 390 85 770 a7 570 3 380 84 770 46 570 8 370 83 760 45 510 7 370 82 760 4 560 6 360 81 750 43 580 5 360 80 750 2 580 4 380 79 70 41 550 3 380 78 40 40 540 2 340 7 740 33 540 T 30 16 730 38 540 0 330 B 730 37 530 4 330 m 720 36 530 2 330 2 720 35 520 3 320 7 710 30 520 4 320 n 710 33 510 5 320 7 700 32 510 8 310 6 700 31 500 7 310 68 690 30 500 8 310 7 690 3 490 3 310 86 680 2 490 10 300 65 680 a 480 it 300 64 670 6 480 2 300 53 660. B 470 13 300 82 660 2 470 4 300 Bt 650 2 480 45 300 60 650 2 460 48 300 59 640 2 450 7 300 58 630 20 450 8 290 57 630 0 440 319 290 56 620 8 430 0 290 Ea 620 v 430 a 230 54 610 16 420 2 290 53 600 5 420 23 290 52 600 4 410 77 7290 51 590 B 400 B 290 50 590 2 400 28 290 2 280 8 280 2 20 529 German with Listening Table D Scaled Score Conversion Table Subject Test in German with Listening Reading Subscore (Form 3¥LC) — 59 58 5 38 46 69 19 49 B 3 4a 67 7 48 410 30 4 65 4 46 13 30 a 61 10 42 a 2 5310 ‘The Official Study Guide for All SAT Subject Tests How Did You Do on the Subject Test in German with Listening? After you score your test and analyze your performance, think about the following question Did you run out of time before reaching the end of the test? If'so, you may need to pace yourself better. For example, maybe you spent too much time on one or two hard questions. A better approach might be to skip the ones you can't answer right away and try answering all the questions that remain on the test. Then if there’s time, go back to the questions you skipped. Did you take a long time reading the directions? You will save time when you take the test by learning the directions to the Subject Test in German with Listening ahead of time. Each minute you spend reading directions during the test is a minute that you could use to answer questions, How did you handle questions you were unsure of? If you were able to eliminate one or more of the answer choices as wrong and guess from the remaining ones, your approach probably worked to your advantage. On the other hand, making haphazard guesses or omitting questions without trying to eliminate choices could cost you valuable points. How difficult were the questions for you compared with other students who took the test? Table A shows you how difficult the multiple-choice questions were for the group of students who took this test during its national administration, The right-hand column. gives the percentage of students that answered each question correctly. A question answered correctly by almost everyone in the group is obviously an easier question. For example, 95 percent of the students answered question 37 correctly. But only 27 percent answered question 80 correctly. Keep in mind that these percentages are based on just one group of students. They would probably be different with another group of students taking the test. If you missed several easier questions, go back and try to find out why: Did the questions cover material you haven't yet reviewed? Did you misunderstand the directions? D532 Chapter 11 Italian Purpose ‘The Subject Test in Italian measures your ability to understand written Italian. The test allows for variation in language preparation and is independent of particular textbooks or methods of instruction. The test measures reading proficiency based on communicative materials authentic to the Italian culture. Format This is a one-hour test with 80 to 85 multiple-choice questions. Test questions are written to reflect current trends in high school curricula and to test reading skills and familiarity with the language structure. Content Questions range in difficulty from elementary through advanced, although most questions are at the intermediate level. The test measures reading proficiency through a variety of questions requiring a broad knowledge of the language. The test covers commonly taught grammatical constructions, and all questions reflect current standard Italian. The test includes three parts: Sentence completion questions test your knowledge of high-frequency vocabulary and appropriate idiomatic expressions in the context of paragraphs. Structure questions test your familiarity with the language structure. Reading comprehension questions test your understanding of the content of various, selections taken from sources such as newspaper and magazine articles, prose fiction, historical works, advertisements, tickets, brochures, forms, and schedules. 533 ‘The Official Study Guide for All SAT Subject Tests Italian or ences pier er Vocabulary in Context 30 ‘Structure in Blank 0 Reading Comprehension a How to Prepare ‘The Subject Test in Italian allows for variation in language preparation. You should develop competence in Italian over a period of years by taking two to four years of Italian language study in high school or the equivalent. Familiarize yourself with the directions in advance. The directions in this book are identical to those that appear on the test. Score ‘The total score is reported on the 200-to-800 scale. Sample Questions Your answer sheet has five answer positions marked A, B, C, D, and E, while the questions throughout this test contain only four choices, Be sure NOT to make any marks in column E. PartA Directions: In each of the following passages there are numbered blanks indicating that words or phrases have been omitted. For each numbered blank, four completions are provided. First read through the entire passage, Then, for each numbered blank, choose the completion that is most appropriate given the context of the entire passage ang fill in the corresponding circle on the answer sheet. Un piccolo villaggio siciliano Baria Dorica @ il nome di un piccolo villaggio estivo situato sulla costa siciliana. Tutte le _1_ del villaggio danno sulla piazzetta, dove si trova l'unico locale pubblico: il bar, che fa anche da panineria, pizzeria e panetteria. D534 ‘The Official Study Guide for All SAT Subject Tests 6. In quest’ annuncio, che cos’ & “Il Venerdi”? (A) Un libro (8) Unsettimanale (©) Un notiziario (D) Un’ inchiesta 7. Come si ottiene “Il Venerdi”? Choice (B) is the correct answer to question 6, This question asks what “I! Venerdi The text mentions tutte le settimane and also “ll Venerdi” ¢ in edicola ogni venerdi, (A) Sideve andare in un negozio. (B)_Sideve comprare la Repubblica. (©) Sidevono spendere due euro. (D) Sideve aspettare la fine del mese. ‘Choices (A), (C), and (D) are incorrect within the context of the ad. Choice (B) is the correct answer to question 7. This question asks about how you obtain “TI Venerdi.” The text mentions ‘I! Venerdi” ) con Repubblica, and also (,..) insieme a Repubblica. Choices (A), (C), and (D) are incorrect within the context of the ad. Questions 8-10 E “Ferragosto”, festa nazionale, ¢ se ne sono andati tutti, Restano solo alcune auto, abbandonate lo scorso inverno, ancora pit solitarie sotto il sole d’agosto. E incredibile, poter attraversare Milano in un quarto dora, da un capo all’altro. E poi fermarsi e parcheggiare dove si vuole. Bellissimo, ma per fare che cosa, se é tutto chiuso da una settimana? ... Vado all’edicola e la trovo sprangata. I] tabaccaio pitt vicino adesso si trova a un chilometro di distanza, e non ha pit francobolli. Se in questo momento si fulmina una lampadina di casa sono perduto, non saprei dove comprarne una. Fortuna che per cibi e bevande mi ero fatto una scorta, Lassedio durera fino al giorno 20, e occorre resistere. Dé resto non mi é mai piaciuto lo spettacolo di questo fuggi fuggi, di questo esodo di massa, come se a Milano fosse scoppiata unlepidemia di peste. 2538 Italian Test Practice Helps The test that follows is an actual, recently administered SAT Subject Test in Italian. To get an idea of what it’s like to take this test, practice under conditions that are much like those of an actual test administration. © Set aside an hour when you can take the test uninterrupted. Make sure you complete the test in one sitting. © Sit ata desk or table with no other books or papers. Dictionaries, other books, or notes are not allowed in the test room. © — Tear out an answer sheet from the back of this book and fill it in just as you would on the day of the test. One answer sheet can be used for up to three Subject Tests. © Read the instructions that precede the practice test. During the actual administration you will be asked to read them before answering test questions. © Time yourself by placing a clock or kitchen timer in front of you © After you finish the practice test, read the sections "How to Score the SAT Subject Test in Italian” and “How Did You Do on the Subject Test in Italian?” © The appearance of the answer sheet in this book may differ from the answer sheet you see on test day. »540 ITALIAN TEST ‘The top portion of the section of the answer sheet that you will use in taking the Ttalian Test must be filled in exactly as shown in the illustration below. Note carefully that you have to do all of the following on your answer sheet. |. Print ITALIAN on the line under the words “Subject Test (print). In the shaded box labeled “Test Code” fill in four cireles: —Fill in circle 1 in the row labeled V. —Fill in cirele 4 in the row labeled W. —Fill in circle 2 in the row labeled X. —Fill in cirele B in the row labeled Y. eos at ¥ @5905900000 eae * 000890000 x0 @000 e050 COMO MOMONGMOMOMONO Please answer either Part I or Part Il by filling inthe specific circle in row Q. You are to fill in ONE and ONLY ONE rcle, as described below, to indicate how you obtained your knowledge of Italian. The information you provide is for statistical purposes only and will not influence your score on the test Part I your knowledge of Italian does not come primarily from courses taken in grades 9 through 12, fill in circle 9 and leave the remaining circles blank, regardless of how long you studied the subject in school. For example, you are to fill in circle 9 if your knowledge of Italian comes primarily from any of the following sources: study prior to the ninth grade, courses taken at a college, special study, living in a ‘home in which Italian is the principal language spoken, or extensive residence abroad that includes, significant experience in the Italian language. Part Il If your knowledge of Italian does come primarily from courses taken in secondary school, fill in the circle that indicates the level of the Italian course in which you are currently enrolled. If you are not now enrolled in an Italian course, fill in the circle that indicates the level of the most advanced course in Italian that you have completed. ‘© First year: first or second half —Fill in circle 1 © Second year: first half Fill in circle 2 second half —Fill in circle 3, ‘© Third year: first half —Fill in circle 4 second half —Fill in circle 5. © Fourth year: first half —Fill in circle 6, second half —Fill in circle 7. Course at a level higher than fourth year, second half or high school course work plus ‘a minimum of four weeks of study abroad Fill in circle 8, ‘When the supervisor gives the signal, tum the page and begin the Italian Test. There are 100 numbered circles on the answer sheet and 82 questions in the Italian Test. Therefore, use only circles 1 to 82 for recording your answers. (Unarised spying oe ruse at [any part of tis pose toga 541) ITALIAN TEST Part A -ctions: In each of the following passages there are numbered blanks indicating that words or phrases have been omitted, For each numbered blank, four completions are provided. First read through the entire passage. ‘Then, for each numbered blank, choose the completion that is most appropriate given the context of the entire passage and fil in the corresponding circle on the answer sheet. :.in_un nuovo paese 1 (A) visitare iavazione degli immigrani che vengono a (2) patire (D)_ in un nuovo paese & qualche volta motto (D) accosti (2) _. AWinizio si sentono _@)_. Ci sono 2. (A) saggia (B) difficile spesso motte differenze fra Ia loro vecehia cultura (C disobitata quella nuova, Sono obbligati ad assumere una (D) irresponsabile nuova identita ed ad imparare una nuova lingua. 3. (A) svegli (B) isola Tbambini (4) _ abbastanza velocemente la (©) indipendenti lingua del paese, ma gli adulti continuano _ (3) (D) congenial 4 parlare il dialeto del loro paese. 4. (A) vivono . (B) imparano (©) discutono (D) consigliano 5. (A) (B) dopo (©) molto (D) spesso 3VAC GO ONTO THE NEXT PAGE > ‘Teasporti Posti in aereo non se ne trovano pi Volete _(11) _ al minimo ta spesa delle vostre vacanze in una capitale europea? Beco la soluzione ai due 2) _: viaggiare in autobus. Da Milano e da Roma, ma anche da Bologna, Firenze ¢ Genova _(13)__ ogni giorno molti pullman geanturismo che collegana le principali citta d’ Europa, Rispetto all'aereo, il biglietto costa meno as) (14) _ 1 viaggio dura diverse ore pi certo. In ogni caso, gli autobus viaggiano quasi sempre in (16) __, ¢ il tragitto & diretto, senza an. I biglieto rimane valido per sei nesi. Ogni passeggero pud portare con s® una valigia e un bagaglio a _(18) Smee p5aq ITALIAN TEST— Continued ul 16. (A) includere (B) dare (©) ridurre (D) giungere (A) soggetti (B) lavori (©) mezzi (D) problemi (A) pagano (B) partono (C) stendono (D) toceano (A) delenergia (B) deieconomia (C) delta gita (D) celta meta (A) del velo (B) del volo (C) della vela (D) della volta (A) stazione (B) autostrada (©) silenzio (D) compagnia (A) direzione (B) deviaziont (©) destinazioni (D) pazienza (A) mano (B) piedi (©) terra (D) voce GO ONTO THE NEXT PAGE > ITALIAN TEST — Continued Part B. Directions: In each sentence or dialogue below you will find a blank space indicating that a word or phrase has been omitted. Following each sentence are four completions. OF the four choices, select the one that best com- pletes the sentence structurally and logically and fill in the corresponding circle on the answer sheet. In some instances, choice (A) consists of dashes; by choosing this option, you are indicating that no insertion is required to form a grammatically correct sentence. 25. sono anche cartoline ¢ giornali 28. A quest’ora domani —---- gia Venezia, . (A) sia (A) illustrata (B) saremo (B) illustrat (©) eravamo (© illusteato, (D) fossimo (D) illustrate 29. Ci sari anche mio cugino. Sono cinque anni che 26. Buon giomo signora, ------ qualche rivista, non ----- vediamo. americana’ ws (YE (B) ne @) Ha © mi © Sia (Da (D) Abbia 30. A che ora mi consighi di ——-—- da casa”? 27 se vuole. (A) parto (A) aiutarmi (B) parti (B) aiutarci (C) partire (© aiutarLa (D) partito (D) aiutarsi 31. La ragazza ha viaggiato in aereo con -—---- amici, (A) dei (B) nessuno, (©) aleuni (D) tut 32. Sei sicura di avere chiuso (A) bene (B) buona (C) migliore (D) buonissima a porta a chiave? 33. Mi piace il colore (A) delle tue aligie. (©) nelle (D) sulle [Gamera pine ama GO ONTOTHE NEXT PAGE > D546 44, Vorrei moderna, di (B) un po" © niemte (D) aleune 48, - registrazioni di musica popolare 49, possono comprare qui? A vi (B) me Os Ou 50. Lo sai Anna, ----~ piacciono i complessi italiani che suonano ad alto volume. (A) mi (B) me © lo (D) la 47. Quel pianista & bravissimo. L"ho sentito ~ alla Seala. OY) B) © ) suonato suonava suond [any part ofthis pege stage D5aB ITALIAN TEST — Continued Al cinema Rex danno ~ Massimo Troisi (Ail (B) | © lo (D) la ultimo film con Laura, - (A) ti (B) tei Ou (D) si - piace la pittura moderna? ‘Nel museo ci sono cataloghi splendid! Li -—~-, se non costassero troppo. (A) comprerei (B) comprerd (©) comprai (D) compro GO ONTO THE NEXT PAGE > ITALIAN TEST—Continued Part Directions: Read the following texts carefully for comprehension. Each text is followed by a number of ques- tions or incomplete statements. Select the answer or completion that is best according to the text and fill in the ‘corresponding circle on the answer sheet. Questions $1 Per una gita diversa lontana dallo stress, in pieno relax ‘a due passi da voi CAMPING CITTA DI ANGERA ristorante tipico con cucina intemazionate € piat tipi local, amp saloni per bbanchet, piscine, tennis, campi bocce, pallavole, ping-pong, 549 © ITALIAN TEST — Continued © up inane eto QUAL E’ LO YOGURT PIU’ NUTRIENTE? LINSETTICIDA PIU’ EFFICACE? IL TONNO PIU’ GUSTOSO? LA SPIAGGIA PIU’ PULITA? 58. Cosa® Qualita? (A) Ungi GO ONTO THE NEXT PAGE > 551) © ITALIAN TEST — Continued © LINFORMATICA A TUA IMMAGINE E SOMIGLIANZA. GO ONTO THE NEXT PAGE > 553) a0) In Italia incombe la paura di gigantesche catastrofi naturali, Acqua, terra e fuoco colpiscono il Belpacse con tragica regolarit’ e straordinaria forza, La prima ‘minaccia sono i vuleani. Tre sono fra i pitt pericolosi: Vesuvio, Campi Flegrei ¢ Vuleano. secondo incubo sono i terremoti. In realta quasi tutte le region’ italiane sono a rischio. Di edifici antisismici, che resistono ai terremoti, perd, ne sono stati costruiti davvero pochi, Un'altra minaceia sono le industrie. In Italia ci sono quattrocento Fabbriche a grave rischio ambientale: stabilimenti chimici, farmaccutici, raffinerie rischiano di trasformarsi in bombe tossiche. Ma non basta. Ogni anno ¢"? un appuntamento fisso: emergenza incendi che ogni estate divora una ‘gran quantita di vegetazione. Negli ultimi quindici anni in Sardegna, in Liguria e in Piemonte intere foreste sono andate in fumo in poche ore. 68. Secondo il brano, gli italiani temono (A) le esplosioni @B) Fincubo (C) idisastri (D) it fumo [Unnutorized copying of rung any poof the page segs D554 ITALIAN TEST—Continued 69. 70. mL. S) Cosa intende Pautore con Ia parola “Belpaes (A) Una regi (B) L’Italia (©) Un vuleano (D) Lacampagna Gili edifici antisismici servono a (A) prevenire i terremoti (B) proteggere l'ambiente (C) segnalare il pericolo (D) ridurre i danni Larticolo sostiene che I’industria italiana (A) da Vallarme (B) costruisee le eentrali nucleari (C) propone un referendum (D) intossiea Pambiente 2. Secondo I'articolo, gli incendi distruggono (A) le sole (B) le spingge (©) i bosch (D) i villaggi GO ONTO THE NEXT PAGE > © ITALIAN TEST —Continued Questions 73-74 Serie S BASILICA DI_SAN MARCO - VENEZIA. in occasione dela visita ata Pala d'Oro Mesias Lire 3.000 'IGUETTI D1 CONTROILO (ON CONSEAVARE) Offers per i euto © 41828 per ti decoro dia Basilica, GRAZIE 73. Questo biglieto serve per (A) pregare nella Basilica (B) vedere la Pala d'Oro (©) controllare le offerte (D) visitare Venezia [any poof hi ge esa 74. 1 soldi pagati per questo biglietio sono usati per (A) acquistare libri sulla Basilica (B) conservare i gioielli (C) decorare la Basili (D) ringraziare i turisti GO ONTO THE NEXT PAGE > 555) ® ITALIAN TEST —Continued ions 7 Que 8 75. Dove si trova la narratrice? (A) Allo 200, (B) Aleinema (©) In strada (D) Incasa Dapprincipio non ho fatto caso al rumore. Veniva dal pianerottolo ed era come un lievissimo rosiechiare 4i topo, mescolato ai tanti cigolii, ronzii tonfi Lincs condominiali, al basso continuo del traiTico cingue (9) pani pid sotto. Uno di quei suoni piccoli,insinuanti, mi si& infilato nelle orecchie senza che me ne a (A) Quando artiva il marito ‘Quel suono era troppo leggero; per la pr. (B) A poco a poco furtivo. (©) Verso le cingue (10) Ho messo V occhio allo spioneino e ho visto il mio (D) Prima di andare a dormire 76. Quando si rende conto del rumore? casa, Ho spalancato; mi & quasi caduto tra le braceia 77. Che stava facendo Vex marito? ‘Teneva in mano un minuscolo eacciavite. Riccardo, che fai?” 1 venisse Ia jn del ladro sorpreso a rubare. Niente. Oh, ciao,” ha detto. “Prendevo la targa.’ Aveva gilt finito di staccare la placea cirelee con il nome: Riccardo Prini. Sono rimasta a guardarlo, senza fiatare. (A) Faceva riparazioni (B) Regolava il traffico. (C) Lucidava la porta, (D) Toglieva la targa. 9 Fa 78. Come reagisce la narratrice alla situazione? (A) Tace per la sorpresa. (B) Chiama immediatamente la polizia. (C) Finisce il lavoro. (D) Stacea la placca. any pa ofthis pags ies GO ONTO THE NEXT PAGE ) D556

You might also like